You are on page 1of 175

2007

2007
MN: Mining
MN: Mining Engineering
Engineering

Duration
Duration :: Three
Three Hours
Hours Maximum
Maximum Marks
Marks ::150
150

Read the
Read the following
following instructions
instructions carefully.
carefully.

1. This
1. This question paper contains
question paper contains 85
85 objective
objective type
type questions.
questions. Q.I
Q.l to
to Q20
Q.20 carry
carry one
one mark
mark
each and
each and 0.21
Q.21 to
to 0.85
Q.85 carry
carry two
two marks each.
marks each.

2. Attempt
2. Attempt all
all the
the questions.
questions.

3. Questions
3. must be
Questions must be answered on Objective
answered on Response Sheet
Objective Response Sheet (ORS)
(ORS) by darkening the
by darkening the
appropriate
appropriate bubble
bubble (marked
(marked A,
A, B,
B, C, D) using
C, D) using IHB
lB pencil
pencil against the question
against the question number
number on
on
the left
the left hand side of
hand side of the ORS. Each
the ORS. question has
Each question only one
has only one correct
correct answer.
answer. In
In case
case you
you
wish
wish to change an
to change an answer,
answer, erase
erase the old answer
the old answer completely.
completely.

4. Wrong
4. Wrong answers
answers will
will carry
carry NEGATIVE
NEGATIVE marks.
marks. In Q]
In to Q20.
Q.I to 0.25 mark
Q.20, 0.25 mark ""ill
will be
be
deducted
deducted for
for each
each wrong
wrong answer.
answer. In
In Q2]
Q.21 to Q76, Q13,
to Q.76, Q.78, Q30,
Q.80, 0.82 and in
Q.82 and 0.34, 0.5
in Q.84, 0.5
mark will be
mark will be deducted
deducted for
for each
each wrong answer. However,
wrong answer. However, there
there is
is no
no negative
negative marking
marking in
in
Q77, Q.79,
Q.77, Q79, Q.81,
0.81, Q.83
Q33 and
and in Q35. More
in Q.85. More than one answer
than one answer bubbled against aa question
bubbled against question
will be
will be taken
taken as
as an incorrect response.
an incorrect response. Unattempted
Unattempted questiOns
questions will
will not carry any
not carry any marks.
marks.

5. Write
5. your registration
Write your registration number,
number, your
your name and name
name and name of
of the examination centre
the examination centre at the
at thl!
Specified locations
specified locations on
on the
the right
nght half of the ORS.
halfofthe ORS.

6. Using
6. Using HB
HR pencil, darken the
pencil, darken the appropriate
appropriate bubble under each
bubble under each digit
digit of
of your
your registration
registration
number and the
number and the letters
letters corresponding
correSponding to
to your
your paper
paper code.
code.

7.
7. Calculator is
Calculator is allowed in the
allowed in the examination
examination hall.
hall.

3. Charts,
8. Charts, graph
graph sheets
sheets or
or tables are NOT
tables are allowed in
NOT allowed in the examination hall.
the examination hall.

9. Rough
9. work can
Rough work can be
be done
done on
on the question paper
the question itself. Additionally
paper 1tself. blank pages
Additionally blank pages are given
are given
at
at the
the end of the
end of question paper
the question paper for rough work.
for rough ~Aork.

10.
10. This
This question
quest1on paper
paper contains 20 printed
contains 20 printed pages
pages including
including pages
pages for
for rough
rough work. Please
v. urk. Plea'ie
check all
check all pages and report,
pages and if there
report, if is any
there is any discn.:pancy.
discrepancy.

MN 1.320
MN J/20
31121 Food/06-MN-lA
S/121 Fuuwoe—MN—m

www.examrace.com
www.examrace.com
Q. 11 —
Q. Q. 20
- Q. carry one
20 carry one mark
mark each.
each.
Q] If the slope of
of aa diagonal
diagonal of
of aa rectangle
rectangle is m the slepe of
of the
Q.l If the slope is m the slope other diagonal
the other diagonal is
is

(A) 31—
(A) _I
m
2m (B) —i
1
(B)-- (C) i
(C) _!_
(D) —i
1
(D)--
2m m
m m
Q2
Q.2 If the
If the rank
rank of
of aa matrix
matrix A
A is
is r, the the matrix
the rank of the matrix A.r
AT is
is

(A) r,r, if
(A) and only
if and only if AT = A
if AT= A (B) r,r, for
(B) for all
all A
A
(C)p,
(C) where p
p, where p¢r
=~: r (D) r-1,
(D) r—1,where r321
where r~l
Q3
Q.3 Bulk modulus
Bulk of rock
modulus of defined as
rock is defined as

shear ~tress
stress .
(A)
(A)
shear (B) hydrostatic
hydrostatic pressure
pr~ssure
volumetric stram
volumetnc strain (B)
shear stram
strain
(C) yvolumetnc. press.ure
shear
h drostatic p ressure
(C) hydrostati~
.
strain
volumetnc stratn (D)
shear
————.—
stress
(D) shear stre~s
shear stram
shear strain
Q4 The magnitude of the
Q.4 The magnitude of the resultant moment about
resultant moment about point
point 00 in
in Nm
Nm of
of the
the two forces acting
two forces acting
on the rod
on the shown below
rod shown below isis

2m lOON
lOON
I...—
~~--------------------~
I‘- ..

0.5 m

50 N

(A) 25
(A) 25 (B) 125
(B) 125 (C) 175
(C) 175 (D) 225
(D) 225
Q.5 Radial stress
stress on the excavation
excavation boundary
Q.5 Radial on the of aa circular
boundary of circular tunnel
tunnel is
is

(A)
(A) always zero
always zero
(B) always positive
(B) always positive
(C) always
{C) negative
always negative
(D) positive in some
some area
area and
and negative
negative in
(D) positive in some area
in some area
Q6 The critical
critical diameter of an
an explosive is defined
Q.6 The diameter of explosive is as the
defined as the diameter
diameter below
below which it
which it
(A) develops
develops the
the optimum
optimum velocity of detonation
(A) velocity of detonation
(B) does not involve
(B) does not involve in
in chemical
chemical reaction
reactiOn
(C) develops
(C) develops the
the max1mum
maximum velocity of detonation
velocity of detonation
(D) deflagrates
(D) deflagrates

Q? Which one
one ofthe
of the following
following supports does NOT
Q.7 Which supports does NOT require
require aa power
power pack
pack for its operation
for its operation

(A) ChOCk
(A) chock Shield
shield support
support (B) open
(B) open circuit
circuit hydraulic
hydraulic prop
(C) close circuit
prop
(C) close circuit hydraulic prop
hydraulic prop (D) Alpine breaker
(D) Alpine line support
breaker line support

MN 2/20
MN 2/20 3/121 Food/06-MN-
S/121 FoodIOB—MN—tB
18

www.examrace.com
www.examrace.com
Q3
Q.8 In aa centri
In centrifugal flow fan
fugal flow fan the conversion of veloc
velocity
ity pressure
pressu re to
the conve rsion of static pressure
to static is
pressu re is
accomplished
accom with the
plishe d with the help
help of
of

(A) impel
(A) impeller
ler (B) curve
curved bladess (C) hub
hub
(B) d blade (C) (D) casing
(D) casing
Q9
Q.9 A 3.3
A 3.3 kV,
kV, 3-pha
3—phase AC motor
se AC motor havin
havingg aa PF of 0.85 draws curren
currentt at
at 95
95 A.
A. The
PF of0.8 5 draws The motor
motor
input powe
input powerr in
in kkW is
W is

(A) 266.5
(A) 266.5 (B) 461.5
(B) 461.5 (C) 543.0
(C) 543.0 (D) 799.5
(D) 799.5
Q.10
Q .1 0 The amou
The amount of total
total stone dust required
required in kg for
for aa secon
nt of stone dust in kg secondary/heavy type stone
dary/h eavy type stone dust
dust
barrierr in
barrie in aa roadw
roadway of size
ay of size 4.0
4.0 m
mx 3.0 m
x 3.0 m is
is

(A) 1320
(A) 1320 (B) 4680
4680 (C) 5200
(B) (C)52 00 (D) 6600
(D) 6600
Q.11
Q. ll In the
In the Gauss
Gaussian plume model,
ian plume model, the dispersion
the disper coefficients
sion coeffi are functi
cients are function of
on of
(A) distan
(A) distance from sourc
ce from sourcee and
and stabil
stability
ity class
class
(B) stack
(B) stack heigh
heightt and
and distan
distance from sourc
ce from sourcee
(C) stabil
(C) stability class and
ity class and sourc
sourcee coord
coordinates
inates
(D) sourc
(D) sourcee coord
coordinates and distan
inates and distance
ce from sourcee
from sourc

Q.12
Q.12 The rache
The rachet-and—pawl arrangement
t-and- pawl arrang in percu
emen t in percussive drill mach
ssive drill machine
ine helps in
helps in
(A) provi
(A) providing
ding required rotational
requir ed rotatio nal Speed
speed
(B) index
(B) indexing at the
ing at the bit
bit rock
rock interf
interface
ace
(C) regula
(C) regulating air flow
ting air flow in forward
in forwa and return
rd and return stroke
strokess of the piston
of the piston
(D) engag
(D) engaging the bit
ing the bit with
with the
the rock
rock betwe
between the blows
en the blows

Q.13
Q.l3 The measu
The measurement of distan
remen t of distances from aa positi
ces from position on the
the earth
earth to
to artific
artificial satellites
on on ial satelli is
tes is
knownn as
know as

(A) astronomical
(A) astron rangingg
omica l rangin (B) pseudo
(B) pseud o ranging
rangin g
(C) satelli
(C) satellite rangingg
te rangin (D) celest
(D) celestial ranging
ial rangin g
Q.l4
Q.l4 In openc
In opencast
ast mining,
mining, the
the width which is
is extrac
extracted from the
width which ted from the worki
working bench is
ng bench IS termed as
termed as
(A) cut
(A) cut (B) bench width
width (C) bank width
(B) bench (C) bank width (D) bench
(D) bench face
face
Q.15
Q .15 Zener barriers
Zener barrie are assoc
rs are associated with
iated with

(A) increa
(A) increased safety appar
sed safety apparatus
atus
(B) statist
(B) statistically safe appar
ically safe apparatus
atus
(C) flame
(C) flame proof
proof appar
apparatus
atus
(D) intrin
(D) intrinsic safety appar
sic safety apparatus
atus

Q.16
Q.l6 The most
The most recen
recentt mode
modell of
of self-c
self-contained compressed-oxygen breathing
ontain ed comp ressed -oxyg en breath apparatus
ing appar atus is
is
(A) Proto
(A) Proto-IV
-IV (B) BG-174
(B) BG-1 (C) BG-4
BG—4
74 (C) (D) BG-l74A
(D) BG-1 74A

NIN 3/20
MN 3/20
www.examrace.com
www.examrace.com
Q.17
Q.l7 The measures
The measures of
of dispersion are
dispersion are

(A) range,
(A) range, variance, standard deviation
and standard
variance, and deviation
(B) mean,
(B) mean, median,
median, and
and variance
variance
(C) mean,
(C) mode, and
mean, mode, and skewness
skewness
(D) mean,
(D) range, and
mean, range, variance
and variance

Q.18
Q .18 In aa single
In single server
server queueing
queueing model
model with
with constant
constant arrival rate, which
arrival rate, one of
which one of the
the
following probability
following distributions is
probability distributions followed by
is followed the inter-arrival
by the inter-arrival times of the
times of the
customers at
customers at the
the service
service facility?
facility? 'I
I
r

(A) binomial
(A) binomial (B) Poisson
(B) Poisson (C) Weibull
(C) Weibull (D) exponential
(D) exponential

Q.19
Q.19 A company
A invested Rs.
company invested Rs. 44 lakh in aa machine
lakh in with an
machine with an expected
expected useful life of
useful life of 12
12 years.
years.
The net
The net income expected from
income expected from the operation of the machine Rs.
the operation of the machine is
is Rs. 80,000
80,000 per
per annum.
annum.
The payback
The payback period for the
period for machine in
the machine years is
in years is

(A) 4
(A)4 (B) 55
(B) (C) 66
(C) (D) 77
(D)

Q20
Q.20 The angular
The angular (horizontal/vertical)
(horizontal/vertical) observation
observation made by aa transit
made by transit theodolite
theodolite with
with the
the
face of
face of the vertical circle
the vertical circle on the right
on the of the
right of the observer is called
observer is called

(A) face
(A) face right
right observation
observation (B) face
(B) face left
lefi observation
observation
(C) normal
(C) normal observation
observation (D) reciprocal
(D) reciprocal observation
observation

Q. 21
Q. 21 to Q. 75
to Q. 75 carry
carry two marks each.
two marks each.

Q21
Q.21 Two sides
Two sides of
ofaa triangle
triangle are
are represented
represented by a = i + 3+
vectors a=
by vectors and bb == -i-
j + k and -—i —}+k.
j + k.
The area
The (magnitude) of
area (magnitude) the triangle
of the triangle is
is

(A) m5
(A) 1/~ (B) 11
(B) . (C)
(C)~fl (D) 2J5
Q22
Q.22
-The cost
·The cost of
of diesel
diesel is
is Rs.
Rs. ([25
25 + :o)
+ ~56) per
per km
km to
to drive
drive aa dump
dump truck at aa speed
truck at speed ofx
of x

Inn/hour. The
kmlhour. The maintenance
maintenance cost of the
cost of the truck
truck is
is Rs. 10 per
Rs. 10 hour. To
per hour. minimize the
To minimize the cost
cost
per
per km, the truck
km, the truck speed
speed in ion/hour is
in km/hour is

(A) 55
(A) (B) 20
(B) 20 (C) 25
(C) 25 (D) 30
(D)30

Q23
Q.23 The functionsj(x)
The functionsfix) and
and g(x) satisfy f(x
g(x) satisfy = 0)
f(x = = 3,
0) = f'(x =
3 , f'(x = -5,
0) =
= 0) g(x =
—5 , g(x = 0) = 22 and
0) = and

g'(x = 0) = —10.. Thevalueof!!__(f(x))


g'(x=0)=-10 The value of 1(1—(3—2] is
dx 3(x)
dx ,___0
g(x) x=O

(A) -35.0
(A) —35.0 (B) —s.0
(B) -5.0 (C) 0.5
(C) 0.5 (D) 5.0
(D) 5.0

4/20
MN 4/20
MN
www.examrace.com
www.examrace.com
Q24
Q.24 A wooden
A wooden block
block of
of 50
50 kg
kg rests
rests on
on the
the floor (shown in
floor (shov.:n in figure
figure below)
below) for
for which
which the
the
coefficient of
coefficient ofstatic friction is
static friction is 0.5.
0.5. The
The smallest
smallest magnitude
magnitude ofthe force Pin
of the force P in kg
kg that
that
will cause
wlll cause impending
impending motion
motion ofofthe
the block is
block is
pP
5
3
4

~
'
l'
I
..

50kg
.4_. 'f‘
'. .5. '.

50 kg
ffffx’r’f/ffff

(A) 50
(A) 50 (B) 40
(B) 40 (C) 30
(C) 30 (D) 25
(D) 25

25
Q'25 The solution
solution of
of yex
ye‘cix = 0 for y(O)
Q. The dx + ((4y ex)dy =
4y + ex)dy y(0) == -1
—1 is

(A)ye-"+2y2—1=0
(A) yex + 2y 2 -1 = 0 (B) ex+
(B) e‘+y2x-2=0
y 2x-2 = 0
(C) {vex
(C) yex- -y2 = O0
y2 = (D) xex
(D) +y2
xex + -1 == 00
y 2 —1

Q26
Q .26 A point
A point PP ((10, 3) MPa
10, 3) MPa on
on the
the Mohr's
Mohr's circle
circle represents
represents normal
normal and shear stresses.
and shear stresses. If
If
the centre of the Mohr’s circle is
the centre of the Mohr's circle is C
C (6,
(6, 0)
0) MPa,
MPa, the
the normal and shear
normal and shear stresses in MPa
on the
on the point diametrically opposite
point diametrically opposite to
toPP are
are

(A) 2,-3
(A) 2, —3 (B) 4,
(B) 4, -3
—3 2, 33
(C) 2,
(C) (D) 4,
(D) 4, 33
Q27
Q.27 A rock
A rock sample
sample with
with aa horizontal
horizontal joint is subjected
joint is subjected to 10 MPa
to 10 MPa of
of normal
normal pressure
pressure as
as
shown
shown in the figure.
in the figure. The
The elastic
elastic modulus
modulus and Poisson’s ratio
and Poisson's ratio ofthe
of the rock
rock are
are 5.0
5.0 GPa
GPa
and 0 respectively. If the normal stiffness
and 0 respectively. Ifthe normal (km) ofthejoint
stiffness (knn) of the joint is
is 50 GPa/m, normal
50 GPa/m, normal
displacement at
displacement at the
the top
top of
of the
the sample
sample ((AA' line) in
AA' line) in mm isis
MPa
10 MPa

1 A f0 A'
F
--—I- E=5GPa,v=0
E = 5 GPa, v= 0
EU .
S km, = 50
knn 50 GPa/m
GPa/m
=
/——4
Joi‘nt/ -—-—r = 5 GPa, v
E =5 V== 0
U

/ //////

(A) 0.2
(A) 0.2 (B) 0.4
(B) 0.4 (C) 0.6
(C) 0.6 (D) 0.8
(D) 0.3
Q28
Q.28 The state
The state of stress (CT
of stress (angry, rxy) at
..u•CT>Y' rxy) at aa point below ground
point below ground is
is found
found to be (5,
to be (S, 15,
15, -3)
—3)
MPa. The
MPa. The angle
angle measured
measured in the counter
in the counter clockwise direction between
clockwise direction between the
the x-axis
.r-axis and
and
the major
the principal axis
major principal axis in degree is
in degree is

(A) 9.52
(A)9.52 (B) 15.48
(B) 15.48 (C) 150.48
(C) 150.43 (D) 164.52
(D) 164.52

MN 5/20
MN 5/20
www.examrace.com
www.examrace.com
of aa cylin
gth of drica l rock
cylindrical ple is
sample MPa . The
90 MPa. The
Q29 The unconfined compressive
unco nfine d com strength
pres sive stren rock sam is 90
Q.29 The
is 30°.
rock is conf ining pressure
If aa confining of 5 MPa is
is applied
appl ied
of internal
anglee of ion of
friction
internal frict the rock
of the 30°. If pres sure of 5 MPa
MPa is
angl
sample, the confined compressrve strength in MPa
gth in is
radially the rock
to the
radia lly to rock samp le, the conf ined com pres sive stren

95.000
(B) 95.0 (C) 105.00
(C) 105. 00 (D) 110.0
(D) 110. 0
(A) 92.888
(A) 92.8 (B)

mad e underground
is made
radiu s aa is stres s condition.
hydr ostat ic stress
in hydrostatic condition.
Q.30 circular
A circu opening
lar open of radius
ing of unde rgro und in
Q.30 A
of the
re of ing, where
opening, the tangential
whe re the stres is twice
tangential stresss is twice
The radia
The distance
radiall dista fiom the
nce from centre
the cent the open
the radial
the stress,
radial stres is
s, is

(A) a
(A) a (B)
(B) Jia
Jia (C) Jia
(C) .fia (D)
(D) 2Jia
2.fia

S= S 1ha wf3,, where


= Slhawfi whe re S 1= insitu
S1= strength
insitu stren of the
gth of the
Q.3l
Q.3l Coal pillar
Coal pilla r stren gth is
strength is repre sented by
represented by S
= mining w=
and w pillar width. h. Two bord and
Two bord pane ls are
pilla r panels
and pillar are
pillar, h =mi
p1llar, h height, and
ning height, =pil lar widt
cond ition s at D 1 and 2
dept hs D1
at depths D2 with mining
mini ng
deve lope d in
developed in the similar
the simi geological conditions
lar geological and D with
If the gallery width and the widt h in
pillarr width both the
in both the
heights hhrand
heights 1and h respectively. If
h;2 respectively. the galle ry widt h and the pilla
the ratio
same,, the of pillar safet y factors,
pilla r safety SF1/
factors, SF SF22 is
1/ SF is
panels rema in the
pane ls remain ratio of

r
the same

(A)[ht]
hl
r
Mei
{A} ( h, l l
D2
D2
(B) (
(ELK)
hi Dr
Dl
ya; r
h, D, (C) Ea.
ll
(C) ( !1_
[’12]
hz
r
D2
D2 (D)[hz]
EB;
(D) ( !1_ D,
h2 D1
Dl

Q32
Q.32 Match the
Match following
the follo wing

conveyor
Belt conv
Belt component
eyor com pone nt Function
Func tion

P
p Pull cord
Pull cord 1
1 Clea ning device
Cleaning devi ce
22 Disc harg ing material
Discharging on the
mate rial on the
Q
Q Snub pulley
Snub pulle y
side of the
side of conveyor
the conv eyor
33 Safety
Safe ty stop ping device
stopping devi ce
R
R Tripper
Trip per
4 Incre asing the
Increasing of wrap
angle of
the angle wrap
sS Rota ry brush
Rotary brus h 4
' S-2
R-l, S-2
8-4
R-3, S-4
Q—Z, R-3,
P-l, Q-2, (B) P-3, Q-4,
(B) P-3, Q-4, R-1,
(A) P-1,
(A)
8—1 (D) P—3,
P-3, Q-4,
Q-4, R-2,
R-2, S-l
S-1
(C) P-4,
(C) Q-2,
P-4, Q-2, RB,
R-3, S-1 (D)

Q33
Q.33 ~atch the
Match following
the follo wing

Equipment
Equi pme nt Acti on I/ Process
Action Proc ess

P
p Dragline
Drag line 1
1 Reaming
Ream ing
Q
Q Bucket
Buck et wheel excavator
wheel exca vato r 22 Key cut
Key cut
R
R Tunnel
Tunn el boring
bori ng machine
mac hine 33 Puls ating impact
Pulsating impa ct
sS raulic monitor
Hydraulic
Hyd mon itor 4
4 Terracing
Terr acin g

8-4 (B) P-2,


(B) Q-4, R-l,
P-2, Q-4, S—3
R-1, S-3
(A) P-1,
(A) Q-2, R-3,
P-l , Q-Z, R-3, S-4
R-3, S-l (D) P-3, Q-4,
tD) P-3, R-2, S-l
Q-4, R-2. S-1
(C) P-Z,
(C) P-2, Q-4,
Q-4, R-3, S-1

MN 6/20
MN 6/20

www.examrace.com
www.examrace.com
Q34
Q.34 Match the
Match the following
following

Mining method
Mining method Face supporting
Face system
supporting system

P
p Mechanised
Mechanised longwall
longwall 11 Cable bolting
Cable bolting
Q
Q Blasting
Blasting gallery
gallery 22 Shield type
Shield type powered
powered supports
supports
R
R Steep seam
Steep seam mechanised longwall
mechanised longwall 33 Alpine breaker
Alpine line supports
breaker line supports
sS Wangawilli
Wangawilli 44 Troika shield supports
Troika shield supports

(A) P-1,
(A) P-l, Q-2,
Q-2, R-3,
R-3, S-4
34 (B)
(B) P-2, Q-l, R-4,
P-2, Q-1, R-4, S-3
S-3
(C) P-3, Q-4,
(C) P-3, Q-4, R-Z, S-l
R-2, S-1 (D) P-2,
(D) P-2, Q-4,
Q-4, R-l, S-3
R-1, S-3

Q.35 A
A 1515 yd3
yd 3 dragline
dragline is deployed in
is deployed an overburden
in an overbUrden bench
bench ofof an
an opencast
opencast mine.
mine. ItIt works
works
for 40
for 40 days
days at
at the
the rate
rate of
of 66 hours
hours per shift and
per shift and 33 shifts
shifts aa day.
day. The
The cycle
cycle time, bucket
time, bucket
fill factor,
fill factor, and
and operating
operating efficiency of the
efficiency of the dragline
dragline are
are respectively 50 s, 0.8,
respectively 50s, 0.8, and 75%.
and 75%.
The total
The total volume
volume of of overburden
overburden inin m
m3 handled
3
handled by
by the
the dragline
dragline is
is
(1
(1 yd = 0.765
yd33 = 0.765 mm3)
3
)

(A) 356918
(A) 356918 (B) 634521
(B) 63452] (C) 557685
(C) 557685 (D) 991440
(D) 991440

Q36
Q.36 The phenomenon
The phenomenon of fretting (necking)
of fretting (necking) of
of pillars
pillars in
in room-and-pillar
room-and-pillar stoping
stoping is
is
common in
common in the
the pillars
pillars formed
formed inin

(A)
(A) massive
massive rock
rock with
with very
very high
high pillar
pillar height
height to
to width
width ratio
ratio
(B)
(B) regularly
regularly jointed
jointed rock
rock with
with high
high pillar
pillar height
height to
to width
width ratio
ratio
(C) massive
(C) massive rock
rock with low pillar
with low pillar height
height to
to width
width ratio
ratio
(D) transversely
(D) transverselyjointed rock with
jointed rock with low pillar height
low pillar to width
height to width ratio
ratio

Q.377
Q.3 In an
In underground opening,
an underground opening, the
the immediate
immediate roof strata consists
roof strata of two
consists of rock layers
two rock layers with
with
the
the following
following properties:
properties:

Property
Property Layer-1
Layer- I Layer-2
Layer-2
Modulus ofelasticity
Modulus of elasticity (GPa)
(GPa) 60.0
60.0 40.0
40.0
Modulus of
Modulus rupture (MPa)
of rupture (-MPa) 20.0
20.0 10.0
10.0
Unit
Unit weight
weight (mm?)
(kN/mJ) 25.0
25.0 20.0
20.0
Thickness (m)
- Thickness (m)
..
5.0
5.0 2.5
2.5

Considering
Considering aa factor
factor of safety of
of safety of 4.0, the length
4.0, the length of safe span
of safe span in
in m
m is
is

(A) 27.82
(A) 27.82 (B) 34.06
(B) 34.06 (C) 36.54
(C) 36.54 (D) 39.34
(D) 39.34

Q38
Q.38 In an
In an opencast
opencast mine,
mine, aa centrifugal
centrifugal pump
pump isis required
required to lift water
to lift at the
water at the rate
rate of
of 60 Us to
60 lis to
aa height
height of
of 80
80 m
m above the pump
above the pump level.
level. The vertical suction
The vertical suction head
head is
is 44 m.
m. The
The total
total
friction
friction head
head including
including shock
shock and
and energy loss is
energy loss is 10
10 m.
m. IfIf the
the pump
pump runs
runs at
at an
an
efficiency
efficiency of 80%, the
of80%, brake power
the brake power of
of the motor in
the motor in kW
kW is is
'
(A) 70.50 .
(A) 70.50 (B) 67.50
(B) 67.50 (C) 63.00
(C) 63.00 (D) 57.55
(D) 57.55

7/20
MN 7/20
MN
www.examrace.com
www.examrace.com
Q.39
Q.39 the following
Match the
Match follow ing

Suppo rt system
Support system Suppo rt principle
Support princi ple

P
p Shotcrete
Shotcr ete 1l rock mass
reinforces rock
reinforces bindin g them
by binding
mass by them
together
togeth er

22 acts link between


as link
acts as layers of
two layers
betwe en two rock to
of rock to
Q
Q Backfill
Backfill
transfe r load
transfer betwe en them
load between them

R
R Bolt
Bolt 33 imposes kinem atic constraints
impos es kinematic constr aints
on key pieces
on key stope boundary
in aa stope
pieces in bound ary

Ss Prop
Prop 44 preven ts spatially
prevents progre ssive disintegration
spatial ly progressive disinte gration
of near
of rock mass
field rock
near field mass

(A) P-3,
(A) R-2, 3-1
Q-4, R-2,
P-3. Q-4, S-1 (B) P-2,
(B) R-4, S-2
Q-1, R—4,
P-2, Q—l, S-2
(C) P-4,
(C) P-4. Q-3,
Q-3, R-l,
R-1, S-2
S-2 (D) P-3, Q-4,
(D) P-3, Q-4, R-l
R-1., S-2
S-2

Q.40
Q.40 Match the
Match following
the follow ing

Drill machine
Drill of drilling
Metho d of
Method drillin g
Stope
Stope machi ne

P
p Shrinkage
Shrink age I1 Drill jumbo
Drill jumbo 1 Fan drilling
Fan drillin g

Q
Q Room-and-pillar
Room-and-pillar JJ Down-the-hole hammer
Down-the-hole Overh and drilling
hamm er 22 Overhand drillin g

R
R Sublevel
Sublev el K
K Hand held
Hand stopperr
held stoppe 33 Parallel drillingg
Paralle l drillin

Mechanised fan drill


Mechanised fan 44 Frontal Ivertical/downward
Frontal /vertic al/downward n
sS Sublev el caving
Sublevel caving L
L dri 11
benching
bench ing

(A) P-I-2,
(A) R-L-3 , S-J—l
Q-K-4 , R-L-3,
P-I-2, QaK-4, S-J-1 (B) P-K-4,
(B) R-J-2. S-L-l
Q-I-3. R-J-Z,
P-K-4 , Q-I-3, S-L-1
(C) P-K-Z,
(C) Q-I-4, R—J-3,
P-K-2 . Q-1-4, R-J-3, S-L—l
S-L-1 (D) PuI-3,
(D) R-J-1,., S—L-2
Q-K-4 . R-J-l
P-1-3. Q-K-4, S-L-2

seam of thickness is
12 m thickness worke d out
is worked mecha nized top
by mechanized
out by caving system.
coal caving
top coal system.
Q41
Q.41 A coal seam
A coal of 12m
bottom slice
the bottom
of the is 33m,
slice is length of
m, length solid coal
the solid is 120
face is
coal face and the
m and the
The thickn
The ess of
thickness ofthe 120m
cut by
of cut
depth of the shearer
by the (web) is
sheare r (web) is 70 em. The
70 cm. densit y of
The density coal is
of coal 1300 kglm3
is 1300 kg/m3
averag e depth
average
extrac tion in
of extraction
percen tage of the slice
in the at 95
slice at and in
95 and the top
in the coal at
top coal 70. The
at 70. The
the percentage
with the
with
produc tion of
production coal per
of coal in tonne
cycle in
per cycle tonne isis

(B) 999
(B) (C) 688
(C) (D) 311
(D) 311
(A) 1008
(A) 1008 999 688

by two
conne cted by
are connected equal length
two equal pipelin es with
paralle l pipelines
length parallel diameters d
with diameters d
0.42
Q.42 Two reservoirs are
Tv.·o reservoirs
resista nce coefficients,
simila r resistance if the
coefficients. if discha rge through
the discharge the smaller
throug h the smaller
and 2d.
and Assum ing similar
2d. Assuming
the discharge
m /s, the
0.04 m3/s,
is 0.04 the other
through the
discharge through pipeline e in
in m3/s
m 3
/s is
is
pipelin e is pipelin
3 other
ter pipeline
diameter
diame

(A) 0.226 (B) 0.426


(B) (C) 1.130
(C) (D) 1.280
(D) 1.280
(A) 0.226 0.426 1.130

MN 8/20
MN 8/20
www.examrace.com
www.examrace.com
Q43
Q.43 The shear
The diagram for
force diagram
shear force the shaft
for the shown below
shaft shown resembles which
below resembles which one of the
one of the
following graphs?
following graphs?

10 kN

lm

Graph-II
Graph- Graph-II

a :2;
uE “E
9.

E B
5‘: (m)
SP3" (m)
Span
8 Span (m)

Graph—IV
Graph-III
Graph-ill

8‘1;; a
5 "I?
.83 —5 s .3‘8.
E E

fi fi
Span (m)
Span (m) Span (m)

(A) Graph-I
(A) Graph-I (B) Graph—II
(B) Graph-Il (C) Graph-III (D) Graph-IV

Q44
Q.44 A 12
A diesel locomotive
tonne diesel
12 tonne locomotive of60of 60 kW
kW is plying in
is plying underground haulage
an underground
in an roadway.
haulage roadway.
The coefficient
The coefficient of adhesion is
of adhesion is 0.25
0.25 and
and the
the maximum
maximum gear
gear efficiency
efficiency is
is 80%.
80%. The
The
speed in
speed mls at
in m/s at which it will
which it haul aa train
will haul at its
train at filll power
its full is
power is

(A) 2.548
(A) 2.548 (B) 2.448
(B) 2.448 (C) 2.038
(C) 2.038 (D) 1.630
(D) 1.630

Q45
Q.45 An air receiver
An air receiver of volume 0.2
of volume 0.2 m3
m3 has an initial
has an temperature of
initial temperature of27°C and pressure
27°C and 1800
pressure 1800
kPa. After use,
kPa. After the air
use, the air pressure falls to
pressure falls to 1200 kPa at
1200 kPa temperature of
at aa temperature of 17°C.
l7°C. The
The
volume of air
vo I ume of consumed in
air consumed in in3 corresponding to
m3 corresponding air pressure
an air
to an pressure of 01.3 kPa
of 1101.3 and
kPa and
temperature
temperature of of 0°C is
0°C is

(A) 0.693
(A) 0.693 (B) 0.895
(B) 0.895 (C) 1.002
(C) 1.002 (D) 1.251
(D) 1.251

Q46
Q.46 Four benches
Four are being
benches are worked by
being worked the opencast
by the mining system.
opencast mining system. Height, width and
Height, width and
face angle for
face angle each bench
for each bench are 15 m, 50
are 15m, In and
50 m and 70°
70° respectively.
respectively. The
The overall
overall slope
slope
angle of
angle the benches
of the in degrees
benches in degrees'is
is

(A) 15.45
(A) 15.45 (B) 19.25
(B) 19.25 (C) 32.65
(C) 32.65 (D) 36.25
(D) 36.25

S/121 FDDdIDE-MN-Z
81'121 Food/06- MN- 2
MN 9/20 www.examrace.com
Q47
Q.47 Match the following
Match the following

condition
mass condition
Rock mass Limiting
Limiting
Rock Shaft sinking
Shaft method
sinking method (m)
depth (m)
depth

P
p Water bearing
Water bearing strata of
strata of II Freezing
Freezing l 40
40
loose
loose sand or gravel
sand or gravel

Q
Q rock with
Competent rock
Competent with JJ Depression
Depression of ground
of ground 22 150
150
fissures
fissures and filled
cracks filled
and cracks water level
water level
with water
with water

R
R Highly permeable
Highly coarse
perrreable coarse K
K Cement grouting
Cement grouting 33 1000
1000
solid or
solid or gravel
gravel with
with heavy
heavy
water flow
water flow

sS All types
All types of water bearing
of water bearing L
L Caissan
Caissan 44 > 600
3* 600
rocks
rocks

(A) Q-K-l, R-J-2,


P-L—4, Q-K-1,
(A) P-L-4, S-I—3
R-J-2, S-I-3 (B) P-L-1,
(B) Q-K-4, R—J-2,
P-L-l, Q-K—4, S-I-3
R-J-2, S-1-3
(C) Q-K-4, R-J-3.
P-L-2, Q-K-4,
(C) P-L-2. S-l-l
R-J-3, S-l-1 (D) P-L-4,
(D) Q—K-B, R-J-2,
P-L—4, Q-K-3, S-I—l
R-J-2, S-I-1

Q48
Q.48 Match the
Match following
the following

System
System Device/ device
Safety device
Device! Safety

pP winding
Drum winding
Drum 1
1 Taper guide
Taper guide
Q winding
Koepe winding
Koepe 2
2 Detaching
Detaching safety hook
safety hook
Q
R
R Inclined Haulage
Inclined Haulage 33 Rider
Rider
sS Winding in
Winding shaft
sinking shaft
in sinking 4
4 Back catch
Back catch

(A) P-1, Q-2,


(A) P-l, Q-2, R-3, 8—4
R—3, S-4 (B) P-4,
(B) Q-3, R-l,
P-4, Q-3, S—2
R-1, S-2
(C) P-2, Q-l,
(C) P-2, Q-1, R—3,
R-3, S-4
S-4 (D) P-2,
(D) Q-1, R-4,
P-2, Q-l, S-3
R-4, S-3

Q49
Q.49 A closed
A container with
closed container 10 kg
with 10 kg of air at
of air ambient pressure
at ambient pressure and specific heat
and specific heat
1020 kJ/kg
1020 kJ/kg°C is cooled
”C is If the
35°C. If
from 35°C.
cooled from the removal of 200 k]
removal of200 of heat
kJ of heat resulted in the
resulted in the
saturation of
saturation the corresponding
air, the
of air, point temperature
dew point
corresponding dew in "C is
temperature in °C is

(A) 33.0
(A) 33.0 (B) 27.3
(B) 27.3 (C) 15.4
(C) 15.4 (D) 12.9
(D) 12.9

Q50
Q.50 Identify
Identify the statement
INCORRECT statement
the INCORRECT

(A) meant to
is meant
(A) Evasee is to minimise exit shock
minimise exit losses
shock losses
(B) Evasee efficiency is primarily
(B) Evasee efficiency is primarily a
a fimction
function of divergence angle
of divergence and area
angle and ratio
area ratio
(C) Evasee produces
(C) Evasee produces an inevitable increase
an inevitable in friction
increase in losses
friction losses
(D) Evasee installation
(D) E\asee installation leads
leads to
to reduction
reduction in
in the
the fan total pressure
fan total pressure

10/20
MN 10/20
MN

www.examrace.com
www.examrace.com
Q51
Q.Sl A single lamp
A single lamp placed
placed centrally at the
the roof
roof provides
provid es 40
centra lly at lux illumination
40 lux illumi nation vertically
vertic ally
below, at the-floor
below , at the floor of
of an
an underground
under groun d workshop. The workshop
works hop is is of
works hop. The dimensions
of dimen sions
20.0
20.0 m
mxx 20.0
20.0 mm with
with height
heigh t 4.0
4.0 m.
m. Assuming
Assum ing uniform spherical
ical dispersion
unifo rm spher of
disper sion of
luminous
lumin ous intensity, the floor
intensity, the floor level
level illumi
illumination
nation in
in lux
lux at any corne
cornerr of
at any the works
of the workshop is
hop is
(A) 23.2
(A) 23.2 (B) 10.9
(B) l 0.9 (C) 3.0
(C) 3.0 (D) 0.8
(D) 0.8
Q52
Q.52 An efflue
An effluent samplee IS
nt sampl is dilute
diluted with fresh
fresh water
water to make up
d with to make up aa soluti
solution
on of 300 mi.
of300 ml. The
The
D0 of
DO of the
the soluti
solution initially
ly is 8.0 mg/l
mg/1 and
and the value falls
falls to
on imtial is 8.0 the value 3.0 mg/1
to 3.0 mg/l after
after 55 days.
days. If
If
the 5-day
the 5—day BOD
BOD of of the originalal effluent
efflue nt is
is known
know n to
to be
be 50
50 mg/l,
the ongin mg/1, the
the amount
amou nt of
of fresh
fresh
water
water added
added m in ml
ml to
to the
the soluti
solutionon is
is

(A) 270
(A) 270 (B) 160
160 54
(B) (C) 54 (D) 30
(D) 30
Q53
Q.53 With respect
With to stack
stack emission the pheno
phenomenon
meno n of
respec t to emiss ion the fumigation
of fumig is notice
ation ts noticedd in
in case
case of
of
(A) atmos
(A) atmospheric
pheric lapse
lapse rate
rate being lower than
than the adiabatic
atic lapse
being lower the adiab lapse rate
rate
(B)
(B) atmospheric
atmos pheric lapse
lapse rate being higher
rate being than the
highe r than the adiabatic
adiab atic lapse
lapse rate
rate
(C)
(C) temperature
tempe rature inversion in the
the atmosphere above the
the stack
invers ion in atmos phere above heightt
stack heigh
(D) temperature
(D) inversion
tempe rature invers in the
ion in the atmos
atmosphere below the
phere below stack height
the stack heigh t
Q54
Q 54 A
A jackhammer operates
jackh amme r opera at aa corner
corne r of
of aa squar
sqiiaree field
tes at of side
field of 50 m.
side 50 m. At
At the
the diagonally
diago nally
opposite
oppos corner,
ite corne the SPL
r, the SPL sense
sensedd is 82.3 dB.
dB. The
The SPLa
SPL att any
any of
is 82.3 the other
of the other two corners
two corne rs
of
of the field in
the field in dB
dB isis

(A) 86.3
(A) 86.3 (B) 85.3
(B) 85.3 (C) 83.6
(C) 83.6 (D) 881.2
(D) 1.2
Q55
Q 55 At aa fan
At fan drifi
drift pressure of 450
pressu re of 450 Pa,
Pa. 50
50 m
mJ/s
3
of air
air flows
flo\VS through
/s of throug h aa mine. When the
mine. \Vhen fan
the fan
stops. 10
stops, 10 m3/s
3
of air
m /s of air still
still flows
flows in
in the same direct
direction. The mine
the same ion. The mine resistance
resist ance in s/m3
in Ns 2
m 8 is
is
(A) 0.
(A) 01731
1731 (B) 0.180
(B) 0.18000 (C) 0.187
(C) 018755 (D) 0.
(D) 02372
2372
Q56
Q.56 In an
In experiment
an exper to determ
determine rock therm
thermal conductivity
iment to ine rock al condu ct1v1ty 3a dtsc
disc of
of rock specimen
rock specim en is
is
placed between
placed two solid
betwe en t\\.'0 solid brass
brass cylind
cylinders and one
one dimen
dimensional
ers and sional heat
heat flow is created
flow 1s create d as
as
shown.
show The readings
n. The readin gs of of the
the thermocouple
thern1ocouple sensors
senso rs with
\'-lth respect to zero
respec t to zero potential are
potential arc
shownn in
show in the
the figure. Brass thermal conductivity
cti\ity is 90 W/m
W/m °C,
figure . Br!iSS thermal condu is 90 ”C, and
and the
the thermocouple
thern1ocouple
constant is 40
consta nt is 40 g V/ DC. The
The rock
rock thermal conductivity
fJ. V/"C. thcrn1al condu cttv1ty inin W/m
W m "C ”C and
and the
the heat
heat flux
flu x in
in
W/m?‘2
respectively
W/m respec tively arcare

2000 J.1V
2000 MV 1920 “V
1920 J.1 V l520 J.IV
1520 tiV
1 \ J
(\0 Brass
BrJSS 1: Brass
if?
L._
~I._ _ _ _ _ _ __ _ \ Roel.: ,_H
rf‘ 10 cm
em ’H Rock
I1cm
em

(A) I1.8.
(A) [300
.8, 1800 (B) 0.6.
(B) 0.6. 1020
1020 (C) 3.2,
(C) 3.2, 5-lO
540 (D) 2.1.
(D}2 . 1, 070
o70

MN
~.1 N I [/20
11 /2 0

www.examrace.com
www.examrace.com
Consider the following for the
data for
following data of iron
grade of working bench
from aa working
ore from past
over past
bench over
Q.57
Q.57 Consider the the grade iron ore
55 weeks
weeks

Week 1 2 3 4 55
Fe)
Grade (% Fe)
Grade(% 62.1
62.1 61.0
61.0 60.5
60.5 62.5
62.5 62.0
62.0

The 3-week
The moving average
3-week moving forecast for
average forecast grade, in
the grade,
for the % Fe,
in % in the
Fe, in the 6th week is
6th week is

(A) 61.66
(A) 61.66 (B) 61.90
(B) 61.90 (C) 62.20
(C) 62.20 (D) 62.50
(D) 62.50

Q58
Q. 58 The variable X
random variable
The random has the
X has the following probability mass
following probability function
mass function
11
P(16) == .
1 1 1
P(4) =%,. P(8)
?(4)= P(l2) zé,, P(l6)
13(8):;— ?(12)=
4 4 4 Z.
4
The expected
The value ofX
expected value is
of X is

(A) 11
(A) (B) 33
(B) (C) 10
(C) 10 (D) 12
(D) 12

Q.59
Q 59 he time
'IThe time bet\\een successive failures
between successive failures (in of aa side
hours) of
(in hours) discharge loader
side discharge operating
loader operating
in mechanised underground
in aa mechanised mine are
coal mine
underground coal as follows:
arc as follows:

58, 57. 50, 53


62, 58, 54, 50, 52. 60, 58.57.50.53
62.58,54.50,52.60,

1fthe failure data


If the failure follow an
data follow distribution, then
exponential distribution.
an exponential then rdiability of the
reliability of equipment
the equipment
for a
for a period
period of 50 hours
of 50 is
hours is

(A) 0.25
(A) 0.25 (B) 0.40
(B) 0.40 (C) 0.60
(C) 0.60 (D) 1.00
(D) 1.00

Q.60 Three jobs A, B, and C are


jobs A, be assigned
to be
are to to three
assigned to machines X, Y and The
Z. The
and Z.
0.60 Three three machines X. Y
processing costs
processing costs are below:
given below:
are g1ven
- Machine
Machine
-
X
X yY z2'.
'A
A 19
19 23
28 31
31

Job
Job BB 11
II 17
17 16
16
,--

13
I
cC 12
12 15
15 13
1

The
1 he minimum total cost
minimum total assigning the
of assigning
cost or to the
jobs to
the jobs is
machines is
the machines

(A)
(A) 60
60 (B) 54
(B) 54 (C) 51
(C) 51 (D) 49
(D) 49

Q61
Q.61 An underground coal
An underground mine employing
coal mine cmp1o) ing 1200 persons experienced
1200 persons l2 roof
experienced 12 fall injuries
roof fall injuries
during
during the
the year
year 2005.
2005. The
1 he roof
roof fall
fall injury
injul)' rate
rate per
per 1000
1000 persons
persons employed
employed during
during the
the
period 2005.
period as per
2005. as the DGMS
per the norms.
DGI\1S norms. is is

(A) 66
(A) (B) 8
(B)8 (C) 10
(C) 10 (D) 12
(D) 12

MN
~1N 12/20
12/20
www.examrace.com
www.examrace.com
Q.62
Q.62 Consider
Consider the following linear
the following linear programming
programming problem:
problem:

Maximize Z
Maximize Z =6X
=6X1+4X2
1 +4X2
Subjectto
Subject to
2X158
2X1 ~8
2X2£12
2X2 ~12
3X1+2X2S18
3X1 +2X2 ~18
X120,X2
x 20
1 ~o.x2 ~o

The multiple
The multiple optimal
optimal solutions
solutions lie
lie on
on the
the line joining the
line joining corner points
the comer points

(A) (0,
(A) (0. 0),
0). (0,
(0, 6)
6) (B) (0,
(B) (0, 6). (2, 6)
6), (2, 6) (C) (2. 6).
(C) (2, (4, 3)
6), (4, 3) (D) (4,
(D) (4, 3),
3), (4,
(4. 0)
0)
Q63
Q.63 Match the
Match the following
following

Problem
Problem Technique
Technique

pP Queueing
Queueing 11 Time series
Time series models
models
Q
Q Project scheduling
Project scheduling and
and monitoring
monitoring 22 Linear progrannning
Linear programming models
models
R
R Transportation
Transportation 33 Waiting line
Waiting line models
models
sS Forecastn of
Forecasting of production
production 44 PERT and CPM
PERT andCPM

(A) P-3,
{A) Q-4, R-2,
P-3 , Q-4, R-2, S-1
5-1 (B) P-2,
(B) Q~3, R-4,
P-2, Q-3, R-4, S-1
S-l
(C) P-3,
(C) P-3, Q-4,
Q-4, R-1,
R-l, S-2
S-2 (D) P-2,
(D) P-2, Q-4,
Q~4, R-3,
R-3, S-1
5-1

Q64
Q .64 The net
The net present value in
present value in Rs.
Rs. of
of aa 33 -year
-year annuity of Rs. 10,000
annuity ofRs. 10,000 discounted
discounted at
at 10%
10% is
is

(A) 9,091
(A) 9,091 (B) 17,355
(B) 17,355 (C) 24,869
(C) 24,869 (D) 26,446
(D) 26,446
Q.65
Q.65 For
For aa track
track gauge of 1.05
gauge of 1.05 m
m and speed of
and aa speed 10 km/hour,
of 10 the super-elevation
km/hour, the super-elevation in
in em
em
from the
from the following
following figure is
figure is
y3’
40°

K /K 2
5
Z
of
40 4.3
0o

1-65
(A) 1.65
(A) (B) 2.76
(B) 2.76
(C) 5.54
(C) 5-54 (D) 6.64
(D) 6.64

Q66
Q.66 In the
In the bubble
bubble tube
tube of
of aa dumpy
dumpy level,
level, the
the bubble
bubble moves
moves 55 mm
mm for
for aa change
change of
of
inclination of
inclination of 40'
40•.. The sensitivity in
The sensitivity in mm and
mm and the radius of
the radius of the
the bubble
bubble tube
tube in m are
in mare
(l
( 1 radian = 206265"
radian = 206265n ))

(A) 0.125,
(A) 0.125, 12.89
12.89 (B) 0.063,
(B) 0.063, 26.78
26.78 (C) 0.125,
(C) 0.125, 25.78
25.78 (D) 0.063,
(D) 0.063, 12.89
12.89

MN 13/20
MN 13/20
www.examrace.com
www.examrace.com
Q.67
Q.67
The value of A•B , if A+ B
ThevalueofA-IB,iffit+B=[3
=[: -1]
1 -1
O]andA-B=[I
3
O and A-B = l [] I] 1
4],.is
4 IS
.

(A) -4[~_0 3I
( A)—4Fl :] (B)-
U 2[~0
B —2] 1
3
:]
. "1
(C)
'cc)
[~_0
1
:]3] (D)_§[0
(D) --
2 0
TI]
1 1 1
3]
3

Q.68 The values of f (x)at


at x0, x11 and x22 are
are99.0,
Q.68 Thev alue sof f(x) x0 ,x andx 12.0
.0.12 andd 15.0
.0an 15.0 respc
respectively. Usingg the
ctive ly. Usin the
x:
Simpson's;
Simp son's ± rule,
rule, the
the value
value of ·'J.r (
of If (x)
x). considering an interval
considering an of 0.1
interval of 0.1 is
is
II I
.t.

(A) 1.2 (B)


(B) 2.4 (C) 1.6 (o) 1.3
(A) 1.2 2.4 (C) 1.6 (D) 1.8 -_u_

Q.69
Q.69 From the
from the following
following page of aa levelling
page of levelling field
field book,
book. the
the missing
-'_'|_

valuess in
miss ing value in F.S.
F.S. and
and 3.8.
B.S.
respectively
respe ctively are
are
_

__

Station B.S. l.S. F.8.


F.S. Rise
Rise Fall
Fall Remarks
Remarks
_

1 4.550
4.550 I Point
Starting Point
2 2.125
2.125 ?? 0.750 int
0.750 Change point
3 2.225
2.225
4 ‘2’? 1.975
1.975 Change point
5 2.445
2.445 1.500
1.500

(A) 3.304,
(A) 3.804, 0.945
0.945 (B)
(B) 3.304,
3.804, 3.945
(C) 5.300
3.945
(C) 5.300,, 0.945
0.945 (D)
(D) 5.300,
5.300. 3.945
3.945
Q70
Q.70 The magnetic
The magnetic bearing
bearing and
and declination
declination of
of aa line
line were
were recorded
recorded in
in the
the year
year 1906
1906 as
as
S43”30'E
S and ZOOO'E
43°30' E and 2°00 ' E respectively.
respectively. If
If the declination
the decli in the
nation in the year
year 2006
2006 is 3000'W
is 3°00 the
'W .,, the
magnetic bearing
magnetic beari ng of
of the line is
the line is ~

(A) 5430307; (B) 5450395 (C) 5410391; (D) $330395

MN 14/20
MN 14/20
'. _-_'_.
www.examrace.com
www.examrace.com
.

h
Common
Common Data
Data Questions
Questions

Common Data
Common for Questions
Data for Questions 71,72,73:
71,72,73: In In aa straight
straight duct of length
duct of 200 In aa fan
length 200m fan operates
operates
50 m
50 away from
m away from the
the inlet such that
inlet such that the mean air
the mean air velocity in the
velocity in duct is
the duct is 8.0
8.0 m/s
m/s at
at a
a density
density of
of
1.1 kg/m3.
1.1 kg/m . The
3
The friction
friction pressure loss per
pressure loss perm length of
111 length of the duct is
the duct is 3.0 Pa and
3.0 Pa and the entry shock
the entry shock
factor
factor is Answer the
1.2. Answer
is 1.2. following in
the follov.:ing in terms
terms ofof guage
guage pressure values in
pressure values in Pa.
Pa.

0.71
Q.7l The
Thl! total
total pressure
pressure at
at the
the outlet the duct
of the
outlet of duct is
is

(A) -35.2
(A) -35.2 (B) 35.2
(B) 35.2 (C) 192.2
(C) 192.2 (D)
(D) 635.2
635.2

Q.72
Q. 72 The total
The pressure at
total pressure at the
the inlet
inlet side of the
side of the fan is
fan is

(AU—192.2
(f;\) -192.2 (B) —150.0
(B) -150.0 (C) 150.0
(C) 150.0 (D) 192.2
(D) 192.2

Q.73
Q. 73 The
The total
total pressure by the
generated by
pressure generated fan is
the fan is

(A) 600.0
(A) 600.0 (B) 635.2
(B) 635.2 (C) 677.4
(C) 677.4 (D) 682.2
(D) 682.2

Common Data
Common Data for Questions 74,
for Questions 75: A
74, 75: A bauxite deposit has
bauxite deposit been intersected
has been intersected by
by 55 drill
drill holes.
holes.
The values
The values of
of alumina (% by
alumina(% by weight)
weight) and
and silica (% by
silica(% by weight) in these
weight) in these drill holes are
drill holes as
are as
follows:
follows:

Drill hole
Drill hole Alumina
Alumina Silica
Silica
number
number (%)
(%) (%)
(%)
I1 46
46 1
2 42
42 5
3 45
45 2
4 43
43 4
5Ln-h -JM 44
44 b3
ah-Muh-

Q.74
Q.74 The
The relationship
relationship between
between alumina
alumina and silica is
and silica is
'
(A) linear
positive linear
(A) positive (B) exponential
(B) exponential
(C) negative
(C) linear
negative linear (D) random
(D) random

0.75
Q. 75 unbiased estimate
The unbiased
The estimate of
of variances of alumina
variances of alumina and silica in
and silica (‘%:1)22 respectively
in (%) are
respectively are

(A)
(A) 2.5,
2.5, 2.5
2.5 (B) 2.0,
(B) 2.0, 2.5
2.5 (C)
(C) 2.5, 2.0
2.5, 2.0 (D) 2.0,
(D) 2.0, 2.0
2.0

15/20
MN 15/20
MN

www.examrace.com
www.examrace.com
Linked Answer
Linked Q.76 to
Questions: Q.76
Answer Questions: carry two
Q.SS carry
to Q.85 marks each.
two marks each.

Statement for
Statement Linked Answer
for Linked Answer Questions 76 &
Questions 76 of aa coarse
Porosity of
77: Porosity
& 77: coarse grain sandstone
grain sandstone
is 15%.
sampleis
sample 15%. The specific gravity
The specific gravity of sandstonets
of sandstone is 2.8
2.8.

Q76
Q.76 What is
What the void
is the void ratio in the
ratio in sandstone sample?
the sandstone sample?

(A) 0.150
(A) 0.150 (B) 0.176
(B) 0.176 (C) 0.850
(C) 0.850 (D) 1.176
(D) 1.176

Q77
Q.77 Ifthe
If sandstone sample
the sandstone is fully
sample is saturated in
fully saturated water, the
in water, the saturated density of
saturated density the sample
of the sample
in kglm is
in kg/m3 3
is

(A) 1590
(A)1590 (B) 2234
(B) 2234 (C) 2438
(C) 2438 (D) 2531
(D) 2531

Statement for
Statement Linked Answer
for Linked Answer Questions 78 &
Questions 78 & 79:
79: AA double outboard chain
double outboard stranded
chain stranded
conveyor
conveyor is is installed in an
installed in an underground
underground coal mine to
coal mine to transport
transport coal. The mass
coal. The of the
mass of chain
the chain
and associated
and flight is
associated flight is 40 kg/m,
40 kglm, the
the coefficients
coefficients of
of kinematic
kinematic friction
friction are
are 0.33
0.33 between
between chain
chain
and
and the pan and
the pan 0.5 between
and 0.5 coal and
conveyed coal
between conveyed and the
the pan. The motor
pan. The efficiency is
motor efficiency 80%. Coal
is 80%. Coal
is
is to
to be conveyed at
be conveyed at the rate of
the rate 120 Uhour
of 120 over aa length
tlhour over length of
of 120m at aa chain
120 In at speed of
chain speed 0.9 rule.
of0.9 rn/s.
The density of
bulk density
The bulk coal Iis
of coal 900 kg/m3.
S 900 kg!m .3

Q.78
Q.78 The power requirement
The power of the
requirement of motor of
the motor the chain
of the chain conveyor
conveyor in kW is
in kW is

(A)
(A) 33.16
33.16 (B) 37.53
(B) 37.53 (C) 42.00
(C) 42.00 (D) 45.94
(D) 45.94

Q79
Q.79 The power
The requirement of
power requirement of the motor of
the motor ofthe conveyor in
chain conveyor
the chain in kW, if it
kW, if moves in
it moves in the
the
uphill direction
uphill gradient of l in 10,
at aa gradient
direction at of 1 in 10, is
is

(A) 46.91
(A) 46.91 (B)
(B) 42.00
42.00 (C) 33.53
(C) 38.53 (D) 30.16
(D) 30.16

Statement for
Statement for Linked Answer Questions
Linked Answer Question.s 80
80 & 81: The
& 81: The observed total time
observed total time of
of drilling
drilling aa
face
face in
in an underground coal
an underground mine is
coal mine 18 min.
is 18 The rating
min. The of the drill
rating ofthe crew performance,
drill crew performance,
expressed in
expressed percentage, is
in percentage, 90. Following
is 90. allowances are
Following alloWances recommended by
are recommended the mine
by the mine
management
management
i)
i) personal needs
personal allowance: 5%
needs allowance: of the
5% of basic time
the basic time
ii)
ii) fatigue allowance: 4%
fatigue allowance: 4% of basic time
ofbasic time
iii)
iii) contingency delay allowance:
contingency delay 1% of
allowance: 1% basic time
of basic time

Q.80
Q.80 The basic
The basic time required for
time required for the drilling job
the drilling job by the crew
by the in min
crew in is
min is

(A) 16.2
(A) 16.2 (B) 17.4
(B) 17.4 (C) 18.0
(C) 18.0 (D)
(D) 20.0
20.0

Q.81
Q.81 standard time
The standard
The time required for the
required for the same drilling job
same drilling by the
job by crew in
the crew in min is
min is

(A) 15.50
(A) 15.50 (B) 17.01
(B) 17.01 (C) 17.82
(C) 17.82 (D) 18.90
(D) 13.90

16/20
MN 16/20
MN

www.examrace.com
www.examrace.com
Statement for Linked
Statement for Linked Answer
Answer Questions 82 &
Questions 82 & 83:
83: The results of
The results of aa theodolite survey are
theodolite survey are
given below
given below .
Points ._-_ Coordinate.1in m I East Coordinate,
Points North igfim in m
A
A 400.5
400.5 I* 5202
620.2
B
B 750.5
750.5 I 3205
320.5

Q.82
Q.82 The length
The length of
of the
the line
line AB
AB in In is
in m is

(A) 460.78
(A) 460.78 (B) 349.70
(B) 349.70 (C) 106.60
(C) 106.60 (D) 50.30
(D) 50.30

(2.83
Q.83 The bearing
The of the
bearing of the line
line AB
AB in degrees is
in degrees is

(A) —-23.17NE
(A) -23.17NE (B) 23.17NW
(B) 23.17NW (C) 40.57NW
(C)40.57NW (D)
(0) 40.57NE
40.57NE

Statement for
Statement for Linked Answer Questions
Linked Answer Questions 84 & 85:
84 & The following
85: The following figure
figure provides
provides the grade
the grade
information.
information.
Cu = 0.56
% Cu=0.56
%

O
l
9:. Cu = 0.64

6’12:
A\O”

(2.84
Q.84 The grade
The grade of
of copper(%)
copper (%) at
at point
point A
A using
using the inverse distance
the inverse distance weighting
weighting method is
method is

(A) 0.47
(A) 0.47 (B) 0.58
(B) 0.58 (C) 0.61
(C) 0.61 (D) 1.20
(D) 1.20

Q.85
Q.85 Assume the
Assume grade at
the grade at A
A to
to be
be the
the average grade of
average grade of copper,
copper, mill recovery to
mill recovery to be
be 85%
85% and
and
the smelting
the smelting 8:
& refining
refining losses
losses to
to be 1.0 kg
be 1.0 of copper
kg of copper per
per tonne
tonne of
of ore. The amount
ore. The of
amount of
saleable copper
saleable copper in kg/tonne of
in kg/tonne of ore
ore is
is

(A) 2.93
(A) 2.93 (B) 3.93
(B) 3.93 (C) 4.93
(C) 4.93 ·+ (D)
(D) 5.93
5.93

END OF THE
END OF THE QUESTION PAPER
QUESTION PAPER

MN 17/20
.MN 17/20

www.examrace.com
www.examrace.com.
2008
2008 MAIN PAPER -— MN
MAIN PAPER MN

MN :: MINING
MN MINING ENGINEERING
ENGINEERING

Duration
Duration :: Three
Three Hours
Hours Maximum
Maxmwm Marks
Marks ::150
150

Read the
Read the foUowing
following instructions
instructions carefully
carefully

1. This
1. question paper
This question contains 20
paper contains 20 printed pages including
printed pages pages for
including pages rough work.
for rough work. Please check all
Please check all pages
pages
and report
and report dtscrepancy,
discrepancy, tf
if any.
any.

2.
2. Write your
Write your registration
registration number,
number, your name and
your name and name
name of
of the
the examination
examination centre
centre at
at the specified
the specified
locations on
locattons on the
the right
right half
half of
of the
the ORS.
ORS.

3.
3. Using HB
Using HR pencil, darken the
pencil, darken the appropriate
appropriate bubble under each
bubble under each digit
digit of
of your
your registration
registration number
number and
and the
the
letters correspondmg
letters corresponding to your paper
to your code.
paper code.

4.
4. All the
All the questions
questions in
in thts
this question
question paper of objective
are of
paper are type.
objective type.

5. must be
Questions must
5. Questions answered on
be answered on Objective Response Sheet
Objective Response Sheet (ORS)
(0R8) by darkening the
by darkening the appropnate
appropriate
bubble (marked A,
bubble (marked A, B,
B, C,
C, D)
D) using HB pencil
using HB against the
pencil against question number
the question number onon the
the left
left hand
hand side
side of
of the
the
ORS. Each
ORS. question has
Each question only one
bas only correct answer.
one correct answer. InIn case
case you
you wish
wish to
to change
change an answer, erase
an answer. erase the
the
answer completely.
old answer
old completely. More
More than
than one answer bubbled
one answer against aa question
bubbled against question will
will be wrong
as aa wrong
treated as
be treated
answer.
answer.

6.
6. Questions II through
Questions through 20
20 are
are 1-mark
1-mark quest1ons
questions and
and questions 21 through
questions 21 85 are
through 85 are 2-mark
2-mark questions.
questions.

7. Questions
7. Questions 71 71 through
through 7373 is
is one
one set
set of
of common
common datadata questions,
questions, questions
questions 74 74 and
and 75
75 is
is another
another pair of
paLr of
common data
common data questions.
questions. The
The question
question pairs (76, 77),
patrs (76, (78, 79),
77), (78, 79), (80,
(80, 8181), (82, 83)
), (82, 83) and
and (84,
(84, 85)
85) are
are
questtons with linked
questions with linked answers.
answers. The
The answer
answer toto the
the second
second question
question of of the above pairs
the above will depend
pairs will depend onon
the answer
the answer to to the
the first
first question
question of
of the
the pair. If the
pair. If the first
first question
question in
in the
the linked
linked pair is wrongly
patr ts wrongly answered
answered
is un-attempted,
or ts
or tin-attempted, then
then the
the answer to the
answer to the second
second question in the
question in the pair will not
patr will be evaluated.
not be evaluated.

8. Un-attempted
8. Lin-attempted questions will carry
questions will carry zero
zero marks.
marks.

9.
9. NEGATIVE MARKING: For
NEGATIVE MARKING: For Q.l to Q.20,
Q.l to 0.20,- 0.25
0.25 mark will be
mark will deducted for
be deducted for each wrong answer.
each wrong answer. For
For
Q.21 to
Q.21 to Q.75,
Q.75, 0.5
0.5 mark
mark will
will be deducted for
be deducted for each
each wrong
wrong answer.
answer. For
For the
the pairs
pairs of with
questions wtth
of questions
linked answers.
linked answers, there
there will
will be negative marks
be negative marks only
only for
for wrong answer to
wrong answer to the
the first
first question,
question, t.e.
i.e. for
for Q.76,
0.76,
Q78, Q.80,
Q.78, Q30, Q.82
Q32 and
and Q.84.
Q34, 0.5
0.5 mark will be
mark w1ll deducted for
be deducted for each
each wrong
wrong answer.
answer. There
There IS
is no
no negattve
negative
marking for
marktng for Q.77,
Q.77, Q.79,
Q79, Q.81,
Q31, Q.83
Q33 and
and Q.85.
Q35.

10. Calculator
10. Calculator without
without data
data connectivity
connectivity is
is allowed
allowed in
in the examination hall.
the examination hall.

1 1. Charts,
II. Charts, graph
graph sheets
sheets and
and tables are NOT
tables are allowed in
NOT allowed the examination
in the examination hall.
hall.

12. Rough work


12. Rough work can
can be done on
be done on the question paper
the question itself. AdditiOnal
paper itself. Additional blank
blank pages are given
pages are given at
at the end of
the end of
the questiOn
the question paper for rough
paper for rough work.
work.

MN
MN [I20
1120
www.examrace.com
www.examrace.com
MAIN PAPER -- MN
MAIN PAPER MN
2008
2008

Q. 11 —
Q. - Q. carry
20 carr
Q. 20 y one mark
one mar k each.
each.

(2.1
Q.l The trace
The trace of ing matrix
following
the follow
of the is
matrix ts

2 2 3
3 2 3
4 1 2

(C) 88
(C) (D) 9
(D)9
(A) 66
(A) (B) 7
(B) 7

its probability
defm es its density then the
function, then
ty function, the
is aa continuous
X is
If X randomm variable andj( x) defines
variable andfix) proba bility densi
()2
Q.2 If conti nuou s rando
expected
expe value of
cted value X is
of X is

(A) j f(x)dx
(A) J
+<lO

f(x) dx (B) L xix.


(B) 2
+«>

(C) -·--
L xaftxi)
(C) 2 x.f(x.) (D)
-
j xf(x)dx
(D) J xf(x)dx

tool used
The tool to corre ct borehole
correct boreh ole devia tion is
deviation is
Q3
Q.3 The used to

(C) Whipstock
(C) (D) Rachet
(D) Rach et
(A) shot
Stringg shot
(A) Strin (B) Kelly
(B) Kelly Whip stock

Q.4
Q.4 A phreatic
A surface
pltreatic surfa ce exper ience s aa pressure
experiences press ure

(B) Equal atmospheric


to atmo pressure
spher ic press ure
Less than
(A) Less
(A) atmospheric
than atmo pressure
spher ic press ure (B) Equa l to
(D) Less than barom etric pressure
than barometric press ure
More than
(C) More
(C) barom etric pressure
than barometric press ure (D) Less

hydra ulic prop’


of aa hydraulic is sente d by
represented the curve
by the curve
Q5
Q.5 load-yield
The load-
The yield chara cteris tic of
characteristic prop is repre

(B) (C)
(C) (D)
(D)
(A)
(A) (B)

z:
E
zE '' zE : zE~ :
3..... r
‘3'.... ——
~

...r;.. ' .., _


I; s—
~

s
~

E 3.
0
_."'0 .3 ..J
0
..J

*— :—
mi {mm} viud [nun]
Yield (mm)
‘flfld {mm}
Yleld(mm) ‘flllfl {mm}
Yleld(m m) Yield (mm)

thickness of diate roof


immediate roof is calculated
is calcu lated from
from
Q.6
Q.6 lon&rwall caving,
In longwall
In the thickness
cavin g, the of imme

Bulking
(A) Bulk
(A) factorr and
ing facto width oflon
and width gwal l face
of longwall face
Seam
(B) Seam
(B) thickness
thick ness and
and width
width of
of longwall
longw all face
face
(C) Seam
(C) Seam thickness
thick ness and
and bulking
bulki ng factor
facto r
(D) ing factor
Bulking
(D) Bulk and length
factor and length of the panel
of the pane l

suspended
safelyy suspe
is safel
cage is headgear
the head
in the due to
gear due to
(3.?
Q.7 During over- wind ing, aa cage
Durin g over-winding, nded in

(B) Rope
(B) capel
Rope cape!
(A) Bull
(A) chain
Bull chain
(D) Detac
(D) hing hook
Dctaching hook
(C) D-link
(C) D-lin k

H)—
2120
MN
MN
www.examrace.com
www.examrace.com
2008
2008 MAIN PAPER -*— MN
MAJN PAPER MN

Q.8
Q.8 Depending on
Depending the decreasing
on the decreasing ability
ability of
of surrounding rock to
surrounding rock to store
store strain
strain energy,
energy, the
the underground
underground
metal
metal mining
mimng methods
methods can
can be ordered as
be ordered as

(A) Cut-and—Fill
(A) stoping, Sublevel
Cut-and-Fill stoping, Sublevel caving,
caving, Sublevel open stoping,
Sublevel open stoping, Block
Block caving
caving
(B) Sublevel
(B) Sublevel open stoping, Cut-and-Fill
open stoping, Cut-and-Fill stoping,
stoping, Sublevel
Sublevel caving,
caving, Block
Block caving
caving
(C) Sublevel
(C) Sublevel caving, Sublevel open
caving, Sublevel open stoping,
stoping, Cut-and-Fill stoping, Block
Cut-and-Fill stoping, Block caving
caving
(D) Block
(D) Block caving,
caving, Sublevel
Sublevel caving,
caving, Sublevel open stoping,
Sublevel open stoping, Cut-and-Fill
Cut-and-Fill stoping
stoping

Q9
Q.9 If swell
If swell factor
factor of ore in
of ore in aa shrinkage
shrinkage stope
stope is
is 1.4,
1.4, the output from
the output the stOpe
from the in percent
stope m percent of
of broken ore
broken ore
is
is
(A) 0
(A) 0 (B) 29
(B) 29 (C) 40
(C) 40 (D) 100
(D) 100

QJO
Q.lO The velocity
The velocity of
of the
the wave
wave type
type that determines the
that determines the ‘ri PPability’ of
'rippability' rockmass is
ofrockmass is

(A) P
{A) P wave
wave (B)
(B) S wave
S wave (C)
(C) Raleigh
Raleigh wave
wave (D) Love
{D) wave
Love wave

Q.1
Q.ll1 In the
In the order
order of
of the chronological development,
the chronological development, the longwall support
the longwall systems are
support systems are arranged
arranged as
as

P
P Powered support
Powered support
Q
Q Link bar
Link bar and
and friction support
friction support
RR Frame support
Frame support
3S Hydraulic support
Hydraulic support

(A) P>Q}R}S
(A) P>Q>R>S (B) RbsbP
(B) R>S>Q>P (C) S>RI>P>Q
(C) S>R>P>Q (D)
(D) Q>S>R>P
Q>S>R>P

Q. 12
Q.l2 Effective temperature
Effective temperature is
is estimated
estimated from
from

(A) Wet-bulb
(A) Wet-bulb temperature,
temperature, relative
relative humidity, and air
hurrudity, and velocity
air velocity
(B) Dry-bulb
(B) Dry-bulb temperature,
temperature. relative humidity, and
relative humidity, air velocity
and air velocity
(C) Dry—bulb temperature.
(C) Dry-bulb temperature, wet-bulb
wet-bulb temperature,
temperature, and
and air
air velocity
velocity
(D)
(D) Dry—bulb
Dry-bulb temperature,
temperature, wet-bulb
wet-bulb temperature, and relative
temperature, and relative humidity
humidity

Q. 13
Q.l3 Pressure-quantity characteristic
Pressure-quantity characteristic of
of aa mine fan is
mine fan given below:
is given below:
2
2 _.
E
:1
39 1
o.
\
1 2
2
Quantity
Quantity
The combmed
The combined characteristic
characteristic of
of two such identical
two such fans installed
identical fans mstalled in
in parallel
parallel is
is
(A)
(A) (B)
(B) (C)
(C) (D)
(D)
22 - 2
2 22 2
2 -

E
QJ
E
QJ
.... 93
....
QJ
93
QJ
....
as~11- a - as~1
L..

aa1
:3
::l :3
::l :i
::l D
::l

- ~1 -
(/) (/) (/) (/)

a.
.....
0.. a:o.a 1
~1
o..... 1
0..
....
o.
0..

\ '
11 22 ‘l 22 1
1 22 1
1 22
Quantity
Quantity Quantity
Quantity Quantity
Quantity Quantity
Quantity

MN
MN 3:20
3/20

www.examrace.com
www.examrace.com
MAIN PAP ER -- MN
MAJN PAPER MN

3st
2008

dam construction, if P,
construction, if and R
Q, and
P, Q, R
and roadway conditions
diti ons for wa ter darn
for water
Q.l4 identical water
Un der identical
Under head
water hea d and roa dw ay con
dam respectively,
enc al clam the thickness
respectively, the thic kne ss
Q.1 4 dam , cylindrical dam and
cylindrical dam spherical
and sph
represent
rep the thickness
resent the flat dam,
of flat
thtckness of
would follow the
wo uld follow order
the ord er

(C) P>Q3’R (D) Q'P'PPR


(D) Q> P>R
(B) PPRPQ
(B) P>R >Q (C) P>Q >R
(A) RfiPPQ
(A) R> P>Q

distribution of soil is known as


kno wn as
0.155
Q.l The grain
The size dist
grain size ribution of soil is

(C) Porosity
(C) Porosity (D) Texture
(D) Tex ture
(A) Permeability
(A) Permeability (B) Structure
(B) Structure

precipitator wo rks on
cipitator works prin cip le of
the principle
on the of
Q.166
Q.1 Electrostatic
Electrostatic pre

(B) Ionization
(B) the particles
of the
Ion iza tion of par ticl es
(A) aci tan ce change
Capacitance change gas eou s molecules
the gaseous
(A) Cap Cen trif ugm g the
(D) Centrifuging
(D)
molecules
(C) Electro
(C) Ele of gases
heating of
ctro heating gases
to be 80 m,
be 80 72 m
and 72
m, and m
distances RP and PS
PS are me asu red to
are measured
shownwn below, the dis tan ces RP and
0.17 figure , the
Q.17 In the
In the figu re sho bel ow
The distance e PQ m is
in m
PQ in is
respectively.
resp ecti vel y. Th e dist anc

0*o',
a. '1.
I

: '\
I

0 '
' \
0 '

' '
'
0
'

80m
80 m

(C) 64.8
(C) (D) 68.4
(D) 68. 4
(A) 60.44
(A) 60. (B) 66.4
(B) 66. 4 64.8

ivit y duration is assumed to follow


assumed to follow
0.18
Q.l 8 In PE
In network, the
RT network,
PERT activity
the act duration is

(B) omtal distribution


Binomial
(B) Bin distribution
(A) Bet a distribution
(A) Beta distribution
Weibull distribution
(D) Weibull
(D) dis tnb utio n
rmal distribution
Normal
(C) No
(C) distribution

the INCORRECT
ntify the
identify statement
INC OR RE CT stat em ent
Q.l9
Q.l 9 For LP problem,
an LP
For an problem, ide

lies in
pom t lies of the
one of the com er points
corner points
Optimal point
(A) Optimal
(A) in one
(B) Objective
(B) function is
ObjeCtive function is linear
line ar
the constraints
All the ints are
are linear
line ar
(C) All
(C) con stra
points of
interior points
the interior
of the feasible region
the feasible
of the region
(D) Op
(D) tim al point
Optimal poi nt lies
lies in
in any
any of

435
MN
www.examrace.com
www.examrace.com
2008
2008 MAlN PAPER. — MN
MAIN PAPER- MN

Q20
Q.20 In aa bi-axial
In stress field
bi-axial stress field the
the vertical stress is
vertical stress 10 MPa
is 10 MPa and
and the Poisson ratio
the Poisson ratio for
for the rock mass
the rock is 0.2.
mass is 0.2.
The horizontal
The stress in
horizontal stress in MPa
MPa IS,
is.
(A) 1.5
(A) 1.5 (B) 2.5
(B) 2.5 (C) 2.0
(C) 2.0 (D) 5.0
(D) 5.0

Q. 21 to
Q. 21 to Q.
Q.75 carry two
75 carry two marks
marks each.
each.

Q.21
Q.21 Given bench
Given bench height:
height: 12m,
12m, burden: 4m, spacing:
burden: 4m, spacing: 5m;
5m; sub-grade
sub-grade drilling:
drilling: 2m;
2m; explosive per hole:
explosive per hole:
120 kg;
120 kg; density
density of
of rock: 2600 kg
rock: 2600 kg fm”,
/m3 , the powder factor
the powder factor in
in tonne/kg
tonne/kg isIS

(A) 2.0
(A) 2.0 (B) 4.6
(B) 4.6 (C) 5.2
(C) 5.2 (D)
(D) 7.3
7.3

(2.22
Q.22 Match the
Match following:
the following:

Equipment
Equipment Slice thickness
Slice thickness Action
Action
(range in
(range in m)
rn)

P Dragline
P Dragline 1I 66 -- 12
12 aa Crowding
Crowding
Q Shovel
Q Shovel 22 30 -- 40
30 40 b Hoisting
b Hoisting
R Surface
R Surface Miner
Miner 33 0.2- 0.4
0.2- 0.4 cc Cutting
Cutting

(A) P-l-b; Q-2-a;


(A) P-1-b; Q-Z-a; R-3-c
R-3—c (B) P-2-b;
(B) P-Z-b: Q-1-a; R-3-c
Q-l-a: R-3-c
(C) Pfi2-a: Q-l-b; R-3-c
(C) P-2-a; Q-1-b; R-3uc (D) P-2-b;
(D) P-Z-b; R-1-a;
R-l-a; Q-3-c
Q-3uc

0.23
Q.23 If the
If the value
value of ore is
of ore is Rs.
Rs. 600 per tonne.
600 per tonne, production
production cost
cost Rs. 400 per
Rs. 400 per tonne,
tonne, and cost of
and cost of overburden
overburden
removal Rs. 50
removal Rs. 50 per m3,3, the
per m the breakweven
break-even stripping
stripping ratio
ratio in m3ftonne
in m 3
is
/tonne is
(A) 4:1
(A) 4:1 (B) 3:1
(B)3:1 (C) 1:3
(C) 1:3 (D) 1:4
(D) 1:4

Q24
Q.24 Four mines
Four mmes A,A, B,
B, C and D
C and are located
Dare located along
along aa road
road as
as shown
shown with
with production
production in Mt per
in Mt per year 1, 2,
year I, 2, 1
l
and 33 respectively.
and In order
respectively. In order to
to handle
handle total
total coal
coal produced.
produced, the
the ideal distance of
ideal distance of aa coal washery along
coal washery along
the
the road from the
road from the mine
nune AA in km ts
tn km is
A
A BB cC 0D

0
1 km
1 km
£
• 2 km
2km •
0
4 km
4km •
O

(A)
(A) 4.01
4.01 (B) 3.91
(B) 3.91 (C) 3.81
(C) 3.31 (D) 3.71
(D) 3.71

Q25
Q.25 A shaft
A shaft inset
inset is
is as
as shown
shown below. To transport
below. To transport aa 15m
15 m long
long object,
object. the
the height ‘H’ of
he1ght 'H' of the
the inset
inset in
in m
m
should be
should be

D=7m

(A) 10.5
(A) 10.5 (B) 7.0
(B) 7.0 (C) 6.5
(C) 6.5 (D) 5.9
(D) 5.9

MN
MN 5:20
5/20

www.examrace.com
www.examrace.com
2003
2008 MAIN PAPER. -— MN
MAIN PAPER MN

Q26
Q.26 Match
Match the
the following:
following:

Blast
Blast Problem
Problem Cause
Cause

P
p Misfire
Misfire I Poor
Poor stemming
stemming
Q
Q Vibration
Vibration 2 Low
Low current
current
RR Blown-out shot
Blown-out shot 3 Excess
Excess charge
charge
sS Cut-off shot
Cut-off shot LI4
AM-— Improper
Improper delays
delays

(A) P-3,
(A) Q-2, R—4,
P-3, Q-Z, R-4, S-l
S-1 (B)
(B) P-4, 11-2, 5-3
0-1, R-2,
P4, Q-1, S-3
(C) P-2,
(C) P-2, Q-3,
Q-3, R-1, 8-4
R-l, S-4 (D)
(D) P-1, Q-2, R-4,
P-l, Q-Z, 5-3
R-4, S-3

Q27
Q.27 From
From the diagram shown
stress-strain diagram
the stress-strain below, the
shown below, the tangent and the
tangent and the seeant
secant moduli
moduli of elasticity in
of elasticity in GPa
GPa
are
90
90 _

73 1---------~
70 1 - - - - - - - - - . 1 '

E
E
2
§
10

10 20
20 30 33
30 33 4E}
40 50
50
Strain (micro units)
Strain (micro units)

(A)
(A) 4.0,
4.0, 22
2.2 (B) 3.3. 2.3
(B) 3.3. 2.3
(C)
(C) 3.3.
3.3, 1.5
1.5 (D)
(D) 4.0,
4.0, 1.5
1.5

Q28
Q.28 A
A herd
bord and
and pillar
pillar operation planned at
is planned
operation is depth of
at aa depth of 300
300 mm inin aa strata of average
strata of average unit weight 24.5
unit weight 24.5
km3
kN/m 3 and compressive strength
and compressive 15.50 MPa.
strength 15.50 MPa. If the width
If the of the
width of the Opening
opening is
is 66 In considering aa factor
m considering factor
of safety of
of safety l, the
of 1, the maximum ratio in
extractiOn ratio
possible extraction
maximum possible percentage is
in percentage IS

(A) 28
(A) 28 (B) 34
(B)34 (C) 45
(C) 45 (D)
(D) 53
53

Q29
Q .29 Match the
Match following:
the following:

Stoping method
Stoping method Ore handling
Ore handling Support system
Support system
systems
systems

P. Breast stoping
P . Breast stoping 1. LHD
I.LHD a.
a. In
In situ
situ pillars
pillars
Q.
Q. Cut
Cut and
and fill stoping
fill sloping 2. Scraper
2. Scraper b.
b. Unsupported
Unsupported
R. Sublevel
R. stoping
Sublevel stoping 3. Gravity
3. flow
Gravity flow c.
c. Mill
Mill railings
tailings

(A)
(A) P-2-a,Q-1-c,
P-2-a. Q-1-c, R-3-b
R-3-b (B) P-1-a, 0-3-2.
(B) P-l-a, 11—24:)
Q-3-c, R-2-b
(C) P-Zub, Q-
(C) P-2-b, l-a. R-3—c
Q-1-a, R-3-c (D) P-l-c, Q-3-a,
(D) P-1-c, R-2-b
Q-3-a, R-2-b

MN 6Q!)
61:!0

www.examrace.com
www.examrace.com
2003
2008 MAIN PAPER MN
MAIN PAPER-
-
MN

0.30
Q.30 Match
Match the following:
the following:

Access
Access Haulage
Haulage Mineralisation location
Mineralisation location

P. Shaft
P. Shafi 1. Track
1. Track 3.
a. Moderate
Moderate depth
depth
Q.Decline
Q.Decline 2. Trackless
2. Trackless b.
b. Deep
Deep seated
seated
R. Adit
R. Adit 3. Hoisting
3. Hoisting c. Hillock
c. Hillock
(A) P-l-a,
(A) P-1-a, Q-3-b, R-2-c
Q-3-b, R-2-c (B) P~3-b, Q-2-a,
(B) P-3-b, Q-2-a, R-1-c
R-l—c
(C) P-2-a,
(C) P-Z-a. Q-1-b,
Q-l-b, R-3-c
R-3—c (D) P-2-b,
(D) P-2-b, Q-3—c.
Q-3-c, R-l-a
R-1-a

Q.31
Q.31 Match
Match the
the following:
following:

Mining method
Mining method Operation
Operation

P
p Bord and
Bord and Pillar
Pillar 1l Longhole radial
Longhole radial drilling
drilling
Q
Q Sublevel caving
Sublevel caving 22 Splitting and slicing
Splitting and slicing
R
R Longwall retreating
Longwall retreating 33 Loosening
Loosening under
under strata
strata pressure
pressure
sS Integrated Caving
Integrated Caving 44 Mechanical
Mechanical cutting
cutting
(A) P-l.
(A) P-1, Q-4, R-3. S-2
Q-4, R-3, 8-2 (B) P-2,
(B) P-2. Q-3,
Q-3, R-l,
R-1, 3-4
S-4
(C) P-4,
(C) P-4, Q—2, R-3, 5-]
Q-2, R-3, S-1 (D) P—2, Q-l,
(D) P-2, R-4, 5—3
Q-1, R-4, S-3

0.32
Q.32 A 30m tape
A 30m tape has an error
has an error of
of±i 0.005
0.005 m. If aa length
m. If length of 1500 m
of 1500 In is measured with
is measured with this
this tape,
tape, the
the
expected total error
expected total error made in the
made in the measurement
measurement in in m is
m is

(A) i
(A) 0.025
± 0.025 (B)
(B) i 0.030
± 0.030 (C)
(C) :I:
± 0.035
0.035 (D)
(D)±i- 0.04
0.04

Q.33
Q.33 Match
Match the following:
the following:

Instrument
Instrument Principal features
Principal features Application
Application

P Tilting
P Tilting level
level 11 Micrometer
Micrometer aa Levelling
Levelling
Q Microoptic
Q Microoptic theodolite
theodolite 22 Magnetic
Magnetic needle
needle bb Traversing
Traversing
R
R Telescopic alidade
Telescopic alidade 33 U-tube
U-tube cc Azimuth
Azimuth (Bearing)
(Bearing)
S
S Compass
Compass 4 Plane table
4 Plane table surveying
surveying dd Contouring
Contouring

(A) P-1-b;Q-2-c;R-4
(A) P-l-b;Q-2-c;R—4-a;S-3-d
-a;S-3-d (B)
(B) P—4-b;Q-3-a;R—l-c;S-2—d
P-4-b;Q-3-a;R-1 -c;S-2-d
(C) P-2-c;Q-3-b;R-4
(C) P-2-c;Q-3-b;R-4—a;S-l -d
-a;S-1-d (D) P-3-a;Q-1-b;R-4
(D) P-3-a;Q- l —b;R-4—d;S-2-c
-d;S-2-c

Q34
Q.34 A confined
A confined aquifer
aquifer of 75 m
of 75 m thickness
thickness has
has 22 monitoring
monitoring wells spaced 2500
wells spaced 2500 m apart along
m apart along the
the
direction of
directiOn of water
water flow.
flow. The
The hydraulic
hydraulic conductivity
conductivity of of the
the aquifer is 40111
aquifer is 40m per day. The
per day. The water
water head
head
difference between the
difference between wells is
the wells is 1.5 m. Applying
1.5 m. Applying thethe Darcy’s law, the
Darcy's law, rate of
the rate flow per
of flow per meter
meter of
of
distance perpendicular
distance to the
perpendicular to the direction
direction of flow in
of flow in m
m3/day
3
/day is
is

(A) 2.!
(A) 2.1 (B) 1.8
(B) 1.8 (C) 1.45
(C) 1.45 (D) 1.21
(D) 1.21

0.35
Q.35 Precipitation of
Precipitation of metallic
metallic ions
ions in
in mine
mine water drainage is
water drainage is carried
carried out by
out by

(A) CaSOi
(A) CaS04 and MgSO4
and MgS04 (B) CaCO;3 and
(B) CaC0 MgCO;
and MgC03
(C) Ca(OI-l)3 and
(C) Ca(OH)2 NaOH
and NaOH (D)
(D) CaCO;
CaC03 and
and MgSO4
MgS04

MN
MN 7f20
7120
www.examrace.com
www.examrace.com
MAIN PAPER — MN
MAIN PAPER MN
2008
2008
carbon
fixed carbo has upper
n has and lower of 49%
limits of
lower limits and 41%
49% and 41%
Q.36 In aa mine, controll chart
mine, aa contro ucted for
constructed
chart constr for fixed upper and
Q.36 In
values of carbonn are
fixed carbo 42%, 43%, 40%, 50% and
tively . On
respectively. the five
day, the
On aa day, five group averagee values
group averag of fixed are 42%, 43%, 40%, 50% and
respec
is to have not
to have than 22 out
more than
not more of 5 samples to be out
out
49.5%.
49 .5%. IfIf the controll rule
processs contro
the proces of the
rule of mine is
the mine out of 5 sampl es to be

of the control
of the chart, the
contro l chart, process
the proce ss on that day
on that is
day is

(A) Above upper


(A) Above and below
upper and below lower contro l limits
lower control limits
(B) Abov
(B) upper
Abovee upper control
contro l limits
limits
(C) Below
(C) contro l limits
lower control
Below lower limits
(D) Within
(D) upper
Withm upper and
and lower
lower contro limits
controll limits

308 kW
of 308 when the
kW when rope speed
maxim um rope
the maximum is 77 m/s.
speed is m!s.
Q.37
Q.37 drum winder
A drum
A of radius
winde r of 2.5 m
radius 2.5 draws aa power
m draws power of
The torque in
RMS torque
The RMS in kNrn
kNm isis

(B) 76 (C) 110 (D) 144


(D) 144
55
(A) 55
(A) (B)76 (C) 110

averag e cross-sectional
of average m , of
of 0.099 1112,
area of0.0
2
bulkk density
densit y
Q38 belt conveyor
A belt conve yor conve mater ial of
ys material
conveys cross- sectio nal area ofbul
Q.38 A
speed 22 oils.
at aa speed The carrying capacity the belt
of the in tonne/hr is
tonne /m , at
1.5 tonne/m3,
3 m/s. The carryi ng capac ity of belt in tonne/ hr is
1.5
(B) 864
(B) (C) 732
(C) 732 (D)
(D) 643643
(A) 972
(A) 972 864

stowi ng pipe 60. If


is 60. densityy is
solids densit
the solids kg/m3 , the
3000 lag/m}. pulp
ater stowing
the pulp
Q39
Q.39 The wt
The wt% % of
of solids sand-water
in aa sand-w
solids in pipe is If the is 3000
of the
densityy of slurry in kglm is
in kg/m3L3
densit the slurry is

(B) 1420 (C) 1560


(C) 1560 (D) 1670
(D) 1670
(A)1380
(A)13 80 (B) 1420

activities uled for


scheduled
is sched 90 days
for 90 at aa cost of Rs.
cost of Rs.l21 200
00
Q40 miningg project
A minin comprising
projec t compr of A,
ising of and C
B, and
A, B, C activi ties is days at
Q.40 A
to reduce the time
reduce the completion
for compl of the projectt to
to 85
85
million. The manager
millio n. The the project
of the
manag er of decidess to
projec t decide time for etiOn of the projec

days. The
days. decision
The decisi taken after
was taken
on was after 45 days.
45 days.
B C
C
A
A B

Activity
Activi ty A
A B
B c
C

40 15 35
35
Duration(days)
Durat ion(da ys) 40 15

Crashing cost/day
ing cost/d 15
ay 15 25
25 20
20
Crash
(milli on rupees)
(million rupees)

in million
cost in
projectt cost after crashing
rupeess after crashi ng by days is
by 55 days is
The minimum
The minim um projec millio n rupee

(B) 1300
(B) (C) 1475
(C) (D) 1825
(D)
(A) 1100
(A) 1100 1300 1475 1825

Q.41
Q.41 The following
The inform ation is
follow ing information is provid ed for
provided for an ore deposit:
an ore deposit:

waste blocks
of waste = 10
= 10
Number
Numb er of block s
ore blocks
of ore
= 5
Number
Numb er of blocks = 5
block , in3 = 600
Volume
Volum e of waste block,
each waste
of each m3 = 600
3 =
Total cost of
Total cost per m3
handl ing per
waste handling
of waste m = Rs.
Rs. 100
I 00
of each ore block = 400
Tonnage
Tonna ge of each ore block = 400
per ton
handl ing per
ore handling
of ore = Rs.
Rs. 150
Total cost
Total cost of ton = 150
per ton
ore per = Rs. 500
Sale price
Sale of ore
price of ton = Rs. 500

The
The net of mining
flow of
cash flow
net cash the deposit
minin g the in lakhs
depos it in lakhs of rupees, is
of rupees, is

(B) 2.5
(B) (C) 1.0
(C) 1.0 (D) 0.3
(D) 0.8
(A) 3.4
(A) 3.4 2.5
0
8120
MN www.examrace.com
www.examrace.coriia
2008
2008 MAIN PAPER
MAIN PAPER-- MN
MN

Q42
Q.42 Determine the
Determine the correctness or otherwise
correctness or otherwise of
of the
the following Assertion [a]
following Assertion (a) and the Reason
and the Reason [1']
(r)

Assertion: While
Assertion: While stonedust
stonedust barrier may be
barner may effective against
be effective against aa coal
coal dust explosion, the
dust explosion, same is
the same is not
not
true in case
true in case of
of firedamp
firedamp explosions.
explosions.

Reason: In
Reason: In general
general firedamp
firedamp explosions are much
explosions are much more powerful than
more powerful than coal
coal dust explosions.
dust explosions.

(A) Both
{A) Both [a)
[a] and [r] are
and [r) false
are false
(B) [a)
(B) [a] is
is true
true but [r] is
but [r] is false
false
(C) Both
(C) Both [a]
[a] and
and [r]
[r] are
are true and [r]
true and [r] is
is the correct reason
the correct reason for
for [a]
[a]
(D) Both
(D) Both [a]
[a] and
and [r]
[r] are
are true
true but [r] is
but [r] is not the correct
not the correct reason for [a)
reason for [a]

Q43
Q.43 Match the
Match the following:
following:

Component of
Component of flame
flame safety
safety lamp
lamp Purpose of
Purpose component
of component

P
P Asbestos rings
Asbestos rings I1 Dissipation of
Dissipation of heat
heat of
of flue
flue gas
gas
Q
Q Wire gauges
Wire gauges 22 Formation
Formation of air-tight joints
of air-tight joints
R
R Outer glass
Outer glass 33 Arrest of explosion
Arrest of explosion inside
inside the lamp
the lamp
S Combustion
S chimney
Combustion chimney 4
4 Separation of
Separation inlet air
of inlet from flue gas
air from flue gas
(A)
(A) 13-2, Q-l. R-3,
P-2, Q-1, R-3, S-4
8-4 (B) P-4,
(B) P-4, Q-l, R-Z, S-3
Q-1, R-2, S-3
(C)
(C) P-Z,
P-2, Q—4, R-3. S-1
Q-4, R-3, 3-] (D) P-1,
(D) P—l, Q-Z, R-4. S-3
Q-2. R-4, S-3

Q44
Q.44 A
A roadheader district produces
roadheader district 20 mg/
produces 20 mg/ m3
m3 of airborne dust
of airborne dust with
with the following size
the following size distribution:
distribution:

Size up to
Size up to Cumulative wt
Cumulative wt %
%

1 pm
lJ..Lm 1
55 um
~un 5
10 J..lm
10 pm 10
10
20 um
20J..Lm 20
20
50 um
50 J..llll 50
50
> 50 J..llll
>50 pm 100
100

The concentration
The concentration of
of respirable
respirable fract1on
fraction of dust in
of dust in mg/m3 is
mg/m3 is

(A) 0.2
(A) 0.2 (B) 2.0
(B) 2.0 (C) 10.0
(C) 10.0 (D) 1.0
(D) 1.0

0.45
Q.45 For aa person
For person working in an
working in an atmosphere
atmosphere containing 21% 002,
containing 21% the exhaled
2, the exhaled air
air contains
contains 4.5%
4.5% C0
CO;2 and
and
16% 0;.2. The
16%0 The respiratory
respiratory quotient
quotient of
of breathing is
breathing is

(A) 0.21
(A) 0.21 (B) 0.9
(B) 0.9 (C) 0.28
(C) 0.23 (D) 1.11
(D) 1.11

Q46
Q.46 Total number
Total number of injuries in
of injuries in an
an opencast
opencast coal
coal mine
mine employing
employing 800 persons is
800 persons is 16
16 in
in aa year.
year. As per
As per
DGMS norms,
DGMS norms. the
the injury
injury rate 1000 persons
per 1000
rate per employed is
persons employed is
(A) 13
{A) 13 (B) 15
(B) 15 (C)
{C) 20
20 (D) 25
(D) 25

Q47
Q.47 The coefficient
The coefficient of
of friction between the
friction between the tub-wheel and haulage
tub-wheel and haulage track is l1N3.
track is /-J3. For
For the applicability of
the applicability of
direct haulage, minimum
direct haulage, minimum inclination (in degrees)
inclination {in of track
degrees} of track is
is

(A) 60
(A) 60 (B) 55
(B) 55 (C) 35
(C) 35 (D) 30
(D) 30
MN
MN 9:20
9/20

www.examrace.com
www.examrace.com
2003
2008
MAIN PAPER -— MN
MAIN PAPER MN

Q.48
Q.48 A surface mine
A surface mine blast
blast pattern
pattern shown
shown below
below has
has the
the following
following details:
details:

Accessory
Accesso ry Resistance
Resistance (in
(in Ohms)
Ohms) Number
Number or
or Length
Length
Detonator
Detonat or 22 per detonatorr
per detonato 40
40 nos
nos
Connecting
Connec ting wire
wire 0.5/m
0.5/m 100
lOOmm
Bus wire
Bus wire 0.5/m
0.5/m 100 m
100m
Firinflte
Firing line 0.01/m
0.01/m 200
200mm

Connecting wire

Bus wire

Firing line

If the
If the explode
exploderr supplies
supplies 440
440 V,
V, the
the current
current in
in the
the blasting
blasting circuit in ampere
ampere is
circuit in is

(A) 5.36
(A) 5.36 (B) 3.51
(B) 3.51 (C) 4.83
(C) 4.83 (D)
(D) 2.57
2.57

Q49
Q.49 In
In aa surface
surface mine
mine blast, the peak
peak particle velocity (V
(V in
in mm/s)
mm/s) is
blast, the particle velocity is estimated
estimated from
from the
the equation
equation
V=
V 120( .JSD
=120(-JSD)_1'°f '- ,, where
0
where SDSD is square root
root scaled distance.. If
If at
at aa distance
is square scaled distance of 100
distance of 100 m
m from the
from the
blast site the
blast site the permissible
permissible peak
peak particle
particle velocity
velocity is
is 25
25 mm/s,
rrun/s, the
the maximum charge per
maximum charge per delay
delay in
in kg
kg is
is

(A) 404
(A) 404 (B) 414 (C) 434
(B) 414 (C) 434 (D)
(D) 464
464

Q50
Q.50 Daily production
Daily product ion measured
measure d for
for aa period
period of
of 50
50 days in aa coal
days in coal mine
mine exhibits
exhibits normal
normal distribution
distribution with
with
mean 1200 tpd
mean 1200 and standard
tpd and standard deviation
deviation I100
00 tpd.
tpd. The 95% confiden
confidence ce interval
The 95% interval of daily production
of daily production
(standard
(standard normal
normal variable
variable Z
Z at
at 0.025
0.025 level of significance
significance is
is 1.96)
1.96) in
level of in tpd
tpd is
is

(A) 1200
(A) 1200 ±
1: 120.5
120.5 (B) 1200 :I: 96.0
1200 ± 96.0 (C) 1200
1200 ±
i 39.6
(B) (C) 39.6 (D) 1200 i
(D) 1200 ± 27.7
27.7

Q.51 In an iron ore


ore deposit
deposit alumina
alumina is
is distributed n == 33 % <:r == 0.5%;
Q.51 In an iron distributed with % and
and 0'
with ~ 0.5%: whereas
whereas silica is
silica is
distributed
distributed withwith u = 2.5
~ = % and 0' == 0.8%. The combine
2.5 % and <:r 0.8%. The combined alumina and
d alumma and silica
silica (as
(as impurities)
impurities) has
has n
~
and 0' ,, in
and <:r in percentage
percentage respectively
respectively as
as

(A)
(A) (5.5,
(5.5. 0.94)
0.94) (B) (5.5,
(5.5, 1.3) (C) (0.5, 0.3)
(B) 1.3) (C) (0.5, 0.3) (D) (5.5,
(D) 0.62)
(5.5, 0.62)

MN
10120
www.examrace.com
www.examrace.c'8iii)
2003
2008 MAIN PAPER — MN
MAIN PAPFR- MN

0-52
Q.52 The inverse
The inverse of
of the
the following
following matrix
matrix is:
is:

4 0 0
0 2 0
0 0 1

(I6 0 0
(A) 0 4 0
[025
(B) 0
(B)
(0.25
0
0
0
0.50
0.50
0
0
(0 0 l 0
K 0 0
0 1.00
(2 0 0 (16 0 0
(C) 0 «5 0 (D)['
(D) 6 0 0
4 1 0 4
K0 0 l K 00 0
0 :J
l

(2.53
Q.53 The solution
The solution of
of the
the following
following system
system of
of linear equations is
linear equations is

xx+4y+3z=0
l-4y+3z=O
3x+5y+22=0
3x-•5y · 2z 0
8x+10y+l2z=0
8x · I Oy+ l2z-c0

(A) (0,0,0)
(A) {0,0,0) (B) (1,-1,1)
(B) (11'111) (C)
(C) (2, *1, -2)
(2, -1, 43) (D)
(D) (-3.0,1)
(-3,0,1)

0.54
Q.54 The volume
The volume of
of aa cone
cone IS given by
is g1ven by
V=
V = g? sin2 Bcos
tr l 3 sin' 6cost9B
3
where,
where, (f? is
IS the slant height
the slant height and 6 is
and (} is the semi-vertical angle.
the sem1-vert1cal The angle
angle. The angle (6 ), for
( 0 }, for which the volume
which the volume
of cone becomes
of cone becomes maximum
maximum is
is

(A) sin
(A) sin"(i)
(~)
J3
1
(B) cos-*(J—o
J5
(C) cos"(x/§)
(C) COS I (.fi) (D) sin"'(\/§)

Q5
Q.55 The d~reclion
The direction of
of gradient vector at
gradient vector at aa point
point (l,
(1 , l,
l, 2)
2) on
on aa surface
surface S(x,
S(x, y, z) =
y, z) +/-- zz ISis
any"1
= x 2

11 . . 1
1 . . +k)
(A) -§(21+2J
(A) Hi)
(2i +2j + k) (3) -(-2i
(B) (-2! +2j
+2] “0
3 *3"
3
(C)
(C) -%(2i
3
1
(2i -2i +k)
-2j +k) (D) gm
I
+2j —k)
(D) -(2i +2j k)
3

Q.S6
Q.56 The solution
The solution of
of the differential equation
the differential equation

22
d—Z+3+d—y—4y=o,is
d y dy
-+3--4y=O, Is
.
dx
dx 2
dx
dx

(A) yy
(A) = cole“
= 4
1e ' (B) yy
(B) = cclez"
= eh
1

(C) y == cclc"
(C) y + cole—4"
1e' + 2e
41
(D) yy
(D) ==c.e
c,e""‘'+
+ ccze“
4
2e x

MN
MN
www.examrace.com
wwéRamracecom
11 120
2008
2008 MAIN PAPER ‘- MN
MAIN PAPER MN

Q57
Q .57 A force vector
A force F=
vector F (2i+3j-k) inN
= (2i+3j-k) in N is
IS acting point, whose
on aa point,
acting on vector 1'r =
position vector
whose position = (i-j+6k)
(i-j+6k) in
in In. The
m. The
magnitude
magnitude of
of the
the torque about the
torque about origtn in
tbe origin Nm is
tn Nm ts

(A)
(A) 20.85
20.85 (B) 21.42
(B) 21.42 (C) 21.97
(C) 21.97 (D) 22.27
(D) 22.27

(3.53
Q .58 If H
If H is the maximum
is the maximum height by aa projectile,
attained by
height attained projectile, the maximum horizontal
the maximum range when
horizontal range fired at
when fired at
45”
45° inclination
inclination from
from ground level is
ground level IS

(A) 4.0H
(A) 4.0H (B)
(B) 1611
3.6H (C) 3.2H
(C) 3.2H (D) 2.7H
(D) 2.7H

0.59
Q .59 Force diagram
Force diagram for
for aa square frame is
square frame shown below.
is shown below. Considering clockwise moment
Considering clockvvise positive, the
as positive,
moment as the
resultant
resultant moment about an
moment about an axis
axis passing
passing through point A
the point
through the A in Nm is
in Nm is

D 4 "i c
3 N 'h v
5N

A B
2N

(A) 88
(A) (B) 55
(B) (C) 33
(C) (D) -2
(D) -2

0.60
Q .60 The local
The mean time
local mean time at longitude 75°30
at longitude is 3hr
75fl30'1 is 8hr 45min. The corresponding
45min. The corresponding standard time with
standard time with
reference
reference to 82° 30
30' meridian is
1
to 82° meridian is

hr 13 min
(A) 83hr13
(A) min (B)9hrl3min
(B) 9 hr 13 min (C) 911:1?
(C) min
9 hr 17 min (D) 10hr17min
(D) 10 hr 17 min

0.61
Q.61 Block economic values
Block economic values in
in aa 2D model are
block model
2D block are shown
shown below. based on
Then based
below. Then on the assumption of
the assumption of 1:1
I :I
slope angle, the
slope angle, the blocks (identified by
blocks (identified by row
row and
and column numbers) that
column numbers) that constitute the ultimate
constitute the pit are
ultimate pit are

1 22 33 4
4

1 -1
-1 -1
-1 1
1 -1
-1

2
2 -1
-1 1
1 3
3 -1
-1

33 -1
-1 -1
-1 -1
-1 -1
-1

(1.1), (1,2),
(A) (U),
(A) (1 ,2), (1,3),
(1 ,3), (212)
(2,2) (B)
(B) (12), (1,3), (1.4).
(1 ,2), (1.3), (1,4), (23)
(2,3)
(C) (1,3),
(C) (1 ,3), (244)
(2,4) (D) (13),
(D) (1,4), (2.4)
(1,3), (1.4), (2,4)

MN
MN IZIZU
12120

www.examrace.com
www.examrace.com
2008
2008
MAIN PAPER - MN
MAIN PAPER MN
062
Q.62 The feasible
The region of
feas1ble region of an
an LP
LP problem
problem is is shown
shown as
as given
given below.
below. The maximum value
The maximum of the
value of the
objective function
objective function Z =1600xl+l200x2_
Z =1600x 1+1200x 2• is
is

(3,13)

x2

(”1”) )(1 (12.0)

(A) 20400
(A) 20400 (B) 20000
(B) 20000
(C) 19200
(C) 19200 (D) 16800
(D) 16800

Q63
Q.63 A
A conveyor
conveyor of
of rated
rated power 100 kW
power 100 kW hauls
hauls coal
coal up-dip
up-dip at 30 kgls
at 30 kg/s along
along an
an inclination of 15°
inclination of 15‘] and
and
distance 300m.
distance 300m. Heat added by
Heat added by the
the conveyor
conveyor to
to the
the air
air in
in kW
kW is
IS

(A)56.4
(A) 56.4 (B) 65.9
(B) 65.9 (C) 77.2
(C) 77.2 (D) 82.3
(D) 82.3

(2.64
Q.64 A cage of
A cage of floor
floor area
area 5.0
5.0 In2 suspended in
m2 suspended in aa shaft
shaft has
has aa drag coefficient 2.5.
drag coefficient 2.5. If
If the
the velocity
velocity of air in
of air in
the
the shaft
shaft is
is 6.0
6.0 m/s,
m/s, the
the drag
drag force (N) experienced
force (N) experienced by by the
the cage
cage is
is

(A) 120
(A) 120 (B)
(B) J170
70 (C) 200
(C) 200 (D) 270
(D) 270

Q65
Q.65 A
A cash flow diagram
cash flow diagram is shown below.
below. Based on NPV,
NPV, at 10 % rate
rate of
of interest,
interest, the
the minimum
is shown Based on at 10% annuity
minimum annuity
‘x' at
'x' at which
which the investmentt becomes
the investmen becomes viable
viable is
is

xX xX xX xX

'
00 1 22 33 44 year
year

200
200

(A) 63
(A) 63 (B) 54
(B) 54 (C) 42
(C) 42 (D) 35
(D) 35

0.66
Q.66 A system of
A system of two
two identical
identical mine pumps connected
mine pumps connected in series has
in series has reliability 0.49. If
reliability 0.49. If the
the pumps
pumps were
were to
to
be connected
be connected in
in parallel,
parallel, the
the system
system reliability
reliability would
would be
be

(A) 0.21
(A) 0.21 (B) 0.6
(B) 0.6 (C) 0.91
(C) 0.91 (D) 0.95
(D) 0.95

MN
MN
[3(20
13/20
www.examrace.com
www.examrace.com
2008
2008 PAPER -- MN
~WN PAPER
MAIN MN

(11.67
Q.67 An working at
SDL working
An SDL at different faces gives
different faces the following
gives the following performance:
performance:

Operating Face
Operating Face Production per blast
Production per blast Muck clearance time
Muck clearance time
(tonne)
(tonne) (hrs)
(hrs)

Development face
Development face 6
I16 I .25
1.25
Splitting face
Splitting face 17
17 1.20
1.20
Slicing face
Slicing face 13
18 1.30
1.30
Heightening face
Heightening face 20
20 1.50
1.50

In 55 hrs
In operation maximum
hrs operation is obtainable
output is
maximum output from the
obtainable from the

(A) Heightening face


(A) Heightening face (B) Slicing
(B) face
Slicing face
(C)
(C) Development face
Development face (D) Splitting
(D) face
Splitting face

Q68
Q.68 In aa coal
In coal handling
handling plant of 8m
wagons of
plant wagons length are
8m length loaded, at
are loaded, rake travel
at rake speed of
travel speed of 0.48 km/hr. The
0.48 kin/hr. The
chute
chute loading rate is
loading rate is 6000 As the
tonne/hr. As
6000 tonne/hr. rake moves
the rake continuously.
moves continuously. the
the chute
chute stops
stops for
for a
a total
total of
of
24s in
24s in between two wagons.
between two The quantity
wagons. The quantity of coal
of coal in
in tonne
tonne loaded
loaded in
in each
each wagon
wagon is
is

(A) 52
(A) 52 (B) 60
(B) 60 (C) 76
(C) 76 (D) 94
(D) 94

Q69
Q.69 Match the
Match the following:
following:

Failure criteria
Failure criteria Relationship
Relationship

P.
P. Drucker- Prager
Drucker- Prager l.
l. a" 0'3 +
=or.
0', = ~m0'3 4—+ s2
+ (into! s2

Q.
Q. Hoek -— Brown
l-loek Brown 2.
2. T =
r = cc + an tan
+ 0'0
p
tan¢

R.
R. Mohr— Coulomb
Mohr - Coulomb 3.3. J%[("' ;2 "')'J ·(
RR CTl *0'I
2

{—7) "·;"' ]
"';"}(+[———2]
{TI “ff3
1
0'3 "UI
)'f
I y:

=.r'5t(t:rl +or2 +03)+B

(A) Q-3, R-2


P-1, Q~3.
(A) P-l. R-2 P-3 , Q-l.
(B) P-3.
(B) R-Z
Q-1, R-2
(C)
(C) Ps3.
P-3 , Q-2.
Q-2, R-l
R- L (D) P-l, Q-Z.
(D) P-1, R-3
Q-2, R-3

0.70
Q.70 An assay
An value of
assay value of alumina in aa borehole
alumina in borehole from bauxite deposit
from aa bauxite is as
deposit is shown below.
as shown If the
below. If cut-off
the cut-off
grade is
grade 40%, the
is 40%. the composite value
composite value of
of ore
ore in
in the
the borehole
borehole in
in percent
percent is
is

44%

m 42%
42%

\ 41%
41%

E a" 39%
im
39%

(A) 31.6
(A) 31.6 (B) 33.9
(B) 33.9 (C) 41.7
(C) 41.7 (D) 42.2
(D) 42.2

MN
MN www.examrace.com
www.9xamracé'i88m
14/20
2003
2008
MAIN PAPER -— t.fN
MAIN PAPER MN
Common
Com Data Que
mon Data Questions
stion s
Common
Comm Data for
for Ques
Questions 71, 72
72 and
and 73:
73: Two
on Data tions 71, Two block
blockss of mass 55 kg
of mass kg and
and 10
10 kg
kg are
are conne
connected
cted with
cords and frictio
cords and frictionless pulleyss as
as show
shown. Friction
with
nless pulley n. Fricti coefficient
on coeff between
icient betwe the 55 kg
en the kg block
block and
and table
table is
is 0.2.
0.2.

5kg

5kg
5 kg 10kg
10 kg
0.71
Q .7 1 The accel
The acceleration of the
the system
system when
when the
the block
blockss are
eration of are releas
released from rest
ed from rest (( •‘g’ is accele
g' is acceleration due to
ration due to
gravity)
gravit is
y) is
(A) 5g
(A) 53 (B) 2g
(B) Zg (C) g/5
3/5
(C) (D) g/10
(D) g/10
Q.72
Q. 72 Tension
Tensi (N) in
in the
the cord
cord conne
connected to the
on (N) cted to theIO kg block
I 0 kg block is
is
(A) 83g
(A)8 (B) 6g
(B) 63 (C) 4g
4s
(C) (D) 23g
(D)2
0.73
Q.73 Tension
Tensi (N) in
in the
the cord
cord conne
connected to the
the 55 kg
on (N) cted to kg block
block is
is
(A) 8g
(A) 83 (B) 6g
(B) 6s (C) 4sg
(C)4 (D) 23
(D) 2g
Common
Comm on DData for Ques
ata for Questions 74 and
and 75
75:: Three
Three boreh
tions 74 boreholes intersect
oles inters ect aa coal
coal seam
seam at
at point
pointss A,
A, B
B and
and Cas
shown. (figuree is
is drawn
drawn to
to scale) C as
show n. (figur scale)::

N
N
‘l

1in3
1 in 3

3cm

c
B

The surve
The surveyy detail
detailss are
are given
given below
below

Line
Line Bearing
Beari ng Gradient
Gradi ent

AB
AB S 40°W I1 in
in 55
AC
AC S 30°13 1l m3
in 3

0.74
Q.74 The direct
The direction of true
ion of true dip
dip of
of the
the seam
seam is
is

(A)S
(A) l5°W
S 15° W (B)S25°W (C) SS 15°
(B) S 25° W (C) lS°EE (D) S
(D) S30°E
30° E
MN
MN
lSr’Zfl
15120
www.examrace.com
www.examrace.com
2008
2008 MAIN PAPER _ MN
MAIN PAPER MN

0.75
Q.75 The gradient of
The gradient of the
the seam
seam is
is
(A) 1l in
(A) in 2.7
2.7 (B) l in
(B) 1 3.7
in 3.7 (C) l i114.7
(C) 1 in 4.7 (D) I1 in
(D) 5.7
in 5.7

Linked Answer
Linked Q.76 to
Questions: Q.76
Answer Questions: Q.85 carry
to Q.85 two marks
carry two each.
marks each.

Statement for
Statement Linked Answer
for Linked Questions 76
Answer Questions 77: An
and 77:
76 and open pit
An open pit mine has aa potential
bench has
mme bench failure
potential failure
plane
plane as indicated below.
as indicated below. The
The unit
unit weight,
weight, cohesion,
cohesion, and
and angle
angle of
of internal
internal friction
friction of
of the
the rock
rock mass
mass are
are
24.5km3, 0.02
24.5kN/m 3
, 0.02 MPa
MPa and
and 30“
30° respectively.
respectively.

T i
25m
645“ :35”

W‘s.
l
Q76
Q.76 The driving force
The driving for failure,
force for failure, on the potential
on the potential failure plane is
failure plane is

(A) 187 N
(A) 187 N (B) 1.87
(B) kN
1.87 kN (C) 18.? RN
(C) 18.7 kN (D) 1.87 MN
(D) 1.87 MN

Q7?
Q.77 The ‘factor of
The 'factor of safety' of slope
safety’ of under given
slope under given conditions is
conditions is

(A) 0.7
(A) 0.7 (B) 0.9
(B) 0.9 (C) 1.1
(C) 1.1 (D) 1.3
(D) 1.3

Statement for
Statement for Linked Questions 78
Answer Questions
Linked Answer and 79:
78 and Mine water
79: Mine flowing at
water flowing 1.5m3fs
at 1.5m 3
with 22 mg]
/s with mg/1
dissolved
dissolved oxygen, joins river
oxygen, joins river water flowing at
water flowing 7m3/s
at 7m 3
/s containing 6mg/1 dissolved
containing (5n) dissolved oxygen.
oxygen.

Q38
Q.78 The dissolved oxygen
The dissolved concentration of
oxygen concentration the mixture
of the in mg]!
mixture in is
mg/1 is

(A) 5.3
(A) 5.3 (B) 4.8
(B) 4.8 (C) 4.2
(C) 4.2 (D) 3.9
(D) 3.9

Q39
Q.79 The saturated value
The saturated of the
value of dissolved oxygen
the dissolved in the
oxygen in mixture is
the mixture given to
is given to be 9.3mg/1. On
be 9.3mg/l. this basis.
On this basis, the
the
deficit of
oxygen deficit
imtial oxygen
initial the
of the mixture
mixture in
in mg/l
mg/1 is
is
(A) 2.4
(A) 2.4 (B) 4.0
(B) 4.0 (C) 6.8
(C) 6.8 (o) 14.6
(D) 14.6

Statement for
Statement for Linked Answer Questions
Linked Answer 80 and
Questions 80 81: Unit
and 81: matrix of
cost matrix
Unit cost of aa transportation problem is
transportation problem is
given below
given below in monetary units.
certain monetary
in certain units.
Destination
11 22 33

1 22 22 8e 15
15

Source
Source 22 11 55 77 40
40 SUPP”
Supply

33 e6 44 33 20
20

1o
10 25
25 4.0
40
Demand
Demand

0.80
Q.80 The total cost
The total cost of based on
transportation based
of transportation initial basic
the initial
on the feasible solution
basic feasible by the
obtained by
solution obtained the North-
North-
West
West corner
corner rule
rule is
IS
(A) 250
(A) 250 (B) 290
(8)290 (C) 330
(C}330 (D) 360
(D)360

MN
MN I tflfl
16F .l0

www.examrace.com
www.examrace.com
2008
2008 PAPER -- MN
MAIN PAPER
MAIN MN

081
Q.81 The optimal
The solution for
optimal solutiOn for the
the transportation
transportation problem
problem has
has allocation
allocation as
as shown
shown below:
below:

Destination
1 2
2 3
3

1 15 15

Source
Source 22 10
10 1o
10 20
20 4o
40 supp”
Supply

3
3 20
20 20
20

1t]
10 25 40
40
Demand

When compared to
When compared to initial
initial basic feasible solution
basic feasible solution from
from the above, the
the above, the optimal
optimal allocation
allocation results
results in
in
savings of
savings of
(A) 10
(A) 10 (B) 20
(B) 20 (C)
(C) 30
30 (D) 40
(D) 40

Statement for
Statement for Linked
Linked Answer Questions 82
Answer Questions 82 and
and 83:
83: In
In aa mine
mine ventilation system, the
ventilation system, the resistances of two
resistances of two
Splits A
splits A and
and BB are 0.5
are 0.5 Ns‘zm'3
2 8
and 2.0
Ns m- and 2.0 sm'3
Ns m- respectively.
2 8
respectively. Combined resistance of
Combined resistance of two shafts and
two shafts and trunk
trunk
airways is
airways is 0.7
0.7 Ns
sm'a.
2
m-8 • A quantity of
A quantity 20 111%
of20 m3/s of air passes
of air passes through
through split A.
split A.

0.82
Q.82 The
The total air quantity
total air quantity passing
passing the
the mine
mine in
in mJIs
m3/s is
is

(A) 30
(A)30 (B) 27
(B) 27 (C)
(C) 25
25 (D) 17
(D) 17

Q83
Q.83 The total
The total air power of
air power the ventilation
of the ventilation system
system in kW is
in kW is

(A) 82.9
(A) 82.9 (B) 48.9
(B) 48.9 (C) 24.9
(C) 24.9 (D) 27.9
(D) 27.9

Statement for
Statement Linked Answer
for Linked Answer Questions
Questions 8484 and
and 85:
85: A loco of
A loco of mass
mass 10000
I 0000 kg
kg has
has aa coefficient of
coefficient of
adhesion to
adheswn to the
the tracks as 0.25.
tracks as 0.25. The
The loco offers aa running
loco offers running resistance equal to
resistance equal 10% of
to 10% of its
its weight.
weight.

0.84
Q.84 The draw-bar-pull
The generated by
draw-bar-pull generated the loco
by the on aa level
loco on level ground
ground in
in kN
kN is
is

(A) 11.3
(A) 11.3 (B) 14.7
(B) 14.7 (C) 15.8
(C) 15.8 (D) 17.2
(D) 17.2

(3.85
Q.85 The draw-bar-pull
The draw-bar-pull generated
generated by
by the
the loco
loco when
when the upward gradient
the upward grad1ent of the track
of the track is
LS 5" in kN
5° in kN is
is

(A) 6.16
(A) 6.16 (B)
(B) 7.9
7.9 (C) 9.5
(C) 9.5 (D) 11.5
(D) 11.5

END OF
END OF THE
THE QUESTION PAPER
QUESTION PAPER

MN
MN I 3'20
17f20

www.examrace.com
www.examrace.com
2009
2009 MN

MN :: MINING
MN MINING ENGINEERING
ENGINEERING

Duration Three Hours


Duration :: Three Hours - Maximum Marks
Maximum Marks :100
:100

Read the
Read the following
following instructions carefully.
instructions carefully.

This question
1. This question paper contains 16
paper contains 16 printed including pages
pages including
printed pages for rough
pages for work. Please
rough work. Please check
check all
all pages
pages and
and
report discrepancy,
repon discrepancy, if
if any.
any.

Write your
2. Write your registration
registration number,
number. your
your name and name
name and of the
name of the examination
examination centre at the
centre at the specified
specified locations
locations
on the
on the right half of
right half of the
the Optical
Optical Response
Response Sheet
Sheet (ORS).
(ORS).

Using HB
3. Using HR pencil. darken the
pencil, darken the appropriate
appropriate bubble under each
bubble under digit of
each digit your registration
of your registration number
number and
and the
the
letters corresponding
letters corresponding to
to your
your paper
paper code.
code.

All questions
4. All questions in
in this
this paper are of
paper are of objective
objective type.
type.

Questions must
5. Questions must be
be answered
answered on
on Optical
Optical Response
ReSponse Sheet
Sheet (ORS)
(ORS) by darkening the
by darkening the appropriate
appropriate bubble
bubble
(marked A,
(marked A. B,
B. C,
C. D)
D) using
using HB
HB pencil
pencil against the question
against the number on
question number on the
the left
left hand
hand side
side of
of the
the ORS.
ORS. E
Each
ach
question has
question only one
has only one correct
correct answer.
answer. In
In case
case you wish to
you wish to change
change anan answer,
answer. erase
erase the old answer
the old answer
completely. More
completely. More than
than one
one answer
answer bubbled against aa question
bubbled against will be
question will as an
treated as
be treated an incorrect
incorrect response.
response.

There are
6. There are aa total of 60
total of 60 questions ca_r[ying 100
questions carrying 100 marks. Questions 11 through
marks. Questions through 20
20 are
are 1-mark
l-rnark questions,
questions.
questions 21
questions 21 through
through 60 are 2-mark
60 are 2-mark questions.
questions.

Questions 51
7. Questions 51 through
through 56
56 (3
(3 pairs)
pairs) are
are common
common datadata questions
questions and question pairs
and question (57. 58)
pairs (57, 58) and
and (59,
(59. 60)
60) are
are
linked answer questions.
linked answer questions. The
The answer
answer to to the
the second
second question of the
question of above 22 pairs
the above depends on
pairs depends on the
the answer
answer
to the first
to the first question
question ofof the
the pair. If the
pair. If first question
the first in the
question in linked pair
the linked is wrongly
pair is wrongly answered
answered or is
or is
un-attempted. then the
un-attempted, then answer to
the answer to the
the second
second question
question in the pair
in the pair will not be
will not evaluated..
be evaluated

8. Un-attempted questions
Un-attempted will carry
questions will carry zero
zero marks.
marks.

9. Wrong answers
Wrong will carry
answers will carry NEGATIVE
NEGATIVE marks.
marks. For
For Q.
Q.ll to 0.20. ~
to Q.20, *2: mark will be
mark will be deducted for each
deducted for wrong
each wrong
answer. For Q.
answer. For Q. 2121 to
to Q.
Q. 56,
56. %
3%: mark will be
mark will deducted for
be deducted for each wrong answer.
each wrong The question
answer. The question pairs
pairs
(Q57. Q.58),
(Q.57, Q58). and
and (Q.59,
(Q59. Q.60) are questions
Q60) are questions with
with linked
linked answers.
answers. There
There will
will be negative marks
be negative marks only
only for
for
wrong answer
wrong answer to
to the
the first
first question
question of
of the
the linked
linked answer
answer question
question pair i.e. for
pair i.e. for Q.57
Q57 and
and Q.59,
Q59. % 55 mark will
mark will
be deducted for
be deducted for each
each wrong
wrong answer.
answer. There
There is
is no
no negative marking for
negative marking for Q.58
Q58 andand Q.60.
Q60.

10. Calculator (without


10. Calculator data connectivity)
(without data connectivity) is
is allowed
allowed in
in the
the examination hall.
examination hall.

ll. Charts. graph


II . Charts, graph sheets
sheets or
or tables
tables are
are NOT allowed in
NOT allowed in the
the examination hall.
examination hall.

12. Rough
12. work can
Rough work can be done on
be done on the
the question paper itself.
question paper itself. Additionally, blank pages
Additionally. blank are given
pages are at the
given at end of
the end of
the question
the question paper for rough
paper for work.
rough work.

1116
1/16

www.examrace.com
www.examrace.com
2009
2009
MN

Q. l—
Q. 1 - Q.
Q. 20 carry one
20 carry mark each.
one mark each.

Q]
Q.l If A
If is an
A is an orthogona
orthogonall matrix, then
matrix, then

(A) AT = A" (3)


(B) AT = .4“
AT =-A-1 (C) A = A" (D)
(D) A = — A-
A=- A"1

Q.2
Q.2 In aa normal
In (Gaussian)) distributio
normal (Gaussian distribution curve, the
the area
area between
between one
one standard
standard deviation
deviation from
n curve, from mean
mean on
on
either side in
either side in percent
percent is
is '

(A) 50
(A) 50 (B) 68
(B) 68 (C) 86
(C) 86 (D) 95
(D) 95

Q.3
Q.3 A measure of
A measure of dispersion
dispersion of
of aa sample
sample data set is
data set is
(A) mean
(A) mean (B)
(B) median
median (C) mode
(C) mode (D) standard deviation
(D) standard deviation

. 2 -— 22
Q4
QA The value
The off 11m ( 2v'4-x
value o ) 1s
x—iz
x-+2 5
llm[-—4—x—]is

(A)- 2 8
2../8 2
(A) ——;—/—— (3)0
(B) 0 (C) {E
(C) 2../8 (D) non-existent
(D) non-existent
5 5

Q5
Q.S and k
{.3 and
i,j I; represent
representthe unit
the unit vectors
vectors in the positive
in the positive x.
x, yy and
and z directions
directions of
of aa Cartesian
Cartesian coordinate
coordinate
system. Using
system. Using the
the right-hand rule, k
right-hand rule, x j represents
E x} represents

(A)
(A)O0 (B) l
(B) 1 (C) -—i
(C) -i
~

(D) i
(D) i
A

Q6
Q.6 The rock
The rock mass classification
mass classificat system that
ion system considers "active
that considers “active stress"
stress" factor is
factor is
(A)
(A) Q-system
Q-system (B) RMR
(B) RMR (C)
(C) RQD
RQD (D) 681
(D) GSI

Q-7
Q.7 In aa triaxial
In triaxial compression
compression test
test if (11'11 is
if cr is axial
axial stress
stress and
and cr
0'22 and
and 0'3 are confining
cr3 are confining stresses,
stresses, then
then

(A) 0'3 >02 =01 (B) 0', >02 =03 (C) 01:02 >03 (D) 0'3 =02 >0,

Q.8
Q.8 In aa longwall
In longwall mining subsidencee phenomenon,
mining subsidenc phenomenon, the “angle of
the "angle of break"
break" is
is the angle between
the angle between
(A) the
(A) the vertical line at
vertical line at the
the panel edge and
panel edge and line connectingg the
the panel
panel edge and zero subsidencee on
line connectin edge and zero subsidenc on the
the
surface
surface
(B) the vertical
(B) the vertical line
line at
at the
the panel edge and
panel edge and line connectingg the
the panel edge and
and point
line connectin panel edge of critical
point of critical
deformation
deformati on the
on on the surface
surface
(C) the
(C) the vertical line at
vertical line at the
the panel
panel edge
edge and
and line
line connecting
connecting the
the panel
panel edge
edge and
and the
the point of the
point of the
maximum tensile
maximum tensile strain on the
strain on the surface
surface
(D) the horizontal
(D) the horizontal line
line and
and the
the line
line connecting
connecting the
the panel
panel edge and zero
edge and zero subsidenc
subsidencee on
on the surface
the surface
MN
2:16
MN
2/16

www.examrace.com
www.examrace.com
2009 ~~~

0.9
Q.9 Pocket and
Pocket and Wing
Wing technique of pillar
technique of extraction is
pillar extraction relevant to
is relevant to

(A) room
(A) room and
and pillar method
pillar method (B) herd
(B) and pillar
bord and pillar method
method
(C) Wongawilli method
(C) Wongawilli method (D) shortwall
(D) shortwall method
method

QJO
Q.IO A non-electric
A non-electric detonating
detonating relay
relay does NOT contain
does NOT contain

(A) delay
(A) delay element
element (B) fuse
(B) fuse head
head
(C) metal
(C) metal sleeve
sleeve (D) neoprene
(D) neoprene connecting
connecting tube
tube

Q.11
Q 11 An iron
An iron ore
ore deposit
deposit has
has aa mean
mean grade
grade of
of 63%
63% Fe. During the
Fe. During the course
course of
of mining,
mining. 30% fines by
30% fines weight
by weight
are generated
are generated at
at aa grade
grade of 72% Fe
of 72% Fe which
which are
are rejected. The effective
rejected. The effective mean
mean grade
grade of
of the
the deposit
deposit in
in Fe
Fe
percentage is
percentage is

(A) 59.1
(A) 59.1 (B) 53.1
(B) 53.1 (C) 50.4
(C) 50.4 (D) 41.4
(D) 41.4

Q. 12
Q.12 Koepe system
Koepe system of
of winding
winding does
does NOT
NOT include
include

(A) tapper
(A) tapper guide
guide (B) limit
(B) switches
limit switches (C) safety
(C) safety hook
hook (D) brake
(D) brake

Q.l3
Q.13 A gas
A gas mask
mask does
does NOT include
NOT include

(A) check
(A) check valve
valve
warning device
(B) warmng
(C) face
(C) piece assembly
face piece assembly
(D) coolant
(D) coolant canister
canister

Q. 14
Q.l4 Resuing stoping
Resuing steping method
method is
is adopted
adopted when
when ore
ore body is
body is

(A) flat
(A) flat and
and thick
thick (B) very
(B) very steep
steep and
and thick
thick
(C) flat
(C) and thin
flat and thin (D) very
(D) very steep
steep and
and thin
thin

Q. l S
Q.l5 Moody diagram
Moody diagram represents
represents resistance
resistance coefficient in terms
coefficient in of
terms of

(A) Reynolds
(A) number and
Reynolds number and asperity
asperity ratio
ratio (B) viscosity and
(B) viscosity ratio
aspect ratio
and aspect
(C) surface
(C) surface tension
tension and viscosity
viscosity (D) Reynolds
(D) Reynolds number
number and
and surface
surface tens1on
tension

Q.16
Q. I6 An area
An area of
of 100 In2 is
100 m 2
measured on
is measured on aa plan
plan having
having aa R.
R. F.
F. of
of 11800.
1/800. If
If the R.F.
I the were to
RF. were to be 1/2000. the
be 1/2000, the
area in
area in 1112
2
would be
m would be

(A) 16
(A) 16 (3)40
(B) 40 250
(C) 250
(C) (D) 625
(D) 625

MN
Ml\ 3H6

www.examrace.com
www.examrace.com
2009 ~1N

Q]?
Q.17 As
As per the DGMS
per the DGMS norms,
norms, the severity index
the severity index is
is aa measure of
measure of

(A)
(A) fatality
fatality rate
rate (B) serious
(B) serious injury rate
injury rate
(C) number
(C) number of
of reportable
reportable injuries
injuries (D) accident
(D) accident proneness of mine
proneness of mine

Q.18
Q.l8 A
A balanced
balanced transportation
transportation problem is charactenzed
problem is characterized by
by

(A)
(A) total supply exceeds
total supply exceeds total
total demand
demand
(B)
(B) total
total demand exceeds total
demand exceeds supply
total supply
(C)
(C) total
total demand
demand is equal to
is equal to total
total supply
supply
(D) total
(D) supply is
total supply is either
either equal
equal to or more
ro or more than
than total demand
total demand

Q. I19
Q. 9 In
In the
the context
context of
of project
project management
management techniques.
techniques, the
the TRUE statement is
TRUE staLement ill

(A) CPM
(A) is stochastic
CPM is and PERT
stochastic and is deterministic
PERT is deterministic
(B) CPM
(B) CPM is
is deterministic and PERT
deterministic and PERT is stochastic
is stochasuc
(C)
(C) Both CPM and
Both CPM PERT are
and PERT deterministic
are deterministic
(D)
(D) Both CPM and
Both CPM and PERT
PERT are stochastic
are stochastic

Q20
Q.20 For mining property
For mining appraisals, typical
property appraisals, typical reports
repOJts prepared
prepared are
are Bankable Feasibility Report
Bankable Feasibility Report (BFR).
(BFR),
Conceptual
Conceptual Plan
Plan Report (CPR), Feasibility
Report (CPR), Feasibility Report (FR) and
Report (FR) and Detailed
Detailed Project
ProJect Report
Report (DPR). The
(DPR). The
chronological order for
chronological order the preparation
for the of these
preparation of these reports
reports is
is

(A)
(A) CPR—+FR—+BFR—rDPR
CPR--+FR--+BFR-+DPR (B)
(B) BFR—+CPR—+DPR—rFR
BFR--+CPR--+DPR-+FR
(C) FR—rBFR—dCPR—erDPR
(C) FR--+BFR--+C PR-+DPR (D) CPR--+BFR-+
(D) CPR—+BFR-fl DPR—+FR
DPR-+FR

Q. 21
Q. 21 to
to Q.
Q. 60 carry two
60 carry two marks each.
marks each.

Q21
Q.21 The
The mean of the
mean of the cubes
cubes of
of the
the first a natural
first 11 natural numbers
numbers is
is

(A) n(n+1) ”(3+ )(fl+ )


22
n(n + 1) B n(n + ll)(n 2
+ 2) 11n44 +1
+1 3

(A) (B)
( ) 8 (C)
(C)
44 3 nn 4

l 2
Q22
Q.22 The sum
The sum of
of the
the eigenvalues
eigenvalues of
of the
the matrix [I 0]
matrix [: ~
is
}s

(A) -3
(A) "3 (B) -1
(B) -1 (C)
(C) 11 (D) 33
(D)

MN
MN #16
4/16

www.examrace.com
www.examrace.com
2009 MN

Q .23
Q.23 . a ..
The value
The value of V'F of
of V·F vector F
of aa vector F = 4x
412i
2
=l ++ 3..\)'
31y2j+ 2
xyz‘ik
} + xyz ~katat the point ((1.
the point l. 2)
1. I. is
2) is

(A) 24
(A) 24 (B) 26
(B) 26 30
(C) 30
(C) (D) 32
(D) 32

Q24
Q.24 The function
The function f
f(x) =x
(x) = x30
3 - x)
(1- is integrated
x) is between 00 and
integrated between and II (both
(both inclusive)
inclusive) using
using closed form method
closed form method
1
and also
and also by
by Simpson's
Simpson's _!_rule. The difference
rule. The in the
difference in values obtained
the values from these
obtained from methods is
these methods is
a;
3

l
1 1l 1l
(M0
(A)O (B) — (C)
(C) —— (D)
( ——
' D) -
(B) 480
480 120
120 20
20

Q25
Q.25 Water starts
Water starts to flow into
to flow into aa sump
sump initially containing 250
initially containing 250 kL
RL of water. The
of water The inflow rate of
inflow rate of water is
water is
4t Umin
4t where tt refers
Umin where refers to elapsed in
time elapsed
to time min. If
in min. the pumping
11" the pumping rate of water
rate of water out
out of
of the
the sump 250
is 250
sump is
Umin. the
Urnin, the total volume of
total volume water in
of water the sump
in the sump after
after 3
3 hours
hours in It]. is
in kL is

(A) 250.5
(A) 250.5 (B) 255.6
(B) 255.6 (C) 269.8
(C) 2698 (D) 280.9
(D) 280.9

Q.26
Q.26 There are
There are 50
50 lemon
lemon trees in aa reclaimed
trees in mine area.
reclaimed mine area. Each
Each tree
tree produces
produces 800 lemons per
800 lemons year. For
per year. For each
each
additional tree
additional tree planted in this
planted in this area.
area, considering all trees,
considering all trees, the
the output number of
output number of fruits
fruits per tree drops
per tree drOps by
by
10 lemons
10 lemons in in aa year.
year. The
The number
number of of trees
trees that to be
that to added to
be added to the existing reclaimed
the existing reclaimed area
area in
in order
order to
to
maximize the
maximize the total number of
total number of lemons
lemons in the year
in the year is
is

(A) 10
(A) 10 ((B) 15
B) 15 (C) 16
(C) 16 (D) 26
(D) 26

Q27
Q.27 The grain
The grain density
density and
and bulk density of
bulk density of aa dry
dry coarse
coarse grained sandstone rock
grained sandstone rock sample
sample are 3.0 grn/cc
are 3.0 gm/cc and
and
2.7 grnlcc
2.7 gm/cc respectively.
respectively. The
The void
void ratio
ratio of
of the sample in
the sample in percentage is
percentage is

(A) 8.4
(A) 8.4 (B) 10.0
(B) 10.0 (C) 11.1
(C) 11.1 30.5
(D) 30.5
(D)

Q28
Q .28 The ratio
The ratio of compressive strength
uniaxial compressive
of uniaxial to uniaxial
strength to uniaxial tensile
tensile strength
strength of
of aa sandstone
sandstone specimen
specimen is
is 8:1.
8:1.
The theoretical
The value of
theoretical value of angle
angle of internal friction
of internal of the
friction of the specimen
specimen inin degree
degree is
is

(A) 51
(A) 51 (B) 41
(B) 41 32
(C) 32
(C) (D) 7
(D) 7

Q29
Q.29 A circular
A circular tunnel is made
tunnel is made underground
underground where where far field vertical
far field vertical and horizontal stresses
and horizontal stresses are Pu0 and
are P KPD0
and KP
00 ) at the boundary of the tunnel for B = 45° from the horizontal
respectively. The
respectively. The tangential
tangential stress
stress (a
(11:11.5.fl ) at the boundary of the LUnnel for 8 = 45° from the horizontal

plane is 3P
plane is 3P0. The value
0 . The of K
value of is
K is

(M00
(A) (BH1
(B) (C) 22
(C) (D) 33
(D)

MN
MN 5m
S/16

www.examrace.com
www.examrace.com
2009
2009
MN
Mlli

Q.30
Q.30 The bending
The bending moment
moment diagram
diagram for
for the shaft shown
the shaft shown below
below resembles which one
resembles which one of
of the following graphs
the following graphs ?‘3
5 kn
5kN

J£-·*-·n"
I• 2m ~· 2m •I

M (kN.m)
M (Wm) M (kNm)
M(kN.m)

5 55

(A)
(A) 0o (B)
(B) o 44
2 2
Length (( m)
Length m) Length (( rn)
Length m)
-5
-5 -5

M (kN.m)
M (kNm)

1” '
(kNm)
M (kN.m)
10
M

55 --n-
(C) 55 —
(C) (D)
(D)

0 .
O 22 4
4
22 4
Length ({ m)
Length m} Length (( rn)
Length m)

Q.31
Q.31 A
A mining
mining equipment
equipment hashas aa life
life of
of 55 years
years with
with no salvage value.
no salvage value. Assuming
Assuming that
that the
the depreciation of the
depreciation of the
equipment is
equipment is calculated
calculated byby thethe straight
straight line
line method,
method. the average annual
the average value of
annual value of the equipment in
the equipment in
percentage
percentage of
of its
its original
original value
value is is
(A) 20
(A) 20 (B) 40
(B) 40 (C)
(C) 50
50 (D) 60
(D) 60

0.32
Q.32 Air flows
Air flows at
at 22 m
m3ls
3
/s through
through aa forcing
forcing fan
fan duct
duct of
of 0.3
0.3 m2
m2 having
having uniform
uniform cross-section. The duct
cross-sectio n. The duct
resistance is
resistance 40 sm'gi
is 40 2
Ns m- and
8
air density
and air density is
is 1.2 kg/m3.. The
1.2 kglm 3
The total
total pressure generated by
pressure generated by the
the fan
fan in
in Pa is
Pais
(A) 186.7
(A) 1867 (B) 160.0
(B) 160.0 (C) 133.3
(C) 133.3 (D) 26.7
(D) 26.7

0.33
Q.33 Match the
Match the following
following in
in the
the context of Indian
context of Indian mining
mining practice
practice ::

Equipment
Equipment Power source
Power source

P. Rocker shovel
P. Rocker shovel 1.
l. Battery
Battery
Q. Locomotive
Q. Locomotive 2.
2. Compressed air
Compressed air
R.
R. Shearer
Shearer 3.
3. Electricity
Electricity (maximum
(maximum voltage
voltage 6.6 kV AC)
6.6 kV AC)
S.
S. Dragline
Dragline (24
(24mm33 bucket capacity)
bucket capacity) 4.
4. _ Electricity
Electricity (maximum
(maximum voltage 1.1 kV
voltage 1.1 kV AC)
AC)
(A) P—l, Q-2,
(A) P-1, Q—2, R-3. 8-4
R-3, S-4 (B) P-2. Q—l.
(B) P-2, R-4. S-3
Q-1, R-4, S—3
(C) P-2, Q—l,
(C) P-2, R-3. S-4
Q-1, R-3, 8-4 (0) P-l, Q-3,
(D) P-1, R-2, S-4
Q-3, R-2, S-4

MN ‘
!\1N
one
6/16

www.examrace.com
www.examrace.com
2009
2009 MN
MN

Q34
Q.34 The planes
The H and
planes H and V
V represent
represent the
the horizontal
horizontal and vertical planes
and vertical planes respectively
respectively as shown in
as shown in the
the figure.
figure.
Which
Which one of the
one of the following Mohr circles
following Mohr circles represents the stress
represents the conditions applied
stress conditions applied in planes H
in planes H and V ?‘?
and V

8 All
All stresses in MPa
are in
stresses are MPa

1-r and
and on refer shear
On refer shear stress
stress and
and
normal stress respectively
normal stress respectively
4
Note: shear stress
Note: shear stress is positive itif itit tries
is positive tries
to rotate
to rotate the element in
the element in clockwise
clockwise
direction
direction

T T

't 't

Q35
Q.35 Two splits A
Two splits and B
A and B are
are ventilated
ventilated from
from an intake airway.
an intake Resistances of
airway. Resistances of the splits are
the splits 0.5 sm"8
are 0.5 and
Ns2 m-8 and
0.8 Ns
0.8 sm'8
2
m-8 respectively.
respectively. A regulator is
A regulator is placed
placed in
in split
split B to maintain
B to maintain aa flow
flow ofof 15
IS m3/s and 10
m3/s and trials
I0 m 3
/s in
in
splits A
splits A and
and BB respectively, as shown
respectively, as shown in
in the figure. The
the figure. The size
size of
of the
the regulator
regulator in
in m2 is
m2 is
Split-A
Split-A

- -
(A) 2.10
(A) 2.10 (B) 1.30
(B) 1.30 (C) 1.20
(C) 1.20 (D) 1.13
(D) 1.13

Q.36
Q.36 The concentration
The of OH‘
concentration of Oir ion in aa mine
ion in mine water
water sample
sample is
is 110'll mol/L. The
o-tt mol/L. The pH of the
pH of sample is
the sample is
(A) 2
(A)2 (B) 33
(B) (C)
(C) 4
4 (D) 11
(D) 11

Q37
Q.37 A mine
A mine having
having aa reserve
reserve of 320 Mt
of 320 Mt produces
produces 44 Mt of ore
Mt of ore at
at the end of
the end of 1st
1“ year. If the
year. If the mine increases
mine increases
production by
production by 10%
10% every year, the
every year, the percentage of the reserve
percentage of the reserve that still remains
that still at the
remains at the end of 215‘1 year
end of21' year is
is
(A) 50
(A) 50 (B) 35
(B) 35 (C) 25
(C) 25 (D) 20
(D) 20
MN 7116
7/16

www.examrace.com
www.examrace.com
2009 MN

Q38
Q.38 Match the
Match the following
following ::

Type of
Type deposit
of deposit Ore, rock
Ore, strength
rock strength Mining method
Mining method

P. Flat,
P. Flat. thin
thin 1. Strong. strong
1. Strong, strong aa. Sublevel stoping
Sublevel stoping
Q. Massive
Q. Massive 2. Weak.
2. Weak. weak
weak b. Room and
b. Room and pillar
pillar
R. Steep,
R. Steep. thick
thick c. Block
c. caving
Block caving

(A) P-1-c,
(A) P-l-c. Q-l-a.
Q-1-a, R-2-b
R-2-b (B) P-1-b,
(B) P—l-b. Q-2-c.
Q-2-c, R-l-a
R-1-a
(C) P-Z-b.
(C) i—a. R-1-c
P-2-b, Q-1-a, R—l-C (D) P—l-c.
(D) Q-l-b. R-2-a
P-1-c, Q-1-b, Rt2—a

0.39
Q.39 Match the
Match the following
following ::

Stoping method
Stoping method Advance of
Advance of stoping
stoping face
face

P.
P. Shrinkage stoping
Shrinkage stoping 1.
1. Sideward vertical
Sideward vertical slices
slices
Q.
Q. Rill stoping
Rill stoping 2.
2. Upward horizontal slices
Upward horizontal slices
R.
R. Blasthole stoping
Blasthole stoping 3.
3. Downward horizontal slices
Downward horizontal slices
s.
S. Top slicing
Top slicing 4.
4. Sideward
Sideward inclined
inclined slices
slices
(A) 11.3,
(A) P-3, Q~l. R-4. S-2
Q-1, R-4, S-2 (B) 13.2.
(B) P-2, Q-3. R-l. 84
Q-3, R-1, S-4
(C) P-Z. Q—4. R-1,
(C) P-2, Q-4, R-1.S-3
S-3 (o) P—4.
(D) Q-3, 12-2,
P-4. Q-3, R-2, 8-]
S-1

Q.40
Q.40 Which one of
Which one of the following graphs
the following graphs typically
typically represents the standard
represents the strain-time creep
standard strain-time creep behaviour
behaviour of an
of an
isotropic
isotropic rock
rock material
material under constant temperature
under constant temperature?? P.
P. Sand
S and TT in the figures
in the figures refer
refer to
to primary creep.
primary creep.
secondary creep
secondary and tertiary
creep and tertiary creep respectively.
creep respectively.

ALE

c
/ .5. 5 i
'(ij
.... U) : .
(i5

P'
Pl ss T P :.
p sS : T
T
'
T
'
'' .. _ s
Time
Time T'me
Time

(Graph 1)I)
(Graph (GmPh 1”
(Graph II)

it E 4k

c c

-cfi
'(ij
....
( /) g '
''
(i5
·~
g i
P?
pl''
ss T
T P;
p ss \‘\T
'
' —r I
Time
Time Time
Time
(Graph Ill)
(Graph III) (Graph IV)
(Graph W)

(A) Graph
(A) GraphlI (B) Graph
(B) 11
Graph II (C)
(C) Graph 11]
Graph 1II (D) Graph 1V
(D) Graph IV

MN
MN $16
8116

www.examrace.com
www.examrace.com
2009 ~IN

Q41
Q.41 The following
The data represent
following data the number
represent the of workers
number of workers suffering
suffering from
from pneumokoniosis
pneumokoniosis in coal mines.
10 coal
in 10 mines.

Mine
Mine I1 II
II III
III IV
IV vV VI
VI VII
VII VIII
VIII IX
IX X
X
Number
Number 10
10 16
16 14
14 15'
15 14
14 12
12 17
17 13
13 15
15 12
12

The number
The of mines
number of mines falling
falling above
above the 50“1
the 50 1
h percentile
percentile in
in terms of the
terms of number of
the number workers suffering
of workers suffering from
from
pneumokoniosis is
pneumokoniosis is

(A) 22
(A) (B) 33
(B) (C) 44
(C) (D) 55
(D)

Q42
Q.42 Cause-wise data
Cause-wise of injuries
data of in an
injuries in an underground coal mine
underground coal for aa five-year
mine for period is
five-year period given below:
is given below:

Cause of injury
Cause of injury Number
Number of
of injuries
injuries

Fall of
Fall of roof
roof 27
27
Fall of
Fall person
of person 22
22

Rope
Rope haulage
haulage l7
17

Explosives
Explosives 55
Other causes
Other causes 44

The cumulative
The probability of
cumulative probability injury due
of injury to fall
due to fall of
of roof
roof and
and fall of person
fall of person is
is

(A) 0.65
(A) 0.65 (B) 0.50
(B) 0.50 0.36
(C) 0.36
(C) (D) 0.29
(D) 0.29

Q43
Q.43 Consider the following
Consider the linear programming
following linear programming problem:
problem:
hdaxinuze
Maximize

zz=3x+2y
=3x+ 2y
Subject to
Subject to

3x+2y215
3x+2y ~ 15

2x+3y$6
2x+3y ~6

xZQyZO
x~O,y~O

The above
The above linear
linear programming
programming problem has
problem has

(A) unique
(A) unique optimal solution
optimal solution (B) multiple
(B) multiple optimal solutions
optimal solutions

(C) unbounded
(C) solution
unbounded solution (D) infeasible
(D) solution
infeasible solution

MN 9116
9/16

www.examrace.com
www.examrace.com
2009
2009 MN
~~~

Q44
Q.44 A mine
A mine workshop
workshop hashas 44 lathe
lathe machines and 4
machines and 4 tasks
tasks for completion. Each
for completion. Each of
of the
the machines
machines can
can perform
perform
each of
each of the
the 44 tasks.
tasks. Each
Each task
task can
can be assigned to
be assigned to one
one and
and only
only one
one machlne.
machine. Estimated
Estimated cost in Ru!Jee'
cost 111 Rupees
to complete
to complete each
each task
task is
is given in the
given in the matrix
matrix below.
below.

Machine
Machine
M1
M1 M2
M2 M3
M3 M4
M4
T1
Tl 61
61 92
92 52
52 72
72
Task
Task T2
T2 42
42 49
49 69
69 85
85
T3
T3 47
47 59
59 80
80 71
71
T4
T4 65
65 70
70 68
68 72
72
The total
The total optimum
cptimum cost in Rupees
cost in Rupees for assigning the
for assigning the tasks to the
tasks to the machines
machines is
is

(A) 210
(A)210 (B) 215
(B) 215 (C)
(C) 220
220 (D) 286
(D) 286

Q45
Q.45 A 1100
A 1100 V,
V, 3CD
3<1> power supply system
power supply system of
of aa mine
mine draws
draws aa load
load of 185 kW.
of 185 kW. The
The ammeter
ammeter reading
reading shows
c,hows
115 A.
115 A. The
The power
power factor
factor of
of the
the system is
system is
(A) 0.84
(A) 0.84 (B) 0.73
(B) 0.73 (C) 0.64
(C) 0.64 (D) 00.48
(D) 48

Q46
Q.46 Two
Two belt conveyors load
belt conveyors load aa ground
ground bunker. each at
bunker, each at aa rate of 400
rate of tph, which
400 tph. which is initially filled
filled with
' is initially w1th
10000 t of coal.
10000 t of coal. Coal is discharged
Coal is discharged from
from the
the bottom of the
bottom of the ground
ground bunker onto aa belt
bunker onto conveyor at
belt conveyor at aa rate
rate
of 1200
of 1200 tph.
tph. The
The time elapsed in
time elapsed in hours before the
hours before the bottom conveyor starts
bottom conveyor starts to
to operate below 1h
operate below its rated
1ared
capacity
capacity is
is
(A) 6.5
(A) 6.5 (B) 8.5
(B) 8.5 (C) 12.5
(C) 12.5 (D) 25.0
(D) 25.0

Q47
Q.47 The
The cash flow table
cash flow table of
of aa manganese
manganese mine
mine for
for aa particular
particular year
year is shown below:
is shown below:

Amount (Rs. in
Amount (Rs. lakhs)
in lakhs)
Item
Item

Revenue
Revenue 900
900
Cost
Cost (other than depreciation)
(other than depreciation) 300
300
Depreciation
Depreciation l 00
100
Profit
Profit before tax
before tax 500
500

If the
If corporate tax
the corporate tax is
is 50%
50% of
of the
the Profit
Profit before
before tax.
tax, the operating cash
the operating cash inflow
inflow in
in lakhs
lakhs of
of Rupees
Rupees is
1'
(A)
(A) 400
400 (B) 350
(B) 350 (C) 250
(C) 250 (D) 200
(D) 200

Q.48
Q.48 In an
In an area
area within
within aa surface
surface mine. under static
mine, under static condition
condition the
the following
following gases
gases are found: l'J0
are found. N02.2• C01.
co~. 0;o,
and 80;.
and Assuming no
S02. Assuming no diffusion,
diffusion, reaction and bonding
reaction and bonding of the gases.
of the gases, the
the concentration of the
concentration of the ga~e..,
gases l'rum
from
bottom upwards
bottom will be
upwards will be in
in the
the order of
order of

(A) N02, C02, 03 and


Mid 502
so2
(B) SO; N02,
(B) so2. N02. C02
C02 and
and 03
03
30:
(C) S02, O3_
03, N02 311d
and C02
(D) N02, C02,
C02. SO;
so2 and 03

MN
MN 1M6
10/16

www.examrace.com
www.examrace.com
2009
2009 MN
MN

Q49
Q.49 In
In aa mine site. the
mine site, cost of
the cost of shaft
shaft sinking in lakhs
sinking in lakhs of
of Rupees
Rupees is given as
is given as 2.64D
2.64D + + 34.8. where Dis
34.8, where D is the
the shaft
shaft
depth in
depth in m.
m. In In the same site,
the same site. the corresponding cost
the corresponding cost of
of driving
driving an
an incline
incline is 0.96L, where
is 0.96L, where L L is
is the
the
length
length ofof the
the incline
incline in
in m.
m. Assuming
Assuming L L by D ratio
by D ratio is
is 3.0. the depth
3.0, the depth in
in m
to beyond which the
beyond which the shaft sinking
shaft sinking
becomes more economical
becomes more economical is
is
(A) 43
(A) 43 (B) 48
(B) 48 (C) 145
(C) 145 (D) 155
(D) 155

Q50
Q.50 Match the
Match the following
following ::

Seam char
Seam characteristics
acteristics Coal mining method
Coal mining method

P.
P. 12 m
12 m thick flat seam
thick flat scam I1. Mechanized
Mechanized longwall
longwal1
Q.
Q. 77 m
m thick
thick seam
seam at
at 65°
65° inclination
inclination 2.
2. Descending shield
Descending shield
R.
R. 33 m
m thick
thick flat
flat seam
seam 3
3. Mechanized integral
Mechanized integral caving
caving
S.
S. 77 m
m thick
thick seam
seam at 25° inclination
at 25° inclination 4.
4. Jankowice
Jankowice

(A) P-4,
(A) P-4, Q-3,
Q-3, R-2,
R-2. S-1
8-1 (B) P-3,
(B) P—3. Q-4,
Q-4. R-1,
R-l. S-2
S-2
(C) P-2,
(C) P—2, Q-3,
Q-3, R-4,
R-4. S-1
S-l (D) P-3,
(D) P-3. Q—2, R-l, S-4
Q-2, R-1, 34

Common Data
Common Data Questions
Questions

Common Data for


Common Data for Questions
Questions 51
51 and 52:
and 52:
Workmen arrive
Workmen arrive at
at aa mine
mine workshop
workshop to to receive
receive tools
tools for maintenance. The
for maintenance. The inter-arrival time of
mter-arrival time workmen at
of workmen at
the service counter
the service counter is exponentially distributed
is exponentially distributed with
with an average time
an average of I10
time of 0 min.
min. The service time
The service time at
at the counter
the counter
is also
is also distributed
distributed exponentially
exponentially with
with aa mean
mean time of 6
time of 6 min.
min.
0.51
Q.51 Probability that
Probability that there
there is
is aa queue
queue (more
(more than
than one
one workman) at the
workman) at the service counter is
sen ice counter is
(A)
(A) 0.24
0.24 (B) 0.36
(B) 0.36 (C) 0.40
(C) 0.40 (D) 0.60
(D) 0.60

Q.52
Q.S2 Average time
Average spent by
time spent by aa workman waiting for
workman waiting for his
his turn
tum to
to be served in
be sened in min is
min is
(A) 99
(A) (B) 12
(B) 12 (C) 15
(C) 15 (D) 18
(D) 18

Common
Common Data
Data for
for Questions 53 and
Questions 53 54:
and 54:
A tacheometer
A tacheometer is set up
is set at aa station
up at station 'B'.
‘B’. The
The RL
RL of
of the
the station
station B
B is
is 150
150 m
m above the MSL.
above the MSL. By
By holding
holding aa staff
staff
vertically at aa station
vertically at station 'A',
‘A‘, the
the following
following readings
readings are
are taken:
taken:

Vertical
readings (m)
(In)
gale
Staff readings
angle
Lower Middle Upper
26°36.
Lower Middle Upper
26°36'
(130
0.80 3.03
3.08 5.36
5.36

The multiplying
The factor and
multiplying factor and additive
additive constant
constant of
of the
the instrument are 100
instrument are 100 and
and 1.9
1.9 m
m respectively.
respectively.
Q.53
Q.53 The horizontal distance
The horizontal between the
distance between stations A
the stations A and B in
and B in m
m is
is
(A) 364.6
(A) 364.6 (B) 366.3
(B) 366.3 (C) 409.4
(C) 409.4 (D)
(D) 457.6
457.6

Q54
Q.54 If the
If the height of the
height of instrument is
the instrument is 1.2
1.2 m.
m, the RL of
the RL of the station 'A'
the station ‘A‘ above
above the MSL in
the MSL in m is
m is
(A) 337.6
(A) 337.6 (B) 334.5
(B) 334.5 (C) 331.5
(C) 331.5 (D) 330.3
(D) 330.3
MN
MN llflfi
11/16

www.examrace.com
www.examrace.com
2009
2009 MN
MN

Common Data
Common Data for
for Questions 55 and
Questions 55 56:
and 56:
A
A turbine
turbine pump
pump of
of efficiency 70% discharges
efficiency 70% discharges water at the
water at the rate
rate of
of 2100
2100 Umin
Umin at
at aa total
total head of 100m.
head of 100 m.

Q55
Q.55 If the
If the pump
pump is
is run
run by
by aa motor of efficiency
motor of efficiency 90%, the input
90%, the input power required for
power required for the
the motor
motor in
in kW
kW is
is
(A) 22.49
(A) 22.49 (B) 34.31
(B) 34.31 (C) 44.11
(C) 44.11 (D)
(D) 54.50
54.50

Q56
Q.56 1f the
If velocity of
the velocity water in
of water in suction and delivery
suction and pipes of
delivery pipes of the
the pump
pump are 1.8 m/s
are 1.8 and 2.5
rnls and 2.5 mls
rnls
respectively,
respectively, the diameter of
the diameter of suction
suction and
and delivery
delivery pipes
pipes in cm are
in em are

(A) 15.73
(A) 15.73 and
and 13.35
13.35 (B) 7.86
(B) 7.86 and
and 6.67
6.67 (C) 5.78
(C) 5.78 and
and 6.02
6.02 (D)
(D) 4.97 and 4.22
4.97 and 4.22

Linked Answer
Linked Questions
Answer Questions

Statement for
Statement Linked Answer
for Linked Answer Questions
Questions 57 and 58:
57 and 58:
A
A fan running at
fan runnmg at aa speed
speed of280
of 280 rpm circulates 105
rpm circulates 105 m3/s of air
m3/s of air in
in aa mine.
mine.
Q.57
Q.57 If the
If the power
power input
input to the motor
to the motor for
for driving
driving the fan is
the fan recorded to
is recorded be 75
to be 75 kW,
kW, with
with the
the combined
combined
efficiency of
efficiency of fan
fan and
and motor
motor at 70%. the
at 70%, fan pressure
the fan pressure in
in Pa
Pa is
is
(A)
(A) 50
50 (B) 350
(B) 350 (C)
(C) 500
500 (D)
(D) 650
650

Q58
Q.58 If the
If the fan
fan pressure
pressure is
is to
to be increased by
be increased 200 Pa
by 200 Pa by
by changing
changing the
the fan Speed, the
fan speed, fan speed
the fan speed in rpm will
in rpm will
become
become

(A) 768
(A) 768 (B) 549
(B) 549 (C) 392
(C) 392 (D) 332
(D) 332

Statement for
Statement for Linked
Linked Answer Questions 59
Answer Questions 59 and
and 60:
60:
A surface
A surface mine
mine blast
blast design
design has
has 99 holes
holes in
in aa row,
row, each
each of
of 88 m
m length
length and
and 200 mm diameter.
200 mm diameter. The
The spacing
spacing and
and
burden are
burden are 66 m
m and
and 55 m
m respectively.
respectively. The
The length
length of
of subgrade
subgrade drilling
drilling is
is 11 m
m and
and the
the density
density of
of in-situ
in-situ rock
rock is
is
2.43 t/m3.
2.43 t/m3.

0.59
Q.59 Assuming no
Assuming no back
back break,
break, the
the output
output per blast in
per blast t is
intis
(A) 4593
(A) 4593 (B) 5905
(B) 5905 (C) 6124
(C) 6124 (D) 6299
(D) 6299

Q60
Q.60 Considering an
Considering an explosive
explosive density
density of 0.9 t/m3
of 0.9 t/m3 and stemming length
and stemming length of 2 m. the
of 2m, powder factor
the powder from the
factor from the
blast in
blast t/kg is
in tlkg is

(A) 4.12
(A) 4.12 (B)
(B) 4.00
4.00 (C) 3.86
(C) 3.86 (D)
(D) 3.01
3.01

END OF
END OF THE QUESTION
THE QUESTIO N PAPER
PAPER

MN
MN 12116
12/16

www.examrace.com
www.examrace.com
...
auto
MN :: MINING
MN MINING ENGINEERING
ENGINEERING aA
“fl

Duration: Three Hunt's Mtaimum dittt‘h:


dfitt'mttm [III
Morb: 100

Jlad the
and doe Following lwei well aat earei'lllr.
l'eJknrt.c hetrueflols cuefally.

1.
I. 'l'hisqueetioap-eroontainslipagesineludinghlsnkpagfiforronghworki'leaseeheokailpages
This question .-p« contaias " pages including blanl pages for rough W<ll"k. Please check. all pages
andreportdiserepaneyji‘any.
and report discrcpulcy, if any.

Write your
2. Write registration manber,
your regifllrl.tion your name
number, your name of
name and name of the examination centre
the examination at the
sentre at speeified
the specified
locations on
locations on the
the right
right halfoftbe
half of the Optical
Optical Response Sheet {DES}.
fleaponse Sheet(ORS).

Using liB
3. Using HE pencil,
peneiL darken
darken the
the appropriate bubble under
swept-late bubble \IDder each digit of
each digit registrlltion number
yoor registration
of your and the
number and the
letters cmespsttaiing
letter~; correeponding to
to you
your paper
paper eode.
code.

4. Allquestionsinthieplperateofobjeetivetype.
All questions in this~ an: of objective type.

S. meanaweredonfisefllls by dukeniog the appropriak bubble (lllill'ked A. B, C, B}


Questioos lllllll be aoswen:d oo the ORS bydarkeuiugtheapproptietebuhbletnmrkedAE,E. D)
using HE
using pencil against
HB peneil nwnber on
question number
the question
againat the on the hand side
left hand
the left of the
side of ORS. For
the ORE. eKh question
For eaeh qae~tloll
darken
darba the
tbe bubble
babble oi of the
the torrent
eorred •-er. In nose
answer. In you wish
case you wish to change an
to change an5wer, erase
an answer. tbe old
erase the old

--
answer oempleteljr. More
answer completely.
response.

6. Therearea
More

There an:: a totslol'tifi


than
lhan one
one answer
answer bubbled
bubbled

carrying lDDntarks.
questions EElTj'iflg
IOU.] of 6~ questin-
against
against

I 00 marlal.
a
a question
question will
will be
be treated
treated as
as an
an ineorreet
inc(II'Te(:l

W Q.\
7. Questions [1.1—- Q.2.S earn; Il-niark
will carry
0.25 will each, andqiiCSiioos
-mark each. {1.26 — Q5S
andquestioue Q.26- 0.55 will can}! 2-mad<s
will carry each.
E-nearlts 1:8ch.

Mom Q.4S-
8. Questioos 0.43 — Q.SI
$.51 (2{2 pairs) oonntton data qumions
are C01D111011
pairs} an:: questi01111 and quantion pairs
and question {0.51, Q.53)
paifs (Q.Sl, and
0.53] and
{1.154. 0.55] are linked
(Q.54, Q.SS) an:: Hnn.:t anew:
answer questions.
questi0011. The
The answer
amwer to
to the
the seeond
8CWIId question
question of
of the
the linked
linked answer
answer
depends on
questions depeods
questions on the
the answer
IUltwe:r 1o the first
to the question of
fiqt question pair. if
the pflr.
of the the first
If the in the
question in
lirst question linked pair
the linked pair
isarronglysnsweredor
ill un~ then the answertotheseoond
is un-atterrqited,thenthe
wrongly lUlllwered or is answer to the aecond question
question inthepairwillnot
in the pair will not be
be
evaluated.
evaluated

Quc:sti0011 (lid
9. Questions Q.56- — 0.65 beloug to
Q.6S belong General Aptiutde
to General Questions {3.5-5
(GA). More
Aptitude {GA}. Q.S6- - Qfll
Q.60 will cany t-ntark
will £13.1i 1-marl.
-.:b, and
web. Q.61 —
questi0011 (Mil
aod questions Q.6S will
- 0.65 cany Ltnarks
will can-3r The Grill.
each. The
2-marlcs each questiOil!l will
GA questions on aa fresh
begin on
will begin msh
page starting
nag starting from
from page
page 11.
11.

10. Lin-attempted
ll]. will eatryaeromarks.
questions will
Ull-"u. ••q tW questions cany zeromaJb.

1]. Wrong
II. answers will
Wrong Bll!wer!i NEGATIVE marks.
learnt:r NEGATIVE
will carry marks. For 0.] -- Q.2S
For Q.l {3.515 -— Q.60,
and Q.56
{1.25 and {160. ~ mark will
ii marl. be
will be
deducted for each
deducted for wrong answer. For 0.26 QSI
eaeh wrong answer. For Q.26- - Q.SI and
and {1.61
Q.61 —
- 0.65.
Q.6S, ii
t; mark
marl. will
will be
be deducted
deducted for
for eseh
each
wrong answer.
wroog answer. The
The que!tion
question pairs {0.51, Q.SJ),
pairs (Q.S2, (1.5 5} are
{112.54. Q.55)
and (Q.S4,
[3.53]. and questions with
are qucsti01111 with linked answers.
linked answers.
There will be
There will negative marks
be negative forwtonganswertothe
only.f for
marks only fn'st questionoi'the
wrong answer to the firSt linked snswerquestion
question of the linked answer question
pair i.e. for
pair i.e. for Q.S2 0.54, ii
and Q.S4,
{3.52 and will be
mart. will
tJ mark deducted for
be deducted each wrong
for each There is
answer. There
wrong answer. oo negative
is no negative
...-king for 0.53 and 0.55.
marking forQ.SJ andQ.SS.

I2. Baleadator
12. data oonneetiirity]
(without data
Calculator [widioatt allowed in
is allowed
OO!'!Metivity) is eumjnatloo hall.
the examination
in the hall.

13. Charts,
13. Charts, graph
graph sheets
sheets or tables are
()!"tabla are NUT allowed in
NOT allowed in the examination hall.
the examination hall.

14. Rough
14. Roogh work
wotk. ean
ca11 be
be done
doPe on the question
on the ilaelf. Additionally.
paper itself.
questioo paper blank pages
Additirumlly, blank are provided
page~ are at the
provided at the
end ofthe
end of the question paper for
qi!Wion paper rough writ.
for mug: work.

I.I'II|'.'I-
1116

www.examrace.com
\MNw.examrace.com
1a
2010

{1.1 -— Q.25
Q.l 0.15 carry
earrg,Ir one mark eaeb.
one mark each.
"'
on
Q.l Ascensionally ventilated coeJ.
J'lesett't:laionall3:r ventilated coal mine
mine inclines
inclines ideally should have
ideally should have higher methane layering
higher methane number
layering nUJDbe!'
when compared
when compared to to descensiona.lly
deseensionallv ventilated
ventilated inclines.
inclines. The
The reason is
reason is
{A} in
(A) in IISCelt!iionally
aaeeosionall},r ventilated
ventilated incline
incline density
densityr ofair
of air is higher
higher
{El-1| ascennionaJ[y
(B) ascenflionallv ventilated
ventilated incline creates conditions
incline creates C<ltlditious for W
improved turbtflent
turbulent nIiiting
mixing ofofmethane
methane
,.,..
latter
{C} methane
(C) methane drainage
drainage is
is not
not practiced in aseensionally
practiced in aseensionallyr ventilated
ventilated incline
inl;line
[D] descensionally
descensionallgllr ventilated
ventilated incline
incline crealcll
creates conditions
conditions for improved rwbulent mining
improved turbulent mixing of methane
(D)
,.,..
lever
on
Q.2 A coolant
A coolant is
is aa desirable
desirable cocaine-ant
~ in in the
the deaign
design of
of aa Sclf-Comained
Self-Contained Breathing
Breathing Apparanis since
Appanllus siD;c
[A] SUITOIIJlding:s
(A) sanrotnidings can
can be
be hot and humid
bot and humid dwingrescue
dto'ing rescue
{B} aa rescue
{B) rescue worker generates large amount
worker generate5 amount of
of metabolic
metabolic heat
{C} exhaled
{C) exhaled air
air C02
CD: abso!plion
absorption is
is an exothermic reaction
an exothermic reaetion
{El} exhaled
{D) exhaled air
air water
water vapour
vapour ha.s
has to
to he condensed
be condensed

on
Q.3 Determine the
Determine the correctness
correctness or
or otherwise
otherwise of
of the following Assertion [a[
the followingAs&ertlon [a] and
and the
lhe Reason [r]
Reuon [r]

Assertion :: Both
ADertioa Both intBke
intake and
and return
rattan side
side stopping:s
stoppings must
must he
be closed Slmmij' in
closed simultanoously the event
in the event of
of
sealing off
sealing off aa COB!
coal mine
mine panel with explosion
panel wilh explosion baard
hazard following
following aa fare.
fire.

Reason :: By
Reaon By continUOU!I\y
continuous]:-r ventilating
ventilating lhc
the area
area till shtarltaneous cl-oattre
till simultaneous closure ofof the stoppings. the
the stoppings, the
possibility of an explosion
possibility ohn explosion hazard
hazard due
due to
to gas
gas build-up
build-up isis avoided.
avoided.
{A} [a]
(A) [a] is
is true
true but [r] is
but [r] is false
False
{B} Both
(B) Both [a]
[a] and
and [r]
[r] arc
are uue
true and
and [[r] is the
r] is tlte correct
COJTOCt reason
tc8SOD for [a]
for [a]
[{3} Both
(C) Both [a]
[a]a.nd [r] arc
and [r] aretnieand
true and [[r] is not
r] is not thccorrrectreason
the COJTOCI reason for [a]
{D} Bolh{a)and[r]arcfalsc
{D) Both [a]and[r]arefalse

on
Q.4 ln as Cartesian
In Carteaiaa coordinate
coordinate sys1e01.
system the
the vcrtiCCI
vertiees of
of aa trtangular'
triangular plate are given
plate arc Irv {-2,
given by {$1, 1}, [3, 4},
l), (3, 4), and
and
(-4, —3}. The
[—4, -8). The coordinates
coordinates of
of the
the centre
centre of gtavitjr of the plate are
of gravity oflhe p\ale arc
{A} 3,
(A) 3, 4
4 {317,12
(B) 7, 12 {C} -1,-l
(C) —l, —l {D} —3.-4
{D) -3,-4
on
Q.5 An air quality
An air quality parameter required to
parameter required to he monitored under
be monitored under the
the Indian
Indian National
Natiooal Ambient
Ambient Air
Air QualityI
Quality
is
Standards is
Stllndards
[Alias
(A) As {B} Pb
(B) Pb {C} Hg
(C) Hg {D} Silica
(D) Silica
QE-
Q.6 mummemmnnnafiesexposedmofcmhcwppwmdhyappmmme
In an underground coeJ. mine., a freshly e1r.posed roof can be supported by a lcn4KJimy support in the
form of
fonnof
(A)

--
{A} triangular
triangular chocks

[Bl Werner
(B)

{Cl safari
(C) flflfidwrtfl
checks

supports
{D} hydraulic
(D) hydraulic props
props

~~ ,,.

www.examrace.com
\MNw.examrace.com
eats
21110 _ _ HE
MN

{2.7
Q.7 At aa surface
At surface mine
mine office
office the
the independent
independent Sound Pressure Levels
Sound Pressure {EFL} measured
Levels (SPL) measured in
in dEliA}
dB(A) onon
account of
accowrt of 3
3 drill
drill machines
machines are
are 85,
35, 88
BE and
and 85.
35. if
If all machines work
the three machines
all the simultaneously, the
work simultaneously, the
combined SPL,
combined EFL, in
in dB(A),
dflifii}, is
is
{A} 91
(A)91 [B191]
(B)90 92
{C} 92
(C) [D] 94
(D) 9‘4
QB
Q.8 Thehcksiglstreidiugnnahurhnmhoffllmflmis1.45mlftlteinversestafi‘readingona
The backsigbt reading on a beo<:h IIINk of RL 100.0 m is 1.45 m. If the inverse staff reading on a
foresightislflnatheflofflsestafl'stafiooinmis
foresight is 2.23 m, the RL of the staffswion in m is
{A} 105.13
(A) “Still {B} 103.68
(B) “13.63 (C) IODJS
{C} 100.78 {D} 98.55
(D) 93.55

[33'
Q.9 For aa mine
For mine ofof production
production tI per year, the
per year, the total
total cost
00111 of production is
of production given by
is given by its1
at' +b. The revenue
+b. The revenue
fi-oin sale is given by et . li' a, b and e, are constants, the hrealtevcn
from &ale is given by c/ . If a, b and c, are C(IIIBtants, the breakeven value
value of
of t
I is
is

(A) [d~{c'-4ab}]/(2a)
to} [Ii-'11.! {421 -4flb}]i{3ttl (Bl
(B) (~{c' *4flblliflvl
Nit‘: 4ab)]/(2a)

[Cl
(C) [-d~{c 1
[-Eiv'ic: -4ab}]filal
4ab}]/(2a) {D} [£11,t
(D) +4ab}].t[2a}
[c±Jfc'+4ab))1(2a)

The value ol'the Il' . ~:""—“]


out Thevalueofthe
Q.IO
' ' ~
1—!
_,. , l
is
' -x ,,, is
_ I
—I.I]

is} m
(A).,. {a} II
(B)
{C} v
(C)O (D) U2
[m 112
EH 1
Q.l1 Two de!enninants
Two determinants of
of order
order n are multiplied.
n are multiplied. The
1111: order of the
order of the resultant detenninanl is
resultant determinant is
in} nn {B} 2n
2:: {c} i"i in} ml?

.,
(A) (B) (C) (D) n/2

(3.12
Q.l2 'l'hepertial
The diflerentiel o:qUBlion,
pallial ditfcrartial iJD =constant, isasolution
eqmfien. rrZ—H=eottetant. for
is a solution for
r

336' l 33
i A 1 —— —— = [I
Hr1 r Br
.,,a'e ao...
(B)-+-=0
313 at?
I: E 1. -—-a- + — =
3:"
31""
ate 1 as ale 11 as
a'o ao
[C}r 1_+___= D —+-—~——={II
(D)-+--=0
3r: rar iJr' rEi'r
{1'31"2 r iJr
{3.13
Q. \3 In Mohr-Coulomb
In failtn'e criterion,
Bohr-Coulomb failllfC the ratio
criterion, the ratio of uniaxial eontpeeesive
the uniaxiel
of the strength to
compressive strenst.h tbe tensile
to the tensile
sis-mgr]: is
strength is
1+ ' l-sin ‘b
”in“
l+sinfl
{A) Il -s1n¢iI
sin; .
{a} !-sin;
1+ sin e
(B) l+sin;
'
[D] 2C{li +sin¢)
C 1+ (D) ZC l+sin m
e21 (I{l{—i‘m‘”
(C) C(l+sin;J
- sin¢)
sin d]
.
(1"3111
(I 9)
sin¢)

{M4
Q.l4 The avt:Illge
The average Young's modulus and
Young‘s modulus and Poisson's ratio values
Poisson‘s ratio of s limestone
values ofa sample are
limestone sample are 60 It'll“ lls
so 1-:x 10' MPa
and {It} respectively.
and 0.3 respedi~ly. The
The shear
shear modulus
modulus in
in NIH
MPa is
is
{A} 23.07
(A) Ill]? (3)1302?
(B)230.7 {C} 2311111
{C) 2307.0 {D} EEtlTllfl
(D) 23070.0
{1.15
Q.IS Theengle
The oi‘dnw
llllgie of inatroughntheidenee
draw in a trough subsidence helps in determiningthe
helps in determining the
{A} maximum
(A) m subsidence subsidence
{B} ex\l:nt
(B) extent of
of surface subsidence
surface subsidence
{E} plane
(C) of fracture
plane of fracture
[[1] critical
(D) critical width
width of
of the
the opening
opening
'
“Hal... Evie
J/16

www.examrace.com
\MNw.examrace.com
Ifl'lfl
2010 H“
MN

[Md
Q.l6 Reesppingaa winding
Recapping windingrope isdonete
rope is done to
{A} increBsc:
(A) inereasetheflettttralstrengmoftherope
the flex.Wlll strength of the rope
[B] increase
{B) inerease the flexibility of the rope
the flexibihtycithe rope
{Cl remove
(C) remove aepra'tionei'theropesubjeetedtedeterioration
portion of the rope subjected to deterior&tion
{D} preventthernpefrotneaeessiverusting
(D) prevent the rope from excessive rusting

on
Q.l7 Match the
Mat.;h following for
the following for standard
standard diamond
diamond drill
drill rods.
rods.

Speeifieation
Specification lDuter Diameter
Outer Diameter in
in mm
mm
P. AW
P.AW 11. 34.9
p. 34.9
{2- BW
Q.BW 11. 44.4
q. 44.4
IL EW
>L<W r.r. 54.0
S4.0
5. NW
S.NW s. 66.7
B. 66.”?

{A} P-r;
(A) ill-q: R.s;
P-r. Q-q; Re: Sop
3-11 {Bl P-I".
{B) Q-P; PH;
P-r, Q-p; 5-4
R.s; S-q
{Cl P—q: EH". R-p: !k
{C) P..q; Q.r, R-p; 3-5 {D} P-q; CH". R-s: Sop
(D) P-q; Q.r, R.s; 51!
one
Q.l8 Payback period
Payback period is
is time
time required
{A} fill"
(A) for the
the cash
sash income
income from
from aa projeet to get
project to get heel:
back the
the initial
initial eash
cash investment
investment
[B] from
{B) from thethe stan
start of
of the
the project to the
project to the time
time to
to tee-over the total
recover the total initial
initial investment
investment
{C} from
(C) from thethe stan
start of
of the
the projeet to the
project to the start
start of
of production
production
[D] to
(D) to the
the period
period during
din-tag which
whieh internal
internal rate
rate of return is
of return is generated
generated

£1.19
Q.l9 ForeleetriesignaiingsvstemsinurederyoundeosttdnenthestatmnentthatisNUTmteis
For electric signllli11f!: systems in underground coal mines, the statement that is NOT true is
{Ajallsipalingequipmentnnrstbeintrinsieallvsafe
{A) all siplingequipnx:11t must be inlrinsically safe
[Blithe
{B) signaling circuit
the signalilli eireuitnnsstheeonneetedtagrumd
must be cormected to ground
{AC}thesottreeofeurrentshouldbeanapproveddryhatter}:r
(C) the SOUR:e of cummt should be an approved dry battery
[DJDCbeflaurretayswhmeonne-ctedinpanllel
{D) OC bells or relays wbe:n connected in ponollel sheafldhesttppliedfrumasinglesnureeuftnrrrent
ohould be oupplied from a single soun:e of current

one A
Q.20 A ladder
ladder of 1sleight SO
of weight 50 N rests against
N rests against aa frictionlesa
fi'ietienlesa wall and floor
walland floor as shown in
as shown the figure.
in the figure. A
A horiaoutal
horizontal
string ties
siring ties the
the base
base ofoftlte ladderto the wall. The tension in the string in N is
the ladder to the wall. The tension in the 8tring in N is

(A) we
{A} 100 as} so
(B)S<l to) as
{C) 72 {n} as
(D)2S
{1.11
Q.21 ThenteenendflwmndardMenefmegradeofhnnereinadqsmitarefifliandfli
The mean 111111 the standanl. deviation of the grade of iron ore in a deposit an: 62"--' and S%
reqeeetivelvTheeoetfieientefvariafion
respectively. The coefficient of variation ofthegradeinfiis
of the grade in% i5
[A]: 14.3
(A)24.8 {B} 12.4
(B) 12.4 {C} 31]
(C) 8.0 {D} 4.11
(D) 4.0
:12:
Q.22 The variarK:C
The varianee of
of failure
failure time
time {time
{time to
to failures)
failures] efan
of an eleett'ie motor in
electric motor shovel is
in shovel [drill] hrz.
is 1600 hr'. lithe
If the
failure time
failun: time follows
fol lows an
an exponential
exponential dislribution,
distribution. the
the expected
expected failure
failure time in hr
time in is
br is
[Ali-Hi
{A)40 {El} 80
{B) Ell {G} Hill]
(C) 800 {D} 1600
{D) [drill]

MIN—A .,.
eis

www.examrace.com
\MNw.examrace.com
IMH_ _ _ _______ __HH

{1.23
Q.23 Much lhe
Match I11: following
fallow
_
"'

'i
22
33
- lnshmmnt
Ahnw‘s
A lml
bney'slevcl
Pmlugraph
PeuiOgr~~ph
Pinninmtcr
•bbI
Pumas: a"I Hmnt
Purpose
hufimntnl
Measun:ment
and Vtrtiml
horimntal and
area
angina
vertical angles
of platted
area uf figure
plotted figure
reductioo flf
and raductiun plotted mm
of plmtcad
44
Planimeter
Hm: Sextant
Box Sultan: '(Ir:d £111a and
enlargement
angle of
angle inclinaflnn
of inclination
maps

{All 1-H;
{A) 1-1:; 3-d;
\-a; 2-c; 3-d;4-I:I
4-b {B}l-¢:1—h:3-d:#-a
(B) 1-c; 2-b; 3-d; 4-a
{[3] l-d;
(C) l-d; 2-a;
1-H; 3-d;
3-11; 4-c
4-: {D} l-d;
(D) I -d; Z-c; 3-1:;4-a
2-c; 3-b; <HI

{1.24
Q.24 Augwflghing
A liflflflkgisraisuiby
cap weighing 12000 fmuchains
kg is raisod by four each making an angle of 30" withthtvcrticai.
cbains mhmkingananglaufafl“ with the vertical.
Th: tension
The min in in each
nah c:hlin
chain in
in kN
1:14 is
is
[A141
(A)41 [B134
(B)34 {C}
{C) 15
25 [D]
(D) 21}
20
[3.25
Q.25 The relationship
The mlntiunship between] 111: dnlwbar
between the drawba: pull and the
pull and speed for
the speed fur difi'ercnt gears of
different gram unlf prupclling
of aa self propelling
‘Ii'fihifiil: is
vehicle is tepie!Mmted
rcprcamtfid by
by

'I
I
-- ~-
"' 'Ii
--

(g k4
w-
!nun-MEN Ifl Ilfl'PI JH

-·- .,
'I
I ~ •I ~(S)

~w- I ~/
! ! ~·--

{A} Q
(A) Q
-- -- {B}
(B) s5 {C} R
(C) R {D} pP
(D)

0.26 — Q.55
Q.lfi- tin-3r two
Q55 c•rry mark: eaeh.
t'l'l'fl marks each.
{1.26
Q.26 A flammable
A flannnahl: mixture
mixture: lias
has 70%
Tfl'ili CU.
CH4 and
and 30%
30% C0.
CO. The fl•mmability limits
lower flammability
The lowcr limits far
for this:
these 335:5
guc:s an:
are
5% and
S% 13% respectively.
and 13% Fur the
wipes-lively. For the mixture,
mixture, the
111:1a
lower flammability limit in
flammability limit in%'34 is
is
{A} 6.13
(A)6.\3 {a} 8.72
(B) 3.12 (C) 10.25
{C} 10.25 {D} 12.16
(D) 11115
1:32?
Q.27 The volwne
The mlm of Wm with
uf lelnbedroD m aa
with vertices u {0,0,0),
mam. (I ,O,Q), {m
(mm, ,u} and
(0,1,0) (0,0,1) is
aod {mail} is
{A} l/2
(A) u: {E} l/4
{B) H4 {C} ms
(C) i/6 (D) 1:3
{in} 118
MN-A ill-I5
5116

www.examrace.com
\MNw.examrace.com
113“]
:WJO MIN
MN

£1.23
Q.28 Ahslsnoedwinoerrsisesflflflfltoonesperdeyfiomsoepthoffiflflm Ions—polirloldofflsewindklg
A balanced win&:r raises 3000 toones per day from a depth ofSOO ltL The payload oftbe winding
eegeisTtoonss.Themeonsumodperdayinkfisfimswiodereflieieoeyis
cage is 7 tonncs. The energy oonsumed per day in .kWh at 70% winder efficieocy is
{A} 6030
(A) 1503!} {H} 5840
(B) 5344] {C} 5750
(C) 57321 {D} 5630
{D) SEED
loss htmssis loadedssshoim inthefigureflhet‘oreetnthememherhfl

·- is
Q.29 A truss is loaded as shown in the figure. The fi:Jfce in the member AC is

'
{All tension
(A) tension 75.9
75.9 N
H 1'B} compression
(B) oomqsrossion 43.3
43.3 N
N
{[2] tension
(C) teflsioo 43.3
43.3 N
H {D} compression
(0) compression 75.9
T53 NN

{1.3!}
Q.30 In the
ln the frictionless
fi-ietiooless pulley
polls}.r system
system shown
shown inin the
the figure,
figure. each
one]: pulley
Wilt-'3" weighs
weighs 20
20 N.
N.
The woight W,
Tits weight W, inN,
in N. that
that can
can be
he lifted
lifted by
by the
thcsystsm tmderthe
system under conditions shown
the conditi0115 is
show is

{ME-:10
(A) 200 us] 170
(B) no {a} ISO
(C) 15s ms
{o} 100
(D)
0.31 A force
foree of
ot' SOi
. . is
Elli—EDEN
Q.31 A -SOjN moved from
is moved item the
the origin
origin to
to as coon1inate(4.0m,2.0m).
coordinate{4.flm,2.flm}. The
The wor1r.
work dooe
done in
in
the process
the pine-Es in
in JJ is
is
is} 75.6
(A) 71s [3} 85.5
(B) ss.s
{C} 913.2
(C)90.2 {D} 100.0
(D) IN.”
on
Q.32 'I‘lteqoeueofoueksstsemshesplsnthoPpcrisknomtohelt-UMHqueue.T1:repmhshilityl]1tIt
The queue of trucks at a crusher plant hopper is known to be MIMI! queue. The probability that
there is
there is no
no truck
truck to
to unload
unload is
is 0.3.
[1.3.
Doetorsinsthe
Due to rains the moan
mean senrieetjmestthehopperis
sesvice time at the hopper is ineressedbyflflfifi.flssoonsoqueooe.tho
increased by 30%. As a consequence, the
expected
expected number
number of trucks to
uftruclcs in the
the queuing system {including
queuing system (including the
the one
one PGSSlbijr tmlomling} boo-amass
possibly•mloading) becomes
{A}
(~00H] {B} [2
(.02 {C}
~M 14 {D}
~01 16
{3.3}
Q.33 Pollfi'omsotmderyoondttmnelblastingismmflydistflhutodwifltsmesnoflmtonnesmd
Pull from an underground IIIIIDel blasting is normally distributed with a Illll8ll of 100 1o1mes and
1naturism-lee
variance 100 lfllflltonttesf. Theprobahilitythatthetonmgevslue
(tonnes)'. The from: blast
probability that the tonnage value fioma exceeds 110
blast exceeds Hill:
is
{A} fltifl {B} 0.3!} {C} 0.16 {D} ELSE
£2.34
Q.34 ~····
The feasibility
The feasibility n-:gion
region of··-
of an
an LP
LP problem
problem in ~···
in variables
1inst-islsles .r and y is
x andy is given ~=
givott by
by the
the following constraints. in
following <:onstmints, in
addition to
addition to the
the non-negativity
non-negativity oonstraiots.
oonso'aints.
ySfil}; xS90;
y$60; ISEIIIJ; x+
I+ySTD.
y$70.
111: number
The number of
of corner
oorner point
point feasible
feasible solutioos
solutions for
for this
this problem
problem me
are
{A} 33 l3] 44 {C} '5 [D] '5
...
(A) (B) (D)I
(C)
mus.- so.
""

www.examrace.com
\MNw.examrace.com
mle
lOIO _ _ HH
MN

0.55
Q.JS The
The tmit eest matrix
unit eost et‘sa hslsneed
matrix of transportation preblem
balanced trsnsponstien sbown belew
is shewn
problem is below

Destinstiun DI I12 D5 Eiupplg.I

iD2~~D~3~Supply
::""~";oo~·:·oo=]~±D~I~

1: I~ IE
51 T 5 5 fit]

So~
Enuree

Demand
Demand
_I~
__
52
I~
_55__
5 4
_E___ Illi__
so 121]
50 120
5' . fit]
2
3!}
30
3t}

comer rule
,,
The COSl iii
transpOrtalion eest
The treuspet'teliesi
is
of the initial hssie
the initial basic feasible obtained by
solution obtained
feuiblc selutien by the Nortb·West enmer
the HerthrWest rule

{A} 1025
(A) “125 (B) 1015
[B] 1075 (C) [130
{C} liJO (D) 1225
{D} 1226

11.55
Q.36 Ahighwltmmsirssrmletisoperatedferflheursinsmineuriththeflewrsteefsitvmingtin-m
A higb volume air saq~ler is operated for 8 hours in a mine with the flow n.!e of air varying from
m1/min in
1.5 mjlmin
1.5 m1Jmjn. The
1.3 mjhnin.
to 1.3 The~ empty weight of the filter
weight efthe 2.30 gg and
is 2.50
paper is
filter pmr the fins]
~ the weight is
final weight is
2.155
2.65 g.
g. The conc.enuation ef
mean menusflm
Tbc menu of the Suspended Particulate
the Suspended Matter [SPMI
Particulate Matter during me
(SPM) dining study pet-ind
the state period
in ,1m
in )J.r/m' isis
(A) 591
{A} 59\ (B) 551'!
{B} 5SO (C) 545
{C} 545 (0) 521
{D} 521

1
[1.32
Q.37 In
In an epeuesst
opencast mine
mine shes-m1n
shown in the
the figure
figure belew the ens]
below the density ef
has sa densityr
coal hss of l1.4 tonne/m . Assuming
4 tetuiea'm]. Assuming
an the given
under the
ratio under conditions m
given eentlitiens'
staned from
mining operation started
mmmgeperstien ftom plane XY, the
plane X‘t’ the epersting stripping rstie
operating shipping in
mEll.I'tlsnnle'
m'ltonne iiss

I
X

2'11 m
20m i

l
l-Eittrseted Coal
"1
\Omm
311
H
20m m

'l'
y

{A1232
(A) 2.32 (B) 2.441r
(B) 2.47 (C) 2.56
{C} 2.56 (D) 2.64
{D} 2.64

QJE
Q.38 A. developed panel
A deseleped panel the coal seem
for aa seal seam Issuing incubation period
having insubstien period sfof 5 months has
6 menths 32 s~ pillsrs
has 52 pillars
under extraction
under estrsetien having
having size
size 25 m,
25m, and
and height
height 3.0
3.0 m.
m. Density»r
Density ei'
of ens]
coal is
is L4
1.4 tennel'm
tonnefm. . Esusetieu
Extraction
mtie during
n.lio depillsriug is
timing dcpillaring expected to
is expected to be
be 7S%.
25%. To
Te depillar the panel
depills: the panel within the incubation
within the ineuhstien peried.
period,
assuming 25
BSI1Iming 25 working
wetting days in aa month,
days in month, the
the production
produetien ftom the panel
fiem the panel inin tennefdsp
tonne/day is
is

{1454211
{A)420 {E4311
{8)480 (C) 560
{C} 560 {D) 53!]
{D} 680

{155'
Q.39 A
A elesed ABCDE ei‘
travcne ABEDE
closed unease of pesimeter
perimeter 425
425 m
m has error +fl+25m
total errer
has sa tots! Jati!Ude and
in latitude
+0.2Sm in -o.44m in
~ —fl+44m in
depsrtme.
departure. Title
The peeeisien
precision ei'
of traverse
traverse is
is
[A]
(A) II in55ti
in 5~ (B) II iu‘IBS
[B] in 785 {C} I1 in
{C) in 355
833 in 1024
(D) 11 in
[D1 1024

Hit-s me
71!6

www.examrace.com
\MNw.examrace.com
Illllil
2010

use
Q.40 The value
The 1luralne of
ct'the given integral
the given integral is
is
"1:'J sinsx __ cb:
sin
'j-
I[sins+ccsx}
_.(sinx+cos.r)
,.5.
(A) sins
sin~r/8 st
< (C) sitts'i'fi
sin~r/5 1e
3<
{All {H _
(B)- it} {D}
( 0)_ -
It!
10 lIltIII
10 til
10 It]
10
{9.41
Q.41 The probabilities
The prchahilities ofat" hilling
hitting aa t1trget
target by
by AA and
and Bare
E are 1/3
LG and
and 215
13.5 rcspmively.
respectively. AA shoots
shoots at
at the
the target
target
mice, followed
once, t‘cllcwetl hy
by BB s.hooting
shooting at at tbe
the target
target <JDCe.
ease. The
The probability
probability of
cf hitting tbe
bitting the target
target is
is
wvo•
[A] 1"15 •• ,,
{B} 5315 {C} HHS
~005 {D} THIS
AMI

{2.42
Q.42 The 1value
The value cf
of i:k fer
for which
which the
the points
points (5, SML l},{l 9,?)
(5,5).(k,l),(IO, 7) lie
lie on
on aa straight
straight line
line is
is

Is] —5
(A) -5 {B} +5
(8)+5 (C) —s
[C] -2 {c}
(D) +2
+2

{1-43
Q.43 A pscject
A netwcrit =nprises
project network comprises five
five activities
activities as strewn below.
u $hown below. The activity durations,
The activity in days.
dunltioos. in are as
days. an: as
indicated- Crashing
indicated. Crashing cf any.r activity
of any activityr costs
ccsts Rs.
Rs. 1000
NIH} per day. If
per day. If the
the project
project is clashed to
is~ tn the shortest
tbc !honest
possible dwation,
possible duraticn. the
the total
tctal crashing
crashing cost
cast in
in Rupees
Rupees isis
1G
"
• ' D

[Aflfiflflfl
(A)l5000 {B} 14000
(B) ldflflfl
'
{C} 13000
(C) lJflflfl {D} 12000
(D) tlflflfl

0.44
Q.44 Ftsteelwirercpect‘25nundiameterweighingflflfmhastish'andscfiwireseechThcdiameter
A steel wire rope of25 mm dimnetel" weighing 37 N/m bas 6 strands of7 wires each. The di•meter
andteusilestrcugthet'eachwireareljnuuahd
and [EMMresp-ectively.Thefactcscfsafetyfur
tensile strength of each wire are 2.5 mm and 1800 MPa, respectively. The factor of safety for
raisingacageci'weightfiflkNfiemad-cpthct‘lflflmis
noising a cage of weight 60 kN from a depth of200 m is

{A} SE]
(A)HO {B} 4.50
(B) 4.50 {C} 4.25
{C) 4.25 {D}
(D) 4.15
4.15

{1.45
Q.45 In block
In blue]: caving
caving operation
eperaticn the
the draw
draw points
paints are
are placed at Ill
placed at 20m in center
center tc
to center,
c~nter, with
with the
the pillar
pillar width
width
3.5 mas
3.5 m as shown
shcwn inin the
the figw-e
figure below.
hetcw. The
The muck
sinusitis assumed to
is assumed to have we cohesion
have z.ero cchesiau and
and 35°
35“ frictioo
fricticn
angle, The
angle. The beight
height of
cf draw
thaw cone
cone (b)
{1:} in
in m
m is
is

[fit] 12.5
(A) L15 {E} 14.6
(B) 1415 [C] 15.8
(C) 15.3 {D} 16.5
(D) “5.5

MH—s't.
MN--A .,.
flflfi

www.examrace.com
\MNw.examrace.com
mm
1010 "H
MN

0.46
Q.46 The shrsiss length
The stroke and pitch
length and uf ihs
pitch of rille bll'
the sins bar of pen:ussive drill
of sa psshussiss 60 mm
I!lliCir.ine are 00
drill muchim‘ar: and mm
nm sssi tn60
respectively.
respectively. lfflh: drill alum-mu
If tbc drill op:rateS 11 blows/minule, the
2000 bluwflnfinutfl,
at 2000 speed III
rotllional speed
the militant] rpm nflhfl
in IJllll drill 51531
of the 1111]] steel
•is
{A} I45
(A) 145 [E1150
(B) 158 (C) 102
{C} 162 (Dl 115
[D] m
(14‘?
Q.47 'I'htnninflnflpflllingpuintnfa If 1111 NPV of
vemilatioo I)'Siml is shown in the figure below. HanHPV
The main faa operating point of a vwfillfimmmisshuwninihtfimhcluw. of
1mPlflliflflthfimfllflfimmMMMIMWIJMMW{WJ}WMW
200 Pa aMists the vcntilllioa system, the resultant pressure (Pa) andquaatil}' (m) generated by tbc
fanrcmuctivtlym
fao respectively are

1000 ---~--

600- -----~--

Pl!BI'II'l!I,Pa
401>--

20Q--~

'

40
'
Quantity. m Is
. .. 100

{A} 500,68
(A) 500.03 000,63
{3) 600,63
(B) {C}
(C) 640.55
640,55 (D) 400.63
{D} 40o,63

Comma]:
Commoa Data
Data Question:
Quesdoa•
Cull-ll“
eo.- DillData far Qllntloas is
fer Que-flan: ud 49:
411 and 49:

mmmmmmmhmmmsrmmmmwmw
The gnoular media ia fiD ore bin is awuned to be ofregulm" spberi<:Bl shape tepa 1!«1 by the geometry
ss
as shssss
sbmm isin Ihe fiSUJI'. The usii
the 531mm wcigbtofsolida is
1111it wsightsi‘shiids 25 Wm}.
is25 kN/m
1
.

Q.411 The void ratio is


{h}
(A) 0.01
0.91 {B} 0.34
(8)0.84 {C} 0.13
(C)0.78 in}
(D) thh-ii
0.69

£3.40
Q.49 The
The dry
dry density
density in
in ishiiss’
kNtm' is is
(A) 13.09
{A} 13.09 (B) 12.50
{a} 12.50 {C} 1 1.14
(C)IL74 (D) 111.3?
in} 10.87

ism—h
' "
...
iiiiii

www.examrace.com
\MNw.examrace.com
III-ID HH
{tumult-
Ceeran fermwudfle—
Dlllll for Qn ·~50 ud Sl: "'
Metch the
Match the elements
elements ofa
nt‘ s simple
simple curve
curve as
as given
given in
in the
the figull:
figure below.
heicw.
P

' ...
W

T1 /' '—- T2

Ft-IIIIII n1

DI

[3.511
Q.SO The tangent
The tangent length
length in
in m
m is
is
{A} 215.5
(A) 215.5 (3)2204
(8)220.4 {C} 228.4
{C) 223.4 {D} 23Q.9
(D) 1311.9

£1.51
Q.5I The lengtb
The length oflbe
cflhe longcbord
lengchcrtl in
in m
m is
is
{A} 375
(A) 315 {B} 40M}
(8)400.0 (C) 415
{(2} 415 {D} 450
(D) 451}

Linked Answer
Uaked Answer Quesdooa
Questions

Sure-meat [er
Slatemeat fer United
IJaknl Answer
AMwer Questions 51 s.cl
Questiou 52 and 53:
53:

A lcngwsll psnelwithsfsce
A longwall panel with a face height nf3.flmsndfaeelength
beigbt of 3.0 m md &ce length nt'lfirflflm
of !SO.O m isis werkedinli
WOfted in 3 shiftsperdsy
sbifts per day
mployingdflmenpershifi.ThedepthnfthewebnftheshesmcutfingccfliisflfimTisetmitwe-ightcf
employing 40 mea per shift. The depth of the web of the sbean:r cutting coal is 0.5 m. The l.Ul.it weight of
fl1eene1isl.4tnnncim3.Twcfisllfececulssreexecutedpetshifi.
the coal is I .4 tonnelm'. Two full face cuts are~ per shill
0.52
Q.S2 The daily
The daily production
productinn from
firm: the
the panel
psnel in
in 1011ne11
tnlmes isis
{A1945
(A) 945 is} 1240
(B) 1240 {C} 1890
(C) less {n} 2530
{D) 25311
{153
Q.53 Thcpnnel 0M5 in tnnnes is
TbcpaoelOMSintonncsis
{A} 12.75
(A) [2.15 {s} 15.75
(B) 1535 {C} 8.75
(C) see {D} 5.25
(0) 5.25
fitntensent t'nr Linked Answer Questions 54 and 55:

Air at
Air st as density
density of
nt‘ 12 kym'i'1 flows
1.2 kglm flnws in straight duct
in as straight duct such that the
sudt that the velocity
velocity st the centre
Bllhc centre is 12.5 mls.
is 12..5 ms. The
The
merited
method faster
factor the
for the
the 1lurellslcit}:r
velocity pmfile prufilo.: is
is known
known toto he 0.31}.
be 0.80.
0.54
Q.S4 The velncity
The velocity pressure
pressun: value in the
value in duct in
the duct in P11 is
Pa is
{e}. 31
(A)31 {s} 41
(8)47 {C160
(C)60 {D} 83
(0) ea
{3.55
Q.SS The air
The sir flow
fins.r encouJ~Ien
cnccunters as S)nmetriC
symmetric expansion
expensinn such
such that the aoa-sectiooal
that lbc crossesectinnsi area
sees of
cf the
the duct
duct
hecnmesdcnhle.
bc:comes TheststicpresetncvsluesttheinletsndnnfletcfflseexpsnsinnsrefiflPssnd
double. The static pre5IIIIR: value at lbe inlet and outlet of the expansion are 60 Pa and
9i} Pa.
90 F's. respectively.
respectively. Neglecting
Neglecting friction.
fi'ictinn, the
the shock~
shack pressure ioSII
loss on
nn account
scents“ of
ct' expansion
expensinn in
in Pais
Fe is
l5
[A] IS
(A) is} 22
(8)22 {C} 38
(C) ss [n] 4s
(0)46

'. - . .. - - - ----..- .-__. .


A
10/16
-..._1m

www.examrace.com
\MNw.examrace.com
Efllfl
.,
_."‘.""
General #pfltude
Geoenl Apdtude (GA) Quufluns
{GA} Qaeltioos
11.5.;—
Q.56 - Q.“
Q.60 any
carry pl: lurk each
OH mark Neb.
{1.55
Q.56
......
HEB}! nflhejbflflwing
Which
Eli-rt
(A) first
{A} urge
options it
of the foiWwitrg aptiam is the in mflgg
c/osQt in
the claim! 111«111ilfg m the nurd
to the word barium:
bf!low:

{E}
(B) candnnm
condemn
(C) ream
{C} restrain
in} scold
(D) scald
(3.5?
Q.S7 The question beluw
The quation cumin: of
hfpw eo~~Sists Q)" aa pair afufmpd
pair of “film
nlaJed ...wdr followed by four pain
byfimr pain of wit. Safer:
of womb. the
Select the
Mir that bat
pair thai has: uj,.esus
W the the reL:uion
”hump In it! the original pair.
the original pair.
M : mm
Prealllblli: ~

{mm
(A) arpendqynr:: law
law
{31mm Play
(B) prolop: : play
{E}
(C) epimd:
episode:: will
serial
{D}
(D) plat
plot;I not}!
story

{153
Q.S8 Chcxne the
Chase the mm: word flow
appropriate W
mtMI W the apriam
from the below m
gfven beiuw
opt/om 3mm to wmpiere the finflawing
COI1lpiete the following
MIME:
seNtence:
Th: wnne aa
comllllttee amt:
Tbe filllllflttlfl report. extolling
report. t1te strength-
only the
eDollla& 0!t tbe proposal.
of the
ltreqlbt uI' J)f'O~

{A}
(A) mush]:
rnsonable
EB} supporti~
(B) Wfliw
{C} biucod
(C) 13M
mind
{D} iiagmenled
(D)
(3.59
Q.59 Chum: tile
Choose the most Hun! from
War-int: >WNd
mast oppmpriiJI£ gtven befaw
options given
the options
firm the m War:
beluw to the Mfmving
complete the following
sentence.-
sentence:
If u.e -.try u.
Il'lhmh'yhuln-chlmrulprmpcrlttII
1o Khleve re111. pro~pn1ty,lt 1e thtefruluup-ngrmmch
tbat 111e frulu or progrea rudro
Ill.
aU. lull in equl
and In mm
e~~ul meai!Te.

{A} inmritnhle
{A) inevitable
{Blmlinaml
(B) contingent
{C}
(C) nblivinus
oblivious
[D]
{D) impn'lliv:
impcntive
{1.45:}
Q.60 hpu'amimm Rs.IOOO at lflfiannualoommdinlflfitfnrlym.mmemdufmmthe
A pc:rsa:1 illvesla lflflflu 10% aDDual compound interest for 2 years. At tbeeod of two~ tho:

m-·
wbole mtiainvmdatanannulnhmleinmnfllfifmfiym.Thctuuh-Iflu¢uffln
whale uoounl i$ in..ested at an 8DIIl.lll. &imple interest of 12% fiH" S yean. The I<,Jtal value oftbe
inmfinfllyis:
lTTfi
{A} 1776
(A)
fiDaUy is:
(B) 1TH}
{E} 1760 1921'}
{C} 1920
(C) {D} 1936
(D) lfilfifi

(2.61 -— Q.65
Q.61 {M5 carry twp marks
curry two each.
Inn-ks eaeh.
{1.51
Q.61 The bu
The bl- on amid-g In deslpated
nn •DGki-.;ID publle pllflflfln
Haunted publll: save aa lira:
pJ- CIID an: aumber ul'
brae IIIIIIHI' from the
people [rpm
of people tile
well Wu am
weB lulown If mflmnmnm
effecb ef mm Inch.
eariron~~~ental ....,._ tiiMike. .._lve uni-Hug
Min tmekiD& Harlan-l3
.eriolllly lmpflrs
lmpaln
bnldL. Tile bJm. rlgtatly Reb to pnlleet llOIHJIIOken froal itl weffeda.
mmqmmmmmmmmmmmmmmmm
rapinltory

H'ir'tichpflhtfifluwiugsm
Which be!/ sum.r up tile mealling cif the Mm:
oftile fo/luwing stato:s f!JJLJ batmuplhemflngqflhe above passage:
{AJEfincuufmvirmmflimha-mnmwfllm
(A) Effects ofmviiOWIIimtlll tobacl:oan: wclllmown..
{E}1hhnmmuflnginwhficplnmmm¢nmm
{B) The biD oo smoking iD public places protectB the non smo1:en..
{fls-mthlgishdfmhmlth.
(q Passive smnkin8 is b&d for bcalth.
immunpnmfinginpuhficplmuchflflpusiwsmfing.
(D) The baa on $~~:~~~king iD public places excludes passive smoking.
Ml'iH'h 'I11116
119175—

www.examrace.com
\MNw.examrace.com
mm
2010 film
MN
{1.52
Q.62 Gimme
Given eeqeenee A,
the sequence A. B,
E. B,
B. C,
:3. C.
{2,13, m, D,
C, D, D. D,
D. D,
n. ••• etc.,
augment-is enemtwe
that is one Be. three
A, two Bs, threefie. fun: Dr!,
Cs, four De,
fiwEamdmumlhcld-fltletterinmcwfillhe:
five Es and so on, the 240"'1etter in the 9CqUCIICe will be:

{AW
{A)V {B}
(B) U
u {C} T
{C) T {D} W
(D) W
[3.63
Q.63 Comidcrmem
Consider the set uf
of human I“, 1,
integers {1, 2, 3,
3, ... , 50110}.
SOOO}. Th:
Tlte mmherufintagera
number of integers thntis
that is divisible:
divisible hynfither
by neither
33 unri-
nor 4 is:
is:
{A} 1668
(A) I663 {E} 2084
(B) 2fl34 {C}
(C) 2301}
2500 [D]
(D) HIE-
2916

QIA
Q.64 Apmifiwintegwminm
A positive integer m in base 10lflwhmmpmunedinhasulhnsflwmprmnuflnnpmdinhHHS
when LtpL ;; nce.t in base 2 has the ~on p and iD base 3
buttEflmq.W:gctp—q=mmmcmhumfiunisdnminbaselfl+Whichuffln
has the Ltpi rnlati011 q. We gctp-q af90 wbcR tbe subtraction is doQe iD base 10. Wbich of !be
fulluwingismfllym:
following is necessarily true:
{$111214
(A) m::: 14 {5195111513
(B)9:Sm:513 (C) GEmEE
{C} 6:Sm:58 {mini-".15
(0)m<6

Elfifi
Q.6S Given
Given the
the fallnwing
following fuur
four fimeeieee
functions 13m} - n‘".
r,(n)- f,(n) =
n'", Mn} = {1.11', f,(o) -
(1.2)', me] • 2"”.
2...,, me} - 3'”
f,(o)- 3oJJ which
which
fimutian
function will
will haw
have the
the [mt
largest min:
value fur sufficiently large
for sufficieotly large values
values if
of nn (Lt. o_,. an}?
(i.e. n—r a;,)?
{Mi}
(~~ {3}
(B)~13 {C}
~~ f: {D}
~~ 1':

END OF
END [IF THE
THE QUESTION
QUESTIDN PAPER
PAPER

' ' " U '


HH—fi lama
12116

www.examrace.com
\MNw.examrace.com
.,.
2am _ __ HF

Splice for
Space Rough Work
I'm; hung]! Work

_. ' '
h ifi
ll/16

www.examrace.com
\MNw.examrace.com
PM
Space for
Space for Rougb
Rough—fiat]:
Work

_..
A.
HJ'Ifi
1-1116

www.examrace.com
\MNw.examrace.com
2q
2010 .,u"
Space for
Space for Rough
Rough Wail-k-
Work

"' "' ' ''


ran—n.
MN-~ um-
1'-'16

www.examrace.com
\MNw.examrace.com
Ifllfl ___ _ _ _ __ HM

51m;1;
Space hihéi m
for Rough Work

' "
LIE-A. [HIE
l(o'l6

www.examrace.com
\MNw.examrace.com
2011
2011 MINING ENGG. —
– MN

MN : MINING ENGINEERING
ONLINE Examination

Duration: Three Hours


Duration: Maximum Marks: 100
Maximum Marks: 100

Read the following instructions carefully.


1. Questions must be answered using computers provided by the GATE at the examination centers. Each
1.
computer shall run specialized examination software that permits a maximum of one answer to be selected
for questions of multiple choice type.
2. Your answers shall be updated and saved on the server periodically and at the end of the examination. The
examination will automatically stop once the duration of the examination is over. 1"
3. There are a total of 65 questions carrying 100 100 marks. All questions are of multiple choice fleiEach type. Each of
these questions carries four choices for the answer labeled A, B, C and D. Only one of the four choices is
the correct answer. ”I. theiouryices 1
'
4. Questions Q.1 —
Q1 – Q.25 carry 1-mark l-mark each, and questions Q.26 – Q.55 — Q55 carry 2-marks each. fch. r
5. Questions Q.48 Q.48— Q. 51 (2 pairs) are common data questions and question pairs (Q.52,
– Q.51 (Q. 52, Q.53)
Q. 53) and (Q.54,
(Q. 54,
Q. 55) are linked answer questions. The answer to the second question of
Q.55) of the linked
linkeciI answer questions
depends on the answer to the first first question of the pair. If the first first question
quest1 n 'in,the linked' pair is wrongly
in the linked
answered or is unattempted, then the answer to the second question que'st n in theéair 111not
the pair will not be evaluated.
6. Questions Q.56 Q56 — – Q.65 belong to General Aptitude (GA). (G Questions Q56 —
uestigns Q.56 Q. 0 carry 1-mark
– Q.60 1-mark each, and
Q.61 —
questions Q.61 – Q.65 carry 2-marks each. .-
7. Unattempted questions will result in zero mark. Wrong Wr g answers
an ers will ill result in NEGATIVE marks. For
Q.1 —
Q.1 Q25 and Q.56
– Q.25 Q56 — Q60, ⅓
– Q.60, 1/3 mark will be deduc
deducted d for e
each h w ong
wrong answer. For Q.26 — Q51 and Q.61
– Q.51 Q61
— Q65, ⅔
– Q.65, 2/3 mark will be deducted for each wrong answer. nswer. The he question
uestion pairs (Q.52,
(Q52, Q.53),
Q53), and (Q.54,
(Q54,
Q55) are questions with linked answers. There wil
Q.55) will be negative
nega marks only for wrong answer to the first first
question of the linked answer question pair, i.e. i. e. for Q.52
Q. 52 and
an Q.54,
Q54,' 3⅔ mark will be deducted for each
wrong answer. ThereThere1s is no negative markin Q. 53 and Q.55.
marking for Q.53 Q. 5
Calculator1s
8. Calculator is allowed whereas charts, graph hhéets
sheets or tables are NOT allowed
arJNOT allowed1n in the examination hall.
9. Rough work can be done done1n spe
in the specified areaa only.
10. Candidates may use the back side of
10. 0 this
1‘s pa ge to record their answers for their own convenience.
page
:fifie
11. To login, type your Registration Number and password as per instructions provided1n
11. provided in the envelope.
12. In order to answer a question,
12. questifioniybuyou may select the question using the left side selection panel on the screen
and choose the correct answer ans er by clicking on the radio button next to the answer. The answered questions
shall be indicated by a solid black ball on the selection panel. In order to change the answer, you may just
soli bla
opt 11. If you wish to leave a previously answered question unanswered, you may click
click on another option.
on DESELECT ANSWER ER button.
13. -You
13. selec
You may also select ANS questions using NEXT and PREVIOUS buttons.
questions for reviewing later using MARK button. All marked questions are indicated
14. You may also mark1alsuifiesttlions
14.
{on
by Km panel. Questions which are answered but are marked for the review are
y a rectangle in the selection
\Btion
indicated by a solid bblack aclk rectangle and questions which are not answered but are marked for the review
are indicated by an outlined rectangle1n
rectangle in the selection panel.
15. You must
15. m'fi'st sign this sheet
slie
lieet and leave it with the invigilators at the end of the examination.

1"
DECLARATION
DEC RATION
I hereby
here declare that I have read and followed all the instructions given in this sheet.
F
Paper Code: MN Registration No: __________________Name:
Name: _____________________________

Signature

MN

www.examrace.com
www.examrace.c0m
2011
2011 MINING ENGG. —
– MN

Q. 1 —
– Q. 25 carry one mark each.
Q.1
Q.1 A scatter plot prepared using a set of
of values of lead and zinc from a lead-zinc deposit is shown in
figure below. The value of correlation coefficient
figure coefficient is

s ‘1
8

lead (%)
0

8 ‘
Pd
0
_ - .
'4.

_. . ,4 a_
1
1

r
I

1 8
I I I | | | | | I | I /‘

1
zinc (%)
zinc (%) 8
r I"
(Not to scale) "' '
u . .J 'l
1.0
(A) 1.0 (B) 0.7 O5
(C) 0.5 “H ..((D)) 0
O
. 1 "" K (
: ..u- .I r
Q2
Q.2 The two vectors are orthonormal, if
1f
. '
I
III
J

(A) vector product is zero and norm of


of each vecto al .0 zero
vector is also
(B) vector product is one and norm of
of each vector
vect is also
als one
of each vect
(C) cross product is zero and norm of vector is one
vectorr is zero
(D) cross product is one and norm of each vect

{m
Q3
1  x  1  x  is
Q.3 1
The value of lim
lim—

mug
x 0 x
x—)0x
(A) 0
(A) (B) 1 (C) 22
(C) ("1" (D) 33
(D)

Q4
Q.4 1l 1 1
 (B)1P:”‘tjz:
infinite series 11+—
The infinite   '+
4—1—1“?  , is
2 4 8
I

oscillatory
(C) oscillatory (D) semi-convergent
semi-convergent
1;
(A)
(A) convergent
convergent (B) divergent
}(B)_,d1vergent (C) (D)

Q5
Q.5 are' of
The largest area of a rectangILilar
rectangular shaft for a given constant perimeter is obtained when length is
timea’o
(A) 2.5 times of breadth
readth 1.5 times of breadth
(B) 1.5
of
(C) 2 times of breadth (D) equal to breadth

Q.6 offi
A drive shaft of en ine develops torque of 500 N-m. It rotates at a constant speed of 50 rpm. The
an dth
engine
power transmitted
transmitte by the shaft
shaftin kW1s
in kW is
by
1.46
(A) 1.46 (B) 2.05
(B) 2.05 (C) 2.62
(C) 2.62 (D) 4.32
(D) 4.32
15.)

MN

www.examrace.com
www.examrace.com
2011
2011 MINING ENGG. —
MINING ENGG. – MN
MN

Q.7 A mine winder cage traveling 450 m from pit bottom to pit top is following a three period duty
figure below. The maximum velocity attained by the cage in m/s is
cycle as shown in the figure

a":

Velocity (m/s)
b
13d)
>

'4.

a
'

10 s
:

40
a
:

40ss —>|‘—10s—>|
10 s
. "'*
1‘ ._ E"

|‘—10s—>|‘— l
.
Tlme
Time 1: ' /1

(A) 7.5 (B) 9.0 11.0


(C) 11.0 12'0
(D) 12.0 r
1" I

Q.8 Stress concentration at a point on the wall of a vertical shaft results-din


results compiiessive stress of
I in a compressive
59.66 MPa. The wall rock mass has an unconfined Mfa. The safety
stre tlf of 89.49 MPa.
unconfined compressive strength
factor of the shaft wall at the point is I‘ x Y?
1
(A) 0.67
(A) 0.67 (B) 0.86
(B) 0.86 .2__3
(C) 1.23 1" (‘D) 1.50
(D) 1.50

Q.9 of 54 mm diameter having Youn


A core sample of Young’ss modulus 0 68.97 GPa fails in uniaxial
mod lus of
loa at failure
compression at 0.1% axial strain. The axial load failu in kN is
158.00
(A) 158.00 (B) 68.97 58.
(C) 58.00 15.80
(D) 15.80

Q. 10
Q.10 The maximum number of coal faces in an underground bo
bord and 'plllar 13.
pillar development district is 13.
The number of in‘dn
of headings in the district is k
(A) 33
(A) (B) 55
(B) 1 (C) 66
(C) . (D) 77
(D)
¥1
Q.11
Q-11 tll'e" line AB is 116
The whole circle bearing of the 0
2020 . If
1 16°20'20”. If there exists an east declination of 20
2000 , the
of line AB is
true quadrantal bearing of
“In .' line
(A) S41
(A) 841059'40”E
0
5940E (Brs43039'40"E
(B) S4303940E (C) S45
(C) S45059'40”W
0
5940W (D) S47
(D) s47°59'40”w
0
5940W
(
Q.12 It is proposed to
t4 connect 'two of a road by a simple circular curve. If the maximum speed
two straights of
1/4, the minimum radius of the
of the vehicle is 60 km/h and the centrifugal ratio for the road is 1/4,
of
- - guwe m is 2 x
in~m
curve in
J
( 113.26
(A) 113.26 \\ (B) 98.18 (C) 25.46 15.50
(D) 15.50
3
Q. 13 _ A centrifugal fan
Q.13 fa rotating at 500 rpm delivers 70 m
m3/s of air. If the speed is reduced to 200 rpm, the
/s of
. Quantity of air
quantity of air delivered in m
d 'livered in m3/s
3
/s will
will be
be
Ir
(A)) 175
175 i (B) 55 (C) 28 11
(D) 11

Q. 14
Q.14 ording to mine regulations, the value of
According of the fleet
fleet angle α,
oz, in degree of a drum winder
in allation lies in the range of
installation
1.’5<0t£2.0
(A) 1.5<α≤2.0 0<01 ≤1.5
(B) 0<α 31.5 (C) 2.0< α(x 32.5
≤2.5 2.5<0t $3.0
(D) 2.5<α ≤3.0

Q. 15
Q.15 Water will not be delivered by a centrifugal pump due to
of priming
(A) lack of (B) too low discharge head
(C) wrong direction of rotation (D) partial obstruction at discharge outlet

MN
MN

www.examrace.com
www.examrace.com
2011
2011 MINING ENGG. —
– MN

Q16
Q.16 Match the following

Mine car type Mode of unloading

P. Granby 1. Bottom opening


1.
Q. Gable bottom 2. Both side tilting
R. Drop bottom 3. Single side opening
S. Rocker dump 4. Both side opening
S-l
(A) P-2, Q-4, R-3, S-1 Q-l, R-3, S-2
(B) P-4, Q-1,
Q-l, R-4, S-2
(C) P-3, Q-1, R-l, S-2
(D) P-3, Q-4, R-1,
1.
*1.
Q.17
Q.17 of a 450 m deep downcast shaft
Mean air temperature of is 29 00C
shaft1s C and that of the upcast shaft is 37 00C.
shal't‘ls C.
of the motive column
The height of column1nin m is .1 I
'
/ 1|
(A) 8.2 (B) 9.5 11.6
(C) 11.6 (D) 12.8 -

12.8 r.
Q18
Q.18 The total pressure and the static pressure measured at a point in a ventilation duct
dfict are 20 mm and
3
10 mm of
10 of water gauge respectively. IfIf density of
of air is 1.2
1.2 kg/m
kg/m3,, the velocity of
of die
the air in m/s is
(A) 14.08
(A) 14.08 (B) 12.78
(B) 12.78 (C) 8.53
(C) 8.53 624
(D) 6.24
K (velocity
{05(-
Q.19
Q.19 fire extinguisher that must NOT be us
of fire
The type of used in case of fire
casedb
.I'
firein1n an electric substation located
in an underground metal mine is 'I

(A) multi-purpose dry chemical extinguisher B) CO2 snow


(B) no extinguisher
(C) dry chemical powder extinguisher e nguisher
foa extinguisher
(D) foam

Q20
Q.20 ISO 9000
ISO 9000 Quality
Quality Systems
Systems DO NOT contain
contain /
(A) legal provisions (B) measurembfit‘L
measurement docufent control
(C) document (D) standardization

Q21
Q.21 Air samples collected from the intakee and the return of a retreating longwall face show
return gates of
methane concentration valuestlilof fl.
0.11 % and 0.8 % respectively. The production from the longwall
face is 2000 tonne/day and thefife air quantity circulating the face is 15 m3/s. The rate of methane
15 m3/s.
m33 per tonne-(of
emission in m tonne of coal produced is
t r

11.0
(A) 11.0 (B)-'9-.5
(B) 9.5 (C) 5.5 (D) 4.5
(
Q22
Q.22 The time study data ofof an equipment
equipment deployed in a mine during a calendar month is given below.
vata
workiii'g dours
Total working hours == 400
Total maintenance
maintenan hours = 100
urs = 100
Total hours of
of actual
ac al work = 240
ork =

The.percentage
_ The _ utilization of the equipment is
percentage of
(A) 85
(A) 85 (B) 80
(B) 80 (C) 65
(C) 65 (D) 60
(D) 60
,
Q23
Q.23 1000 ml of
of waste water is allowed to evaporate in a dish weighing 48.6232 g. The weight of the dish
waste”water
with dry solids is 48.6432 g. The concentration of dry solids in waste water in mg/l is
(ATZPO
(A) 200 (B) 220
(B) 220 (C) 260
(C) 260 (D) 320
(D) 320
Q24
Q.24 A longwall face cut by double back shuffle
shuffle method can be only worked with
(A) fixed drum shearer (B) single ended ranging drum shearer
(C) double ended ranging drum shearer (D) plough

MN
MN

www.examrace.com
www.examrace.com
2011
2011 MINING ENGG. —
MINING ENGG. – MN
MN

Q.25 of 50 g of
Proximate analysis of of a coal sample shows the following:

Moisture = 0.80 g
=
Ash = 7.85 g
=
Volatile matter = 15.90
= 15.90 g

fixed carbon in percentage on a dry, ash free basis is


The fixed
(A) 83 (B) 66 (C) 55 (D) 45

'4. .
Q. 26 to Q. 55 carry two marks each. *
!
Q.26 of striking an oil reservoir is 11 out of 15.
For an oil exploration drilling, chance of 15. If an oil exploration
company decides to explore 5 sites, the probability of striking at least one successful oi'l'expgation
oil res rvo‘ir is
reservoir
(A) 0.292 (B) 0.250 (C) 0.034 succe?ful
(D) 9.; 024
0.0024 I r
. . J 1
Q.27 Product of
of the eigen values of
of the matrix A is --. .3 ‘-
' "1 r-
 33 22 55  ., (
_—
AA =  22 22 11  1K
. ' 1
'II
J

 11 5 4  '-
 
(A) 6 (B) 8 C) 10
(C) 10 (D) 35

Q-28
Q.28 .y=
dy = 0.1 r .
equatlon
For the equation  22xx 
+33y ofyy at xx =
y , the value of 0.1 in
1 one step using Runge-Kutta fourth
d—
dxx ste‘plusmg
-

order method for the condition yy  fiixx 


=11 when = 0 ,, is
t
(A) 0.3608 (B) 1.2508 ( 1.3608
(C) 1.3608 1.4625
(D) 1.4625

Q.29 1 11+ )2" '1


1.25‘08
Q29 1

‘f '
x
Value of
of the integral J-  1  xx
dx is
no0 fidx
:1:
7r 
(A) 1 (3)125“
(B)  1 (C) 72—1
(C) 1 (D) n+1
(D) 1
(ME—1
2 2
1
Q.30- A
Q.30 A '11 tonne mine
miné carr traveling at a constant speed of 1010 km/h collides with a stationary buffer and

(.
comes to rest. If uffer spring stiffness is 200 kN/m, the maximum compression in the spring in
If thee buffer
mm is R
__(A)
(A) :19
49 . (B) 98
(B) 98 (C) 196
(C) 196 (D) 247
(D) 247

.1

MN

www.examrace.com
www.examrace.com
2011
2011 MINING ENGG. —
– MN

Q.31
Q.31 In an iron ore handling port, a barge is pulled by ropes using two tugboats as shown in the figure.
figure. In
equilibrium, the resultant of
of the forces TT11 and TT22 along the axis
aXis of the barge in the direction of its
travel is 5000 N. The tensions T T11 and T
T22 in N respectively are
Tugboat 1

T1
300
0
30
Barge Q — — — — -> Direction of travel
45°0
45
T
T22
'4.
. 1.
Tugboat 2 C 'l .

'I
1: / 1.
''
(A) 9700
(A) 9700 and
and 6831
6831 (B) 6831
(B) 6831 and
and 9700
9700 r‘
(C) 3660
(C) 3660 and
and 2588
2588 (D) 2588
(D) 2588 and
and 3660
3660 .. r .
4| 1
.1 3
Q32
Q.32 flat belt conveyor is carrying coal of bulk density 11 tonne/m
A flat tonne/m3 at rate of 400 tofi'ne/h.
af‘aa rate'of tonne/h. The belt
speed is 3 m/s. Coal is spread over the belt covering 80% of the bel width in a shape of
belt widt' of a triangle.
If the pile height is 1/4
If 1/4 of
of the belt width, the width of
of theeibel
belt in mm is
(fithe I
J

1109
(A) 1109 (B) 909 (
(C) 0.
709 (D) 609

Q.33 Match the following.

Hydraulic system components . Symbols


'
P Fixed displacement unidirectional flow pump 1
*3:
r
"
Q Fixed displacement unidir‘itro'nlflow
unidirectional flow motor 2
<5

1I
1'
R Accumulator 1.. 3 D
'.
Ii
1'.

.
S Filter ‘ 4
a

if i $
' (A)P-4,‘Q-2,R-3, —1
(A) P-4, Q-2, R-3, S-1 (B) P-2,
(B) 13—2, Q-4,
Q-4, R-3,
R3, S-1
81
( (C)
(C) P-3,
13—3, Q-2,
Q-2, R-1,
Riga-1‘
S-4 (D) P-2,
(D) P-2, Q-3,
Q—3,R—1,s—4
R-1, S-4

Q.34 ‘.Match follow ng


Match the following
*
Method of mining
mi ing Stope support Ore loading

P.. Shrinkage stoping


oping 1. Insitu pillar
1. a. Overhead mucker
Q. Blasthole stoping 2. Broken ore b. Pneumatic autoloader
R. Top slicing 3. Timber mat c. Load haul dumper
ReTfip
Q-l-c, R-3-b
(A)P-2-a, Q-1-c, R-l-b
(B) P-2-a, Q-3-c, R-1-b
R-l-a
(C) P-2-b, Q-3-c, R-1-a R—l-b
(D) P-3-c, Q-2-a, R-1-b

MN

www.examrace.com
www.examrace.com
2011
2011 MINING ENGG. —
MINING ENGG. – MN
MN

Q35
Q.35 flat strata is shown in the figure
A typical case of gravity loading under complete lateral restraint in flat figure
below. The physico-mechanical parameters of the strata are given in the table. The in in situ
situ stresses
 Z ,, 
((OZ 6HH ) on the top of
of the coal seam in MPa are

Surface
/ /><><><\><><// /
10 MPa
10

/////////////////////.39998199///W
Sandstone
_ ‘ a,
Shale
Coal r '
1' I
of the strata
Cross-section of /‘

Strata Thickness Specific


Specific Young’s
l
Shear 4' "-
(m)
(m) Gravity
Gravity Modulus (GPa)
Modulus (GPa)‘ .' Modulus
Modulus (GPa)
(G'Pa)
Sandstone
Sandstone 50
50 2.35
2.35 26.40 1
26.40 f 12.5
1235
Shale
Shale 25
25 2.15
2.15 .51) ‘1
20.50
' 8.25
8.125
Coal
Coal 20
20 1.52
1.52 .41 “' 1
2.41 0.95
0.95
I
(A) (10.17, 2.54) (B) (10.17, 3.69) (C)) (11.68,
(11 68, 3.69)
3 9) (D) (11.68, 2.54)

Q.36 The sale value of


of chromite ore from an open
ope pit mine
mi i Rs. 6500 per tonne. Cost of mining,
is
- t of stripping is Rs. 1150
excluding stripping cost, is Rs. 2450 per tonne. If the cost m3, the
1150 per m3,
3
m3/tonne
breakeven stripping ratio in m /tonne is _
(A) 2.18
(A) 2.18 (B) 3.52
(B) 3.52 '1 K. (C) 3.65
(C) 3.65 I (D) 4.25
(D) 4.25
Q.37 10% yearly1
An investment at 10% 6%t rate,
yearly interest ate, compounded quarterly, accumulates to a sum of
120,000in
Rs. 120,000 in 5 years. The pre it value of the sum in rupees is
present
(A) 72,233 74,511
(B) 74,511 (C) 88,232 106,063
(D) 106,063
I- .
qt _ L
Q38
Q.38 A toxic gas flows
flows into
int aamine m3/min. The concentration of the
mine working place at the rate of 2.52 m3/min.
3
i 0.25%.
gas in the intake air is 0.25 0. The minimum quantity ofof intake air in m
m3/min
/min required to dilute the
gas to its threshifld of 1.0
threshold limit value of 1.0 % is
(A) 123
(A) 123 '
i j“ (B) 252
(B) 252 (C) 295
(C) 295 (D) 333
(D) 333
.— "~.._
Q.39 An exhaust fan attached
g to an evasee ofof 18
18 mm22 cross-sectional area at the outlet circulates 150 m3/s
150 m3/s
of air at the pres
of aayd
re off 1000
pressure 1000 Pa in a mine ventilation system. The ratio of the inlet to outlet area of
3
1:4 and the density of air is 1.2
the evasee is 1:4 1.2 kg/m
kg/m3. . The quantity of air circulated in the mine in
3
_ absence ofof evasee is 120
120 mm3/s. efficiency in % is
/s. The evasee efficiency
J")576
(A) 57.6 evasje (B) 43.2
(B) 43.2 (C) 39.06
(C) 39.06 (D) 37.7
(D) 37.7
3
Q.40 an circulates 24 m
A fan m3/s of air at a pressure of 1200
/s of 1200 Pa in a ventilation district. It is intended to
3
re ce the air quantity to 16
reduce 16 mm3/s/s by placing a regulator. Assuming the pressure remains
unctged, the size of
unchanged, of the regulator
regulatorin mby
in m22
is
1.48
(A) 1.48 (B) 0.74 (C) 0.37 (D) 0.18

MN
MN

www.examrace.com
www.examrace.com
2011
2011 MINING ENGG. —
– MN

Q.41
Q.41 An air sample taken from the return airway of a district contains the following gases. The Graham’s
ratio for the district is

Gas Concentration (%)


C022
CO 0.40
H44
H 1.17
1.17
022
O 19.92
19.92
N2
N2 78.49
CO 0.02
(A) 5.6 (B) 4.8 (C) 3.0 (D) 2.3
1.
“H.
Q.42
Q.42 vehicle1s
An incandescent headlight of a mining vehicle is of spot beam type with a beam angle of
0 30
0
0OgThe
. The
2
surface1n
spherical surface in m2
m subtended by the lighted beam at a distance of
of 5 m from the headlight is I
the headliglyé 1|
(A) 7.5 15
(B) 15 21
(C) 21 (D) 25
25'r -r.
Q.43
Q.43 semi--variogr'5'm m
The thickness of a coal deposit is represented by a spherical semi-variogram (1‘21 with sill of
model
The22 spherical
2
5mm2.. If the semi-variogram value at lag distance h
h1s
is 3 m2,
m , the correlogram valu
value at the same lag
distance1s
distance is correlogxam
(A) 0.4
(A) 0.4 (B) 2.0
(B) 2.0 (C) 2.55'
(C) k ((D) 5.0
(D) 5.0
all
'
Q.44
Q.44 C (lakh rupees) of
The total cost C len h L
of a longwall face of length L in m is given by the equation
2 0.1
1562.5
1 562.5 '
C
C L
0.1L + 
+ 300 . Length of the face in m for thee m' imum total cost is
minimum
L
(A) 40
(A) 40 (B) 125
(B) 125 (C) 156
(C) 156 * (D) 300
(D) 300
Q.45
Q.45 20 plain detonators
detonators1n of 2
in series, each of He resistance, are fired by a DC exploder supplying a current
of 1.25
of 1.25 A. If
If 250 mJ
111] energy is spent to fire
gsistance,
e the detonators,
detonat{fed
s the time required1n
required in millisecond after
$1.130
detonator initiation is
(A) 44
(A) (B) 88).:
(B) (C) 12
(C) 12 (D) 16
(D) 16
Q.46
Q.46 A sudden increase of
of‘COi cidence has occurred in an underground mine section. A man at point A
CO incidence
starts to run out to th
the n'1ain._intake
main intake of of the mine where he will be safe. Refer figure
figure below for the
mine section and the logic
ogic diagram. The probabilities that he will successfully cross the gallery
sections A, B, C, D, E, and F are 0.9, 0.8, 0.7, 0.8, 0.7 and 0.9 respectively. The probability that he
will successfully
successfu11. reach the main intake is
-I
3,
fl Intake
I
F

.- __ I" It
D
<—o(:]I :
H
J E

l E /‘\
B

- —- I-I-I
'1 c

.fl.
C f
A I3]
B E
D
A F
C E

of the mine
Section of Series-parallel logic diagram

(A) 0.40 0.51


(B) 0.51 (C) 0.66 (D) 0.77

MN

www.examrace.com
www.examrace.com
2011
2011 MINING ENGG. —
– MN

Q.47 In an underground correlation survey by the Weisbach triangle (figure


(figure below) the following data
are obtained.
AB == 3.50
AB 3.50 m, BC == 2.75
m, BC 2.75 m, CA == 6.20
m, CA 6.20 m,
m, ACD
ˆ 
ACD = 179
179°14'33",
1433 , BCD
ˆ 
BCD = 179
179°10'17" and bearing
1017 and of
bearing of
AB ==115°23'49”
1152349 . The bearing of
of traverse CD is

D
D
North
B
B
W

A
A C
C
'4. .

(A) 102
102°27'16"
2716 (B) 114
114041'49”
41 49 -* I
1 .
(A) (B)
(C) 115
115°27'16” (D) 179
179014'16" /
.
(C) 2716 (D) 1416 ,
r. 1':
III" I

Common Data Questions . 4 1


..._ 1-

1x
- ‘- F
Common Data for Questions 48 and 49: "
1
(
A concentrator pilot plant is fed with 11 tonne of
of copper ore
or att ROM
ROMlgrade of 1.5
grade of 1.5" % Cu. Metal recovery in
the concentrator pilot plant is 90% and the grade of copper
copp in concentrate
c" ncent te is 20%.

Q.48 of copper in concentrate in kg is


The amount of
L
(A) 13.5
(A) 13.5 (B) 14.0
(B) 14.0 (C) 14.5
(C) 14.5 (D) 15.0
(D) 15.0
_/
Q.49 of concentrate produced from
Amount of 1 tonne of ore in kg
fromfitfine kg is
(A) 75.0
(A) 75.0 (B) 72.0
(B) 72.0 1 J 70.0.
(C) 70.0
(C) (D) 67.5
(D) 67.5
Common Data for Questions 50 and 51:
and—516

Ns2m-88 and 3.2 sm


. . . '1 1 . . . _ _
m1ne ventilation
A mine vent11atlon system consists of two splits
spl1ts A and B w1th
with res1stances of 0.8 Ns2m
resistances of Ns2m-88,,
2 -8
respectively as shown in figure. IITrunk
Trunk airways have resistance ofof 0.2 sm'g.
Ns m . The main mine fan is
generating pressure of
of.500 fiqire. (-
Pa..
500 Pa.
Split A
' I
1' '
. . ‘ Trunk airway Trunk airway
{runk>
-

—>—
J

( \\

’ Split B
3
Q.50 lhre m3/s
The air quantities in m /s circulated in the splits A and B respectively are

(A)) 20 and 30
quantitijs
(B) 30 and 20 10
(C) 20 and 10 (D) 40 and 10
10

Q.51
Q.51 T flows
The flows in the two splits are equalized by placing a booster fan in split B. Assume that the fan
prE'Ssrre of the booster fan. The size of
pressure does not change after installation of of the booster fan in Pa is
(A) 749.05
(A) 749.05 (B) 850.08
(B) 850.08 (C) 950.02
(C) 950.02 (D) 1000.50
(D) 1000.50

MN

www.examrace.com
www.examrace.com
2011
2011 MINING ENGG. —
– MN

Linked Answer Questions


Statement for Linked Answer Questions 52 and 53:

A 400 V, 3 phase, star connected induction motor takes a line current of 10


10 A with 0.86 p.f. lagging. Total
stator losses are 5 % ofof the input. Rotor copper losses are 4 % of
of the input to the rotor, and mechanical
of the input to the rotor.
losses are 3 % of
Q52
Q.52 The input power to the rotor in Watts is

(A) 5958 (B) 5788 (C) 5660 (D) 5532


4 1.
Q53
Q.53 The shaft output power in Watts is (1 1
(A) 5562
5562 (B) 5490
5490 (C) 5434
5434 (D) 5264 '*
(A) (B) (C) (D) 5264 , /1' .'
1
Statement for Linked Answer Questions 54 and 55: ( r
1" |

The bolts are spaced at 1.5


1. 5 m centre-to-centre in a square pattern as shown
shown1n in theeflgure
figure belbw.
below. The tensile
stress in 22 mm diameter bolt rod
rod1s 193. 35 MPa. The unit weight of
is 193.35 of the roof layer is kN/m3.
(S 25 kin3.

+ + '51
\rooila'yg'r
[<— 1.5m #r-‘é

+ +
1.5m

a"
4L

+ *£+ f
“31:“;
Plan of rock bolting pattern
view of
Q54
Q.54 The axial load in the bolt rod'l'h
rod in kN is

(A) 294.0
(A) 294.0 a (B)173.5
(13)“173.5 147.0
(C) 147.0
(C) (D) 73.5
(D) 73.5
Q. 55
Q.55 thtknes‘;of
At equilibrium, the thickness ofthe
the roof
roof layer supported by the bolt1n
bolt in m is
1.31
(A) 1.31
. J
,‘he

(B) 2.4 (C) 2.62 (D) 3.08

_(_ "x

l'

MN

www.examrace.com
www.cxamracecom
2011
2011

General Aptitude (GA) Questions

Q. 56 —
– Q. 60 carry one mark each.
Q56
Q.56 Choose the word from the options given below that is most nearly opposite in meaning to the given
word:
Deference
(A) aversion
(B) resignation
(B) resignation - *1.
(C) suspicion r 1
(D) contempt 'I *5
"I
/ 1.
Q57
Q.57 Choose the most appropriate word(s) from the options given below to complete
corfiflete the following
folling
sentence. .r I
We lost confidence
confidence in him because he never __________ the grandiose promises
promis$ heh'e had made.
grandiose
: F
(A) delivered -" -.

(B) delivered on 1 k (
(C) forgot I. . ‘
(D) reneged on I

Q58
Q.58 Choose the word or phrase that best completes thee sentence
sente ce b low.
below.
______________ in the frozen wastes of ArcArcticc takes special
ec 1 equipment.
"
(A) To survive
(B) Surviving _ {f
(C) Survival ‘4‘
(D) That survival k ‘
1
Q.59
Q.59 In how many ways 3 scholarships
scholarshi‘flcan
'
can bee awarded to 4 applicants, when each applicant can receive
any number of scholarships? .-
(A) 44
(A) (B112
(B) 12 (C) 64
(C) 64 (D) 81
(D) 81
Q.60
Q.60 ap‘rozprifte word from the options given below to complete the following
Choose the most appropriate
sentence.
The _________ . ‘ of eviden ce was on the side of the plaintiff since all but one witness testified
testified
that —storfias
his story was correct.
"h.
(A) paucity ""
(B) propensity \\
(C) preponderan
preponderance
' (D) accuracy
I
IF

_.I"

Q. 61 to
t Q. 65 carry two marks each.
carry,rtwo

Q.61
Q.61 If -.. /(y+2) < 1,
y+1)/(y+2)
(2y+1) 1, then which of the following alternatives gives the CORRECT range of y?

(A)—E<y<2
(A) -2<y<2 (B)-2<y<1
(B) -2<y<1 (C)-3<y<1
(C) -3<y<1 (D)-4<y<l
(D) -4<y<1

Q.62
Q.62 A student attempted to solve a quadratic equation in xx twice. However, in the firstfirst attempt, he
incorrectly wrote the constant term and ended up with the roots as (4, 3). In the second attempt, he
incorrectly wrote down the coefficient
coefficient of
of xx and got the roots as (3, 2). Based on the above
information, the roots of the correct quadratic equation are

(A) (-3,
(A) (-3, 4)
4) (B) (3,
(B) (3, -4)
-4) (C) (6,
(C) (6, 1)
1) (D) (4,
(D) (4, 2)
2)

www.examrace.com
www.examrace.com
2011
2011

Q.63 L, M and N are waiting in a queue meant for children to enter the zoo. There are 5 children between
L and M, and 8 children between M and N. If there are 3 children ahead of N and 21 21 children
behind L, then what is the minimum number of children in the queue?
(A) 28 (B) 27 41
(C) 41 (D) 40

Q.64 Four archers P, Q, R and S try to hit a bull’s eye during a tournament consisting of seven rounds.
figure below, a player receives 10
As illustrated in the figure 10 points for hitting the bulls’
bulls’ eye, 5 points for
hitting within the inner circle and 11 point for hitting within the outer circle.

Outer circle
11 point

Inner circle
5 points

Bull’s eye
10 points
10

_/
final scores received by the player-alking
The final players during the tournament
tournatfient are listed in the table below.

R0 d
Round PP Q
Q 'R
R s
S
__ i 11 11 55 11 1o
10
2
2 55 1o
10 10
10 11
1 14;. 3
3 11 11 11 55
w. 4
4 1o
10 10
10 11 11
. ( 5 11 5 5 1o
- 1'
5 5 5 10
1 6
6 1o
10 55 11 11
7
7 55 1o
10 11 11
"1‘
The most accurate
accur and the most consistent players during the tournament are respectively
( (A) P and S 7"hang (B) Q and R (C) Q and Q (D) R and Q

Nimbus clouds are dark and ragged, stratus clouds appear dull in colour aand
Q.65 ',_Ninibus nd cover the entire
sky.
s15. Cirrus clouds
clofids are thin and delicate, whereas cumulus clouds look like cotton balls.

It can be inferred
inferrd from the passage that
(A A cumulus cloud on the ground is called fog
(A)
1t
(B) It is easy to predict the weather by studying clouds
(C) Clouds are generally of of very different shapes, sizes and mass
(D) There are four basic cloud types: stratus, nimbus, cumulus and cirrus

END OF THE QUESTION PAPER

www.examrace.com
www.examrace.com
GATE 2011 -- Answer
GATE 2011 Key -- Paper
Answer Key Paper :: MN
MN

Paper
Paper Question no.
Question no. Key
Key Paper Question no.
Question no. Key
MN
MN 11 AA MN 41
41 D
MN
MN 2
2 MarkstoAll
MarkstoAll MN 42
42 C
MN
MN 3
3 B
B MN 43
43 A
MN
MN 4
4 AA MN 44
44 B
MN
MN 5
5 DD MN 45
45 A
MN
MN 6
6 CC MN 46
46 C
MN
MN 7
7 B
B MN 47
47 B
MN
MN 8
8 DD MN 48
48 A
MN
MN 9
9 AA MN 49
49 U>UJ
D
MN
MN 10
10 B
B MN 5O
50 C
MN
MN 11
11 MarkstoAll
MarkstoAll MN 51
51 A
A
MN
MN 12
12 AA MN 52 C
C
MN
MN 13
13 CC MN 53" '-
53 D
D
MN
MN 14
14 B
B MN . 54
54 D
D
MN
MN 15
15 AA MN 55
55 A
A
MN
MN 16
16 DD MN 56
56 D
D
MN
MN 17
17 CC MN 57
57 B
B
MN
MN 18
18 B
B MN 58
58 A
A
MN
MN 19
19 DD MN 59
59 C
C
MN
MN 20
20 AA MN 6O
60 C
C
MN
MN 21
21 DD MN 61
61 B
B
MN
MN 22
22 B
B MN 62
62 C
C
MN
MN 23
23 AA MN 63
63 A
A
MN
MN 24
24 CC MN 64
64 B
B
MN
MN 25
25 Markstfi
MarkstoAll MN 65
65 D
D
MN
MN 26
26 AA IL
MN
MN 27
27 DBI"
MN
MN 28
28 J CC
MN
MN 29
29 .. _' i B B
MN
MN 30
30 “'GC
MN
MN 31-
31 A {CC
MN
MN 321
32 'D
D
MN
MN 33-1
33 B
B
_MN
MN ’34!
34 AA
MN'
MN 35 1:1
35 CC
MIN
MN 36 I
36 B
B
‘MN
MN 37 I
37 AA
MN
MN 38 J
38 DD
MN._
MN 39 1'
39 AA
MN
MN 40 1
40 B
B
1
1'

www.examrace.com
www.examrace.com
GATE 2012 Online Examination
MN : MINING ENGINEERING
Duration: Three Hours
Duration: Maximum Marks: 100
Maximum Marks: 100

Read the
Read the following
following instructions
instructions carefully.
carefully.

1.
I. The computer allotted to you at the examination center runs a specialized software that permits only
one answer to be selected for multiple choice questions using a mouse. Your answers shall be updated
and saved on a server periodically and at the end of
of the examination.
2. To login, enter your Registration Number and password provided in the envelope. Go through the
symbols used in the test and understand the meaning before you start the examination.
examination.‘ You can view
an View
all questions by clicking on the View All Questions button in the screen after the th start of the
stai't .of
"F '
examination. 1 1
3. To answer a question, select the question using the selection panel on the screen een and choose
oh se the
correct answer by clicking on the radio button next to the answer. To change t];
the answer, just click on
answer,Ijust
another option. If
If you wish to leave a previously answered question unanswered, click
gliok on the button
next to the selected option. "I .' "
F
I! .

4. The examination will automatically stop at the end of


of 3 houlé‘
hours. {

5. There are a total of
of 65 questions carrying 100
100 marks.
m as. Except Q26 —
Exce'p questions Q.26 – Q.30, all the other
questions are of
of multiple choice type with only onee correct
co ect answer. Q26 -— Q.30 require a
a wer. Questions Q.26
numerical answer, and a number should be entered
enter (1 using the virtual
he v' ual keyboard on the monitor.
Q.l —
6. Questions Q.1 – Q.25 carry 11 mark each. Questions Q.26 Q. 6 – Q.55 carry 2 marks each. The 2 marks
questions include two pairs of common data questions and a two 0 pairs of
of linked answer questions. The
answer to the second question of of the linked answer questions depends on the answer to the first first
question of
of the pair. If
If the first
first question in the linked pair is
questionfll i wrongly answered or is unattempted, then
the answer to the second question in the pair will not be evaluated.
atwill ev luated.
7. Questions Q.56 —– Q.65 belon
belong to General
neral Aptitude (GA) section and carry a total of 15
15 marks.
Questions Q.56 —
– Q.60 carry 11 mark
_ k each, and questions Q.61
Q.6l —
– Q.65 carry 2 marks each.

8. Unattempted questions will result in zero mark and wrong answers will result in NEGATIVE NEGATIVE marks.
There is no negative marking
l'lilarkiilflgJ for questions of Q26 —
of numerical answer type, i.e., for Q.26 – Q.30. For all 11
mark questions, ⅓ % mark
ma(k wil will be deducted for each wrong answer. For all 2 marks questions, ⅔ 2/3 mark
will be deducted
deducte for each wrong
'ong answer. However, in the case of the linked answer question pair,
there will be negative
ne ative marks only for wrong answer to the first first question and no negative marks for
wrong answer
answer! to the second question.
tithe
9.
9 Calculator is allowed.
alloy?" Charts, graph sheets or tables are NOT
NOT allowed in the examination hall. Do the
( rough work in the Scribble
cri ble Pad provided.
10.
10. You must sign this
thi sheet and leave it with the invigilators at the end of
of the examination.
________________________________________________________________________________________
. I

DECLA TION: I herglby


DECLARATION: hereby declare that I have read and followed all the instructions given in this sheet.
I
Registration Number MN
Name
Name

Signature
Signature

Verified
Verified that the above entries are correct.
Invigilator’s
Invigilator’ s signature:

www.examrace.com
\MNw.examrace.com
2012
2012 MINING ENGG. —
MINING ENGG. – MN
MN

Q. 1—
Q. 1 Q. 25
– Q. 25 carry
carry one
one mark
mark each.
each.

Q.1
Q.1 A 30 m
A 30 m steel
steel tape having an
tape having an area
area of
of cross-section
cross-section of
of 5
5 X 10'6 m2
x 10-6
is standardized
m2 is standardized at
at 20
200C,
0
supported
C, supported
under
under aa tension
tension of
of 5.45
5.45 N. It is
N. It is used to measure
used to measure aa horizontal distance of
horizontal distance of 81.15
81.15 m
m under an applied
under an applied
tension
tension of of 9.09
9.09 N. The error,
N. The error, due
due to incorrect pulling
to incorrect arrangement in
pulling arrangement in this in m
observation, in
this observation, m is
is
(Esme):
(E 200 GPa)
steel = 200 GPa)
(A) 0.148
(A) 0.148 (B) 0.295
(B) 0.295 (C) 1.820
(C) 1.820 (D) 3.640
(D) 3.640
Q2
Q.2 The coefficient
The coefficient of
of variation of aa dataset
variation of dataset is
is measured
measured by
by
mean mean 1 -
(A) - - B
(B) .— . “a
dev1atlon
standard deviation varlance
variance r 1
. . . a-r |
standard deviation variance ,
(C)
(C) (D)
(D) /
standaldecieylatlon
mean v:1::::e
mean ‘_"
':
Q3
1-" I
Q.3 11
sin −1 ( cos x ) dx is
∫ ‘- 1.
II I!

The value
The value of
of [sin—1(cosx)dx is .. *1
0

((yr—1)
)
π −1 (mi)
(
π +1 ) ( 2π + 1) 1 ((272—1)
2π − 1)
(A ) —
(A) (B ) —
(B) (C) (D)
(D)
22 22 2 22

Q4
Q.4 Assuming Sin(l) = 0.841
Assuming sin(1) 0.84land Sin(3) = 0.141
and sin(3) 0.141 ,, the
he Lagrangian
Lag ngi linear
linear interpolating
interpolating polynomial, for
polynomial, for
the function f(x)
the function = sin(x)
f ( x) sin( defined on
x) defined on the
the interval
int rval [1,
[1, 3] and
a passing
passing through
through the end points
the end of the
points of the
1nterval, is
interval, 1s
/
(A)
(A) − 0.35 x + 1.19
—O.35x+1.19
(B)
(B) —3.05x+11.92
− 3.05 x + 11.92 it l
(C)
(C) —35.00x+ll9.lO
− 35.00 x + 119.10 1 J
(D)
(D) —40.50x+219.19
− 40.50 x + 219.19 L;
Q5
Q.5 If Poisson’s
If Poisson’s ratio
ratio of
of aa rock
rqclg sample
sample is
is 0.25,
0.25, then the relationship
then the among the
relationship among the modulus of elasticity
modulus of elasticity
(E), modulus
(E), modulus of
of rigidity
rigidity (G)
(G2 and
and bulk
bulk modulus (K) is
modulus (K) is

(A)E=K=G
(A) E=K=G
.(B)(E>G>K
(B) E > G > K (C) EE=G>K
(C) =G>K (D) EE>K>G
(D) >K>G

Q6
Q.6 The 22nd
The nd
ordefd'
order ferential equation
differential equation having solution yy = ((A
having aa solution x) + B
A // x) B ,, where
where A and B
A and B are
are constants,
constants,
.iS
is ‘ x
J'
d == 00
(A) dd yy ++ 32 dy
22
(A)
dydx xx dx d
'
dydx
ddzy
2
y — 22 dy
d = 00
(ET
(B) +
a’x2 xx dx
dx 2
d

< —5 ——=o
2
⎛ dd22 y ⎞ - 2 dy
d
(C) ⎜ 2 ⎟ + =0
⎝ dx ⎠ xdx x dx
‘-
d 22 y d 22 y dy
(D) 5y ++dy+fl20
(D) + =0
dydx dx
dydx a’x22 dxdx

Q.7
Q.7 A cylindrical rock
A cylindrical rock specimen
specimen is is uniaXially loaded under
uniaxially loaded compression and
under compression and fails
fails at
at 50
50 MPa.
MPa. The
The
fracture plane
fracture is inclined
plane is inclined at
at an
an angle
angle of
of 45
450 with
0
with the
the axial
axial direction.
direction. The
The normal and shear
normal and shear stresses
stresses
respectively on
respectively on the
the failure
failure plane in MPa
plane in are
MPa are
(A) 50,
(A) 50, 50
50 (B) 0,
(B) O, 50
50 (C) 50,
(C) 50, 0
O (D) 25,
(D) 25, 25
25

MN
MN 2/14
2/14

www.examrace.com
www.examrace.com
2012
2012 MINING ENGG. —
MINING ENGG. – MN
MN

Q.8
Q.8 A
A uniformly distributed load
uniformly distributed load of
of 20
20 kN/m
kN/m isis acting on aa 15
acting on 15 m long cantilever
m long cantilever beam
beam AB of area
AB of area of
of
cross section
cross section 22 m
m x X 2
2 m,
m, as
as shown
shown inin the
the figure.
figure. The
The beam is fixed
beam is fixed at
at point
point A. The modulus
A. The of
modulus of
elasticity of
elasticity of the
the material
material is
is 1.0
1.0 GPa.
GPa.

2O kN/m
20 kN/m
F \l/ \l/ \l/ \l/ \l/ \l/ \l/ \l/ \l/ \l/ \L v
\L _
/ AA 22A Q
Bl 2m
B
a
“m
I< 15m
15m >1 2ml<
>| 2m
'4.
. . - ”H.
The maximum vertical
The maximum vertical displacement
displacement of
of the
the beam 1n m
beam in m is
1s 1
0004 0.071 (D) 0.190 "*v I
(A) 0.004
(A) (B) 0020
(B) 0.020 (C) 0.071
(C) (D) 0.1;“). (i
Q.9
Q.9 In aa surface
In surface mine,
mine, sound
sound pressure level at
pressure level at aa location
location generated
generated by operation of
by operation (Iaa dozer
dozer and
and E drill
a drill
respectively are 80
respectively are 80 dBA
dBA and
and 60 dBA, when
60 dBA, operated independently.
when operated independently. The
The sound
sou'fid pressure generated
pressure generated
by the dozer
by the dozer compared
compared to
to the drill1s
the drill higher by
is higher by aa factor
factor of
of J
.I"
(A) 10
(A) 10 (B) 20
(B) 20 (C) 100
(C) 109 ( 200
(D)
‘\ {
Q.10
Q.10 As per
As per the Indian Electricity
the Indian Electricity Rules
Rules 1956,
1956, the
the max' m perfi'fssible
maximum length of
permissible length of aa flexible
flexible cable
cable used
used
with an electric
with an electric rope shovel in
rope shovel m is
in m is
(A) 100
(A) 100 (B) 200
(B) 200 C) 300
(C) 30 (D) 500
(D) 500

Q.11
Q.11 The equipment
The equipment that is NOT
that is NOT used in hard
used in hard rock etal mining
rock metal min1 drivage
rivage is
1s
(A) road
(A) road header
header (B) drill
(B) drill jujumbo _/
(C) jack
(C) hammer
jack hammer i (D) dint
(D) h ader
dint header
k
Q.12
Q.12 The roof
The roof bolt that follows
bolt that the pprinciple
follows the in‘p(of of point anchorage1s
point anchorage is
(A) expansion
(A) expansion shell
shell bolt
bolt _. =1 (B) full
(B) full column
column grouted
grouted bolt
bolt
(C) split
(C) split set
set bolt
bolt (D) swellex
(D) swellex bolt
bolt
11.. ,' ‘1;
Q.13
Q.13 Equipment used
Equipment used in (Sf-placer
in mining of deposits is
placer deposits is
' mifing
(A) auger
(A) auger (B) (vagon
(B) drill
wagon drill (C) rope
(C) saw
rope saw (D) riffle
(D) riffle box
box

Q-_14.
Q.14 A dump truck
A dump trufk‘owered
powered by 350 kW
by 350 kW engine
engine is
is running at aa speed
running at speed of
of 35
35 km/h. Considering the
km/h. Considering the
transmission yof
transmission efficiency the truck
of the truck as
as 85%, the rim
85%, the rim pull of the
pull of the truck1n
truck in kN1s
kN is
(A) 21
(A) 21 fifi)
effifi‘laq\ 31
(B) 31 (C) 41
(C) 41 (D) 51
(D) 51

Q.15 - Nystagmus
Q.15 is a miner’s
Nystagmus is iner’ s disease
disease associated
associated with
with
(A)) lever
lever (B) lung
(B) lung (C) eye
(C) eye (D) stomach
(D) stomach
a}
Q.16
Q.16 art from
Apart from mining
min'in g of
of coal,
coal, the
the longwall
longwall mining method has
mining method has been
been practiced for mining
practiced for mining the deposits
the deposits
o
of
in.

(A) copper
(A) cUpper (B) lead
(B) lead and
and zinc
zinc (C) manganese
(C) manganese (D) pyrite
(D) and phosphate
pyrite and phosphate

Q. 17
Q.17 The three
The three segments,
segments, whose synchronous functioning
whose synchronous functioning is
is essential
essential for
for GPS
GPS operations,
operations, are
are
(A) space,
(A) space, control
control and
and user
user (B) signal,
(B) signal, control
control and
and user
user
(C) space,
(C) space, control
control and
and geo-registration
geo-registration (D) signal,
(D) signal, control
control and
and geo-registration
geo-registration

MN
MN 3/14
3/14

www.examrace.com
www.examrace.com
2012
2012 MINING ENGG. —
MINING ENGG. – MN
MN

Q.18
Q.18 When aa double
When double ended
ended ranging
ranging drum
drum shearer
shearer cuts
cuts coal
coal in
in aa longwall
longwall face,
face,
(A) both
(A) the drums
both the drums rotate in the
rotate in the same
same direction
direction keeping
keeping the drum up
front drum
the front and the
up and the rear drum down
rear drum down
(B) both
(B) the drums
both the drums rotate in the
rotate in opposite direction
the opposite direction keeping
keeping the front drum
the front drum up and the
up and the rear drum
rear drum
down
down
(C) both
(C) both the drums rotate
the drums rotate in
in the opposite direction
the opposite direction keeping the front
keeping the front drum
drum down
down andand the
the rear drum
rear drum
up
up
(D) both
(D) the drums
both the drums rotate
rotate in
in the
the same
same direction keeping the
direction keeping front drum
the front drum down
down and
and the
the rear drum up
rear drum up

Q.19
Q.19 The match
The match the
the following
following
Mine gas
Mine gas Principal constituent
Principal constituent 1.,

P
P damp
Stink damp
Stink 11 CO
CO v " .
... I
'
Q
Q damp
White damp
White 22 HZS
H 2S " v/ 1.
R
R Black damp
Black damp 33 CH44
CH r r
SS Fire damp
Fire damp 44 C022
CO l J ‘5
.
F
I! I.

(A) P-1,
(A) P-l, Q-2,
Q-2, R-3,
R-3, S-4
S-4 (B) P-3, Q-4, R-1, S-2 . l
(C) P-2,
(C) P-2, Q-1,
Q-l, R-4,
R-4, S-3
S-3 (D) P-2, Q-1, R-3, S-4
'
Q20
Q.20 Continuous miner
Continuous and shuttle
miner and shuttle car
car combination
combination is NOT a\plic ble in
NOT applicable mining with
in mining with
(A) rib
(A) rib pillar technique
extraction technique
pillar extraction .
(B) Wangawilli
(B) Wangawilli system
system
' (I
(C) room
(C) and pillar
room and method
pillar method =
(D) longwall
(D) longwall method
method ‘4‘ ‘

1'
Q21
Q.21 Contours in
Contours in aa topographic
topographic map 1
map‘
(A) are
(A) are not closed upon
not closed themselVes although
upon themselves although the earth is
the earth is aa continuous
continuous surface
surface
(B) are
(B) are not
not perpendicular.L
perpendicular to to the
the direction
direction of
of maximum slope
maximum slope
(C) provide
(C) provide anan indication
indicationt'o‘l?
of presence of valley
presence of or ridge
valley or in the
ridge in the area
area
(D) are
(D) are the
the lines
lines jjoining the
the"points of same
points of same declination at different
declination at different elevations
elevations
oinir‘g
Q22
Q.22 A Dragger Gas
A Dragger Gas Mask DOES NOT
Mask DOES filter
NOT filter
(A) water
(A) vapo‘r
water vapour (B) nitrous
(B) fumes
nitrous fumes
(C) carbon monoxide
(C) carbon mon (D) carbon
(D) carbon dioxide
dioxide

Q23
Q.23 A system consists
A system consis of of four
four elements
elements A, B, C
A, B, C and
and D
D which are connected
which are connected functionally
functionally in
in aa parallel
parallel
- configuration. The
configuration. individual reliability
T_ e individual of the
reliability of elements is
the elements is 0.80,
0.80, 0.82,
0.82, 0.85
0.85 and
and 0.90
0.90 respectively.
respectively.
uThre
The reliability the system
reliability of the system is
is
i
(A)) 0.498
0.498 0‘ (B) 0.602
(B) 0.602 (C) 0.750
(C) 0.750 (D) 0.999
(D) 0.999
Q.24
Q.24 The blasting
blasting technique
technique used for controlled
used for controlled throw overburden is
of overburden
throw of is known as
known as
(ATCFt
(A) cast blasting
blasting (B) coyote
(B) coyote blasting
blasting
(C) plaster
(C) shooting
plaster shooting (D) pop
(D) shooting
pop shooting

Q25
Q.25 The stoping method,
The stoping method, where
where aa large
large part of blasted
part of ore is
blasted ore is allowed
allowed to
to accumulate
accumulate in
in the stope to
the stope serve
to serve
the
the purpose of providing
purpose of working platform
providing working platform for stoping as
for stoping as well as to
well as support the
to support the wall-rock, is known
wall-rock, is known
as
as
(A) shrinkage
(A) shrinkage stoping
stoping (B) cut
(B) cut and
and fill
fill stoping
stoping
(C) square-set
(C) square-set stoping
stoping (D) sublevel
(D) sublevel stoping
stoping

MN
MN 4/14
4/14

www.examrace.com
www.examrace.com
2012
2012 MINING ENGG. —
MINING ENGG. – MN
MN

Q. 26
Q. 26 to
to Q.
Q. 55
55 carry
carry two
two marks
marks each.
each.

Q.26
Q.26 The injury
The injury rates
rates of
of mine
mine workers in an
workers in an underground coal mine
underground coal mine based on age
based on age group
group are
are given
given below:
below:

Age group of
Age group of mine
mine Age-specific injury rate
Age-specific injury rate Age-specific
Age-specific population
population
workers
workers (per 1000
(per 1000 persons)
persons) in the
in the mine
mine
18 — 32
18 − 32 1.8
1.8 1000
1000
33 — 46
33 − 46 2.5
2.5 500
500

47 − 60
60 4.5 300
47 4.5 300 i

The injury
The injury rate
rate per 1000 persons
per 1000 employed in
persons employed in the
the mine for the
mine for the total
total population is
population is r 1
(A) 0.24
(A) 0.24 (B) 2.44
(B) 2.44 (C) 8.80
(C) 8.80 (D) 24.40
(D) 24.40 . /, -'
Q27
Q.27 A shearer is
A shearer is deployed
deployed inin aa mine
mine where
where thethe specific energy consumption
specific energy consumptionfior cutting coal
for cutting ogal is
is
800 kJ/m3.
800 The specific
kJ/m3. The specific gravity
gravity of
of coal 1s 1.2.
coalis 1 2. If
If the machine produces
the machine 700 te7h,
produces 700 te/h, therelfict'rical
the electrical power
power
consumption in kW of the shearer at 65% motor
kW of efficiencyis
motor efficiency 1s .‘5 i i._
(A) 149.4
(A) 149.4 (B) 199.4
(B) 199.4 (C) 219.4
(C) 219.4 ( (D) ‘2394
239.4
K (1')")
Q.28
Q.28 The figure
The figure shows
shows aa weightless
weightless beam PQ of
beam PQ of length
leng 8 m m reslfi
restingg on
on aa hinge support at
hinge support at P
P and
and on
on aa
roller support at
roller support at R.
R. A
A vertical force of
vertical force of 40
40 N
N isis acting
ctin 'at
at aa distance
stance of
of 44 m from P.
m from P. A
A uniformly
uniformly
distributed load
distributed load of
of 10
10 N/mis acting on
N/m is acting length off 2
on aa length 2mm of the eam from
the beam from Q.Q.

8m

4m 40 N 10 N/m
R
P Q

2m

The magnitude of reaction force at R in N is


(A) 20
(A) 20 (B) 30 (C) 40
(C) 40 (D) 50
(D) 50
a.
I.-

Q.29
Q.29 The figure
The figure shows theilistar‘ce
the distance vs
vs time graph of
time graph of aa moving
moving particle. The tangents
particle. The tangents to
to the at A
curve at
the curve A
and B
and B make
make angles
an es of
of 45
450a'nd
0
600 respectively
and 600
with the
respectively with time axis.
the time axis.
“'1 it
shogs
A
30
B
25
Distance, m

20
600
15

10 A

5
0
: N450;
45 . : : V
11 22 33 44 55 66
Time, sec
Time, sec

The ratio
The ratio of
of the
the speeds
speeds of
of the
the particle at B
particle at B and
and at
at A is
A is
(A) 0.72
(A) 0.72 (B) 1.38
(B) 1.38 (C) 1.58
(C) 1.58 (D) 1.75
(D) 1.75
MN
MN 5/14
5/14

www.examrace.com
www.examrace.com
2012
2012 MINING ENGG. —
MINING ENGG. – MN
MN

Q.30
Q.30 The gear
The gear ratios
ratios of
of the first gear,
the first gear, transfer
transfer case
case and
and differential
differential of
of aa four
four wheel drive vehicle
wheel drive are
vehicle are
3.81:1, 2.72:1 and
3.81:1, 2.72:1 and 4.11:1
4.11:1 respectively. If the
respectively. If engine is
the engine is rotating at 1000
rotating at 1000 rpm and the
rpm and the wheel diameter
wheel diameter
is 1.2
is 1.2 m,
m, the
the speed
speed of
of the
the vehicle in first
vehicle in gear in
first gear in km/h is
km/h is

Differential
Differential

Arr/
—\
Gear Box
Gear Box 1
Transfer
‘/Wheel
Transfer Wheel _
Engine
Engine Case
Case q:
.4 .
a-I- I
"I ' (1.
(A) 5.31
(A) 5.31 (B) 3.68
(B) 3.68 (C) 2.42
(C) 2.42 (D) 1.68
(D) _
1.? r.
Q.31
Q.31 An iron ore
An iron ore mine
mine recorded an average
recorded an average ofof 33 accidents
accidents per month. The
per month. The n‘u'fnber
number of of accidents
accidents is
is
distributed according
distributed according to
to Poisson
Poisson distribution.
distribution. The
The probability
probability that there: will
that there will be exilctl'y 2
be exactly 2 accidents
accidents
per
per month is 3 ,.
-.
(A) 0.22
(A) 0.22 (B) 0.30
(B) 0.30 (C) 0.43
(C) 0 3r K { 0.67
(D) 0.67
(D)
I

1
J

Q.32
Q.32 Match the
Match the following:
following:

Equipment
Equipment Component
Component

P
P Scraper
Scraper 11 Dribble belt
Dribble belt
Q
Q Dragline
Dragline ‘4‘ 22 stick
Dipper stick
Dipper
k
R
R Bucket wheel
Bucket wheql excavator
egcavator _ 33 Fair
Fair lead
lead
S
S Rope shovel
Rope sho’e‘ 44 Bowl
Bowl

.. T-. i..—
1
(A)
(A) P-2,
P_29 Q-4,
Q‘49 R-3,
13—39 S-1
8" (B)
(B) P-4,
P_49 Q-2,
Q‘za R-1,
R'la S-3
8'3
- ‘I.
(C) P-4,
(C) P-4, Q-3,
Q-3, R-1, S-2
S-2 (D) P-2,
(D) P-2, Q-4,
Q-4, R-1,
R-1, S-3
S—3
Ril,
an" '
Q.33.
Q.33 The torque
The torque in
in i—m of aa winder
N-m of winder motor is described
motor is described by the relationship
by the 1450 —
relationship TT = 1450 − 3.2ω , where,
3.2a) where, ω
wis
is
the angular
the angular speed
speed of the
he motor in rad/s.
motor in If the
rad/s. If the shaft
shaft is
is rotating at aa speed
rotating at speed of
of 1450
1450 rpm,
rpm, the
the power of
power of
( the motor
the in kW
motor in kW is
(A) 112.4
(A) 112.4 (B) 146.4
(B) 146.4 (C) 184.4
(C) 184.4 (D) 212.4
(D) 212.4

Q.34 .1411" 10,000, compounded


An investment of Rs. 10,000, com19 ounded annually,
annually , is estimated toto return ears
return Rs. 20,000 after 6 Yyears
om the
from the date
date of
0 investment. The expected
investment. The expected rate of return
rate of on this
return on investment in
this investment in percentage is
percentage is
( 8.75
(A) 8.75 (B) 10.50
(B) 10.50 (C) 12.25
(C) 12.25 (D) 16.6
(D) 16.6

Q.35
Q.35 Asplprical
A droplet of
spherical droplet of water,
water, with density 1000
with density 1000 kg/m3 and diameter
kg/m3 and of 11 µm,
diameter of um, is
is falling
falling in
in air.
air. The
The
viscosity of
viscosity air is
of air is 1.85
1.85 xX 10 '5
10 kg/m
-5
⋅ s . Neglecting
kg/m-s. air density
Neglecting air density and
and assuming
assuming that
that the settling of
the settling of
droplet in air
droplet in air follows
follows Stokes’ Law, the
Stokes’ Law, the settling
settling velocity in m/s
velocity in is
m/s is
(A) 0.98
(A) 10 -5‘5
0.98 xx 10 (B) 2.95
(B) 10 -5‘5
2.95 xx 10 (C) 8.04
(C) 10 -5‘5
8.04 xx 10 (D) 53.03
(D) 10 -5‘5
53.03 xx 10

MN
MN 6/14
6/14

www.examrace.com
www.examrace.com
2012
2012 MINING ENGG. —
MINING ENGG. – MN
MN

Q36
Q.36 A mining company
A mining company hashas three
three mines
mines (M1,
(M1, M2
M2 and
and M3)
M3) that supply coal
that supply coal to
to three
three power
power plants
plants
(P1, P2
(P1, P2 and
and P3).
P3). The
The three
three mines
mines produce 900, 1000
produce 900, 1000 and
and 1200
1200 te
te of
of coal
coal per day respectively.
per day The
respectively. The
power
power plant
plant requirements from these
requirements from these three mines are
three mines are 1200,
1200, 1000
1000 and
and 900
900 te
te per day respectively.
per day respectively.
The unit
The cost of
unit cost of transporting coal from
transporting coal from the three mines
the three mines to
to the
the three
three power
power plants in Rs.
plants in Rs. is
is given
given
below
below
Power plants
Power plants
P1
P1 P2
P2 P3
P3

M1
M1 88 10
10 12
12
Mines
Mines M2
M2 l2
12 l3
13 l2
12 1
M3
M3 14
14 10
10 11
11 *1. -
1
Based on
Based on the
the initial
initial basic feasible solution,
basic feasible solution, using
using Vogel’s approximation‘methfi’d,
Vogel’s approximation method, t3!
the total
1total
transportation cost in
transportation cost in Rs.
Rs. is
is
(A) 31200
(A) 31200 (B) 31400
(B) 31400 (C) 32800
(C) 32800 (D) 49600
(D) 40000 1
G
—» A
I:

Q37 R = ttzi
A 1': J
Q.37 The angle
The angle between the tangents
between the tangents to
to the curve R
the curve i + 22tj

at the
tjˆ at point tt =
the point ±1 is
Fiil is f

π7f π7: L‘ k
(A) —
(A) (B)
(B) a ' l
22 3 .
π
7T πif
(C) —
(C) (D)) _
44 66

Q.38 The
Q.38 The chip
chip sampling
sampling data,
data, spaced
spaced irregularly
irregularly for
for aa gold vei - deposit,
gold vein eposit, are shown in
are shown in figure.
figure. The
The sample
sample
points have equal
points have equal influence
influence onon both the sides.
both the sides. -

i
Distance of
Distance of sample from starting point (IE) 2
point (m) 2 88 16
10 16
10
samale I
Width (cm)
Width (cm) I; fim‘starting
70 80
70 80 60
60 100
100
Assay (g/te)
Assay (g/te) 55 77 66 44
11.. r";

60 cm 100 cm
Startirig point
Starting point 70 cm 80 cm
I
1"

Sampling distance →
Sampling —> 2m 8m 10 m 16 m
distanpe
-

The mean
The mean assay
assay V 11:11
value g/te is
in g/te is
(A) 6.52
(A) 6.52 _ (B) 5.50
(B) 5.50 (C) 5.19
(C) 5.19 (D) 4.50
(D) 4.50
Ir

Q39
Q.39 A'rseries of triaxial
series of tria al tests of sandstone
tests of sandstone samples
samples reveal
reveal the cohesion and
the cohesion and the
the angle
angle of
of internal friction
internal friction
21.65 MPa
as 21.65 MPa anda d 30˚
30° respectively. Based on
respectively. Based on the assumption that
the assumption that the sandstone samples
the sandstone samples follow
follow the
the
ohr-Coulomb’s failure
Mohr-Coulomb’s failure criteria,
criteria, the
the tensile
tensile strength in MPa
strength in MPa is
is
(A) 12.50 (B) 18.75
(B) 18.75 (C) 21.65
(C) 21.65 (D) 25.00
(D) 25.00
(A).1%r.50
Q.40
Q.40 The adjusted
The adjusted values
values of
of departure
departure and
and latitude for aa traverse
latitude for line AB
traverse line obtained in
AB obtained in aa field
field survey
survey of
of aa
mine are 225.520
mine are 225.520 m
m and
and 388.835
388.835 mm respectively. The length
respectively. The in m
length in and azimuth
m and azimuth of
of line
line AB are
AB are
(A) 449.50,
(A) 449.50, 30.11˚
30.11° (B) 614.36,
(B) 614.36, 30.11˚
30.11°
(C) 614.36,
(C) 614.36, 45.11˚
45.11° (D) 449.50,
(D) 45.11°
44950, 45.11˚

MN
MN 7/14
7/14

www.examrace.com
www.examrace.com
2012
2012 MINING ENGG. —
MINING ENGG. – MN
MN

Q.41
Q.41 The figure
The figure shows
shows the
the values
values of
of seven
seven perpendicular offsets and
perpendicular offsets and the
the respective locations along
respective locations along the
the
line XY
line as observed
XY as observed While carrying out
while carrying out aa traverse survey. The
traverse survey. area of
The area of the plot XABCDEFGY
the plot XABCDEFGY in in m2
m2
is
is
C
C

/ ‘

G
/ O
B/
B \ F/
F ‘
i¢ 14.4
14.4 ‘
/E

A
A D
E
124'
11.9
12.4
11.9 6.0 D./ 11.8
11.8
7.2
7.2 6.
6.1 '1.
XX .Y Y
> C 1,
7'1.

0.00
0.00 0.60
0.60 1.40
1.40 2.40
2.40 2.70
2.70 3.75
3.75 4.35
4.35 .1 I
"I ' (1.
(All measurements
(All measurements are
are in meters)
in meters) '.
'1- r-
(A) 26.10
(A) 26.10 (B) 43.38
(B) 43.38 (C) 44.92
(C) 44.92 (D) 62.50
(D) '
1
.. . :‘- I- 62.504
i! II F
Q.42
Q.42 In aa longwall
In longwall panel,
panel, the main gate
the main gate roadis 1000 m
road is 1000 long“, 4.
m long, 4.5131
m wide and 22 m high.
wide Gnd The gate
high. The gate road is
road is
to be
to be used
used for airflow at
for airflow at the
the rate
rate of 17 m
of 17 m3/s.
3
Cons ring aa‘poefficient
/s. Considering o Am
coefficient of resistance of airways
resistance of airways of
of
0.01, the
0.01, the pressure in Pa
pressure in Pa required
required to
to maintain
maintain the a Win
the airflow in the ate road
the gate is
road is
(A) 51.83
(A) 51.83 (B) 463.84
(B) 463.84 (C)) 875.98
87 98 (D) 7885.32
(D) 7885.32

Q43
Q.43 ⎡ 33 11 2 ⎤ _
The cofactor
The cofactor matrix
matrix of P = ⎢⎢ 22 33 11 ⎥⎥ is
of P is f
⎢⎣11 22 3 ⎥⎦
k
'
1 _.
21 −—55 22 ⎤
⎡ 21 21122 −—55⎤
⎡ 21 ⎡− —55 22 21
21 ⎤ ⎡1515 77 22 ⎤

(A) ⎢ 22 21
(A)

21 −—55⎥ (B) ⎢
(B) ⎢'22 77 15 15 ⎥⎥ ⎢
(C) ⎢15
(C) 15 77 22 ⎥⎥ ⎢ 21 ⎥⎥
(D) ⎢ −—55 22 21
(D)
—55 22 21
⎢⎣ − 21111—5
⎥⎦
'. I,
2122 ⎥⎦
⎢⎣ −5 21 —22 21
⎢⎣ − 21—5
−5⎥⎦ —22 21
⎢⎣ − 21—5
−5⎥⎦
1

'
Q.44
Q.44 Match the
Match the following:
follo ing: -. 1
Miningg system
system Face supports
Face supports
Mini:
P Mech in flat
flat seam
seam 11 Cable bolting
-

P Mechanized longwall in Cable bolting


(
Q Blasting
Q Blastin gallery
ga
bilongwall
alerymethod
method 22 Shield support
Shield support

. 'R
R Mecha ized longwall
Mechanized longwallin
1n steep
steep seam
seam 33 Alpine
Alpine breaker line support
breaker line support
__r S
S illi method
Wangawilli method for
for 3
3mm thick
thick coal
coal seam
seam 44 Troika shield
Troika shield support
support
Wang}w

Q- 1, R-
(A) P-2, Q-1, R— 4,
4, S-3
S-3 (B) P-4,
(B) P-4, Q-1,
Q-1, R-3,
R-3, S-2
S-2
(A943?)
(C)
(C) P-4,Q-2,
P-4, R—3, S-1
Q-2, R-3, S- 1 (D) P-2,
(D) P-2, Q-3,
Q-3, R-
R— 4,
4, S-1
S-1

MN
MN 8/14
8/14

www.examrace.com
www.examrace.com
2012
2012 MINING ENGG. —
MINING ENGG. – MN
MN

Q.45
Q.45 An opencast mine
An opencast mine bench has aa potential
bench has failure plane
potential failure plane AC
AC asas indicated
indicated in
in figure.
figure. Bolts
Bolts are
are installed
installed
to stabilize the
to stabilize the failure
failure plane
plane providing
providing aa resultant
resultant bolting force of
bolting force of 300
300 kN. The area
kN. The area of
of sliding
sliding block
block
2 3
ABC m2.. The unit
ABC is 37.45 m unit weight,
weight, cohesion and angle of internal friction of rock
rock are 25 kN/m3,
kN/m , 20
kPa and 40
400 respectively.
respectively.
0
kPa and
B
B C
C
//(\\

20m
300 kN

700 .
A 800 a-I- I
/7{\ 1. ' (1.

The factor
The factor of
of safety
safety of
of slope
slope when
when bolts are installed
bolts are installed perpendicular
perpendicular to
to the failulgplane
the failure isI
plane is
(A) 0.79
(A) 0.79 (B) 1.08
(B) 1.08 (C) 1.78
(C) 1.78 .. 1.09) 3.46 J ‘5
(D) 3.46
: .— -. _
Q.46
Q.46 Figure shows
Figure shows aa two
two pulley system for
pulley system for hoisting
hoisting aa load
load of
ofil10 kN. The{coeff
kN. The ient of
coefficient friction between
of friction between
each pulley
each and the
pulley and the rope is 0.2.
rope is 0.2. The
The vertical and h
vertical and izonta distances
horizontal distances be ween the
between the centers
centers of
of the
the
pulleys are 25
pulleys are 25 m and 16
m and 16 m
m respectively.
respectively. I

T2 T3
25m

16m

W = 10 kN
T1

The tensions
The tensions T
T11 and
and T
122 respectivelyin kN are
respectively in kN are
(A) 6.00,
(A) 6.00, 5.38
5'38. ( (B) 12.37, 11.06
11.06 (C) 18.74,
(C) 16.73
18.74, 16.73 25.11, 22.41
(D) 25.11,
(D) 22.41
(B)‘ll2.37,
Q47
Q.47 A circular
A circular tunril of 1.85
tunnel of 1. 85 m
m radius is driven
radius is drivenin rock1n
in rock in aa hydrostatic
hydrostatic stress
stress field
field of
of 20
20 MPa.
MPa. TheThe
tunnel lining1is provided
tunnel lining rovided before occurrence of
before occurrence of any
any rock deformation. The
rock deformation. The shear
shear modulus
modulus ofof rock1s
rock is
‘2 GPa and
2 GPa and the
the modulus
m lus of
of elasticity
elasticity of
of lining
lining material is 3
material is 3 GPa.
GPa. Assuming
Assuming bothboth rock and lining
rock and lining
( behave elasticall the
behave elastically, th radial
radial pressure on the
pressure on the rock and lining
rock and interface in
lining interface in MPa
MPa isis

(A) 8.19
(A) 8.19 (B) 9.91
(B) 9.91 (C) 11.62
(C) 11.62 (D) 13.33
(D) 13.33
Ir

Comm 'rData
Common Que¥ions
Data Questions
Commo Data
Common Data for
for Questions
Questions 4848 and
and 49:
49:
A 2.5 m
A 2.5 mtthick
ck coal
coal seam
seam lying
lying at
at an
an average
average depth
depth ofof 100
100 mm has
has been developed by
been developed by bord and pillar
bord and pillar method.
method.
The width
The widthhof
of thee square
square pillars is 30
pillars is 30 m (centre to
m (centre centre) and
to centre) and the gallery width
the gallery is 4
width is 4 m. The average
m. The average density
density
of the
the overlying
overly1ng strata
strata is
is 26
26 kN/m
kN/m3 andand the
the pillar strength is
is 4500
4500 kN/m2.
3
of pillar strength kN/m2.
Q.48
Q.48 Extraction ratio
Extraction during the
ratio during the development
development of
of the
the pillar is
pillar is
(A) 0.129
(A) 0.129 (B) 0.148
(B) 0.148 (C) 0.218
(C) 0.218 (D) 0.249
(D) 0.249

Q.49
Q.49 The safety
The safety factor
factor of
of the
the pillar is
pillar is
(A) 1.1
(A) 1.1 (B) 1.3
(B) 1.3 (C) 1.5
(C) 1.5 (D) 1.7
(D) 1.7

MN 9/14
9/14

www.examrace.com
www.examrace.com
2012
2012 MINING ENGG. —
MINING ENGG. – MN
MN

Common Data
Common Data for
for Questions
Questions 50
50 and
and 51:
51:
The following
The following data are provided
data are for aa surface
provided for surface mine to be
mine to excavated by
be excavated by aa shovel:
shovel:

Production target
Production target :: 10000
10000 te/shift
te/shift
Available hours per
Available hours shift
per shift :: 66 hrs
hrs
Shovel loading
Shovel loading cycles
cycles per hour
per hour :: 106
106
Bank density
Bank density of of the material mined
the material mined :: 2400
2400 kg/m3
kg/m3
Swing factor
Swing factor at
at 120
1200 swing
0
swing :: 0.91
0.91
Bucket fill
Bucket fill factor
factor :: 0.64
0.64 1
”R
Utilization ofof available
available time
time :: 83%
83%
r _
Utilization
No of working
No of working days
days in in aa year
year :: 300
300 .1 I
No of shifts
shifts per day :: 33 1' / ‘_'
No of per day _
Q.50 The annual
annual production target in
in Mte
Mte is
is r r
Q.50 The
(A) 5.76
5.76
production target
(B) 7.00
7.00 (C) 8.19
8.19 (D) 9.00 '
J .5
(A) (B) (C) (D) 9.00
q._ f I-
'
Q.51
Q.51 The size
The size of
of bucket of the
bucket of shovel in
the shovel m33 is
in m is _ #- -. r
{
(A) 5.55
(A) 5.55 (B) 9.33
(B) 9.33 (C) 11.22
(C) .22 E (D) 13.55
(D) 13.55
.I'

Linked Answer
Linked Questions
Answer Questions
Statement for
Statement for Linked
Linked Answer Questions 52
Answer Questions 52 and
and 53:
53:
A mining project
A mining is composed
project is composed of
of five
five activities
activities whose tim estimates
hree time
whose three 1mates in
in months are given
months are given below:
below:
' /
_
Activity
Activity E ated duration
Estimated duration (months)
( fith's)
Optimistic time
Optimistic time Mostt likely
likely time
time F Permissible
Permissible time
time
1 -2
1-2 11 1 11 _ 77
1-3
1-3 22 '| 55 88
2—4
2-4 11..'- ,1 11 77
3-4
3-4 . 2 2 55 14
14
4—5
4-5 . ""
.. 4‘- £33 66 15
15

(
Q.52
Q.52 The expected
The expected duration
(1 ration of
of the! mining project
the inining in months
project in is
months is
(A) 5
(A) 5 (B) 16
(B) 16 (C) 18
(C) 18 (D) 29
(D) 29

Q.53 ‘The standard deviation


The standard deyi of the
of the project length in
project length in months is
months is
Q5? (A) 22
(A) mil (B)
(B) 33 (C) 66
(C) (D) 99
(D)

Statement for
Statement f6r Linked
Linked Answer
A swer Questions
Questions 54 54 and
and 55:
55:
In aa mine
In mm: between
between up ast shaft
upcast shaft and and downcast
downcast shaft,
shaft, two
two airways
airways are
are connected
connected inin parallel and their
parallel and their
resistan
resistancess are
are 100 and 120
100 and 120 NN ss"2-2 m'8
m-8 respectively. The resistance
respectively. The resistance ofof upcast shaft, downcast
upcast shaft, downcast shaft
shaft and
and the
the
fan drifts
fan drift are
are 10,
10, 20
20 and
and'S5 N
N ss"2-2 m
m'8 respectively.
-8
respectively. The fan drift
The fan drift air
air pressure is 15
pressure is 15 MN/m
MN/mz. 2
.
Q54
Q.54 T rate
The rate of airflow through
of airflow mine in
the mine
through the in m
m3/s
3
is
/s is
(AT4'1-6
(A) 4.16 (B) 18.26
(B) 18.26 (C) 240.35
(C) 240.35 (D) 333.33
(D) 333.33
Q.55
Q.55 The rate
The rate of
of airflow
airflow through
through the
the split airway having
split airway having resistance 100 N
of 100
resistance of N ss'2-2 m"8 in m
m-8 in m3/s
3
is
/s is
0.42
(A) 0.42
(A) (B) 0.79
(B) 0.79 (C) 2.19
(C) 2.19 (D) 7.90
(D) 7.90

END OF
END THE QUESTION
OF THE QUESTION PAPER
PAPER

MN 10/14
10/14

www.examrace.com
www.examrace.com
2012
2012 GENERAL APTITUDE -— GA_AN_Online
GENERAL APTITUDE GA_AN_Online

General Aptitude
General (GA) Questions
Aptitude (GA) Questions

Q. 56 —
Q. 56 Q. 60
– Q. 60 carry
carry one
one mark
mark each.
each.

Q.56
Q.56 Choose the
Choose the most
most appropriate
appropriate alternative
alternative from
from the
the options
options given
given below
below to complete the
to complete following
the following
sentence:

II _ to have
___ to have bought a diamond
bought a diamond ring.
ring.
(A) have
(A) have aa liking
liking (B) should
(B) should have liked
have liked
(C) would
(C) like
would like (D) may
(D) like
may like 1.
”R
Q.57
Q.57 Choose the
Choose the most
most appropriate
appropriate alternative
alternative from
from the
the options
options given
given below
below to the fpfiwi
complete th
to complete 'g
following
I
sentence:
sentence: 1. - 21.

Food prices
Food prices _ this month.
again this
___ again month. r r

(A) have
(A) have raised
raised (B) have
(B) been raising
have been raising
l J g
1‘
(C) have
(C) been rising
have been rising (D) have
(D) arose
have arose : fl. . F

Q.58
Q.58 Choose the
Choose the most
most appropriate
appropriate alternative
alternative from
from the
the options
°
op ns given
1
given below
k
below to
to pdmplete
complete the following
the following
sentence:

The administrators
The administrators went
went on to implement
on to implement ye unr asonable measure,
anothe unreasonable
yet another measure, arguing
arguing that
that
the measures
the measures were already _
were already and one
___ and one more
mor would r
would hardly make a
make a difference.
difference.
'
(A) reflective
(A) reflective (B) utopian
(B) utopian (C) luxuriant
(C) luX nt (D) unpopular
(D) unpopular

Q.59
Q.59 Choose the
Choose the most
most appropriate the options
appropriate alternative from the options given
ven below
below to complete the
to complete following
the following
sentence: alternativ‘frgm
1
To those
To those of
of us
us who ht him
had always thought
who had him timid,
timid, his
his"_ came as
___ came as a
a surprise.
surprise.
alway‘tjiou

(A) intrepidity
(A) intrepidity (B) inevitability (C) inability
(C) inability (D) inertness
(D) inertness
"
(Bhinevitability
'
r

.i-
Q.60
Q.60 ((f five
The arithmetic mean of five different natural numbers
different natural is 12.
numbers is 12. The
The largest
largest possible
possible value among the
value among the
'
numbers is
numbers is _ l
l
(A) 12
(A) 12 i: (B) 40
(B) 40 (C) 50
(C) 50 (D) 60
(D) 60
' i
1..
.l'

Q. 61
Q. 61-- Q.
Q. 65
65 carry
carry two
t 0 marks
marks each.
each.

Q.61 HT
Q.61 Two0 policemen
policemen, IA and B,
A and B, fire
f1re once
once each
each at
at the same time
the same at an
time at an escaping
escaping convict. The probability
convict. The probability
'at A
that A hits
hits the
the convict
c nvict is
is three times the
three times the probability that B
probability that B hits
hits the If the
convict. If
the convict. the probability of the
probability of the
nvict not
convict not getting injured is
getting injured is 0.5,
0.5, the
the probability
probability that B hits
that B the convict
hits the convict is
is
( 0.14
(A) 0.14 (B) 0.22
(B) 0.22 (C) 0.33
(C) 0.33 (D) 0.40
(D) 0.40

GA_AN_Online
GA_AN_Online 1/2
1/2

www.examrace.com
www.examrace.com
2012
2012 GENERAL APTITUDE -- GA_AN_Online
GENERAL APTITUDE GA_AN_Online

Q.62
Q.62 The total
The total runs scored by
runs scored four cricketers
by four cricketers P,
P, Q,
Q, R,
R, and
and S in years
S in 2009 and
years 2009 and 2010
2010 are
are given in the
given in the
following table:
following table:
Player
Player 2009
2009 2010
2010
P
P 802
802 1008
1008
Q
Q 765
765 912
912
R
R 429
429 619
619
S
S 501
501 701
701

The player
The player with
with the lowest percentage
the lowest increase in
percentage increase in total
total runs is
runs is

(A) PP
(A) (B) Q
(B) Q (C) R
(C) R (D) SS
(D)
1 _
. "a.
Q.63
Q.63 If aa prime
If number on
prime number division by
on division by 44 gives
gives aa remainder of 1,
remainder of 1, then
then that number can
that number can be exp sseGL as
be expressed as
fl I
(A) sum
(A) sum of
of squares
squares of
of two natural numbers
two natural numbers 1| . (1'
(B) sum
(B) sum of
of cubes
cubes of
of two
two natural
natural numbers
numbers - -. -
(C) sum
(C) sum of
of square
square roots of two
roots of natural numbers
two natural numbers r r
(D) sum
(D) sum of
of cube
cube roots of two
roots of two natural numbers
natural numbers 1r |
1f 1
.. E I-
Q-64
Q.64 TWO points
Two (4, I?)
points (4, p) and q) lie
(0, q)
and (0, lie on
on aa straight line having
straight line having aa slope
slope of 3/ . The
of 3/4. The. value (1') —– q)
of (p
value of is
q) is
1 K
I'll
-3
(A) -3
(A) (B) 00
(B) (C) 33
(C) (D5 44
(D)
I
J

Q.65
Q.65 In the
In the early
early nineteenth
nineteenth century,
century, theories
theories of
of social
soc' l evolution
ev'l lutio were inspired less
were inspired less by Biology than
by Biology than
by the
by the conviction of social
conviction of social scientists
scientists that
that there was aa growing
ther was row' g improvement
improvement in in social
social
institutions. Progress
institutions. Progress was
was taken for granted
taken for granted and
nd social
soci scientists
sci ntists attempted
attempted to
to discover
discover its
its
laws and
laws and phases.
phases.

Which one
Which one of
of the
the following inferences may
following inferences may be drawn with
be drawn with the reatest accuracy
the greatest accuracy from
from the above
the above
passage?
passage? i
k
r
Social scientists
Social scientists 1
(A) did
(A) did not question that
not question that progrels‘vas
progress was aa fact.
fact.
(B) did
(B) did not
not approve
approve ofof Biology.
Biology
(C) framed the
(C) framed laws of
the laws of pflo ress.
progress.
(D) emphasized
(D) emphasized Biology
Biold‘ y ov‘er. Social Sciences.
over Social Sciences.

_ l
1-“- END OF
END THE QUESTION
OF THE QUESTION PAPER
PAPER

GA_AN_Online
GA_AN_Online 2/2
2/2

www.examrace.com
www.examrace.com
GATE 2012 ‐- Answer Key ‐- Paper : MN

Paper Question no. Key Paper Question no. Key


MN
MN 11 BB MN 41 C
MN
MN 22 Cc MN
MN 42
42 Marks to
Marks to All
All
MN
MN 33 A
A MN
MN 43
43 Marks to
Marks to All
All
MN
MN 44 A
A MN
MN 44
44 A
A !
MN
MN 55 D D MN
MN 45
45 BB _ ‘3
MN
MN 66 BB MN 46 C (3‘ 15
MN
MN 77 DD MN
MN 47
47 Cc _ «-
MN 8 Marks to All MN
MN 48
48 DD : Al
MN
MN 99 A
A MN
MN 49
49 BB r f
MN
MN 10
10 C6 MN
MN 50
50 DD" 4 1 '
MN
MN 11
11 D D MN
MN 51 ‘.
51
' D D '4'
MN
MN 12
12 A
A MN
MN 52 Marks to
521 .4 Marks tcfAII
All
MN
MN 13
13 D D MN 1
MN 5i
53 [ Marks
Marks to
to All
All
MN
MN 14
14 BB MN .1- 54
54 ‘ rMarkstoAII
Marks to All
MN
MN 15
15 C(2 flN“MN
MN 55
55 MarkstoAII
Marks to All
MN
MN 16
16 D D MAL
MN 56
56 C6
MN
MN 17
17 A
A MN!
MN 57
57 Cc
MN
MN 18
18 BB MNI
MN 58
58 DD
MN
MN 19
19 C6 MN”
MN 59
59 A
A
MN 20 D MN 4+ $0 6
‘ 5..
MN 20 D MN 60 C
MN
MN 21
21 C6 MN 61 A

9"
MN
MN 22
22 BB MN 62 B
MN
MN 23
23 D D MN'
MN 63 A
MN
MN 24
24 A
A MN 64 C
MN
MN 25
25 A
A MN 65 A
MN
MN 26
26 2.3 to
2.3 t9 2.5
2'5 1'
MN
MN 27
27 198 to
198 200* -
tp 200
MN
MN 28
28 49 to
J 49 to‘51
51 1-
MN
MN 29
29 1.35 to
1.35 to 1.4
1.4
MN
MN 30 a 5 to
30 to 5.5
5.5
1'5
MN
MN 31
31 5511
A
MN
MN 32
32 C 1

MN
MN 33
33 'i
B
MN
MN 34
34 I:C
MN
MN .1
r 35
35 | BB
MN
MN 36
36 1BB
MN
MN 37
37 3A A
MN
MN IL 38
38 C6
MN
MN ‘4' 39
39 D D
MN
MN F 40
40 A
A

www.examrace.com
www.examrace.com
2013
2013 MINING ENGG.
MINING ENGG. —MN
–MN

MNzMINING ENGINEERING
MN:MINING ENGINEERING
Duration: Three Hours
Duration: Three Hours Maximum
Maximum Marks: 100
Marks:100

Please read
Please the following
read the instructions carefully:
following instructions carefully:

General Instructions:
General Instructions:

1. Total duration of examination is 180 minutes (3 hours). '1. _

2. The clock will be set at the server. The countdown timer in the top right corner of screen will
display the remaining time available for you to complete the examination. When; Whenscfinii/vill
the timer
erlI
reaches zero, the examination will end by itself. You will not be required to end or submit
submi yourour
- .
examination.
r r.-
3. The Question Palette displayed on the right side of screen will show the status of each question
symbols.
using one of the following symbols: I. Jeaglh
..
f

{
‘ You have
You have not
not Visited the question
visited the quest' 1.13
yet.
yet.

uestio .
You have not answered the question.

You have answered


You have answered the
the question.
q estion.

You have
You have NOT answered the
NOT answered the question, ut have
question but arked the
have marked the
for review.
questlon for
question

QQDG
revie‘v.
You have
You have answered t‘e question, but
the question, but itarked it for
marked it for review.
review.

answeId
The Marked for Review status for a question simply indicates that you would like to look at that
stams-dér
question again. If
If a question is answered and Marked for for Review, your
your answer for for that
question
question will be considered
canSIde'red in
1n the evaluation.

to aa Question
Navigating to
Navigating
Qué$i0_n
, .
4. .To answelI aa‘uestion,
To answer question, do the following:

a. Click on the question number in the Question Palette to go to that question directly.
{Nuestion
( b. Select an answer wer for a multiple choice type question. Use the virtual numeric keypad to
enter a number as answer for a numerical type question.
' c. Click on Save and Next to save your answer for the current question and then go to the
'* next question.
uestion.
".I'"
d.
(1. Click on11 Mark for Review and Next to save your answer for the current question, mark
it for review,
eview, and then go to the next question.
e. Caution: Note that your answer for the current question will not be saved, if you
navigate to another question directly by clicking on its question number.
h

5. You
Yeti can view all the questions by clicking on the Question Paper button. Note that the options
for multiple choice type questions will not be shown.

MN
MN 1/15
1/15

www.examrace.com
www.examrace.com
2013
2013 MINING ENGG.
MINING ENGG. —MN
–MN

Answering a
Answering a Question
Question

6. Procedure for answering a multiple choice type question:


a. To select your answer, click on the button of one of the options
b. To deselect your chosen answer, click on the button of the chosen option again or click
on the Clear Response button
c. To change your chosen answer, click on the button of another option
d.
(1. To save your answer, you MUST click on the Save and Next button
e. To mark the question for review, click on the Mark for Review and Next button. If an
butto . If
answer is selected forfor a question that is Marked for for Review, that answer will ill be
considered in the evaluation. a, 'l
-*
'I
7. Procedure for answering a numerical answer type question: 1. ' /1.
' '~
a. To enter a number as your answer, use the virtual numerical keypad
b. A fraction (eg.,‐0.3
(eg.,-0.3 or ‐.3)
-.3) can be entered as an answer with or without ‘0’
0’ before the r
decimal point "' '
1
c. To clear your answer, click on the Clear Response button 4. ”a i. ‘r
d. To save your answer, you MUST click on the Save and Next button uttoj
n
r
e. To mark the question for review, click on the Mark for Review
then-Ma R iew and nd Next button. If If an
answer is entered forfor a question that is Marked for Review, that answer will be
drke for
considered in the evaluation. ""
I'

8. To change your answer to a question that has


ha already been answered,
lread ”been nswered, first select that question
for answering and then follow the procedure forr answering
answ ing that
at type of question.

save or marked for review after answering


9. Note that ONLY Questions for which answers are saved
will be considered for evaluation. (I

fit f"

MN
MN 2/15
2/15

www.examrace.com
www.examrace.com
2013
2013 MINING ENGG.
MINING ENGG. —MN
–MN

specific instructions:
Paper specific

1. There
1. There are
are aa total
total of
of 65
65 questions
questions carrying
carrying 100
100 marks.
marks. Questions
Questions areare of
of multiple
multiple choice
choice type or
type or
numerical answer type.
numerical answer A multiple
type. A multiple choice
choice type question will
type question have four
will have four choices
choices for
for the answer with
the answer with
only one
only one correct
correct choice.
choice. For
For numerical answer type
numerical answer type questions,
questions, the answer is
the answer is aa number and no
number and choices
no choices
will
will be given. A
be given. number as
A number as the
the answer
answer should
should be
be entered
entered using the virtual
using the virtual keyboard on the
keyboard on the monitor.
monitor.

2. Questions
2. Questions Q.1Q. l — Q.25 carry
– Q.25 carry 1mark
lmark each.
each. Questions Q. 26 —
Questions Q.26 Q. 55 carry
– Q.55 carry 2marks
2marks each.
each. TheThe 2marks
imarks
questions include
questions include two
two pairs of common
pairs of common data
data questions
questions and
and two
two pairs of linked
pairs of linked answer ques
answer questions. s‘il‘he
The
answer to
answer to the second question
the second question of
of the linked answer
the linked answer questions
questions depends
depends on on the
the answer
answer tfic to the first
first
question of
question of the
the pair. If the
pair. If the first
first question
question inin the linked pair
the linked pair isis wrongly answered or
wrongly answered or isis not attempted, (1,
not-attem
then the answer
then the answer toto the second question
the second question in the pair
in the pair will
will not
not be evaluated.
be evaluated. 1' 1'

3. Questions
3. Q.56 —
Questions Q.56 Q.65 belong
– Q.65 belong to General Aptitude
to General (GA) section
Aptitude (GA) section and
and carry
carry aa (ital of 15
total of 15 mags.
marks.
Questions Q.56 —
Questions Q.56 Q.60 carry
– Q.60 carry 1mark
lmark each,
each, and
and questions Q.6l —
questions Q.61 Q.65 carry
– Q.65 carry 2marks
2marksreach.
each. I
.. 1
J
4. Questions
4. Questions not attempted will
not attempted will result
result1n zero mark.
in zero mark. Wrong answers for
Wrong answers fof‘rnultiple choice trpe
multiple choice questions
type questions
will result1n
will result NEGATIVE marks.
in NEGATIVE marks. For
For all
all 11 mark
mark questions,
questions,13 mark will be
⅓ mark deducted for
be deducted for each
each wrong
wrong
answer. For all
answer. For all 2
2 marks questions, ⅔
marks questions, mark will
2/3 mark will be each wgl
ded te'd for each
be deducted w ong answer.
wrong However, in the
a swer. However,1n the
case of
case of the linked answer
the linked answer question
question pair,
pair, there
there will negative marks
will be negative arks only
only' for
for wrong answer to
wrong answer to the
the
first question
first question andand no
no negative
negative marks
marks forfor wrong a wer to the
wrong answer s cond question.
the second question. There
There is
is no negative
no negative
marking for
marking for questions
questions of
of numerical
numerical answer
answer type.
type.

5. Calculator
5. Calculator is
is allowed.
allowed. Charts,
Charts, graph
graph sheets
sheets or
or tables
ta les are
are N T allowed
NOT in the
owed in examination hall.
the examination hall.

6. Do
6. Do the
the rough
rough work in the
work in the Scribble
Scribble Pad
Pad provided.
provided. -f

‘1;
:1
(’9
1

MN
MN 3/15
3/15

www.examrace.com
www.cxamrace.com
2013
2013 MINING ENGG.
MINING ENGG. —MN
–MN

Q. 11 —
Q. Q. 25
– Q. 25 carry
carry one
one mark
mark each.
each.

Q.1
Q.1 In the
In the Coward
Coward flammability
flammability diagram,
diagram, the
the respective
respective percentages of methane
percentages of and oxygen
methane and oxygen at
at the
the
nose limit
nose limit are
are
(A) 14.2,
(A) 14.2, 0.0
0.0 (B) 14.1,
(B) 14.1, 18.2
18.2 (C) 5.8,
(C) 5.8, 12.1
12.1 (D) 5.0,
(D) 5.0, 19.2
19.2
Q.2
Q.2 If the
If the transpose
transpose of
of aa matrix
matrix is
is equal
equal to its inverse,
to its inverse, then
then the matrix is
the matrix is

(A) symmetric
(A) symmetric (B) orthogonal
(B) orthogonal (C) skew
(C) skew symmetric
symmetric (D) singular
(D) singular 1
*1.‘
Q3
Q.3 In the
In the Moh’s
Moh’s scale
scale of
of hardness,
hardness, the
the minerals
minerals in
in increasing
increasing sequence
sequence of
of hardness are
hardness are r 1
'
(A) calcite,
(A) calcite, gypsum,
gypsum, topaz, diamond
topaz, diamond , /1. .I
1|
(B) topaz,
(B) topaz, gypsum,
gypsum, calcite,
calcite, diamond
diamond . .1
(C) calcite,
(C) calcite, gypsum,
gypsum, diamond,
diamond, topaz
topaz r r
(D) gypsum,
(D) gypsum, calcite,
calcite, topaz,
topaz, diamond
diamond .r 1
.f "I
a! .
Q4
Q.4 A
A ball of weight
ball of weight W
W is
is supported
supported onon smooth
smooth walls
walls as
as shown in the
shown in the following
following figure.
figure. R1 R22 are
R1 and R are
reactions from the
reactions from walls 11 and
the walls 2. The
and 2. The free
free body diagram_0f
body diagram by a1rd
is represented by
of the ball is
1
tltball r(prése'nt'ed
J
III
l
1

(A) RR11
(A) (B)
(B) RR11 AI. ‘1 (C)
(C) (D)
(D) RR22
R2
RR22 RR22 R1 RR11
1‘ .
(
w
W I‘' I w
W
—"
W W
1 .
t _ I;
Q.5 For aa 25
25 mm
mm diameter
diameter spherical
spIi i'i-cal charge,
charge, the maximum allowable
the maximum allowable charge
charge length in cm
length in cm is
is
'
Q.5 For
'
(A) 15.0
(A) 15.0 (Bi 25.0
(B) 25.0 (C) 30.0
(C) 30.0 (D) 150.0
(D) 150.0

9.6 - Long-hole
Long-hol‘éflilling
drilling with crater blasting is used for the construction of
of
_ Q.6 with crater blasting is used for the construction

(Q.7
(A) winze
(A) winze J (B) shaft
(B) shaft (C) raise
(C) raise (D) decline
(D) decline

Q.7 Rill stoping


Rill stoping m hod is
method is aa form of
form of
(A)
(A) block caving
block caving
"'
.-"" (B)
(B) artificiall
artificially supported stoping
supported stoping
(C) underhan
(C) stoping
underhand stoping
(D) overhand
(D) overhan stoping
stoping

Q.8
Q.8 Transit theodolite
Transit theodolite is
is aa
I.-
(A) micro-optic
(A) micro-optic theodolite
theodolite
(B) theodolite
(B) theodolite with face left
with face left and
and face
face right
right reading facilities
reading facilities
(C) theodolite
(C) theodolite with stadia hairs
with stadia hairs
(D) theodolite
(D) theodolite with
with two
two vertical
vertical circles
circles

MN
MN 4/15
4/15

www.examrace.com
www.examrace.com
2013
2013 MINING ENGG.
MINING ENGG. —MN
–MN

Q.9
Q.9 Incubation period
Incubation is NOT
period is NOT related to
related to
(A) crossing
(A) crossing point temperature of
point temperature of coal
coal
(B) panel
(B) size
panel size
(C) seam
(C) seam thickness
thickness
(D) explosibility
(D) explosibility of
of coal
coal dust
dust

Q10
Q.10 The rotational
The rotational speed
speed and
and cutting
cutting velocity of aa drill
velocity of drill are
are 350
350 rpm and 71.50
rpm and m/min respectively.
71.50 m/min The
respectively. The
diameter of
diameter of the
the rotary drill bit
rotary drill in mm
bit in mm is
is

t‘ - .
(A) 65
(A) 65 (B) 67
(B) 67 (C) 68
(C) 68 (D) 70
(D) 70 . ‘k.
Q11
Q.11 The pressure
The on aa phreatic
pressure on surface is
phreatic surface is .4 I
'
1' / 1|
(A) less
(A) less than
than atmospheric
atmospheric pressure
pressure _H _
(B) greater
(B) greater than
than atmospheric
atmospheric pressure
pressure r r
(C) equal
(C) equal to atmospheric pressure
to atmospheric pressure 1.. I
(D) independent
(D) independent ofof atmospheric
atmospheric pressure
pressure . +1
J
.. "' 1-
Q12
Q.12 Events A
Events A and B are
and B are independent
independent but
but NOT mutually exclusive.
NOT mutually exclusive. If
"
If the P(A) and
the probabilities P(A) agd P(B)
P(B) are
are
0.5 and
0.5 and 0.4
0.4 respectively,
respectively, then P(A ∪
then P( U B)
B) is
is 1 prqbabil'l'ties
(A) 0.6
(A) 0.6 (B) 0.7
(B) 0.7 (C 0.88 .
(C) ,. (1))” 0.9
(D) 0.9
Q.13
Q.13 Among the
Among the following
following options,
options, the
the specific
specific energy
energ for
for ro -drill° g is
rock-drilling is lowest in
lowest in

(A) rotary
(A) rotary diamond drilling
diamond drilling
(B) rotary
(B) rotary roller
roller drilling
drilling
(C) percussive drilling
_/
(C) percussive drilling
(D) jet piercing
(D) jet piercing
I

Q14
i
Q.14 Identify the
Identify the correct
correct statement
statement for
for aa ‘normal
‘no madistribution’.
distribution’.
(A) Mean
(A) Mean is
is greater
greater than mod bEl
than mode but less than median
than median
(B) Mean
(B) Mean is
is less
less than
than mode
mode his geater than
but greater than median
median
(C) Mean
(C) Mean is
is greater
greater than
than mode and median
mode and median
(D) Mean,
(D) Mean, median
median and
and mode are
are equal
equal
imode
1. ' .l ‘1;
Q15
Q.15 An emulsion
An emulsion explosive
explo e‘of'specific gravity 1.25
of specific gravity 1.25 is
is used for blasting
used for in an
blasting in iron ore
an iron ore formation
formation having
having
P-wave velocity
P-wave velocity of
of 3000
000 m/s fs and
and specific gravity of
specific gravity of 3.20.
3.20. For
For an
an explosive
explosive impedance
impedance to
to rock
rock
impedance ratio of
impedance of 0.5,
0.5,..th desired velocity
the desired velocity of
of detonation
detonation ofof the
the explosive
explosive in
in m/s
m/s is
is
refio
(A) 3840
(A) 3840.; (B) 4000
(B) 4000 (C) 4200
(C) 4200 (D) 7680
(D) 7680
i
Q.16 The number
The number of ways in
in which
which the
the letters in the
letters in MINING can
word MINING
the word can be arranged is
be arranged is
'(61'6 (A) 90
(A) 90 of\a\s (B) 180
(B) 180 (C) 360
(C) 360 (D) 720
(D) 720

Q17 Under
Q.17 standar temperature
Under standard temperature and
and pressure conditions the
pressure conditions the theoretical maximum height
theoretical maximum in m
height in m to
to which
which
'* rwater can be
water can ifted using
be lifted air-lift pump
an air-lift
using an is
pump is
(A) 10.33
(A) 10.33 (B) 9.61
(B) 9.61 (C) 7.45
(C) 7.45 (D) 6.05
(D) 6.05
Q. 18
Q.18 In aa belt
In conveyor system,
belt conveyor function of
system, function of the
the snub
snub pulley is to
pulley is to

"((A) clean
clean the
the inner
inner surface
surface of
of the
the belt
belt
( ) clean
(B) clean the
the outer
outer surface
surface of
of the
the belt
belt
(C) increase
(C) increase the angle of
the angle of contact
contact of
of belt
belt with drive drum
with drive drum
(D) decrease
(D) decrease the
the belt tension
belt tension

MN 5/15
5/15

www.examrace.com
www.examrace.com
2013
2013 MINING ENGG.
MINING ENGG. —MN
–MN

Q.19
Q.19 In the
In the following
following figure,
figure, the
the coefficient
coefficient of
of kinetic friction between
kinetic friction between the trolley and
the trolley and the
the surface
surface is
is 0.04.
0.04.
When the
When the block is released
block is from rest,
released from the acceleration
rest, the acceleration of
of the
the trolley in m/s2
trolley in m/s2 becomes
becomes

80kg pulley
80kg

////////////

10kg

.311
(A) 9.65
9.65 (B) 1.23
1.23 (C) 1.09
1.09 (D) 0.74
0.74 _/.
r
(A) (B) (C) (D)

Q20
Q.20 Two meshing
Two meshing spur
spur gear
gear wheels of Module
wheels of Module 66 have
have 24
24 and
and 42
42 teeth. The distance
teeth. The distancemmm
in mm between
between the
the
centres of
centres of the gear wheels1s
the gear wheels is 1 II
1'
(A) 1000
1000 t (D) 72
(A) (B) 198
(B) 198 (C) 126
(C) 126 (DI->321 I"
*1
Q21
Q.21 In an
In an experiment
experiment to
to study
study coal
coal dust
dust explosibility,
explosibility, it
it is
is found
mil that
thgt at
at least
least 3.0
3.0‘g
g 13f limestone dust
of limestone dust
should be
should added to
be added to aa sample
sample of
of 2.0
2.0 g
g of
of coal
coal dust
dust to sure tha'f
to ensure ropagation of
that propagation of flame
flame does
does not
not take
take
place. The explosibility
place. The explosibility factor
factor of
of coal
coal dust
dust is
is

(A) 60.00
(A) 60.00 (B) 20.00
(B) 20.00 (C) 6.70 (D) 1.50
(D) 1.50
Q22
Q.22 A 20
A 20 m
m steel
steel tape
tape used in aa mine
used in survey is
mine survey is found
found to
to be 20 cm
be 20 c" short
short when compared
compared with
with aa standard
standard
tape. If
tape. If the
the measured
measured volume
volume ofof aa dump
dump using the
the tape
tape is
is 4000 m , _iits
3
vynactual volume
actual in m3
volume in is
m3 is
using
(A) 3881
(A) 3881 (B) 3902
(B) 3902 (C) 3920
(C) 3920 (D) 4121
(D) 4121
4%
Q23
Q.23 A mine
A mine worker
worker inhales
inhales norma ereas, the
normal air; whereas, exhaled air
the exhaled air contains
contains 16.65%
16. 65% O
022 and
and 3.83%
3. 83% CO
C02.
2.
The respiratory
The respiratory quotient
quotient of
of brsitting or the
air‘l for
breathing the worker1s
worker is
(A) 0.23
(A) 0.23 (B) 0.89
(B) 0.89 (C) 0.99
(C) 0.99 (D) 1.13
(D) 1.13
'1: 1'
Q.24
Q.24 Block economic
Block economic Values of Rupees
values in Lakhs of Rupees for
for aa section
section of
of aa block economic model
block economic are shown
model are shown
below.
below. in.Lakhs

-1
-1 -1
-1 11 -1
-1 00 —1
-1
.
it -1
I
p—A 0O 0O 00 -1
-1 -2
-2
had-n:

-5
-5 -3
-3 -2
-2 55 -2
-2 -3
-3
1\\
At
At aa permissiblee slope
slope angle
angle of
of 1:1,
1:1, the
the optimum
optimum pit
pit value of the
value of section in
the section in Lakhs
Lakhs of
of Rupees
Rupees is
is
permissi.
r04)
(A) 00 (B) 11
(B) (C) 22
(C) (D) 33
(D)

Q2
Q.25 The bounda
The boundary ofof aa mine
mine is
is plotted on aa scale
plotted on scale of
of 1:2000.
1: 2000. If
If aa planimeter
planimeter measures
measures the
the plotted area as
plotted area as
5 8 cm
58 cm2,the
2
actual mine
, the actual mine area in m
area in m22 is
is
‘(A) 5 800
(A) 5800 (B) 11600
(B) 11600 (C) 23200
(C) 23200 (D) 29000
(D) 29000
F

MN
MN 6/15
6/15

www.examrace.com
www.examrace.com
2013
2013 MINING ENGG.
MINING ENGG. —MN
–MN

Q. 26
Q. 26 to
to Q.
Q. 55
55 carry
carry two
two marks
marks each.
each.

Q.26
Q.26 For aa shrinkage
For shrinkage stope
stope the
the following
following data
data values
values are
are given
given
Insitu tonnage 9000 tonne
Insitu grade 5.2 g/tonne
Average grade of waste 1.4 g/tonne
Loss of ore in the stope 10%
Dilution 20%

The grade
The grade at
at the mill-head in
the mill-head in g/tonne
g/tonne is
is ________ . "I...
r 'I
Q.27
Q.27 In an
In an experiment
experiment to
to determine
determine specific
specific gravity
gravity of
of aa soil
soil sample,
sample, the
the following
following data data is
iiobtairTéd:
obtained: / .'
1|
Mass of empty pycnometer 20.4 g 1
Mass of pycnometer with soil sample 51.6
{.6 g r
Mass of pycnometer with soil sample filled with water 88.6 g
“88.6 '
1
Mass of pycnometer filled with water It 1: III. 70. g
70.4
'. .- I.
The specific
The specific gravity of the
gravity of the sample
sample is
is __________ (
$
Q28
Q.28 A cylindrical
A cylindrical rock
rock specimen
specimen of
of diameter 54 mm
diameter 54 mm has Young’s modulus of 68.97 GPa and Poisson’s
ratio of 0.35. The rock specimen fails in uniaxial compression at a lateral strain of 0.01%. The axial
load at failure in kN is __________

Q.29
Q.29 An open
An open belt drive connects
belt drive connects two
two pulleys on par
pulleys on llel shafts
parallel shaft tthat are
are 3.6
3.6 m
m apart
apart as
as shown in the
shown in the
figure. The
figure. The diameters
diameters of
of the
the pulleys are 2.4
pulleys are 2.4 m
m and
and 1.6
1.6 m.
m. The
T e angle
angle of
picontact on the
contact on the smaller
smaller pulley
pulley
in degrees
in degrees is
is __________ "

r'

3.6m

Q30
Q.30 A two
A tonne mine
two tonne car is
ine car is released from the
released from the top of an
top of an incline
incline at
at aa height of 33 m
height of m as
as shown in the
shown in the
figure. The
figure. The mine
'ne car
car travels
travels 4545 m along the
m along inclined track
the inclined and another
track and another 85
85 m along the
m along the horizontal
horizontal
track before coming to
track to rest.
rest.
befq‘ei'oming

3.0 m
-------------------------------------------------------------- Jig
- /
85.0 m
h.
The specific rolling resistance of the car in N/tonne is ____________

MN
MN 7/15
7/15

www.examrace.com
www.examrace.com
2013
2013 MINING ENGG.
MINING ENGG. —MN
–MN

Q31
Q.31 A surface
A miner with
surface miner with 2.0
2.0 m cutting drum
m cutting drum width excavates coal
width excavates in windrowing
coal in mode from
windrowing mode from aa bench
bench
effective face
with effective
with face length
length 200
200 m.
m. The
The cutting
cutting speed
speed of
of the miner is
surface miner
the surface is 10
10 m/min and the
m/min and the
cutting depth
cutting depth 25
25 cm.
cm. The
The density
density of
of coal
coal is
is 1.4
1.4 tonne/m3. If the
tonne/m3. If the average turning time
average turning of the
time of machine
the machine
at the
at the face
face end
end is
is 5
5 min,
min, the
the rate of production
rate of production inin tonne/hour
tonne/hour becomes
becomes ____________

Q32
Q.32 A core
A core sample
sample ofof aa rock, having diameter
rock, having 54 mm
diameter 54 mm and
and length
length 108
108 mm,
mm, is
is subjected
subjected to
to axial
axial loading.
loading.
If the
If axial strain
the axial strain and
and Poisson’s
Poisson’s ratio are 2000 X 10
ratio are and 0.28 respectively,
10—6 and respectively, the
the value of
value of
volumetric strain
volumetric strain , represented in micro-strain
represented in micro-strain is
is ___________ 1

f‘ - .
- ‘k

Q33
Q.33 A flat
A flat bauxite deposit has
bauxite deposit thickness of
has thickness of 10
10 m
m with an average
with an average density
density of
of 2200
2200 kg/m3.1.The grade /1.
kg/m3. The grade
values and
values and the
the sample
sample coordinates
coordinates are
are as
as shown in the
shown in the table.
table. To
To carry
carry out
out reserve using -
reserve estimation using
triangular method,
triangular method, the
the triangles
triangles are
are constructed
constructed as
as shown in the
shown in figure.
the figure. erimation r
1..- |

Sample No 1 2 3 4 5 2
1
Alumina % 35 4O
40 39 47 42
x coordinate, m 0 200 0 200 5001
500
y coordinate, m 300 300 0O 0 0
3 4 5

The alumina
The alumina content
content in
in million
million tonnes, in the
tonnes, in the re ion comprising
region com isin the
the three
three triangles is _________
triangles is

Q34
Q.34 If the
If following linear
the following linear system
system of
of equations
equations has
has non-trivial
non-trivial solutions /
‘:
0
kpx+y+z= sf‘ums
fl! 2x+y—2z=_0
2 2 0
x+2y—3zé0
2 3 0
the value
the ofp
value of is
p is ‘1

(A) 11
(A) B 0
(B) 0 (C) -1
(C) -1 (D) -7
(D) -7
1(1)
Q35
i r

A bucket wheel excavator


ex vator'with 20 buckets of capacity
capacity 0.5
0.5 m
m3 each,
each, rotates
rotates at
at 5
5 rev/min. The bucket
3
Q.35 A bucket wheel with 20 buckets of rev/min. The bucket
fill factor
fill is_80%.
factor is T e excavator loads
80%. The loads on
on toto 1200 mm wide
1200 mm wide belt The cross-section
conveyor. The
belt conveyor. cross-section area
area
(m2)) ofof the on exctyator
terial on the elt is
is 0.1B
0.1B2,, where
where BB is
is the
the belt
belt width in m. The minimum speed of
of the
2 2
(m the material the belt width in m. The minimum speed the
belt
belt in m/s, to
in m/s toI
. avoid
avoid spillage
spillage of
of material
material is
is
1|
(A) 7.23
(A) 7 .23 “L (B) 5.79
(B) 5.79 (C) 4.63
(C) 4.63 (D) 3.70
(D) 3.70
J
(Q3;
Q.36 A simplex
A simplex tabl au\s\own
tableau shown below is generated
below is during the
generated during maximization of
the maximization of aa linear
linear programming
programming
problem
problem using implex method
using simplex method
'
Variable
Variable 2
Z x1
X1 x2
X2 x3
X3 x4
X4 RHS
RHS
z
Z 11 —1
-1 00 11 00 66
x2
X2 00 11/3
⁄3 11 11/3
⁄3 00 22
x4
X4 00 77/3
⁄3 00 —2/3
2⁄3 11 22
in.
After one
After one iteration,
iteration, the
the value of the
value of the objective
objective function
function becomes
becomes

(A)?
(A) (13113—1
(B) (C)
(C) % (D)

MN 8/15
8/15

www.examrace.com
ViANw.examrace.com
2013
2013 MINING ENGG.
MINING ENGG. —MN
–MN

Q.37
Q.37 The value
The value of

of fat/2 log (cosx) dx is
is

(A) —§log22
(A) (B) —§log22
(B) (C) glogz2
(C) (D) E10g22
(D)

Q38
Q.38 Given the
Given following,
the following,

Machine Component
Component

P. Dint
P. Dint header
header 1. Cowl
1. COWl 1
Q. Coal
Q. Coal plough
plough 2. Cutting
2. chain
Cutting chain . K
R. Road
R. Road header
header 3. Loading
3. apron
Loading apron ( .-
S. Shearer
Shearer 4. Static set
set of
of bits .-
.|
S. 4. Static bits
1| ,
/

I"
the correct match is
(A) P-4,Q-2,R-3,S-1
(A) P-4,Q-2,R-3,S-l (B) P-3,Q-4,R-2,S-1
(B) P-3,Q-4,R-2,S-l (C) P—2,Q—3,R—4,S-1
(C) P-2,Q-3,R-4,S-1 (D) P-2,Q-4,R-3,S-1
(D) P—2,Q_4,R—3,S-1.
. , ¢ 4:
Q.39
Q.39 Given the
Given following,
the following,

Rescue apparatus
Rescue apparatus Chai‘act 'stic
Characteristic (
. 'II
I! I.
P. Draeger
P. Draeger BG-4
BG-4 0 en circuit
1. Open circ it chemical
chemical oxygen
oxygen self-rescuer
self-rescuer
Q. MSA
Q. IW-65
MSA IW-65 2. Filter
2. Fi er type
typ self-rescuer
self-rescuer
R. Draeger
R. Draeger Pulmotor
Pulmotor 3. Self-contained
3. Se -cont ined breathing apparatus
breathing apparatus
S. Oxyboks
S. Oxyboks 4. Resuscitation
4. Re sci tion apparatus
apparatus

the correct match is


(A) P-3,
(A) P-3, Q-2,
Q-2, R-1,
R-l, S-4
S-4 (B) P-4,
(B) P-4, Q-1, — ,s‘g
Q- , R-2, S-3
(C) P-3,
(C) P-3, Q-2,
Q-2, R-4,
R-4, 8-1
S-1 I‘ 4‘ (D) P-1,
(D) P-l, Q-4,
og, R-3,
R-3, S-2
s—2
Q.40
Q.40 Given the
Given the following,
following,
‘:
Equation/formulzfl'fa'w
Equation/formula/law Application
Application

.‘L .
P. Bernoulli
P. Bemoulll _ecfuatlon
equation 1. Pressure
1. Pressure loss
loss in
in laminar
laminar flow
flow ofof fluid
fluid
Q. Poiseuille
Q. Poiseu' e Iequation
equation 2. Drag
2. Drag loss
loss due
due to
to regular obstructions in
regular obstructions in fluid
fluid flow
flow
R._Bromil
R. . ’s formula
Bromilow’s 3. Energy
3. Energy conservation
conservation inin ideal
ideal fluid
fluid flow
flow
ft‘mnula
S. Stokes
S. law a
okes law 4. Terminal
4. Terminal settling
settling velocity of fine
velocity of fine particles in fluid
particles in fluid

'
c,orree!tII match
the correct itch is

(A) P-3,
(A) P—3,Q—l, 2, S-4
Q-1, R-2, -4 (B) P-1,
(B) P-1, Q-3,
Q-3, R-2,
R—2, S-4
s—4
(C) P-2,
(C) P—2,Q—3, 4, S-1l
Q-3, R-4, (D) P-3,
(D) P-3, Q-1,
Q-1,R-4, s—2
R-4, S-2

MN 9/15
9/15

www.examrace.com
www.examrace.com
2013
2013 MINING ENGG.
MINING ENGG. —MN
–MN

Q.4l
Q.41 Four psychrometric
Four psychrometric processes P, Q,
processes P, Q, R
R and
and S
S are
are shown in the
shown in the psychrometric chart below.
psychrometric chart below.

S Q
S
Q 'L .
+ K
R '4 I.
.. I
r
These processes
These respectively represent
processes respectively represent 1' /
_ 1'

(A) dehumidification,
(A) dehumidification, humidification, sensible heating,
humidification, sensible heating, sensible
sensible cooling
cooling r r
(B) sensible
(B) sensible heating,
heating, humidification,
humidification, dehumidification,
dehumidification, sensible
sensible cooling
cooling 1" '
(C) dehumidification,
(C) dehumidification, sensible
sensible heating,
heating, sensible
sensible cooling,
cooling, humidification
humidification ", i J 1
(D) humidification,
(D) humidification, sensible
sensible heating,
heating, dehumidification,
dehumidification, sensible coolir'i‘g --
sensible cooling f'
f '-
"1-
Q.42
Q.42 Given the
Given the following differential equation
following differential equation 1 \ $
. '
'II
J

dzy +7
dy + 12 _0
12y —
r-
a a

the general
the general solution
solution is
is

(A) y = A6“ + Be'3x


(A) (B) y = _' + Be‘3x
(B)
(C) y = Ae3x + 86"”
(C) (D) y = A x +
(D)
Be?
Q.43
Q.43 Given the
Given the following,
following, i {t
t
Mining method
Mining metho‘l . Technique
Technique

P
P Thick seam
Thick sea-m}extraction
raction 11 Double unit
Double face
unit face
Q
Q Bord and
Bord and pillar extraction
pillar extraction 22 Jet cutting
Jet cutting
R
R face development
Longwall face development 33 slicing
Inclined slicing
Inclined
S
S mining
Longxgall mining
Hydraulic 44 Half moon
Half moon method
method
an
the correct
the correct match
match is'Hfrdra'lic
is .
1 ‘a
(A) P-3,
(A) P-3, Q-4, R-2, S-1
S-1 (B) P-1,
(B) P-l, Q-4,
Q-4, R-2,
R—2, S-3
S—3
(C) P-3,
. (C) P-3, Q-2,
--,2‘R1
3,-4,‘R-2, s—4
R-1, S-4 (D) P-3,
(D) P-3, Q-4,
Q-4, R-1,
R-l, S-2
3—2

Q.44 Given the


Given fol W
the following,
(.144
Excavati g/loading machine
Excavating/loading machine Transportation scheme
Transportation scheme

.. P
P Bucket Wheel
Bucket heel Excavator
Excavator 11 Mine tub
Mine tub
'F Q
Q Continu us Miner
Continuous Miner 22 Armoured flexible chain
Armoured flexible chain conveyor
conveyor
R
R Shearer
Shearer 33 Shiftable Conveyor
Shiftable Conveyor
SS Load Haul
Load aul Dumper
Dumper 4
4 Shuttle car
Shuttle car

Inn-the correct match


the correct match is
is

(A) P-3,
(A) P-3, Q-2,
Q-2, R-4,
R-4, S-1
8-1 (B) P-3,
(B) P-3, Q-4,
Q-4, R-2,
R-2, S-1
S-l
(C) P-3,
(C) P-3, Q-2,
Q-2, R-1,
R-l, S-4
S-4 (D) P-1,
(D) P-l, Q-4,
Q-4, R-3,
R-3, S-2
S—2

MN
MN 10/15
10/15

www.examrace.com
www.examrace.com
2013
2013 MINING ENGG.
MINING ENGG. —MN
–MN

Q45
Q.45 A sub-critical
A sub-critical subsidence
subsidence profile is shown
profile is in the
shown in figure below.
the figure below. The points A,
The points B, C,
A, B, C, and
and D
D represent
represent
respectively the
respectively the points of
points of

. ,
.1
a . i

‘5‘...
l | 1 -&
\\ 'l
* | 6.. b"
1 | i Of. '51?
‘1‘
l I I g," $19

..
\
1‘ it t A i 5." 1 Q‘/
1'

1}
\ l -' .E

/‘1
\ '.
‘1 {I

r- "r
i I ”I! ‘H t
,6 angle of draw \
‘ 1 ,' , / \ ; 1”
W3“! '\ I' l h
\ 1 1;. / ‘\ l') I
I
‘5. ll 1 K , WU “I
\ a 61' ‘~.
U ll!)

Excavation J 'I

l"
":K/I/K/f,./.- y-Zz/ :-‘:;/:/I_/

(A) zero
(A) zero vertical maximum tension,
displacement, maximum
vertical displacement, inflexion, maximum compression
tension, inflexion, is'sion
(B) inflexion,
(B) inflexion, maximum
maximum tension, maximum compression,
tension, maximum compress n,‘ze gaximu
zero vertical dis 1 cement
vertical displacement
maximum tension,
(C) maximum
(C) tension, inflexion,
inflexion, maximum
maximum compression,
compr on, zero
zeroll vertical
'ertical displacement
displacement
(D) maximum
(D) compression, maximum
maximum compression, maximum tension, i exio ' zero
tension, inflexion, zero vertical
ertical displacement
displacement

Q.46
Q.46 The uniaxial
The compressive strength
uniaxial compressive strength of
of aa limestone sample is3 80
limesto sample 80 MPa.
Pa. The
The sample
sample is
is confined
confined atat aa
pressure of 20
pressure of 20 MPa
MPa inin aa triaxial
triaxial compressive
compressive strength
st ength test.
tes Based
B ed on
on Hoek-Brown failure criteria
Hoek—Brown failure criteria
the maximum
the maximum principal stress at
principal stress at failure
failure inin MPa
MPa isis
(consider rock
(consider constants as
rock constants as m
m= = 7.88,
7.88, ss == 1.0
1.0 and = 0.5)
and aa = 0.5) (I

(A)117.9
(A) 117.9 (B)132.3
(B) 132.3 (C) 137.9
(C) 137.9 (D) 157.9
4“
Q47
Q.47 A wire
A of length
wire of length L
L1s cut into
is cut into two pieces to construct aa circle
to construct circle and
and an
an equilateral
equilateral triangle such that
triangle such that
the combined
the combined area
area is
is minimu
minimum. twlfiieésngth of
The length of the
the wire
wire used
used to construct the
to construct circle1s
the circle is

(A)
finL

( ) 9+\/§7t
(B) fljhe (C) _ (D)
√ (B)FQ9+\/§1I
√ (C)L2 9—\/§n

q. 1 11"
Data Questions
Common Data
Common ”a.
Questittns
Common Data
Common Data for
for Questions
Q estions 4848":and 49:
and 49:
Pressure characteristic
Pressure charact st' of mine-fan
of aa mine is given
fan is by, P = —0.06Q2
given by, 0.06 + 400, where
where P is
is the
the pressure in Pa
pressure in Pa and
and Q
the quantity in
the quantity m3/s.
in m he resistance
/s. The of the
resistance of the mine is 0.19
mine is 0.19 s/mg.
Ns2/m8.

(2.48
Q.48 The mine
The mine quantity
qua1\ in m3/s is
m3/s is

.- (A) 160.0
(A) 160.0 (B) 53.5
(B) 53.5 (C) 45.9
(C) 45.9 (D) 40.0

Q.49 rAn identical fan


An identical n is
is installed
installed in
in the
the mine
mine to operate in
to operate in series
series with the existing
with the fan. The
existing fan. The new
new mine
mine
quantity in m3/s
quantity in is
/s is
A) 75.6
(A) 756 (B) 56.7
(B) 56.7 (C) 50.8
(C) 50.8 (D) 30.2

MN 11/15
11/15

www.examrace.com
www.examrace.com
2013
2013 MINING ENGG.
MINING ENGG. —MN
–MN

Common Data
Common Data for
for Questions
Questions 50
50 and
and 51:
51:

The following
The following observations are taken
observations are during aa closed
taken during closed traverse.
traverse.

Side
Side Length (m)
Length (m) WCB
WCB
AB
AB 100
100 90°
90°
BC
BC 173
173 180°
180°
CA 200
200 3300
330°
* in.
Q.50
Q.50 The closing
The error of
closing error of the in mm
traverse in
the traverse is
mm is
'I

205 (B) 20.5


20-5 2.05
(C) 2.05 0.205
(D) 0.205 . -*r .1
(A) 205
(A) (B) (C) (D)
11 /‘
'‘
Q.51
Q.51 The reduced
The reduced bearing of the
bearing of the closing
closing error
error in
in degrees
degrees is
is

(A) 87.21 (13) 64.03 (C)14.04 (D) 0 _.


r“
. r"

.r 1
(A) 87.21 (B) 64.03 (C) 14.04 (D) 0
...._
i"‘
j 1'._
f'
Linked Answer
Linked Answer Questions
Questions

Questions 52
1
Statement for
Statement for Linked
Linked Answer
Answer Questions 52 and
and 53:
53: 1

Economic analysis
Economic of an
analysis of an iron
iron ore
ore deposit
deposit reveals that the
reveals that et vale
the net of th
value of ore is
the ore is related
related to
to the grade mined
the grade mined
as shown
as in the
shown in the table.
table.

Grade (%Fe)
Grade (%Fe) Net
Net value of ore (Rs/tonne)
value of onne)
64.5
64.5 3200
3200 orelf
60.2
60.2 1800
1800
Q.52
Q.52 Assuming linear
Assuming linear relationship
relationship betweentfieket value and
between the net value and grade,
gra‘e, the
the break-even cut-off grade
break-even cut-off grade
in %
in Fe is
% Fe is

(A) 52.2
(A) 52.2 (B) 54.7
(B) 54‘]; (C) 58.0
(C) 58.0 (D) 62.2
(D) 62.2
Q5 3
Q.53 Assuming that
Assuming the grade
that the gradefollows
follows normal distribution with
normal distribution mean 62.7%,
with mean 62. 7%, and
and standard
standard deviation
deviation
10. 0% (A
10.0% (A portion cf the
portion of standard normal
the standard distribution table1s
normal distribution given below),
table is given below),
' " *
f
zz .- 0.00 [4
0.00 0.01
0.01 0.02
0.02 0.03
0.03 0.04
0.04
0.6
0.6 0.72575
0.72575 0.72907
0.72907 0.73237
0.73237 0.73565
0.73565 0.73891
0.73891
0.7
0.7 1- 0.75803
0.75803 0.76115
0.76115 0.76424
0.76424 0.76730
0.76730 0.77035
0.77035
- 0.81 J,‘ 0.78814 0.79103 0.79389 0.79673 0.79954
_ _ 0.8 0.78814 0.7 9103 0.79389 0.79673 0.79954
- 0.9
0.9 0.81594 0.81859
0.81859 0.82121
0.82121 0.82381
0.82381 0.82639
0.82639
( 1.0
1.0 J\£81594
0.84134 0.84375
0.84375 0.84613
0.84613 0.84849
0.84849 0.85083
0.85083
1.84134
the percentag of waste
percentage of waste in the deposit
in the deposit based on the
based on the break-even cut-off grade
break-even cut-off grade is
is
the
(A) 78.8
(A) 78. 8 (B) 71.2
(B) 71.2 (C) 28.8
(C) 28.8 (D) 21.2
(D) 21.2
.1

(II—K
1r

MN 12/15
12/15

www.examrace.com
www.examrace.com
2013
2013 MINING ENGG.
MINING ENGG. —MN
–MN

Statement for
Statement for Linked
Linked Answer Questions 54
Answer Questions 54 and
and 55:
55:

A 4.6 m
A 4.6 m wide vein dipping
wide vein dipping at 8000 is
at 80 is mined
mined byby horizontal cut-and-fill stoping
horizontal cut-and-fill stoping method. The fill
method. The fill is
is to
to be
be
placed in the
placed in the stope
stope along
along the length of
the length of 46
46 mm and
and to
to aa height of 3.0
height of 3.0 m. If the
m. If the specific
specific weight of the
weight of fill
the fill
material is 15.86
material is 15.86 kN/m3 and the
kN/m3 and the porosity
porosity isis 35%,
35%, under fully saturated
under fully saturated conditions
conditions

Q.54 the volume


volume of
of water in the
water in the fill
fill in
in m
m3 is
is
3
Q.54 the

(A) 222.18
(A) 222.18 (B) 332.40
(B) 332.40 (C) 336.44
(C) 336.44 (D) 634.80
(D) 634.80
'4.
Q.55
Q.55 the mass
the mass of
of solids
solids in
in saturated
saturated fill
fill in
in tonnes is
tonnes is . K

(A) 820.00
(A) 820.00 (B) 804.10
(B) 804.10 (C) 799.30
(C) 799.30 (D) 788.80
(D) 788.80 r
_.
' 'I
"I 2'1.

r
Aptitude (GA)
General Aptitude
General (GA) Questions
Questions fr .
a J ‘l
I I.
56 —
Q. 56 Q. 60
60 carry
carry one
one mark
mark each.
each. “h.
Q. – Q.
1... F
1.
Q.56
Q.56 If 3 S X S 5 and
If and 8 S Y S 11 then
then which of the
which of follow'mggptions
the following is(TRUI£?
options is TRUE?
3 X 8 J"
(A)-S-S-
(A) _ r.
5 Y 5

3
(B) —< <
_
(B)11_ “<|>< OOIU'I

(ofis “<|>< S
(C)
U'l oo
_/
in
(D)
(D) S SE ‘(4‘ If“
U'lIUJ "<|><

Q.57
Q.57 The Headmaster
The Headmaster ___________
“‘ to speak to
to speak to you.
you.

Which of
Which the following
of the followihgloptions is incorrect
options is incorrect to
to complete
complete the
the above
above sentence?
sentence?
I i
(A) is
(A) is wanting
wanting 1*-
I f
(B) wants
(B) wants i ‘ P

'
. ,
' (C) want
(C) wantJ i

( (D) was
(D) want1
was wanting
a“
Q.58
Q.58 Mahatama Gandhi
Mahatama Ga dhi was for his
known for
was known humility as
his humility as
.- -r .
‘~ (A) hehe playe an important
played an important role in humiliating
role in humiliating exit
exit of
of British
British from
from India.
India.
JIIII._(A)
(B) he
(B) he worked for humanitarian
worke for causes.
humanitarian causes.

(C) he
(C) he displayed
displayed modesty
modesty in
in his interactions.
his interactions.
1.-
(l!) he
(D) he was fine human
was aa fine human being.
being.

MN
MN 13/15
13/15

www.examrace.com
www.examrace.com
2013
2013 MINING ENGG.
MINING ENGG. —MN
–MN

Q.59
Q.59 All engineering students should learn mechanics, mathematics and how
All how to
to do computation.
I II III IV
of the
Which of
Which the above
above underlined
underlined parts of the
parts of the sentence
sentence is
is not appropriate?
not appropriate?
(A) II
(A) (B) II
(B) 11 (C) III
(C) 111 (D) IV
(D) 1v
Q60
Q.60 Select the
Select the pair
pair that
that best expresses aa relationship
best expresses similar to
relationship similar that expressed
to that expressed in
in the
the pair:
pair:
water: pipe::
water: pipe::
(A) cart:
(A) cart: road
road (B) electricity:
(B) electricity: Wire
wire 'I.

a
(C) sea:
(C) sea: beach
beach (D) music:
(D) music: instrument
instrument
(C I.
-*
' 'I
'41 /1

Q. 61
Q. 61 to
to Q.
Q. 65
65 carry
carry two
two marks
marks each.
each. 1'"
r I
II
Q.61
Q.61 Velocity of
Velocity of an
an object
object fired
fired directly
directly in
in upward direction is
upward direction given by
is given by V {803—1132
80 32 t,, wh'gr?
where t (time)
(time)
is in
is in seconds.
seconds. When
When will
will the velocity be
the velocity be between 32 m/sec
between 32 64 m/sec? .- .,
and 64
m/sec and

(A) (1, 3/2)


(A) (1, 3/2) (B) (1/ :1") KJ,
(B) (1/2, 1) INST?“-
.1
(C) (1/2,
(C) (1/2, 3/2)
3/2) (D) (1, 3)

Q.62
Q.62 In aa factory,
In factory, two
two machines M1 and
machines M1 and M2
M2 manufacture
m ufactur 60%
600 and
and 40% of the
40% of the autocomponents
autocomponents
respectively. Out
respectively. Out of
of the
the total
total production, 2% off M1
production, 2% M1 and
and 3% M2 are
(V off M2 are found
found to
to be If aa
defective. If
be defective.
randomly drawn
randomly drawn autocomponent from the
autocomponent from the combined
combined lot
lot is
is found
ound defective, what is the
what is the probability
probability
d6}?ctive,
'
that it
that it was manufactured by
was manufactured M2?
by M2?
‘1
K
r
(A) 0.35
(A) 0.35 (B) 0.45
(B) 0.45:;‘ (C) 0.5
(C) 0.5 . (D) 0.4
(D) 0.4
Q.63
Q.63 Following table
Following table gives
gives data
data on
o'uists from different
tourists from different countries
countries Visiting India in
visiting India in the
the year 2011.
year 2011.

11 .' Country
Country Number of
Number of
i" I" Tourists
" ‘-- USA
Tourists
2000
{ USA 2000
- - 1 .- England
England 3500
3500
"
i Germany
Germany 1200
1200
'
I J- Italy
Italy 1100
1100
' - 1 Japan
Japan 2400
2400
__
Australia
Australia 2300
2300
( 1\ France
France 1000
1000

,Which
Which two co_ ntries contributed
two countries contributed to
to the
the one
one third
third of
of the
the total number of
total number of tourists
tourists who
who Visited in
India in
visited India
. ., 201 1?
2011?
F"
(A) USA
(A) USA and
an Japan
Japan

(B) USA
(B) and Australia
USA and Australia
‘-
((l') England
(C) England and
and France
France

(D) Japan
(D) Japan and
and Australia
Australia

MN
MN 14/15
14/15

www.examrace.com
www.examrace.com
2013
2013 MINING ENGG.
MINING ENGG. —MN
–MN

Q.64
Q.64 If |I—2X
If 9| = 3 then
2 + 9| then the
the possible
possible value of |I—XI| — X 2 would
value of would be:
be:
(A) 30
(A) 30 (B) -30
(B) —30 (C) -42 (D) 42

Q.65
Q.65 All professors
All are researchers
professors are researchers
Some scientists
Some scientists are
are professors
professors

of the
Which of
Which the given
given conclusions is logically
conclusions is logically valid
valid and is inferred
and is inferred from
from the
the above
above arguments:
- K
arguml‘ents:
(A) All
(A) All scientists
scientists are
are researchers
researchers
'
r.
(B) All
(B) All professors are scientists
professors are scientists
.‘I.’ {VI}
(C) Some
(C) Some researchers are scientists
researchers are scientists . LT-
(D) No
(D) follows
conclusion follows
No conclusion

END OF
END THE QUESTION
OF THE QUE PAPER

2:4 ' i

MN 15/15
15/15

www.examrace.com
www.examrace.com
2013 :: Answer
GATE 2013
GATE keys for
Answer keys MN -- Mining
for MN Mining Engineering
Engineering

Paper Q.No
Paper Q.No Key(s)/Value(s)
Key(s)/Value(s) Paper Q.No
Paper Q.No Key(s)/Va|ue(s)
Key(s)/Value(s)
MN
MN 11 C
C MN
MN 36
36 AA
MN
MN 2
2 B
B MN
MN 37
37 AA
MN
MN 3
3 D
D MN
MN 38
38 DD
MN
MN 4
4 AA MN
MN 39
39 CC
MN
MN 5
5 AA MN
MN 40
40 AA
MN
MN 6
6 CC MN
MN 41
41 DD
MN
MN 7
7 DD MN
MN 42
42 B
B
MN
MN 8
8 B
B MN
MN 43
43 DD 1
MN
MN 9
9 DD MN
MN 44
44 B
B r
MN 10 Marks to
to All MN 45 C -' J
~
MN 10 Marks All MN 45 C
1
MN 1.74 1 A“
MN
MN 11
11 C
C MN
MN 46
46 4 _'DD"
MN
MN 12
12 B
B MN 47 A
MN
MN 13
13 C
C ME’ 48 .1:
MN 48 D' D
MN
MN 14
14 D
D MN"
MN 'L49
49 C
C
MN
MN 15
15 AA [MNMN ‘0
50 I AA
MN
MN 16
16 B
B MN
If MN .1}
51 DD
MN
MN 17
17 AA MN
MN 5!
52 I B
B
MN
MN 16
18 C C MN
MN 5{
53 D
D
MN
MN 19
19 D D MN
MN 54
54 AA
MN
MN 20
20 B
B LL MN
MN 55
55 B
B
"
MN
MN 21
21 D
D MN
MN 56
56 B
B
MN
MN 22
22 ‘AE
A L MN
MN 57
57 C
C
MN
MN 23
23 (B.
B '1 MN
MN 58
58 C
C
MN 24 C - ‘ MN 59 D
'
MN 24 C MN 59 D
MN
MN 25 ‘1
25 C MN
MN 60
60 B
B
MN
MN 26
26 to 4.518
4.5 to 4.518 MN
MN 61
61 C
C
MN
MN 27
27 i€35": 2.45
2.35 to 2.45 MN
MN 62
62 C
C
MN . 28
MN 28 _,'44.8
44.8 to
to 45.5
45.5 MN
MN 63
63 C
C
MM
MN 29
29 i 166
166 to
to 169
169 MN
MN 64
64 B
B
MN
MN 30
30 j 225
225 to
to 231
231 MN
MN 65
65 C
C
M
MN 31
31 335 to
335 to 337
337
MN-
MN 32
32 0.000875 to
0.000875 to 0.000885
0.000685
'
MN
MN 33
33 0.9 to
0.9 to11
MN
MN 34
34 D
D
MN
MN 35
35 C
C

www.examrace.com
www.examrace.com
2013
2013 MINING ENGG.
MINING ENGG. —MN
–MN

MNzMINING ENGINEERING
MN:MINING ENGINEERING

Duration: Three Hours


Duration: Three Hours Maximum
Maximum Marks: 100
Marks:100

Please read
Please the following
read the instructions carefully:
following instructions carefully:

General Instructions:
General Instructions:

1. Total duration of examination is 180 minutes (3 hours).

2. The clock will be set at the server. The countdown timer in the top right corner of screen will
display the remaining time available for you to complete the examination. When the timer
reaches zero, the examination will end by itself. You will not be required to end or submit your
examination.

3. The Question Palette displayed on the right side of screen will show the status of each question
using one of the following symbols:

‘ You have not


You have not visited the question
visited the question yet.
yet.

You have not


You have answered the
not answered the question.
question.

You have answered


You have answered the
the question.
question.

You have NOT


You have answered the
NOT answered the question,
question, but
but have
have marked
marked the
the
for review.
question for
question review.

QQDG You have


You have answered
answered the
the question,
question, but
but marked it for
marked it for review.
review.

The Marked for Review status for a question simply indicates that you would like to look at that
question again. If
If a question is answered and Marked for for Review, your
your answer for for that
question will be considered in the evaluation.

Navigating to a Question

4. To answer a question, do the following:

a. Click on the question number in the Question Palette to go to that question directly.
b. Select an answer for a multiple choice
Choice type question. Use the virtual numeric keypad to
enter a number as answer for a numerical type question.
c. Click on Save and Next to save your answer for the current question and then go to the
next question.
d.
(1. Click on Mark for Review and Next to save your answer for the current question, mark
it for review, and then go to the next question.
e. Caution: Note that your answer for the current question will not be saved, if you
navigate to another question directly by clicking on its question number.

5. You can view all the questions by clicking on the Question Paper button. Note that the options
for multiple choice type questions will not be shown.

MN
MN 1/15
1/15
2013
2013 MINING ENGG.
MINING ENGG. —MN
–MN

Answering a Question

6. Procedure for answering a multiple choice type question:


a. To select your answer, click on the button of one of the options
b. To deselect your chosen answer, click on the button of the chosen option again or click
on the Clear Response button
c. To change your chosen answer, click on the button of another option
d. To save your answer, you MUST click on the Save and Next button
e. To mark the question for review, click on the Mark for Review and Next button. IfIf an
answer is selected for
for a question that is Marked for for Review, that answer will be
considered in the evaluation.

7. Procedure for answering a numerical answer type question:


a. To enter a number as your answer, use the virtual numerical keypad
b. A fraction (eg.,‐0.3
(eg.,-0.3 or ‐.3)
-.3) can be entered as an answer with or without ‘0’ before the
decimal point
c. To clear your answer, click on the Clear Response button
d. To save your answer, you MUST click on the Save and Next button
e. To mark the question for review, click on the Mark for Review and Next button. If If an
answer is entered forfor a question that is Marked for for Review, that answer will be
considered in the evaluation.

8. To change your answer to a question that has already been answered, first select that question
for answering and then follow the procedure for answering that type of question.

9. Note that ONLY Questions for which answers are saved or marked for review after answering
will be considered for evaluation.

MN 2/15
2/15
2013
2013 MINING ENGG.
MINING ENGG. —MN
–MN

specific instructions:
Paper specific

There are
1. There are aa total
total of
of 65
65 questions
questions carrying
carrying 100
100 marks.
marks. Questions
Questions areare of
of multiple
multiple choice
choice type or
type or
numerical answer type.
numerical answer A multiple
type. A multiple choice
choice type question will
type question have four
will have four choices
choices for
for the answer with
the answer with
only one
only one correct
correct choice.
choice. For
For numerical answer type
numerical answer type questions,
questions, the answer is
the answer is aa number and no
number and choices
no choices
will
will be given. A
be given. A number
number as as the
the answer
answer should
should be
be entered
entered using the virtual
using the virtual keyboard on the
keyboard on the monitor.
monitor.

Questions Q.1
2. Questions Q1 — Q.25 carry
– Q.25 carry 1mark
lmark each.
each. Questions Q.26 —
Questions Q.26 Q.55 carry
– Q.55 carry 2marks
2marks each.
each. The
The 2marks
2marks
questions include
questions include two
two pairs of common
pairs of common data
data questions
questions and
and two
two pairs of linked
pairs of linked answer questions. The
answer questions. The
answer to
answer to the second question
the second question of
of the linked answer
the linked answer questions
questions depends
depends on on the
the answer
answer to to the first
the first
question of
question of the
the pair. If the
pair. If the first
first question
question inin the linked pair
the linked is wrongly
pair is answered or
wrongly answered or is
is not attempted,
not attempted,
then the answer
then the answer toto the second question
the second question in the pair
in the pair will
will not
not be evaluated.
be evaluated.
3. Questions Q.56 —
Questions Q.56 Q.65 belong
– Q.65 belong to General Aptitude
to General Aptitude (GA)
(GA) section
section and
and carry
carry aa total of 15
total of 15 marks.
marks.
Questions Q.56 —
Questions Q.56 Q.60 carry
– Q.60 carry 1mark
lmark each,
each, and
and questions Q.61 —
questions Q.61 Q.65 carry
– Q.65 carry 2marks
2marks each.
each.

Questions not
4. Questions attempted will
not attempted will result
result in
in zero
zero mark.
mark. Wrong answers for
Wrong answers for multiple choice type
multiple choice questions
type questions
will
will result in NEGATIVE
result in NEGATIVE marks. For all
marks. For all 11 mark questions, ⅓
mark questions, mark will
1/3 mark will be deducted for
be deducted for each
each wrong
wrong
answer. For all
answer. For all 2
2 marks questions, ⅔
marks questions, mark will
2/3 mark will be deducted for
be deducted for each
each wrong answer. However,
wrong answer. However, in in the
the
case of
case of the linked answer
the linked answer question
question pair,
pair, there
there will
will be negative marks
be negative marks only
only for
for wrong answer to
wrong answer to the
the
first question
first question andand no
no negative
negative marks
marks forfor wrong answer to
wrong answer to the second question.
the second question. There
There is
is no negative
no negative
marking
marking forfor questions
questions of
of numerical
numerical answer
answer type.
type.

Calculator is
5. Calculator is allowed.
allowed. Charts,
Charts, graph
graph sheets
sheets or
or tables
tables are
are NOT in the
allowed in
NOT allowed examination hall.
the examination hall.

Do the
6. Do the rough
rough work in the
work in the Scribble
Scribble Pad
Pad provided.
provided.

MN 3/15
3/15
2013
2013 MINING ENGG.
MINING ENGG. —MN
–MN

Q. 11 —
Q. Q. 25
– Q. 25 carry
carry one
one mark
mark each.
each.

Q.1
Q.1 In the
In the Coward
Coward flammability
flammability diagram,
diagram, the
the respective
respective percentages of methane
percentages of and oxygen
methane and oxygen at
at the
the
nose limit
nose limit are
are
(A) 14.2,
(A) 14.2, 0.0
0.0 (B) 14.1,
(B) 14.1, 18.2
18.2 (C) 5.8,
(C) 5.8, 12.1
12.1 (D) 5.0,
(D) 5.0, 19.2
19.2
Q.2
Q.2 If the
If the transpose
transpose of
of aa matrix
matrix is
is equal
equal to
to its
its inverse,
inverse, then the matrix
then the is
matrix is
(A) symmetric
(A) symmetric (B) orthogonal
(B) orthogonal (C) skew
(C) skew symmetric
symmetric (D) singular
(D) singular

Q3
Q.3 In the
In the Moh’s
Moh’s scale
scale of
of hardness,
hardness, the minerals in
the minerals in increasing
increasing sequence
sequence of
of hardness
hardness are
are

(A) calcite,
(A) calcite, gypsum,
gypsum, topaz,
topaz, diamond
diamond
(B) topaz,
(B) topaz, gypsum,
gypsum, calcite,
calcite, diamond
diamond
(C) calcite,
(C) calcite, gypsum,
gypsum, diamond,
diamond, topaz
topaz
(D) gypsum,
(D) gypsum, calcite,
calcite, topaz,
topaz, diamond
diamond

Q4
Q.4 A ball
A of weight
ball of W is
weight W is supported
supported onon smooth
smooth walls as shown
walls as in the
shown in figure. R
following figure.
the following R11 and
and R
R22 are
are
reactions from the
reactions from walls 11 and
the walls 2. The
and 2. The free
free body diagram of
body diagram of the
the ball is represented
ball is represented by
by

(A) RR11
(A) (B)
(B) RR11 (C)
(C) (D)
(D) RR22
RR 22
RR22 RR22 RR11 RR11

W W
W W

Q5
Q.5 For aa 25
For 25 mm
mm diameter
diameter spherical
spherical charge,
charge, the maximum allowable
the maximum allowable charge
charge length in cm
length in cm is
is

(A) 15.0
(A) 15.0 (B) 25.0
(B) 25.0 (C) 30.0
(C) 30.0 (D) 150.0
(D) 150.0
Q6
Q.6 Long-hole drilling
Long-hole drilling with
with crater
crater blasting is used
blasting is for the
used for the construction
construction of
of
(A) winze
(A) winze (B) shaft
(B) shaft (C) raise
(C) raise (D) decline
(D) decline

Q7
Q.7 Rill stoping
Rill stoping method is aa form
method is form of
of
(A) block
(A) caving
block caving
(B) artificially
(B) artificially supported
supported stoping
stoping
(C) underhand
(C) stoping
underhand stoping
(D) overhand
(D) overhand stoping
stoping

Q.8
Q.8 Transit theodolite is a
(A) micro-optic
(A) micro-optic theodolite
theodolite
(B) theodolite
(B) theodolite with
with face
face left
left and
and face
face right
right reading facilities
reading facilities
(C) theodolite
(C) theodolite with
with stadia
stadia hairs
hairs
(D) theodolite
(D) theodolite with
with two vertical circles
two vertical circles

MN 4/15
4/15
2013
2013 MINING ENGG.
MINING ENGG. —MN
–MN

Q.9
Q.9 Incubation period
Incubation is NOT
period is NOT related to
related to
(A) crossing
(A) crossing point temperature of
point temperature of coal
coal
(B) panel
(B) size
panel size
(C) seam
(C) seam thickness
thickness
(D) explosibility
(D) explosibility of
of coal
coal dust
dust

Q.10
Q.10 The rotational
The speed and
rotational speed and cutting
cutting velocity of aa drill
velocity of drill are
are 350
350 rpm and 71.50
rpm and m/min respectively.
71.50 m/min The
respectively. The
diameter of
diameter of the
the rotary drill bit
rotary drill in mm
bit in mm is
is
(A) 65
(A) 65 (B) 67
(B) 67 (C) 68
(C) 68 (D) 70
(D) 70

Q.11
Q.11 The pressure
The on aa phreatic
pressure on surface is
phreatic surface is
(A) less
(A) less than
than atmospheric
atmospheric pressure
pressure
(B) greater
(B) greater than
than atmospheric
atmospheric pressure
pressure
(C) equal
(C) equal to atmospheric pressure
to atmospheric pressure
(D) independent
(D) independent ofof atmospheric
atmospheric pressure
pressure

Q. 12
Q.12 Events A
Events and B
A and B are
are independent
independent but
but NOT mutually exclusive.
NOT mutually exclusive. If
If the
the probabilities P(A) and
probabilities P(A) P(B) are
and P(B) are
0.5 and
0.5 and 0.4
0.4 respectively,
respectively, then P(A ∪
then P( U B)
B) is
is

(A) 0.6
(A) 0.6 (B) 0.7
(B) 0.7 (C) 0.8
(C) 0.8 (D) 0.9
(D) 0.9

Q. 13
Q.13 Among the
Among the following
following options,
options, the specific energy
the specific energy for
for rock-drilling is lowest
rock-drilling is in
lowest in

(A) rotary
(A) rotary diamond drilling
diamond drilling
(B) rotary
(B) rotary roller
roller drilling
drilling
(C) percussive
(C) drilling
percussive drilling
(D) jet piercing
(D) jet piercing

Q. 14
Q.14 Identify the
Identify the correct
correct statement
statement for
for aa ‘normal
‘normal distribution’.
distribution’.

(A) Mean
(A) Mean is
is greater
greater than
than mode
mode but less than
but less median
than median
(B) Mean
(B) Mean is
is less
less than
than mode
mode but greater than
but greater than median
median
(C) Mean
(C) Mean is
is greater
greater than
than mode and median
mode and median
(D) Mean,
(D) Mean, median
median and
and mode
mode are
are equal
equal

Q.15
Q.15 An emulsion
An emulsion explosive
explosive ofof specific gravity 1.25
specific gravity 1.25 is
is used for blasting
used for in an
blasting in iron ore
an iron ore formation
formation having
having
P-wave velocity
P-wave velocity of
of 3000
3000 m/s
m/s and
and specific gravity of
specific gravity of 3.20.
3.20. For
For an
an explosive
explosive impedance
impedance to
to rock
rock
impedance ratio
impedance ratio of
of 0.5,
0.5, the desired velocity
the desired velocity of
of detonation
detonation ofof the
the explosive
explosive in
in m/s
m/s is
is

(A) 3840
(A) 3840 (B) 4000
(B) 4000 (C) 4200
(C) 4200 (D) 7680
(D) 7680

Q. 16
Q.16 The number
The number of
of ways in which
ways in which the
the letters in the
letters in MINING can
word MINING
the word can be arranged is
be arranged is
(A) 90
(A) 90 (B) 180
(B) 180 (C) 360
(C) 360 (D) 720
(D) 720

Q. 17
Q.17 Under standard temperature
Under standard temperature and
and pressure conditions the
pressure conditions the theoretical height in
maximum height
theoretical maximum in m
m to
to which
which
water can
water can be lifted using
be lifted air-lift pump
an air-lift
using an is
pump is
(A) 10.33
(A) 10.33 (B) 9.61
(B) 9.61 (C) 7.45
(C) 7.45 (D) 6.05
(D) 6.05

Q. 18
Q.18 In aa belt
In conveyor system,
belt conveyor function of
system, function of the
the snub
snub pulley is to
pulley is to
(A) clean
(A) clean the
the inner
inner surface
surface of
of the
the belt
belt
(B) clean
(B) clean the
the outer
outer surface
surface of
of the
the belt
belt
(C) increase
(C) increase the angle of
the angle of contact
contact ofof belt
belt with drive drum
with drive drum
(D) decrease
(D) decrease the
the belt tension
belt tension

MN
MN 5/15
5/15
2013
2013 MINING ENGG.
MINING ENGG. —MN
–MN

Q.19
Q.19 In the
In the following
following figure,
figure, the
the coefficient
coefficient of
of kinetic friction between
kinetic friction between the trolley and
the trolley and the
the surface
surface is
is 0.04.
0.04.
When the
When the block is released
block is released from
from rest, the acceleration
rest, the acceleration of
of the
the trolley in m/s2
trolley in m/s2 becomes
becomes

80kg pulley
80kg

////////////

I 0kg
10kg

(A) 9.65
(A) 9.65 (B) 1.23
(B) 1.23 (C) 1.09
(C) 1.09 (D) 0.74
(D) 0.74

Q20
Q.20 Two meshing
Two meshing spur
spur gear
gear wheels of Module
wheels of Module 66 have
have 24 and 42
24 and 42 teeth. distance in
The distance
teeth. The in mm
mm between
between the
the
centres of
centres of the gear wheels
the gear is
wheels is
(A) 1000
(A) 1000 (B) 198
(B) 198 (C) 126
(C) 126 (D) 72
(D) 72

Q21
Q.21 In an
In an experiment
experiment to
to study
study coal
coal dust
dust explosibility,
explosibility, it
it is
is found
found that
that at
at least
least 3.0
3.0 g
g of
of limestone
limestone dust
dust
should be
should added to
be added to aa sample
sample of
of 2.0
2.0 g
g of
of coal
coal dust
dust to ensure that
to ensure that propagation
propagation ofof flame
flame does
does not
not take
take
place. The explosibility
place. The explosibility factor
factor of
of coal
coal dust
dust is
is

(A) 60.00
(A) 60.00 (B) 20.00
(B) 20.00 (C) 6.70
(C) 6.70 (D) 1.50
(D) 1.50

Q22
Q.22 A 20
A 20 m
m steel
steel tape
tape used in aa mine
used in survey is
mine survey is found
found to
to be 20 cm
be 20 cm short
short when compared with
when compared with aa standard
standard
tape. If
tape. If the
the measured
measured volume
volume ofof aa dump
dump using the tape
using the tape is
is 4000
4000 m3, its actual
m3, its actual volume in m3
volume in is
m3 is
(A) 3881
(A) 3881 (B) 3902
(B) 3902 (C) 3920
(C) 3920 (D) 4121
(D) 4121

Q23
Q.23 A mine
A mine worker inhales normal
worker inhales air; whereas,
normal air; whereas, the exhaled air
the exhaled air contains
contains 16.65%
16.65% O
022 and
and 3.83%
3.83% CO
C02.
2.
The respiratory
The respiratory quotient
quotient of
of breathing for the
breathing for the worker is
worker is
(A) 0.23
(A) 0.23 (B) 0.89
(B) 0.89 (C) 0.99
(C) 0.99 (D) 1.13
(D) 1.13
Q24
Q.24 Block economic
Block economic values in Lakhs
values in of Rupees
Lakhs of Rupees for
for aa section
section of
of aa block economic model
block economic are shown
model are shown
below.
below.
-1
-1 —1
-1 11 —1
-1 00 —1
-1

—1
-1 00 00 00 —1
-1 —2
-2

—5
-5 —3
-3 —2
-2 55 —2
-2 —3
-3

At aa permissible
At slope angle
permissible slope angle of
of 1:1,
1:1, the
the optimum
optimum pit
pit value of the
value of section in
the section in Lakhs
Lakhs of
of Rupees
Rupees is
is

(A) 00
(A) (B) 11
(B) (C) 22
(C) (D) 3

Q25
Q.25 The boundary
The boundary ofof aa mine
mine is
is plotted on aa scale
plotted on scale of
of 1:2000.
122000. If
If aa planimeter
planimeter measures
measures the
the plotted area as
plotted area as
22 . . 22 .
58 cm
58 cm , the
the actual
actual mine
m1ne area 1n m
area in m is 1s

(A) 5800
(A) 5800 (B) 11600
(B) 11600 (C) 23200
(C) 23200 (D) 29000

MN 6/15
6/15
2013
2013 MINING ENGG.
MINING ENGG. —MN
–MN

Q. 26
Q. 26 to
to Q.
Q. 55
55 carry
carry two
two marks
marks each.
each.

Q.26
Q.26 For aa shrinkage
For shrinkage stope
stope the
the following
following data
data values
values are
are given
given
Insitu tonnage 9000 tonne
Insitu grade 5.2 g/tonne
Average grade of waste 1.4 g/tonne
Loss of ore in the stope 10%
Dilution 20%

The grade
The grade at
at the mill-head in
the mill-head in g/tonne
g/tonne is
is ________

Q.27
Q.27 In an
In an experiment
experiment to
to determine
determine specific
specific gravity
gravity of
of aa soil
soil sample,
sample, the
the following
following data
data is
is obtained:
obtained:
Mass of empty pycnometer 20.4 g
Mass of pycnometer with soil sample 51.6 g
Mass of pycnometer with soil sample filled with water 88.6 g
Mass of pycnometer filled with water 70.4 g

The specific
The specific gravity of the
gravity of the sample
sample is
is __________

Q28
Q.28 A cylindrical
A cylindrical rock
rock specimen
specimen of
of diameter
diameter 5454 mm
mm has
has Young’s
Young’s modulus
modulus of of 68.97
68.97 GPa
GPa and
and Poisson’s
Poisson’s
ratio of
ratio of 0.35.
0.35. The
The rock
rock specimen
specimen fails
fails in
in uniaXial compression at
uniaxial compression at aa lateral
lateral strain
strain of
of 0.01%.
0.01%. The
The axial
axial
load at
load at failure
failure in
in kN is __________
kN is

Q.29
Q.29 An open
An open belt drive connects
belt drive connects two
two pulleys on parallel
pulleys on shafts that
parallel shafts are 3.6
that are 3.6 m
m apart
apart as
as shown in the
shown in the
figure. The
figure. The diameters
diameters of
of the
the pulleys are 2.4
pulleys are 2.4 m
m and
and 1.6
1.6 m.
m. The
The angle
angle of
of contact
contact on
on the
the smaller
smaller pulley
pulley
in degrees
in degrees is
is __________

3.6m

Q30
Q.30 A two
A tonne mine
two tonne car is
mine car is released from the
released from the top of an
top of an incline
incline at
at aa height of 33 m
height of m as shown in
as shown in the
the
figure. The
figure. The mine
mine car
car travels
travels 4545 m along the
m along inclined track
the inclined and another
track and another 85
85 m along the
m along the horizontal
horizontal
track before
track coming to
before coming to rest.
rest.

3.0 m

85.0 m

The specific rolling resistance of the car in N/tonne is ____________

MN
MN 7/15
7/15
2013
2013 MINING ENGG.
MINING ENGG. —MN
–MN

Q31
Q.31 A surface
A miner with
surface miner with 2.0
2.0 m cutting drum
m cutting drum width excavates coal
width excavates in windrowing
coal in mode from
windrowing mode from aa bench
bench
effective face
with effective
with face length
length 200
200 m.
m. The
The cutting
cutting speed
speed of
of the miner is
surface miner
the surface is 10
10 m/min and the
m/min and the
cutting depth
cutting depth 25
25 cm.
cm. The
The density
density of
of coal
coal is
is 1.4 tonne/m3.. If
1.4 tonne/m3 If the
the average turning time
average turning of the
time of machine
the machine
at the
at the face
face end
end is
is 5
5 min,
min, the
the rate
rate of
of production
production inin tonne/hour
tonne/hour becomes
becomes ____________

Q32
Q.32 A core
A core sample
sample of rock, having
of aa rock, having diameter 54 mm
diameter 54 mm and
and length
length 108
108 mm,
mm, is
is subjected
subjected to axial loading.
to axial loading.
If the
If the axial
axial strain
strain and
and Poisson’s
Poisson’s ratio are 2000 X 10
ratio are and 0.28 respectively,
10—6 and respectively, the
the value of
value of
volumetric strain
volumetric strain , represented in micro-strain
represented in micro-strain is
is ___________

Q33
Q.33 A flat
A flat bauxite deposit has
bauxite deposit thickness of
has thickness of 10
10 m
m with an average
with an average density
density of
of 2200
2200 kg/m3. The grade
kg/m3. The grade
values and
values and the
the sample
sample coordinates
coordinates are
are as
as shown in the
shown in table. To
the table. To carry
carry out
out reserve estimation using
reserve estimation using
triangular method,
triangular method, the
the triangles
triangles are
are constructed
constructed as
as shown in the
shown in figure.
the figure.

Sample No 1 2 3 44 5 2
1
Alumina%
Alumina % 35 40 39 47 42
x coordinate, m 0 200 0 200 500
yycoordinate,
coordinate, m 300 300 0 0 0
33 Z4 5U1(

The alumina
The alumina content
content in
in million
million tonnes, in the
tonnes, in the region comprising the
region comprising the three
three triangles is _________
triangles is

Q34
Q.34 If the
If following linear
the following linear system
system of
of equations
equations has
has non-trivial
non-trivial solutions
solutions

px+y+z=00
2 2 0
2x+y—22=0
2 3 0
x+2y—3z=0
the value
the ofp
value of is
p is

(A) 11
(A) (B) 00
(B) (C) -1
(C) -1 (D) -7
(D) -7
Q35
Q.35 A bucket
A wheel excavator
bucket wheel excavator with 20 buckets
with 20 buckets ofof capacity
capacity 0.5
0.5 m
m3 each,
3
each, rotates
rotates at
at 5
5 rev/min. The bucket
rev/min. The bucket
fill factor
fill is 80%.
factor is 80%. The
The excavator
excavator loads
loads on
on toto 1200 mm wide
1200 mm wide belt The cross-section
conveyor. The
belt conveyor. cross-section area
area
(m2)) ofof the material on
on the
the belt is 0.1B
0.1B2,, where
where BB is
is the
the belt
belt width in m. The minimum speed of
of the
2 2
(m the material belt is width in m. The minimum speed the
belt
belt in m/s to
in m/s to avoid
avoid spillage
spillage of
of material
material isis

(A) 7.23
(A) 7.23 (B) 5.79
(B) 5.79 (C) 4.63
(C) 4.63 (D) 3.70
(D) 3.70

Q36
Q.36 A simplex
A simplex tableau shown below
tableau shown is generated
below is during the
generated during maximization of
the maximization of aa linear
linear programming
programming
problem
problem using simplex method
using simplex method

Variable
Variable Z
Z X11
X X2
X2 X3
X3 X4
X4 RHS
RHS
Z
Z 11 -1
-1 00 11 00 66
X2
X2 00 1⁄/3 11 /3
1⁄ 00 22
X4
X4 00 77/
⁄3 00 — 2⁄/3 11 22

After one
After one iteration,
iteration, the
the value of the
value of the objective
objective function
function becomes
becomes
48 11 22 2
(A) 7
(A) (B) 3
(B) (C) 7
(C) (D) g
(D)

MN 8/15
2013
2013 MINING ENGG.
MINING ENGG. —MN
–MN

Q.37
Q.37 The value
The value of

of fat/2 log (cosx) dx is
is

(A) —§log22
(A) (B) —§log22
(B) (C) glogz2
(C) (D) E10g22
(D)

Q.38
Q.38 Given the
Given following,
the following,

Machine Component
Component

P. Dint
P. Dint header
header 1. Cowl
1. Cowl
Q. Coal
Q. Coal plough
plough 2. Cutting
2. chain
Cutting chain
R. Road
R. Road header
header 3. Loading
3. apron
Loading apron
S. Shearer
S. Shearer 4. Static
4. Static set
set of
of bits
bits

the correct match is


(A) P-4,Q-2,R-3,S-1
(A) P-4,Q-2,R—3,S-1 (B) P-3,Q-4,R-2,S-1
(B) P-3,Q-4,R—2,S-l (C) P-2,Q-3,R-4,S-1
(C) P-2,Q-3,R—4,S-l (D) P-2,Q-4,R-3,S-1
(D) P-2,Q-4,R—3,S-l

Q.39
Q.39 Given the
Given following,
the following,

Rescue apparatus
Rescue apparatus Characteristic

P. Draeger
P. Draeger BG-4
BG-4 1. Open
1. Open circuit
circuit chemical
chemical oxygen
oxygen self-rescuer
self-rescuer
Q. MSA
Q. IW-65
MSA IW-65 2. Filter
2. Filter type
type self-rescuer
self-rescuer
R. Draeger
R. Draeger Pulmotor
Pulmotor 3. Self-contained
3. Self-contained breathing apparatus
breathing apparatus
S. Oxyboks
S. Oxyboks 4. Resuscitation
4. Resuscitation apparatus
apparatus

the correct match is


(A) P-3,
(A) P-3, Q-2,
Q-2,R-1, s—4
R-1, S-4 (B) P-4,
(B) 13—4, Q-1,
Q-1,R-2, s—3
R-2, S-3
(C) P-3,
(C) Q-2, R-4,
P-3, Q-2, R—4, S-1
s—1 (D) P-1,
(D) 13—1, Q-4,
Q-4, R-3,
R—3, S-2
s—2
Q.40
Q.40 Given the
Given the following,
following,

Equation/formula/law
Equation/formula/law Application
Application

P. Bernoulli
P. Bernoulli equation
equation 1. Pressure
1. Pressure loss
loss in
in laminar
laminar flow
flow ofof fluid
fluid
Q. Poiseuille
Q. Poiseuille equation
equation 2. Drag
2. Drag loss
loss due
due to
to regular obstructions in
regular obstructions in fluid
fluid flow
flow
R. Bromilow’s
R. Bromilow’s formula
formula 3. Energy
3. Energy conservation
conservation inin ideal
ideal fluid
fluid flow
flow
S. Stokes
S. Stokes law
law 4. Terminal
4. Terminal settling
settling velocity of fine
velocity of fine particles in fluid
particles in fluid

the correct match is


(A) P-3,
(A) P-3, Q-1,
Q-1,R-2, 34
R-2, S-4 (B) P-1,
(B) P-l, Q-3,
Q-3, R-2,
R—2, S-4
s—4
P-2, Q-3,
(C) P-2,
(C) Q-3, R-4,
R-4, S-1
s—1 (D) P-3,
(D) P-3, Q-1,
Q-l, R-4,
R—4, S-2
s—2

MN 9/15
9/15
2013
2013 MINING ENGG.
MINING ENGG. —MN
–MN

Q.41
Q.41 Four psychrometric
Four psychrometric processes P, Q,
processes P, Q, R
R and
and S
S are
are shown in the
shown in the psychrometric chart below.
psychrometric chart below.

P
S
Q

These processes
These respectively represent
processes respectively represent

(A) dehumidification,
(A) dehumidification, humidification,
humidification, sensible
sensible heating,
heating, sensible
sensible cooling
cooling
(B) sensible
(B) sensible heating,
heating, humidification,
humidification, dehumidification,
dehumidification, sensible
sensible cooling
cooling
(C) dehumidification,
(C) dehumidification, sensible
sensible heating,
heating, sensible
sensible cooling,
cooling, humidification
humidification
(D) humidification,
(D) humidification, sensible
sensible heating,
heating, dehumidification,
dehumidification, sensible
sensible cooling
cooling

Q.42
Q.42 Given the
Given the following
following differential
differential equation
equation

7 d_y 12 _ 0
dz_y +7dx+12y—0
dx2

the general
the general solution
solution is
is

(A) y = A6” + Be‘3x


(A) (B) y = Ae—4x + Be‘3x
(B)
(C) y = Ae3x + 86"”
(C) (D) y = A6“ + Be3x
(D)

Q.43
Q.43 Given the
Given following,
the following,
Mining method
Mining method Technique
Technique

P
P Thick seam
Thick seam extraction
extraction 11 Double unit
Double face
unit face
Q
Q Bord and
Bord and pillar extraction
pillar extraction 22 Jet cutting
Jet cutting
R
R Longwall face
Longwall face development
development 33 Inclined slicing
Inclined slicing
S
S Hydraulic mining
Hydraulic mining 44 Half moon
Half moon method
method
the correct
the correct match
match is
is
(A) P-3,
(A) P-3, Q-4,
Q-4, R-2,
R-2, S-1
8-1 (B) P-1,
(B) P-l, Q-4,
Q-4, R-2,
R-2, S-3
S—3
(C) P-3,
(C) P-3, Q-2,
Q-2, R-1,
R-l, S-4
S-4 (D) P-3,
(D) P-3, Q-4,
Q-4, R-1,
R—l, S-2
S-2

Q.44
Q.44 Given the
Given following,
the following,

Excavating/loading machine
Excavating/loading machine Transportation scheme
Transportation scheme

P
P Bucket Wheel
Bucket Wheel Excavator
Excavator 11 Mine tub
Mine tub
Q
Q Continuous Miner
Continuous Miner 22 Armoured flexible chain
Armoured flexible chain conveyor
conveyor
R
R Shearer
Shearer 33 Shiftable Conveyor
Shiftable Conveyor
SS Load Haul
Load Haul Dumper
Dumper 44 Shuttle car
Shuttle car

the correct
the correct match
match is
is
(A) P-3,
(A) P-3, Q-2,
Q-2, R-4,
R-4, S-1
8-1 (B) P-3,
(B) P-3, Q-4,
Q-4, R-2,
R-2, S-1
S—l
(C) P-3,
(C) P-3, Q-2,
Q-2, R-1,
R-l, S-4
S-4 (D) P-1,
(D) P-l, Q-4,
Q-4, R-3,
R—3, S-2
S-2

MN 10/15
10/15
2013
2013 MINING ENGG.
MINING ENGG. —MN
–MN

Q45
Q.45 A sub-critical
A sub-critical subsidence
subsidence profile is shown
profile is in the
shown in figure below.
the figure below. The points A,
The points B, C,
A, B, C, and
and D
D represent
represent
respectively the
respectively the points of
points of

\ IL
. i
\I ‘l I ll ll."
13 = angle of draw —__L I. , .1”
M3 M. I. | . ,1’
\~
I 1" I
1

a—
II

gr"'1 3! .'I .-
\

1i 1" 7“ angle of break


\a.
-.a I ‘
i—

I“
~ '
. i '
I I... -’ Q
_ (I ‘5‘
. .
III

ofl'fiz’fx/ ‘ -
\Illl-IE \ui: .'

@999»s '/ I
\‘-.,.,_.. "I“ ,
/

Excavation --- -:-c-r~ .


"‘-':'r:' '-"-":‘.'|5'-'

- -'
fix-”xix: ..»../;.-< - : c. :- :.--:.-’. r.., x

(A) zero
(A) zero vertical
vertical displacement, maximum tension,
displacement, maximum inflexion, maximum
tension, inflexion, maximum compression
compression
(B) inflexion,
(B) inflexion, maximum
maximum tension,
tension, maximum compression, zero
maximum compression, zero vertical displacement
vertical displacement
(C) maximum
(C) maximum tension,
tension, inflexion,
inflexion, maximum
maximum compression,
compression, zero
zero vertical displacement
vertical displacement
(D) maximum
(D) maximum compression,
compression, maximum
maximum tension, inflexion, zero
tension, inflexion, zero vertical displacement
vertical displacement

Q.46
Q.46 The uniaxial
The compressive strength
uniaxial compressive strength of limestone sample
of aa limestone sample is
is 80
80 MPa.
MPa. The
The sample
sample is
is confined
confined atat aa
pressure of 20
pressure of 20 MPa
MPa inin aa triaxial compressive strength
triaxial compressive strength test.
test. Based
Based on
on Hoek-Brown failure criteria
Hoek-Brown failure criteria
the maximum
the maximum principal stress at
principal stress at failure
failure inin MPa
MPa isis
(consider rock
(consider constants as
rock constants as m
m= = 7.88,
7.88, ss == 1.0
1.0 and = 0.5)
and aa = 0.5)

(A) 117.9
(A) 117.9 (B) 132.3
(B) 132.3 (C) 137.9
(C) 137.9 (D) 157.9
(D) 157.9
Q47
Q.47 A wire
A of length
wire of length L
L is
is cut
cut into
into two
two pieces to construct
pieces to construct aa circle
circle and
and an
an equilateral
equilateral triangle such that
triangle such that
the combined
the combined area
area is
is minimum. The length
minimum. The length of
of the
the wire
wire used
used to construct the
to construct circle is
the circle is
finL
√ 9L 5 18L
(A) (B)
(B) 9+\/§7r
(C)
(C) 2 (D)
(D)
(A) 9+\/§7r
√ √ 9—\/§7T

Data Questions
Common Data
Common Questions

Common Data
Common Data for
for Questions
Questions 4848 and
and 49:
49:
Pressure characteristic
Pressure of aa mine
characteristic of mine fan
fan is
is given by, P = —0.06Q2
given by, 0.06 + 400, where
where P is
is the
the pressure in Pa
pressure in Pa and
and Q
the quantity in
the quantity in m
m3/s.
3
The resistance
/s. The of the
resistance of the mine is 0.19
mine is 0.19 s/mg.
Ns2/m8.

Q.48
Q.48 The mine
The mine quantity in m
quantity in m3/s
3
is
/s is

(A) 160.0
(A) 160.0 (B) 53.5
(B) 53.5 (C) 45.9
(C) 45.9 (D) 40.0
(D) 40.0

Q.49
Q.49 An identical
An identical fan
fan isis installed
installed in
in the
the mine
mine to operate in
to operate in series
series with the existing
with the fan. The
existing fan. The new
new mine
mine
quantity in m
quantity in m3/s
3
is
/s is
(A) 75.6
(A) 75.6 (B) 56.7
(B) 56.7 (C) 50.8
(C) 50.8 (D) 30.2
(D) 30.2

MN
MN ll/15
11/15
2013
2013 MINING ENGG.
MINING ENGG. —MN
–MN

Common Data
Common Data for
for Questions
Questions 50
50 and
and 51:
51:

The following
The following observations are taken
observations are during aa closed
taken during closed traverse.
traverse.

Side
Side Length (m)
Length (m) WCB
WCB
AB
AB 100
100 90°
90°
BC
BC 173
173 180°
180°
CA
CA 200
200 330°
330°

Q.50
Q.50 The closing
The error of
closing error of the in mm
traverse in
the traverse mm is
is

(A) 205
(A) 205 (B) 20.5
(B) 20.5 (C) 2.05
(C) 2.05 (D) 0.205
(D) 0.205

Q.51
Q.51 The reduced
The reduced bearing of the
bearing of closing error
the closing error in
in degrees
degrees is
is

(A) 87.21
(A) 87.21 (B) 64.03
(B) 64.03 (C) 14.04
(C) 14.04 (D) 00
(D)

Linked Answer
Linked Answer Questions
Questions

Statement for
Statement for Linked
Linked Answer Questions 52
Answer Questions 52 and
and 53:
53:

Economic analysis
Economic of an
analysis of an iron
iron ore
ore deposit
deposit reveals that the
reveals that the net
net value of the
value of ore is
the ore is related to the
related to grade mined
the grade mined
as shown
as in the
shown in the table.
table.

Grade (%Fe)
Grade (%Fe) Net
Net value of ore
value of (Rs/tonne)
ore (Rs/tonne)
64.5
64.5 3200
3200
60.2
60.2 1800
1800
Q.52
Q.52 Assuming linear
Assuming linear relationship
relationship between
between the net value
the net value and
and grade,
grade, the
the break-even cut-off grade
break-even cut-off grade
in %
in Fe is
% Fe is

(A) 52.2
(A) 52.2 (B) 54.7
(B) 54.7 (C) 58.0
(C) 58.0 (D) 62.2
(D) 62.2

Q5 3
Q.53 Assuming that
Assuming the grade
that the grade follows
follows normal
normal distribution
distribution with mean 62.7%,
with mean 62.7%, and
and standard
standard deviation
deviation
10.0% (A
10.0% (A portion of the
portion of the standard
standard normal distribution table
normal distribution is given
table is given below),
below),

zz 0.00
0.00 0.01
0.01 0.02
0.02 0.03
0.03 0.04
0.04
0.6
0.6 0.72575
0.72575 0.72907
0.72907 0.73237
0.73237 0.73565
0.73565 0.73891
0.73891
0.7
0.7 0.75803
0.75803 0.76115
0.76115 0.76424
0.76424 0.76730
0.76730 0.77035
0.77035
0.8
0.8 0.78814
0.78814 0.7 9103
0.7 9103 0.79389
0.79389 0.79673
0.79673 0.79954
0.79954
0.9
0.9 0.81594
0.81594 0.81859
0.81859 0.82121
0.82121 0.82381
0.82381 0.82639
0.82639
1.0
1.0 0.84134
0.84134 0.84375
0.84375 0.84613
0.84613 0.84849
0.84849 0.85083
0.85083

the
the percentage of waste
percentage of waste in the deposit
in the deposit based on the
based on the break-even cut-off grade
break-even cut-off grade is
is

(A) 78.8
(A) 78.8 (B) 71.2
(B) 71.2 (C) 28.8
(C) 28.8 (D) 21.2
(D) 21.2

MN 12/15
12/15
2013
2013 MINING ENGG.
MINING ENGG. —MN
–MN

Statement for
Statement for Linked
Linked Answer Questions 54
Answer Questions 54 and
and 55:
55:

A 4.6 m
A 4.6 m wide vein dipping
wide vein dipping at 8000 is
at 80 is mined
mined by horizontal cut-and-fill
by horizontal cut-and-fill stoping
stoping method. The fill
method. The fill is
is to
to be
be
placed in the
placed in the stope
stope along
along the length of
the length of 46
46 mm and
and to
to aa height of 3.0
height of 3.0 m. If the
m. If specific weight
the specific of the
weight of fill
the fill
material is
material is 15.86
15.86 kN/m3 and the
kN/m3 and the porosity
porosity isis 35%,
35%, under fully saturated
under fully saturated conditions
conditions

Q.54 the volume


volume of
of water in the
water in the fill
fill in
in m
m3 is
is
3
Q.54 the

(A) 222.18
(A) 222.18 (B) 332.40
(B) 332.40 (C) 336.44
(C) 336.44 (D) 634.80
(D) 634.80

Q5 5
Q.55 the mass
the mass of
of solids
solids in
in saturated
saturated fill
fill in
in tonnes is
tonnes is
(A) 820.00
(A) 820.00 (B) 804.10
(B) 804.10 (C) 799.30
(C) 799.30 (D) 788.80
(D) 788.80

Aptitude (GA)
General Aptitude
General (GA) Questions
Questions

Q. 56 —
Q. 56 Q. 60
– Q. 60 carry
carry one
one mark
mark each.
each.

Q.56
Q.56 If 3 S X S 5 and
If and 8 S Y S 11 then
then which of the
which of following options
the following options is
is TRUE?
TRUE?

(m—s—s—
3
(A)
5
X
Y
8
5

3
(B) —< <
_
(B)11_ "<|>< OOIU'I

3
(C) —< <
_
(C)11_ "<|>< U'l oo

X 8
(D)
(D) S; S
U'lICJJ 11

Q.57
Q.57 The Headmaster
The Headmaster ___________ to speak to
to speak to you.
you.

Which of
Which the following
of the following options
options is
is incorrect
incorrect to
to complete
complete the
the above
above sentence?
sentence?
(A) is
(A) is wanting
wanting

(B) wants
(B) wants

(C) want
(C) want

(D) was
(D) wanting
was wanting

Q.58
Q.58 Mahatama Gandhi
Mahatama Gandhi was
was known for his
known for humility as
his humility as

(A) he
(A) he played an important
played an important role in humiliating
role in exit of
humiliating exit of British
British from
from India.
India.

(B) he
(B) he worked for humanitarian
worked for causes.
humanitarian causes.

(C) he
(C) he displayed
displayed modesty
modesty in
in his interactions.
his interactions.

(D) he
(D) he was fine human
was aa fine human being.
being.

MN
MN 13/15
13/15
2013
2013 MINING ENGG.
MINING ENGG. —MN
–MN

Q59
Q.59 All engineering
All engineering students
students should
should learn
learn mechanics,
mechanics, mathematics
mathematics and
and how
how to do computation.
to do computation.
II II
II III
III IV
IV
of the
Which of
Which the above
above underlined
underlined parts of the
parts of the sentence
sentence is
is not appropriate?
not appropriate?
(A) I1
(A) (B) II
(B) 11 (C) III
(C) 111 (D) IV
(D) 1v
Q.60
Q.60 Select the
Select the pair
pair that
that best expresses aa relationship
best expresses similar to
relationship similar that expressed
to that expressed in
in the
the pair:
pair:
water: pipe::
water: pipe::
(A) cart:
(A) cart: road
road (B) electricity:
(B) electricity: wire
wire
(C) sea:
(C) sea: beach
beach (D) music:
(D) music: instrument
instrument

Q. 61
Q. 61 to
to Q.
Q. 65
65 carry
carry two
two marks
marks each.
each.

Q.6l
Q.61 Velocity of
Velocity of an
an object
object fired
fired directly
directly in
in upward direction is
upward direction is given by V = 80 — 32 t,, where
given by where t (time)
(time)
is in
is in seconds.
seconds. When
When will
will the velocity be
the velocity be between 32 m/sec
between 32 and 64
m/sec and 64 m/sec?
m/sec?

(A) (1,
(A) (1, 3/2)
3/2) (B) (1/2,
(B) 1)
(1/2, 1)
(C) (1/2,
(C) (1/2, 3/2)
3/2) (D) (1,
(D) (1, 3)
3)
Q.62
Q.62 In aa factory,
In factory, two
two machines M1 and
machines M1 and M2
M2 manufacture
manufacture 60%
60% and 40% of
and 40% of the
the autocomponents
autocomponents
respectively. Out
respectively. Out of
of the
the total
total production, 2% of
production, 2% M1 and
of M1 and 3%
3% of M2 are
of M2 are found
found to
to be If aa
defective. If
be defective.
randomly drawn
randomly drawn autocomponent from the
autocomponent from the combined
combined lot
lot is
is found
found defective,
defective, what is the
what is the probability
probability
that it
that it was manufactured by
was manufactured M2?
by M2?

(A) 0.35
(A) 0.35 (B) 0.45
(B) 0.45 (C) 0.5
(C) 0.5 (D) 0.4
(D) 0.4

Q.63
Q.63 Following table
Following table gives
gives data
data on
on tourists from different
tourists from different countries
countries visiting India in
visiting India in the
the year 2011.
year 2011.

Country
Country Number
Number ofof
Tourists
Tourists
USA
USA 2000
2000
England
England 3500
3500
Germany
Germany 1200
1200
Italy
Italy 1 100
1100
Japan
Japan 2400
2400
Australia
Australia 2300
2300
France
France 1000
1000

Which two countries contributed to the one third of the total number of tourists who visited India in
201 1 ?
2011?
(A) USA
(A) USA and
and Japan
Japan

(B) USA
(B) and Australia
USA and Australia

(C) England
(C) England and
and France
France

(D) Japan
(D) Japan and
and Australia
Australia

MN 14/15
14/15
2013
2013 MINING ENGG.
MINING ENGG. —MN
–MN

Q.64
Q.64 If |I—ZX
If 9| = 3 then
2 + 9| then the
the possible
possible value of |I—XI| — X 2 would
value of would be:
be:
(A) 30
(A) 30 (B) -30
(B) -30 (C) -42
(C) -42 (D) 42
(D) 42

Q.65
Q.65 All professors
All are researchers
professors are researchers
Some scientists
Some scientists are
are professors
professors

Which of
Which of the
the given
given conclusions is logically
conclusions is logically valid and is
valid and is inferred
inferred from
from the
the above
above arguments:
arguments:
(A) All
(A) All scientists
scientists are
are researchers
researchers

(B) All
(B) All professors are scientists
professors are scientists

(C) Some
(C) Some researchers are scientists
researchers are scientists

(D) No
(D) follows
conclusion follows
No conclusion

END OF
END THE QUESTION
OF THE QUESTION PAPER
PAPER

MN
MN 15/15
15/15
2013 :: Answer
GATE 2013
GATE keys for
Answer keys MN -- Mining
for MN Mining Engineering
Engineering

Paper Q.No
Paper Q.No Key(s)/Value(s)
Key(s)/Value(s) Paper Q.No
Paper Q.No Key(s)/Va|ue(s)
Key(s)/Value(s)
MN
MN 11 C
C MN
MN 36
36 A
A
MN
MN 2
2 B
B MN
MN 37
37 A
A
MN
MN 3
3 D
D MN
MN 38
38 D
D
MN
MN 4
4 A
A MN
MN 39
39 C
C
MN
MN 5
5 A
A MN
MN 40
40 A
A
MN
MN 6
6 C
C MN
MN 41
41 D
D
MN
MN 7
7 D
D MN
MN 42
42 B
B
MN
MN 8
8 B
B MN
MN 43
43 D
D
MN
MN 9
9 D
D MN
MN 44
44 B
B
MN
MN 10
10 Marks to
Marks to All
All MN
MN 45
45 C
C
MN
MN 11
11 C MN
MN 46
46 D
D
MN
MN 12
12 B
B MN
MN 47
47 A
A
MN
MN 13
13 C
C MN
MN 48
48 D
D
MN
MN 14
14 D
D MN
MN 49
49 C
C
MN
MN 15
15 A
A MN
MN 50
50 A
A
MN
MN 16
16 B
B MN
MN 51
51 D
D
MN
MN 17
17 A
A MN
MN 52
52 B
B
MN
MN 18
18 C
C MN
MN 53
53 D
D
MN
MN 19
19 D
D MN
MN 54
54 A
A
MN
MN 20
20 B
B MN
MN 55
55 B
B
MN
MN 21
21 D
D MN
MN 56
56 B
B
MN
MN 22
22 A
A MN
MN 57
57 C
C
MN
MN 23
23 B
B MN
MN 58
58 C
C
MN
MN 24
24 C
C MN
MN 59
59 D
D
MN
MN 25
25 C
C MN
MN 60
60 B
B
MN
MN 26
26 4.5 to
4.5 to 4.518
4.518 MN
MN 61
61 C
C
MN
MN 27
27 2.35 to
2.35 to 2.45
2.45 MN
MN 62
62 C
C
MN
MN 28
28 44.8 to
44.8 to 45.5
45.5 MN
MN 63
63 C
C
MN
MN 29
29 to 169
166 to
166 169 MN
MN 64
64 B
B
MN
MN 30
30 225 to
225 to 231
231 MN
MN 65
65 C
C
MN
MN 31
31 335 to
335 to 337
337
MN
MN 32
32 0.000875 to
0.000875 to 0.000885
0.000885
MN
MN 33
33 0.9 to
0.9 to 11
MN
MN 34
34 D
D
MN
MN 35
35 C
C
GATE 2014
Answer Keys for MN ‐- Mining Engineering
Section Q.. No. Key / Range Marks Section Q. No. Key / Range Marks
GA 1 D 1 MN 24 1320 to 1320 1
GA 2 A 1 MN 25 C 1
GA 3 C 1 MN 26 D 2
GA 4 25 to 25 1 MN 27 3.4 to 3.5 2
GA 5 A 1 MN 28 14.25 to 14.65 2
GA 6 A 2 MN 29 C 2
GA 7 C 2 MN 30 A 2
GA 8 D 2 MN 31 D 2
GA 9 C 2 MN 32 B 2
GA 10 0.48 to 0.49 2 MN 33 B 2
MN 1 B 1 MN 34 C 2
MN 2 A 1 MN 35 2.70 to 2.75 2
MN 3 D 1 MN 36 0.90 to 0.94 2
MN 4 15 to 15 1 MN 37 A 2
MN 5 C 1 MN 38 C 2
MN 6 B 1 MN 39 D 2
MN 7 135 to 142 1 MN 40 A 2
MN 8 C 1 MN 41 D 2
MN 9 C 1 MN 42 C 2
MN 10 A 1 MN 43 21.80 to 21.95 2
MN 11 B 1 MN 44 B 2
MN 12 6 to 6 1 MN 45 502 to 503 2
MN 13 A 1 MN 46 258 to 263 2
MN 14 0.25 to 0.25 1 MN 47 874 to 876 2
MN 15 B 1 MN 48 4.20 to 4.35 2
MN 16 D 1 MN 49 1.0 to 1.2 2
MN 17 A 1 MN 50 D 2
MN 18 D 1 MN 51 D 2
MN 19 B 1 MN 52 A 2
MN 20 B 1 MN 53 2.2 to 2.4 2
MN 21 30 to 30 1 MN 54 40 to 40 2
MN 22 1.25 to 1.25 1 MN 55 0.50 to 0.55 2
MN 23 C 1
GATE 2014: General Instructions during Examination

1. Total duration of the GATE examination is 180 minutes.


2. The clock will be set at the server. The countdown timer at the top right corner of
screen will display the remaining time available for you to complete the examination.
When the timer reaches zero, the examination will end by itself. You need not
terminate the examination or submit your paper.
3. Any useful data required for your paper can be viewed by clicking on the Useful
Common Data button that appears on the screen.
4. Use the scribble pad provided to you for any rough work. Submit the scribble pad at
the end of the examination.
5. You are allowed to use a non-programmable type calculator, however, sharing of
calculators is not allowed.
6. The Question Palette displayed on the right side of screen will show the status of
each question using one of the following symbols:

1 Ten have net 1ii'isited the quesfien yet-

Teu have net answered the quesfien-

a Ten have answered the quesfien-

0 Ten have NOT answered the Iqlztestien= but have marked the quesfien fer review-

H"
0 Ten have answered the quesfien= but marked it fer review-

The Marked for Review status for a question simply indicates that you would like to look at
that question again. If a question is answered,
answered, but marked for
for review,
review, then the answer will
be considered for
for evaluation unless the status is modified by the candidate.
candidate.

Navigating to a Question :

7. To answer a question, do the following:


a. Click on the question number in the Question Palette to go to that question
directly.
b. Select an answer for a multiple choice type question by clicking on the bubble
placed before the 4 choices, namely A, B, C and D. Use the virtual numeric
keypad to enter a number as answer for a numerical type question.
c. Click on Save & Next to save your answer for the current question and then go
to the next question.
d. Click on Mark for Review & Next to save your answer for the current question
and also mark it for review, and then go to the next question.
Caution: Note that your answer for the current question will not be saved, if you navigate
to another question directly by clicking on a question number without saving the answer to
the previous question.

You can view all the questions by clicking on the Question Paper button. This feature is
provided, so that if you want you can just see the entire question paper at a glance.

Answering a Question :

8. Procedure for answering a multiple choice (MCQ) type question:


a. Choose one answer from the 4 options (A,B,C,D) given below the question,
click on the bubble placed before the chosen option.
b. To deselect your chosen answer, click on the bubble of the chosen option again
or click on the Clear Response button.
c. To change your chosen answer, click on the bubble of another option.
d. To save your answer, you MUST click on the Save & Next button.

9. Procedure for answering a numerical answer type question:


a. To enter a number as your answer, use the virtual numerical keypad.
b. A fraction (e.g. -0.3 or -.3) can be entered as an answer with or without '0'
before the decimal point. As many as four decimal points, e.g. 12.5435 or
0.003 or -932.6711 or 12.82 can be entered.
c. To clear your answer, click on the Clear Response button.
d. To save your answer, you MUST click on the Save & Next button
10.
10. To mark a question for review, click on the Mark for Review & Next button. If an
answer is selected (for MCQ) or entered (for numerical answer type) for
for a question
that is Marked for Review,
Review, that answer will be considered in the evaluation unless
the status is modified by the candidate.
candidate.

11. To change your answer to a question that has already been answered, first select
that question for answering and then follow the procedure for answering that type of
question.
12. Note that ONLY Questions for which answers are saved or marked forfor review after
answering will be considered for evaluation.

Choosing a Section :

13. Sections in this question paper are displayed on the top bar of the screen. Questions
in a Section can be viewed by clicking on the name of that Section. The Section you
are currently viewing will be highlighted.
14. A checkbox is displayed for every optional Section, if any, in the Question Paper. To
select the optional Section for answering, click on the checkbox for that Section.
15. If the checkbox for an optional Section is not selected, the Save & Next button and
the Mark for Review & Next button will NOT be enabled for that Section. You will
only be able to see questions in this Section, but you will not be able to answer
questions in the Section.
16. After clicking the Save & Next button for the last question in a Section, you will
automatically be taken to the first question of the next Section in sequence.
17. You can move the mouse cursor over the name of a Section to view the answering
status for that Section.

Changing the Optional Section :

18. After answering the chosen optional Section, partially or completely, you can change
the optional Section by selecting the checkbox for a new Section that you want to
attempt. A warning message will appear along with a table showing the number of
questions answered in each of the previously chosen optional Sections and a
checkbox against each of these Sections. Click on a checkbox against a Section that
you want to reset and then click on the RESET button. Note that RESETTING a Section
will DELETE all the answers for questions in that Section. Hence, if you think that you
may want to select this Section again later, you will have to note down your answers
for questions in that Section. If you do not want to reset the Section and want to
continue answering the previously chosen optional Section, then click on the BACK
button.

19. If you deselect the checkbox for an optional Section in the top bar, the following
warning message will appear: "Deselecting the checkbox will DELETE all the answers
for questions in this Section. Do you want to deselect this Section?” If you want to
deselect, click on the RESET button. If you do not want to deselect, click on the BACK
button.

20. You can shuffle between different Sections or change the optional Sections any
number of times.
GATE 2014
GATE Examination
2014 Examination
MN: Mining
MN: Engineering
Mining Engineering
Duration: 180 minutes Maximum Marks: 100

Read the
Read the following
following instructions
instructions carefully.
carefully.

To login,
1. To login, enter
enter your
your Registration
Registration Number
Number and password provided
and password provided to to you. Kindly go
you. Kindly go through
through the
the various
various
symbols used
symbols used inin the
the test
test and
and understand their meaning
understand their meaning before
before you start the
you start examination.
the examination.
Once you
2. Once you login
login and
and after
after the
the start
start of
of the
the examination,
examination, you can view
you can view all
all the
the questions
questions in
in the
the question
question
paper,
paper, by clicking on
by clicking on the
the View
View All
All Questions
Questions button in the
button in the screen.
screen.
This question
3. This question paper consists of
paper consists of 2
2 sections,
sections, General
General Aptitude
Aptitude (GA)(GA) for for 15
15 marks
marks and
and the
the subject
subject
specific GATE
specific GATE paper for 85
paper for 85 marks.
marks. Both
Both these
these sections
sections areare compulsory.
compulsory.
The GA
The GA section
section consists
consists of
of 10
10 questions.
questions. Question
Question numbers
numbers 11 to to 5 5 are
are of
of 1-mark
1-mark each,
each, while question
while question
numbers 6
numbers 6 toto 10
10 are
are of
of 2-mark
2-mark each.
each.
The subject
The subject specific
specific GATE
GATE paper section consists
paper section consists of
of 55
55 questions,
questions, out
out of
of which question numbers
which question numbers 11 toto
25 are
25 are of
of 1-mark
l-mark each,
each, while
while question
question numbers
numbers 26 26 to
to 55
55 are
are of
of 2-mark
2-mark each.
each.
Depending upon
4. Depending the GATE
upon the GATE paper, there may
paper, there may be be useful common data
useful common data that
that may
may be
be required for
required for
answering the
answering the questions.
questions. If
If the
the paper has such
paper has such useful data, the
useful data, the same
same cancan be viewed by
be viewed clicking on
by clicking on the
the
Useful Common
Useful Common Data Data button that appears
button that appears atat the
the top,
top, right
right hand
hand side
side of
of the
the screen.
screen.
The computer
5. The computer allotted
allotted to
to you at the
you at examination center
the examination center runs specialized software
runs specialized software that
that permits only one
permits only one
answer to
answer to be selected for
be selected for multiple-choice questions using
multiple-choice questions using aa mouse
mouse andand to
to enter
enter aa suitable
suitable number
number forfor
the numerical
the numerical answer
answer type
type questions
questions using the virtual
using the virtual keyboard
keyboard andand mouse.
mouse.
Your answers
6. Your shall be
answers shall be updated
updated andand saved
saved on on aa server
server periodically
periodically andand also
also atat the
the end
end ofof the
the
examination. The
examination. The examination
examination will
will stop
stop automatically
automatically at at the
the end
end of
of 180
180 minutes.
minutes.
7. InIn each
each paper
paper aa candidate
candidate can
can answer
answer aa total
total of
of 65
65 questions
questions carrying
carrying 100100 marks.
marks.
The question
9°.“ The
8. question paper
paper maymay consist
consist of
of questions
questions of of multiple
multiple choice
choice type
type (MCQ)
(MCQ) andand numerical
numerical answer
answer
type.
Multiple choice
9. Multiple choice type
type questions
questions will have four
will have four choices
choices against
against A,A, B,
B, C,
C, D,
D, out
out of
of which only ONE
which only ONE is is the
the
correct answer.
correct answer. The candidate has
The candidate has toto choose
choose thethe correct
correct answer
answer by clicking on
by clicking on the
the bubble
bubble ((Q)
⃝) placed
placed
before
before the choice.
the choice.
10. For
10. For numerical answer type
numerical answer questions, each
type questions, each question will have
question will have aa numerical answer and
numerical answer and there
there will
will not
not be
be
any choices.
any choices. For
For these
these questions,
questions, thethe answer
answer should
should bebe enteredby
enteredby usingusing the
the virtual
virtual keyboard
keyboard that
that
appears on
appears on the
the monitor
monitor andand the
the mouse.
mouse.
11. All
11. questions that
All questions are not
that are attempted will
not attempted will result
result inin zero
zero marks.
marks. However,
However, wrong answers for
wrong answers for multiple
multiple
choice type
choice questions (MCQ)
type questions (MCQ) will
will result in NEGATIVE
result in NEGATIVE marks. marks. For
For all
all MCQ
MCQ questions
questions aa wrong
wrong
answer will
answer will result in deduction
result in deduction of ⅓ marks
of1/3 marks forfor aa 1-mark
l-mark question and ⅔
question and 2/3 marks for aa 2-mark
marks for 2-mark question.
question.
12. There
12. There is
is NO
NO NEGATIVE
NEGATIVE MARKING MARKING for for questions
questions of of NUMERICAL
NUMERICAL ANSWER ANSWER TYPE. TYPE.
13. Non-programmable
13. Non-programmable type type Calculator
Calculator is
is allowed.
allowed. Charts,
Charts, graph
graph sheets,
sheets, and
and mathematical
mathematical tables are NOT
tables are NOT
allowed in
allowed in the Examination Hall.
the Examination Hall. You
You must
must use the Scribble
use the Scribble pad
pad provided to you
provided to at the
you at examination
the examination
centre for
centre for all
all your
your rough
rough work.
work. The
The Scribble
Scribble Pad
Pad has
has to
to be returned at
be returned at the
the end
end of
of the
the examination.
examination.

Declaration by
Declaration by the
the candidate:
candidate:

“I have
“I have read
read and
and understood all the
understood all the above instructions. II have
above instructions. have also
also read and understood
read and clearly the
understood clearly the
instructions given
instructions given on the admit
on the admit card
card and
and shall
shall follow
follow the same. II also
the same. also understand
understand that in case
that in case II am
am found
found toto
violate any
violate any of
of these
these instructions,
instructions, my
my candidature
candidature isis liable
liable to
to be cancelled. II also
be cancelled. also confirm
confirm that
that at
at the
the start
start of
of
the examination
the examination all
all the
the computer
computer hardware
hardware allotted
allotted to
to me
me are
are in
in proper
proper working condition”.
working condition”.
2014
GATE 2014
GATE SET- 2
SET- 2 General Aptitude -GA
General Aptitude -GA

Q. 1—
Q. 1 Q. 5
– Q. 5 carry
carry one
one mark
mark each.
each.

Q.1
Q.1 Choose the
Choose the most
most appropriate
appropriate word from the
word from the options
options given
given below to complete
below to complete the
the following
following
sentence.

A
A person suffering from
person suffering from Alzheimer’s
Alzheimer’s disease
disease short-term memory
short-term loss.
memory loss.
(A) experienced
(A) experienced (B) has
(B) has experienced
experienced
(C) is
(C) is experiencing
experiencing (D) experiences
(D) experiences

Q2
Q.2 Choose the
Choose the most
most appropriate
appropriate word from the
word from the options
options given
given below to complete
below to complete the
the following
following
sentence.

is the
____________ is the key
key to
to their
their happiness; they are
happiness; they are satisfied
satisfied with
with what they have.
what they have.

)
14
(A) Contentment
(A) Contentment (B) Ambition
(B) Ambition (C) Perseverance
(C) Perseverance Hunger
(D) Hunger
(D)

Q3
Q.3 Which of
Which of the
the following
following options
options is
is the
the closest
closest in
in meaning
meaning to
to the
the sentence
sentence below?
below?

“As a woman
woman, I have no country.”
count .”

20
9

(A) Women
(A) Women have
have no country.
no country.
(B) Women
(B) Women are
are not citizens of
not citizens of any
any country.
country.
(C) Women’s
(C) Women’s solidarity
solidarity knows
knows no
no national
national boundaries.
boundaries.
(D) Women
(D) Women of
of all
all countries
countries have
have equal
equal legal
legal rights.
rights.

Q4
Q.4 In any
In any given
given year, the probability
year, the of an
probability of
E
an earthquake
earthquake greater
greater than Magnitude 6
than Magnitude 6 occurring in the
occurring in the
Garhwal Himalayas is
Garhwal Himalayas is 0.04.
0.04. The
The average
average time
time between successive occurrences
between successive occurrences of
of such
such earthquakes
earthquakes
AT
is ____ years.
is years.

Q5
Q.5 The population
The of aa new
population of new city is 5
city is 5 million and is
million and is growing
growing at
at 20%
20% annually.
annually. How
How many
many years
years would
would
it take
it take to
to double
double at
at this
this growth
growth rate?
rate?
(G

(A) 3-4
(A) 3-4 years
years (B) 4-5
(B) 4-5 years
years (C) 5-6
(C) 5-6 years
years (D) 6-7
(D) 6-7 years
years

6—
02

Q. 6
Q. Q. 10
– Q. 10 carry
carry two
two marks
marks each.
each.

Q.6
Q.6 In aa group
In group of
of four
four children,
children, Som
Som is
is younger
younger to Riaz. Shiv
to Riaz. Shiv is
is elder
elder to
to Ansu.
Ansu. Ansu
Ansu isis youngest in the
youngest in the
group. Which
group. of the
Which of the following
following statements is/are required
statements is/are required to find the
to find eldest child
the eldest child in
in the group?
the group?
A

Statements
Statements
1. Shiv
1. Shiv isis younger
younger to
to Riaz.
Riaz.
G

2. Shiv
2. Shiv isis elder
elder to
to Som.
Som.
(A) Statement
(A) Statement 1by
lby itself
itself determines
determines the
the eldest
eldest child.
child.
(B) Statement
(B) Statement 2
2 by itself determines
by itself determines the
the eldest
eldest child.
child.
(C) Statements
(C) Statements 11 and
and 22 are
are both
both required
required toto determine
determine the
the eldest
eldest child.
child.
(D) Statements
(D) Statements 11 and
and 22 are
are not sufficient to
not sufficient determine the
to determine the eldest
eldest child.
child.

GA
GA l/2
1/2
2014
GATE 2014
GATE SET- 2
SET- 2 General Aptitude -GA
General Aptitude -GA
Q.7
Q.7 Moving into
Moving into aa world
world ofof big data will
big data will require
require us to change
us to change ourour thinking about the
thinking about merits of
the merits of
exactitude. To
exactitude. To apply
apply the
the conventional mindset of
conventional mindset of measurement
measurement to to the
the digital,
digital, connected
connected world of
world of
the twenty-first
the twenty-first century
century isis to
to miss
miss aa crucial
crucial point. As mentioned
point. As mentioned earlier,
earlier, the
the obsession
obsession with
with
exactness is
exactness is an
an artefact
artefact of the information-deprived
of the information-deprived analog
analog era.
era. When
When data
data was sparse, every
was sparse, every data
data
point was critical,
point was critical, and
and thus
thus great
great care
care was taken to
was taken to avoid
avoid letting any point
letting any point bias the analysis.
bias the analysis.

From “BIG DA
From “BIG TA Viktor
DATA” Viktor Mayer-Schonberger
Mayer-Schonberger and and Kenneth Cukier
Kenneth Cukier

The main
The main point of the
point of the paragraph is:
paragraph is:
(A) The
(A) The twenty-first
twenty-first century
century isis aa digital
digital world
world
(B) Big
(B) Big data
data is
is obsessed
obsessed with exactness
with exactness
(C) Exactitude
(C) Exactitude is
is not
not critical
critical in dealing with
in dealing with big data
big data
(D) Sparse
(D) Sparse data
data leads
leads to
to aa bias in the
bias in the analysis
analysis

Q.8 The total


total exports
exports and
and revenues from the
revenues from the exports
exports of
of aa country
country are
are given
given inin the
the two
two pie charts below.

)
Q.8 The pie charts below.
The pie
The chart for
pie chart for exports
exports shows
shows the
the quantity
quantity of
of each
each item
item asas aa percentage
percentage of of the
the total
total quantity
quantity of
of

14
exports. The
exports. The pie chart for
pie chart for the
the revenues
revenues shows
shows the
the percentage
percentage of of the
the total
total revenue
revenue generated
generated through
through
export of
export of each
each item.
item. The
The total
total quantity
quantity of
of exports
exports ofof all
all the
the items
items is is 5
5 lakh
lakh tonnes
tonnes and
and the
the total
total
revenues are 250
revenues are 250 crore
crore rupees. What is
rupees. What is the
the ratio the revenue
of the
ratio of generated through
revenue generated export of
through export Item 11
of Item
per kilogram to
per kilogram to the
the revenue
revenue generated
generated through
through export
export of
of Item
Item 44 per
per kilogram?
kilogram?

20
Exports Revenues

Item 6
16%
Item 1
11%
E Item 6
19%
Item 1
12%
AT
Item 5 Item 2
12% 20% Item 2
Item 5 20%
20%
Item 3
Item 4 19%
Item 4 Item 3
22%
(G

6% 23%

(A) 1:2
1:2 (B) 2:1
2:1 (C) 1:4
1:4 (D) 4:1
4:1
02

(A) (B) (C) (D)

Q.9
Q.9 X
X isis 11 km
km northeast of Y.
northeast of Y. Y
Y is
is 11 km
km southeast
southeast of
of Z.
Z. W
W is
is 11 km
km west of Z.
west of is 11 km
P is
Z. P south of
km south of W. Q is
W. Q is
11 km east of
km east of P.
P. What
What is
is the
the distance
distance between
between X and Q
X and Q in
in km?
km?


(B) √2 fl
(C) √3 (D) 22
A

(AH1
(A) (B) (C) (D)

Q.10 10% ofof the


the population in aa town
town isis HIV
HIV+.
+
A new
new diagnostic
diagnostic kit
kit for
for HIV
HIV detection
detection is
is available;
available; this
G

Q.10 10% population in .A this


kit correctly identifies
kit correctly HIV+ individuals
identifies HIV
+
individuals 95%
95% ofof the
the time, HIV_− individuals
and HIV
time, and individuals 89%
89% of
of the time. A
the time. A
particular
particular patient is tested
patient is tested using this kit
using this kit and
and is
is found
found toto be
be positive. The probability
positive. The probability that
that the
the
individual is
individual is actually
actually positive
positive isis _______

END OF
END THE QUESTION
OF THE QUESTION PAPER
PAPER

GA
GA 2/2
2/2
GATE 2014
GATE 2014 MINING —
MINING MN
– MN

Q. 1—
Q. 1 Q. 25
– Q. 25 carry
carry one
one mark
mark each.
each.

Q.1
Q.1 A
A block
block ofof weight
weight 100
100 kN
kN rests
rests on
on aa floor
floor as
as shown
shown in the figure.
in the figure. The
The coefficient
coefficient of
of static
static friction
friction
between
between thethe block and the
block and the floor
floor isis 0.5.
0.5. A force of
A force of 45
45 kN is applied
kN is applied horizontally on the
horizontally on the block. The
block. The
static frictional
static frictional force in kN
force in is
kN is

100
100 kN
W
Coefficient of static friction == 0.5
45kN—9
45 kN

/////////

(A) 22.5
(A) 22.5 (B) 50.0
(B) 50.0 (C) 55.0
(C) 55.0 (D) 100.0
(D) 100.0
Q2
Q.2 A spring of
A spring of constant
constant stiffness
stiffness kk is
is stretched
stretched from
from point A to
point A to point B (displacement
point B (displacement u
u in
in the
the figure)
figure)

)
by
by aa force
force F.
F. The
The potential energy of
potential energy of the spring is
the spring is expressed
expressed by
by

14
u
k A B F

20
1 1
(A) £s
(A) —Fu
ku 2 − Fu (B) éku2
(B) +Fu
ku 2 + Fu
2 2

(C) −F
ICU—F
(C) ku (D) ku+F
(D) ku +F
E
Q3
Q.3 If σ
If 0'ss is
is the
the induced
induced stress and σ
stress and 0'1.i is
is the insitu stress
the insitu stress at
at aa point
point below ground, the
below ground, ‘stress
the ‘stress
AT

concentration’ at
concentration’ at that
that point is
point is
σ01.i σ0ss
(A) (A) σσa. USs

il
(B) σσ0
(B)
al-i

Ss
(>05
(C)
C
σs ()0.
D
(D)
σi
(G

Q4 as σ
The components
components of
of state
state of
of stress
stress at
at aa point x−y plane
in x—y are given
given as xx = 5 MPa,
O'xx MPa, σ yy = 10 MPa
10 MPa
Q.4 The point in plane are O'yy
and τTxy
and = —2
xy = −
The sum
MPa. The
2 MPa. of the
sum of the principal stresses acting
principal stresses acting on
on the x−y plane
the x—y MPa is
in MPa
plane in is _______

Q5
Q.5 The angle 5500 15
15’′ 25
25"′′ is expressed in hours,
hours, minutes,
minutes, and seconds as
N

(A) 11h20min
(A) ℎ 1.673𝑠𝑠
20𝑚𝑚𝑚𝑚𝑚𝑚 1.67 (B) 11h20mm
(B) ℎ 16.005𝑠𝑠
20𝑚𝑚𝑚𝑚𝑚𝑚 16.00 (C) 00h21mi"
(C) ℎ 1.675𝑠𝑠
21𝑚𝑚𝑚𝑚𝑚𝑚 1.67 (D) 00h21mi"
(D) ℎ
16.005𝑠𝑠
21𝑚𝑚𝑚𝑚𝑚𝑚 16.00

Q6
Q.6 A circular
A circular curve
curve has
has aa radius of 200
radius of 200 m and deflection
m and deflection angle
angle of
of 65
650.. The
0
The length
length of
of the
the curve
curve in
in m is
m is
M

(A) 221
(A) 221 (B) 227
(B) 227 (C) 235
(C) 235 (D) 262
(D) 262

Q7
Q.7 The weight
The weight strength
strength of
of ANFO
ANFO of of specific
specific gravity
gravity 0.8
0.8 is
is 912
912 kcal/kg.
kcal/kg. The
The weight
weight strength of an
strength of an
emulsion explosive of
emulsion explosive of specific gravity 1.2
specific gravity 1.2 is
is 850
850 kcal/kg.
kcal/kg. Bulk
Bulk strength
strength of
of the
the emulsion
emulsion explosive
explosive
relative to
relative to ANFO
ANFO inin percentage is ________________
percentage is

MN
MN 1/8
1/8
GATE 2014
GATE 2014 MINING —
MINING – MN
MN

Q.8
Q.8 In aa cut-and-fill
In cut-and-fill stope,
stope, the
the main
main purpose of back
purpose of filling is
back filling is to
to
(A) reduce
(A) reduce ore
ore dilution
dilution

(B) prevent
(B) prevent high stress concentrations
high stress concentrations in
in far
far field domain
field domain

(C) prevent
(C) displacement due
prevent displacement due to
to dilation
dilation of
of fractured
fractured wall
wall rock
rock

(D) improve
(D) improve ore rehandling
ore rehandling

Q.9
Q.9 Bypass valve
Bypass valve in
in aa compressed
compressed oxygen
oxygen type
type self-contained
self-contained breathing apparatus is
breathing apparatus is meant
meant to
to
(A) release
(A) release accumulated
accumulated nitrogen in the
nitrogen in the breathing
breathing bag
bag
(B) release
(B) release excess
excess pressure in the
pressure in the breathing
breathing bag
bag

4)
(C) supply
(C) supply oxygen
oxygen directly
directly to
to wearer in case
wearer in case pressure reducing valve
pressure reducing does not
valve does not function
function
(D) flush
(D) flush out
out the
the apparatus
apparatus with oxygen on
with oxygen on opening
opening the
the cylinder
cylinder valve
valve

1
Q. 10
Q.10 Given S
Given S is
is the
the setting
setting load
load and
and Y
Y is
is the
the yield
yield load
load of
of aa hydraulic
hydraulic prop, the correct
prop, the correct relationship
relationship is
is

20
(A)S<Y
(A) S<Y (B)S>Y
(B) S>Y (C)S=Y
(C) S=Y (D)S=Y2
(D) S = Y2

Q11
Q.11 . . . . dy :
dy . .
Solut1on of
Solution of the
the differential
differential equation
equation = ky
ky follows
follows exponential
exponential decay
decay (where
(where kk is
is aa constant) for
constant) for
d—
x
dx
6 [[0,00]
xx ∈ 0, ∞ ] if
if
E
(A)k>0
(A) k >0 (B)k<0
(B) k <0 (C)k=0
(C) k =0 (D)k=e
(D) k =e
AT

Q. 12
Q.12 The value
The value of for which
of kk for the vectors
which the =2
vectors aa = 2ii -- 3
3jj and
and b
b== kkii + 4
4jj are
are orthogonal
orthogonal to
to each
each other
other is
is _
___

Q.l3
Q.13 Which one
Which one of
of the
the following
following is the most
is the most likely
likely mode
mode of
of slope
slope failure
failure for
for waste
waste dump
dump
(A) Circular
(A) Circular (B) Wedge
(B) Wedge
(G

(C) Plane
(C) Plane (D) Toppling
(D) Toppling

Q.l4
Q.14 The occurrence
The occurrence of
of head in aa single
head in single toss
toss of
of an
an unbiased coin is
unbiased coin given by
is given by aa random
random variable
variable X.
X. The
The
variance of X
variance of is _______________
X is
N

Q.15
Q.15 The divergence
The of the
divergence of the vector V == ((xx + y)(—yi
vector v y )(− yi + xj) is
xj) is

y−x x− y
M

(A) y—x
(A) (B) x—y
(B) (C) x2 −
(C) x2 —y2
y2 (D) y2 −
(D) y2 —x2
x2

Q.l6
Q.16 . Ix‘
x .
The lim
The 11m— is1s
x →0 x
x—>0

(A) −1
(A) —1 (B) 0
(B) O (C) 1l
(C) (D) non-existent
(D) non-existent

Q.l7
Q.17 For Indian
For Indian coal
coal mines,
mines, the ‘maximum allowable
the ‘maximum allowable concentration’
concentration’ of
of respirable dust containing
respirable dust containing 7.5%
7.5%
free silica
free silica in
in mg/m
mg/m3 is
3
is
(A) 2.0
(A) 2.0 (B) 2.2
(B) 2.2 (C) 2.5
(C) 2.5 (D) 2.7
(D) 2.7

MN
MN 2/8
2/8
GATE 2014
GATE 2014 MINING —
MINING – MN
MN

Q.18
Q.18 Given 𝑘𝑘
Given k is
is the
the thermal
thermal conductivity,
conductivity, p is density
𝜌𝜌 is density and
and 𝑐𝑐C is
is specific
specific heat of aa rock
heat of rock sample,
sample, the
the thermal
thermal
diffusivity of
diffusivity of the
the rock
rock sample
sample is
is

(A):
(A) kp
𝑘𝑘𝑘𝑘
𝑐𝑐 (13),.
(B) fl
𝜌𝜌𝜌𝜌
𝑘𝑘 (C) p
(C) k_C
𝑘𝑘𝑘𝑘
𝜌𝜌 (D) W
(D) i
𝑘𝑘
𝜌𝜌𝜌𝜌

Q.19
Q.19 Cyclone, bag
Cyclone, filter and
bag filter and scrubber
scrubber can
can be
be used for control
used for control of
of
(A) water
(A) water pollution
pollution (B) air
(B) air pollution
pollution

(C) soil
(C) soil pollution
pollution (D) noise
(D) noise pollution
pollution

Q20
Q.20 A mine waste
A mine waste dump of pH
dump of 5.2 can
pH 5.2 can be
be neutralized
neutralized by adding
by adding
(A) urea
(A) urea (B) calcium
(B) calcium carbonate
carbonate

)
(C) sulphuric
(C) sulphuric acid
acid (D) sodium
(D) sodium chloride
chloride

14
Q21
Q.21 A flat coal
A flat coal seam
seam of
of thickness
thickness (t)
(t) 3
3 m is excavated
m is excavated and
and broken
broken roof
roof rock
rock has completely filled
has completely filled the
the
space created
space created due
due to
to extraction
extraction as
as shown
shown in
in the figure. If
the figure. If the
the bulking factor of
bulking factor of roof
roof rock is 1.2,
rock is 1.2, the
the
height (H)
caving height
caving (H) in
in m
m is
is ______________

flux-Hf1"““$1132:-

20
19%H
.i'1- -I

.:.-:-.'-:-'?:-¢TH=1: void
void -- -
t space
E
Q22
Q.22 A
A piece of coal
piece of coal sample
sample weighs 10 kg
weighs 10 kg in air and
in air and 2
2 kg
kg when immersed in
when immersed in water.
water. The
The specific gravity
specific gravity
of the
the coal
coal sample
sample is
is ___________
AT

of

Q23
Q.23 In aa borehole
In log of
borehole log of 1.2 m in
1.2 m in length,
length, recovery
recovery of
of rock cores in
rock cores in cm is given
cm is given below
below

20, 8,
20, 8,15, 8, 8,
15, 8, 8, 4,
4, 3,
3, 9,
9,10,1, 5,10
10, 1, 5, 10
(G

The RQD
The RQD in
in percentage is
percentage is
(A) 29.2
(A) 29.2 (B) 31.8
(B) 31.8 (C) 45.8
(C) 45.8 (D) 50.0
(D) 50.0

Q24
Q.24 An
An underground
underground coal mine panel
coal mine panel produces 520 tonnes
produces 520 tonnes per day deploying
per day deploying 220,
220, 200
200 and
and 192
192 persons
persons
N

in three
in three shifts. As per
shifts. As CMR 1957,
per CMR 1957, the minimum quantity
the minimum quantity of
of air
air in
in m3/min
m3/min to
to be delivered at
be delivered at the last
the last
ventilation connection of
ventilation connection of the
the panel is ____________
panel is
M

Q25
Q.25 In a PERT network,
network, the
the activities on the
the critical path a, b
path are a, [2 and c. The standard deviations of the
the
durations of
durations of these
these activities are 2,
activities are 2, 2
2 and
and 11 respectively. The variance
respectively. The of the
variance of the project duration is
project duration is
(A) 33
(A) (B) 55
(B) (C) 99
(C) (D) 12
(D) 12

MN
MN 3/8
3/8
GATE 2014
GATE 2014 MINING —
MINING – MN
MN

Q. 26 —
Q. 26 Q. 55
– Q. 55 carry
carry two
two marks
marks each.
each.

Q.26
Q.26 A
A particle P is
particle P is in
in equilibrium
equilibrium as
as shown
shown in
in the figure. The
the figure. The magnitude in kN
magnitude in and the
kN and orientation θ6’ in
the orientation in
degrees of
degrees of the
the force F respectively
force F are
respectively are
yy
FF

pP
θ(9 x
X
150 kN
30°o
30
200 kN

(A) 52.1,
(A) 52.1, 16.1
16.1 (B) 221.2,
(B) 221.2, 23.2
23.2 (C) 102.3,
(C) 102.3, 53.4
53.4 (D) 180.3,
(D) 180.3, 73.9
73.9

)
Q27
Q.27 A distributed load
A distributed load of 4 kN/m
of 4 acts on
kN/m acts on aa beam of 6
beam of 6 m length supported
m length supported by
by aa hinge and aa roller
hinge and as
roller as

14
shown in
shown in the
the figure.
figure. The
The distance
distance in
in m
m of
of the
the point of zero
point of zero shear
shear in the beam
in the from the
beam from the point A is
point A is _
__
44kN/m
kN/m

20
A B

66m
m

Q28
Q.28 A dry rock
A dry sample of
rock sample of diameter
diameter 50
50 mm
mm and
and length
length 100
100 mm
mm weighs
weighs 300
300 g.
g. After
After saturating in brine
saturating in brine
E
solution of
solution of specific
specific gravity
gravity 1.05,
1.05, its
its weight
weight increased
increased to
to 330
330 g.
g. The
The porosity of the
porosity of the rock
rock sample
sample inin
percentage is __________
percentage is
AT

Q29
Q.29 A
A joint
joint plane
plane ofof length
length L L and dip δ5 intersects
and dip intersects the
the toe of aa slope
toe of slope as
as shown
shown inin the figure. The
the figure. The weight
weight
of the
of the shaded
shaded block
block isis W.
W. Uniform
Uniform water
water pressure
pressure PP acts
acts normal
normal to
to the
the joint
joint plane. If the
plane. If cohesion
the cohesion
and angle
and angle of
of internal
internal friction
friction of the joint
of the surface are
joint surface and φ
are cc and ¢respectively,
respectively, then
then the for
expression for
the expression
‘safety factor’
‘safety factor’ of
of the
the shaded
shaded block is
block is
(G

L
W
¢ xJoint
Joint plane
N

δ Pp
M

Lc + ((W
Le W sin — LP)
sin δ5 − tan φ¢
LP ) tan Lc + ((W
Lc cos δ5 + LP)
W cos tan φ¢
LP ) tan
(A)
(A) (B)
(B)
cos δ5
W cos
W W sin δ5
W sin

Lc + ((W
Le W cos — LP)
cos δ5 − tan φ¢
LP ) tan Lc + ((W
Lc sin δ5 + LP)
W sin tan φ¢
LP ) tan
(C)
(C) (D)
(D)
W sin δ5
W sin cos δ5
W cos
W

MN
MN 4/8
4/8
GATE 2014
GATE 2014 MINING —
MINING – MN
MN

Q30
Q.30 The lengths
The lengths and
and standard
standard errors
errors of
of three
three sections
sections AB,
AB, BC,
BC, and
and CD of aa straight
CD of straight line
line AD
AD are
are given
given
below
below

AB = 125.85±0.021
AB = l25.85i0.02l m; BC= 205.72±0.029
m; BC= 205.72i0.029 m; CD=246.2li0.025 m
m; CD=246.21±0.025 m

The standard
The standard error
error in
in total length AD
total length in m
AD in is
m is
(A) ±0.0436
(A) i0.0436 (B) ±0.0350
(B) i0.0350 (C) ±0.0250
(C) i00250 (D) ±0.0019
(D) i0.0019
0
Q31
Q.31 The bearing
The bearing ofof side
side AB of aa regular
AB of regular hexagon
hexagon ABCDEF
ABCDEF is is 𝑆𝑆
S 50 10′ 𝐸𝐸 .. If
5001OIE If the
the station
station C
C is
is easterly
easterly
from the
from the station
station B,
B, the
the whole circle bearing
whole circle of the
bearing of the side
side BC
BC is
is

(A) 65
(A) 65°15’25”
0
15′ 25′′ (B) 69
(B) 69°50’25”
0
50′ 25′′ (C) 69
(C) 69°15’25”
0
15′ 25′′ (D) 69
(D) 69050’0"
0
50′ 0"

Q32
Q.32 In aa room-and-pillar
In room-and-pillar stope,
stope, bench
bench blasting is conducted
blasting is conducted using having density
ANFO having
using ANFO density of
of 800
800 kg/m3.
kg/m3.

)
The specific
The specific gravity
gravity of
of rock is 2.5,
rock is 2.5, hole diameter is
hole diameter is 100
100 mm and spacing
mm and spacing to
to burden
burden ratio is 1.3.
ratio is 1.3. The
The
charge length
length of
of each
each blast hole is
is 80%
80% of of the
the hole
hole length.
length. For
For aa desired
desired powder of 0.48
factor of 0.48

14
charge blast hole powder factor
kg/tonne, the spacing
kg/tonne, the spacing and
and burden
burden of of the
the blast
blast pattern in m
pattern in m respectively are
respectively are
(A) 2.0,
(A) 2.0, 2.6
2.6 (B) 2.3,
(B) 2.3, 1.8
1.8 (C) 5.2,
(C) 5.2, 4.0
4.0 (D) 1.3,
(D) 1.3, 1.0
1.0
Q33
Q.33 Match the following
Match the following for
for ore
ore handling
handling operations in an
operations in an underground metal mine
underground metal mine

20
Arrangement
Arrangement Description
Description

(P) Drawpoint
(P) Drawpoint (I) arrangement
(I) that prevents
arrangement that oversized rock
prevents oversized rock to
to pass
pass

(Q) Ore
(Q) Ore pass
pass (II) aa system
(II) of vertical
system of or near
vertical or near vertical for
openings for
vertical openings
E
transferring ore from
transferring ore from aa stope
stope to
to aa single
single delivery
delivery point
point

(R) Grizzly
Grizzly (III) aa place
place where ore can
can be loaded and
and removed
AT

(R) (III) where ore be loaded removed

(S) Finger
(S) Finger raise
raise (IV) aa vertical
(IV) vertical or
or inclined
inclined opening
opening used for transferring
used for ore
transferring ore

(A) P-IV,
(A) P-IV, Q-III,
Q-III, R-II,
R-II, S-I
S-I (B) P-III,
(B) P-III, Q-IV,
Q-IV, R-I,
R-I, S-II
S-II
(G

(C) P-II,
(C) P-II, Q-IV,
Q-IV, R-I,
R-I, S-III
S-III (D) P-III,
(D) P-III, Q-I,
Q-I, R-II,
R-II, S-IV
S-IV

Q34
Q.34 The following
The following characteristic
characteristic curves
curves (P, Q, R,
(P, Q, R, S)
S) pertain
pertain to rotary drilling
to rotary drilling in
in rock.
rock.
yY yV yV yY
N
M

xX xX xX xX
P Q R S
of the
Title of
Title the curve
curve
1:
I: Torque versus
Torque RPM
versus RPM
11:
II: Rate of
Rate of penetration
penetration versus
versus uniaXial compressive strength
uniaxial compressive of rock
strength of rock
111:
III: Rate of
Rate of penetration
penetration versus
versus weight on bit
weight on bit
IV:
IV: Specific energy versus
Specific energy versus weight on bit
weight on bit

Match the curves with


with their titles
(A) P-III, Q-IV, R-II, S-I (B) P-II, Q-IV, R-I, S-III

(C) P-IV,
(C) P-IV, Q-III,
Q-III, R-II,
R-II, S-I
S-I (D) P-I,
(D) P-I, Q-III,
Q-III, R-II,
R-II, S-IV
S-IV

MN
MN 5/8
5/8
GATE 2014
GATE 2014 MINING —– MN
MINING MN

Q35
Q.35 The height
The height H
H of
of aa drawpoint
drawpoint in
in aa sublevel
sublevel caving
caving stope
stope is
is 3.0
3.0 m. If the
m. If angle of
the angle repose (𝜑𝜑)
of repose ((0) of
of
broken ore is
is 35
350,, the
the digging depth )2y of
digging depth of the loader as
as shown
shown inin the figure in
in m is _______
0
broken ore the loader the figure m is

Ore Waste
rock

4500 −—5'ϕ1
45
</ 2
H
4
%IH ϕ(0
///////Fyal////////////////////
y

)
Q36 an explosives
For an explosives company, the probability
company, the of producing
producing aa defective
defective detonator
detonator is
is 0.02.
0.02. The
The

14
Q.36 For probability of
probability that aa lot
probability that lot of
of 50
50 detonators
detonators produced
produced by the company
by the company contains
contains at
at most
most 22 defective
defective
detonators is
detonators is __________

Q37
Q.37 The area
area enclosed
enclosed by curves yy = x2
the curves and yy = x3
x 2 and x3 for 6 [[0,00]
for xx ∈ 0, ∞ ] is
is

20
The by the

(A) 1/12
(A) 1/12 (B) 1/6
(B) 1/6 (C) 1/2
(C) 1/2 (D)11
(D)

Q38
Q.38 The value
The value of
of a,
a, for
for which the function
which the function below
below isis continuous
continuous at = 11 is
at xx = is
E
22x+ax2,
x + ax , x31
2
x ≤1
f ( x) = 
f(x)—{ x + 3, x>1
 44x+3, x >1
AT

(A) -5
(A) -5 (B) 0
(B) 0 (C) 5
(C) 5 (D) 10
(D) 10

Q.39
Q39 infinite series a + ar + ar
of the infinite
The sum of air2 + ar
2 3 - - - + ar
ar3 + 
n −1 - - - for lr‘
l'n_1 +  r <<11 is

(A) aa(1+r)
(A) (1 + r ) (B) aa(l—r)
(B) (1 − r ) (C)
(C) fl
a
(D)
(D) fi
a
(G

1+ r 1− r

Q40
Q.40 A centrifugal
A centrifugal pump has aa discharge
pump has discharge rate
rate of
of 2000
2000 LL of water per
of water per min against aa total
min against total head of 200
head of 200 m. If
m. If
the pump
the efficiency is
pump efficiency is 75%,
75%, the
the input
input power to the
power to the pump in kW
pump in is
kW is
(A) 87.20
(A) 87.20 (B) 49.05
(B) 49.05 (C) 13.33
(C) 13.33 (D) 7.50
(D) 7.50
N

Q41
Q.41 A dragline
A dragline is
is required
required to to remove
remove 3,00,000
3,00,000 m m3 of
3
of rock
rock per month on
per month on the
the bank
bank volume
volume basis.
basis.
Consider the following
Consider the following data
data for
for the
the dragline
dragline operation.
operation.
M

Effective working
Effective working hours
hours per month =
per month = 450
450
Bucket fill
Bucket fill factor
factor = = 0.8
0.8
Cycle time
Cycle time == 65
65 ss
Swell factor
Swell factor of
of the
the rock
rock = = 1.25
1.25
The minimum
The minimum bucket
bucket capacity
capacity of of the
the dragline
dragline in
in m
m3 is
3
is
(A) 7.70
(A) 7.70 (B) 9.63
(B) 9.63 (C) 12.04
(C) 12.04 (D) 18.80
(D) 18.80

MN
MN 6/8
6/8
GATE 2014
GATE 2014 MINING —
MINING – MN
MN

Q42
Q.42 A direct rope
A direct rope haulage
haulage pulls
pulls 88 tubs
tubs loaded
loaded with
with coal
coal through
through an
an incline
incline of length 500
of length 500 m
m having
having an
an
inclination of
inclination of 11 in
in 6.
6. Consider
Consider the
the following additional data.
following additional data.

Capacity of
Capacity of tub
tub == 1.0
1.0 tonne
tonne
Tare weight
Tare of tub
weight of tub == 500
500 kg
kg
Hauling speed
Hauling speed ==99 km
km per
per hour
hour
Coefficient of
Coefficient of friction
friction between
between wheel and rail
wheel and = 1/60
rail = 1/60
Coefficient of
Coefficient of friction between rope
friction between and drum
rope and =
drum = 1/10
1/ 10
Mass of
Mass of rope
rope per meter =
per meter = 1.5
1.5 kg
kg

The minimum
The minimum power
power required to haul
required to haul the
the tubs in kW
tubs in is
kW is
(A) 345.50
(A) 345.50 (B) 348.60
(B) 348.60 (C) 350.10
(C) 350.10 (D) 365.50
(D) 365.50
Q43
Q.43 A coal mine
A coal mine receives two bids
receives two for purchase
bids for purchase ofof aa new
new dragline.
dragline. The
The first
first bid quotes Rs.
bid quotes Rs. 150
150 crore
crore as
as

)
aa price to be
price to be paid in full
paid in full on
on delivery.
delivery. The
The second
second bid quotes Rs.
bid quotes Rs. 180
180 crore
crore as
as aa price
price payable at the
payable at the

14
end of
end of the
the third
third year
year after
after delivery.
delivery. If
If the
the discount
discount rate
rate is 12%, the
is 12%, the difference
difference inin NPV
NPV between the
between the
first and
first and second
second bids
bids in
in crore
crore of
of rupees is __________
rupees is

Q44
Q.44 Match the following
Match the following in
in the
the context
context of
of underground environment
mine environment
underground mine

20
Instrument
Instrument Measuring parameter
Measuring parameter
P. Haldane
P. Haldane apparatus
apparatus I.
I. Humidity
Humidity
Q. Godbert-Greenwald
Q. Godbert-Greenwald apparatus
apparatus II.
II. Air velocity
Air velocity
R. Hygrometer
Hygrometer III. Mine air
air composition
composition
E
R. III. Mine
S. Anemometer
S. Anemometer IV. Ignition
IV. Ignition point
point temperature
temperature
AT

(A) P-II,
(A) P-II, Q-I,
Q-I, R-III,
R-III, S-IV
S-IV (B) P-III,
(B) P-III, Q-IV,
Q-IV, R-I,
R-I, S-II
S-II

(C) P-IV,
(C) P-IV, Q-II,
Q-II, R-III,
R-III, S-I
S-I (D) P-I,
(D) P-I, Q-III,
Q-III, R-IV,
R-IV, S-II
8-11

Q45
(G

Q.45 A mine airway


A mine airway having cross-section of
having cross-section 2.2 m
of 2.2 m ×X 2.2
2.2 m and length
m and length 500
500 m
m contains
contains aa bend. Given that
bend. Given that
2 −4
the airway
the airway friction
friction factor
factor is
is 0.01
0.01 Ns2m_4, shock loss
Ns m , shock loss factor
factor for
for the
the bend is 0.07
bend is 0.07 and
and density
density of air is
of air is
1.2 kg/m3,
1.2 the equivalent
kg/m3, the equivalent length
length of
of the
the airway
airway inin mm is
is __________

Q.46
Q.46 In order to NVP in a mine,
to estimate the NVP mine, measurements
measurements are made
made at the
the main
main fan as shown below.
below.
N

Fan speed
Fan speed (RPM)
(RPM) Fan drift
Fan drift pressure (Pa)
pressure (Pa) Fan quantity
Fan quantity (m
(m3/s)
3
/s)
800
800 655
655 82.2
82.2
950 730 85.5
M

950 730 85.5

The NVP
The in Pa
NVP in Pa is
is ____________

Q47
Q.47 The resistances
The resistances ofof two
two splits
splits AA and B are
and B are 0.35 Ns2m−8 and
0.35 Ns2m_8 and 0.05 Ns2m−8 respectively.
0.05 Ns2m_8 The combined
respectively. The combined
2 −8
resistance
resistance of of the
the shafts
shafts and
and trunk
trunk airways
airways is
is 0.4
0.4 Ns2m_8.
Ns m . A A booster fan is
booster fan is planned to be
planned to installed in
be installed in
split A
split to increase
A to increase the
the quantity
quantity flowing
flowing through
through it.it. Assuming
Assuming that
that the surface fan
the surface fan continues
continues to
to
operate at
operate at aa constant
constant pressure of 1000
pressure of 1000 Pa,
Pa, the critical pressure
the critical of the
pressure of the booster fan in
booster fan in Pa
Pa is
is _______

MN
MN 7/8
7/8
GATE 2014
GATE 2014 MINING —
MINING – MN
MN

Q.48
Q.48 A
A pitot tube is
pitot tube is inserted
inserted in
in aa ventilation duct with
ventilation duct with the
the nose facing the
nose facing air flow.
the air flow. A A vertical
vertical U-tube
U-tube
manometer filled with
manometer filled alcohol (specific
with alcohol gravity 0.8)
(specific gravity 0.8) has
has been
been used for pressure
used for pressure measurements such
measurements such
that 10.2
that 10.2 mm
mm is
is read as the
read as the total
total pressure and 8.8
pressure and 8.8 mm
mm asas the static pressure.
the static Given the
pressure. Given of air
density of
the density air
to be
to 1.2 kg/m3,
be 1.2 kg/m3, the air velocity
the air velocity at
at the
the nose of the
nose of the pitot
pitot tube in m/s
tube in is ____________
m/s is

Q.49
Q.49

S
E

3m
m

A?
A B
B

)
/////L///////////////////////<l///
|\ 4
4mm ’1

14
An illumination source
An illumination source S
S shown
shown inin the figure emits
the figure emits light
light equally
equally in
in all
all directions.
directions. At point A
At aa point on
A on
the floor,
the floor, the
the illuminance
illuminance is
is 5.0
5.0 lux.
luX. The
The illuminance
illuminance atat point B on
point B on the in lux
floor in
the floor luX is
is ________

Q50
Q.50 Two machines
Two machines A and B
A and B while operating simultaneously
while operating simultaneously produce
produce aa sound
sound pressure level of
pressure level of 85
85 dBA
dBA

20
at aa point.
at When the
point. When the machine
machine AA stops,
stops, the
the sound
sound pressure level at
pressure level at that
that point
point reduces
reduces to 80 dBA.
to 80 dBA. The
The
sound pressure
sound level at
pressure level at the same point
the same due to
point due to machine
machine A operating alone
A operating in dBA
alone in dBA is
is
(A) 70.0
(A) 70.0 (B) 75.0
(B) 75.0 (C) 80.0
(C) 80.0 (D) 83.3
(D) 83.3
Q51
Q.51 A
A waste
waste water effluent has
water effluent BOD55 of
has BOD of 80
80 mg/L and the
mg/L and the reaction
reaction rate constant is
rate constant is 0.16
0.16 per day. The
per day. The
E
ultimate BOD in
ultimate BOD in mg/L
mg/L is
is
(A) 85
(A) 85 (B) 100
(B) 100 (C) 120
(C) 120 (D) 145
(D) 145
AT

Q52
Q.52 A series of
A series of tri-axial
tri-aXial compression
compression tests conducted on
tests conducted on sandstone
sandstone samples
samples reveal
reveal the following
the following
relationship
relationship between and minor
major and
between major minor principal stresses
principal stresses

𝜎𝜎 = 50
011 = 50 +
+ 3𝜎𝜎
3033 [stresses are
[stresses in MPa]
are in MPa]
(G

The cohesion
The cohesion in
in MPa
MPa and
and angle
angle of
of internal
internal friction
friction in
in degrees
degrees of
of sandstone
sandstone respectively are
respectively are
(A) 14.43,
(A) 14.43, 30.0
30.0 (B) 14.43,
(B) 14.43, 60.0
60.0 (C) 0.21,73.9
(C) 0.21,73.9 (D) 0.21,16.1
(D) 0.21,16.1

Q53
Q.53 SiX detonators
Six detonators each having resistance
each having resistance of
of 1.5
1.5 ohm
ohm are
are connected
connected inin parallel. A 15
parallel. A 15 V
V exploder
exploder isis
connected to
connected to the detonators by
the detonators two single-core
by two single-core cables
cables of
of resistance
resistance 3
3 ohm
ohm each.
each. The
The current in the
current in the
N

circuit in
circuit in Ampere
Ampere isis __________

Q54 The failure


failure and
and the
the repair
repair rates of aa shovel
shovel are
are 0.06 hr−1 and
0.06 hr‘1 and 0.04 hr−1 respectively.
0.04 hr‘1 The availability
availability
M

Q.54 The rates of respectively. The


of the
of shovel in
the shovel in percentage is ____________
percentage is

Q55
Q.55 The individual
The individual reliability
reliability values of four
values of four sub-systems
sub-systems are
are given in the
given in figure below.
the figure The reliability
below. The of
reliability of
the system
the system is
is _________

0.6 0.6
A B

C D
0.5 0.5

END OF
END THE QUESTION
OF THE QUESTION PAPER
PAPER
MN
MN 8/8
8/8
Graduate Aptitude Test in Engineering

Notations :
LDIDtlDIlS SHONE-Ell lIl ETEEI-l CDlOI‘ ElIlCl with 1’ lCDIl ElI'E CDI‘I‘ECt.

EDptiona shown in red color and oath 3‘ icon are incorrect.

Question Paper Name: MN: MINING ENGINEERING 1st


lst Feb shift2
Number of Questions: 65
Total Marks: 100.0
100.0

IWrong
Wrong answer for MCQ will result in negative marks, (-1/3) for 11 mark Questions and (-2/3) for 2 marks Questions.
Questionsl

General aptitude
Number of Questions: 10
10
Section Marks: 15.0
15.0

IQ.1 to Q.5
Q.1 to carry 11 mark
0.5 carry mark each
each &
& Q.6 to Q.10
0.6 to 0.10 carry
carry 22 marks each.|
marks each.

Question Number : 1 Question Type : MCQ


Choose the appropriate word-"phrase- out of the four options given below, to complete the following
eentence:

Apparent lifeleaaneae dormant life-

(A) harbour-3. (B) leads to (C) aupporte (D) affecte.

Options
Options::

1. fa
a. it a
a. it c
a. it D

Question Number : 2 Question Type : MCQ


Fill in the blank with the correct idiom-"phrae e.

That boy; from the town was a in the sleep}.r tillage-


(A) dog out of herd (B) sheep fiorn the heap
(C) fish out of water (13] bird flow the flock

Options
Options::
1. it a
a. it a
3. ate
a. it D

Question Number : 3 Question Type : MCQ


Cheese the statement where underlined werd is us ed cerrectly.

(A) When the teacher eludes te diflirent authers, he is being elusiye.


(B) When the thief keeps eluding the pelice, he is being elusive-
(C) Matters that are difficult te understand, identify er remember are allusiye-
(D) Mirages can be allusiye. but a better way te express them is illusery-

Options
Options::
1. fl .4
2.13s
3. 3 c
4. 3 :1

Question Number : 4 Question Type : MCQ


Tanya is elder than Eric-
Cliff is elder than Tanya.
Eric is elder than CHE

If the first twe statements are true. then the third statement is:

(A) True
(B) False
(C) Uncertain
(D) Data insufficient

Options
Options::

1. fl .4
2.13s
3. 3 c
4. 3 :1

Question Number : 5 Question Type : MCQ


Five teams have te cempete in a league. with every team playing eyery ether team exactly ence,
befere geing te the nest reund Hew many matches will have te be held te cemplete the league
reund ef matches?r

{A} 20 (B) 10 (E) s (D) 5


Options
Options::

1. fl .4
s. s" s
s. 3 c
4. 3 :1

Question Number : 6 Question Type : MCQ


Select the appropriate option in place of underlined part of the sentence-

Increased productivity necessar‘i,r reflects greater efforts made by the employees-

(A) Increase in productivityr necessaryr


(B) Increase productivity is necessary
(C) Increase in productivity necessarily
(D) No improvement required

Options :
1. 3 a
s. 3 s
s. v" c
4. 3 o

Question Number : 7 Question Type : MCQ


Given below are two statements followed by two conclusions- Assuming these statements to be
true decide which one logically follows-

Statements:
1 No manager is a leader-
II All leaders are executives-

Conclusions:
I- No manager is an executive-
II No executive is a manager-

(A) Unly conclusion I follows.


(E) 'Ilnl‘j.r conclusion I[ follows-
(C‘) Neither conclusion I nor II follows-
(IJ) Both conclusions I and II follow-
Options
Options::
1. 3 a
s. 3 s
3. ate
4. 3 o

Question Number : 8 Question Type : NAT

./:
In the y'ven figure angle Q is a right angle- PSflS = 3:1- RT:QT = 5:2 and PU:UE. = l: l- Ifarea of
triangle ors is so cm:- then the area of triangle ms in cm: is

/\/.
Correct Answer :
280
Question Number : 9 Question Type : MCQ
Right triangle PQE- is to be constructed in the 143* — plane so that the right angle is at P and line PR is
parallel to the t-t-aJ-ti-s- The 1: and v coordinates of P- Q- and R are to he integers that satisfy the
inequalities: —4 :33 :4 1i 5 and Er :33 3’ E lfi. How many different triangles could be constructed with
these properties?

(14} 11:) (B) 1.1013 (c?) snot: (D) lfl-flflfl

Options
Options::

1. fl -4
a. 1* a
s. v" c
4. 3 :1

Question Number : 10 Question Type : MCQ


A coin is tossed thrice- Let X he the event that head occurs in each of the first two tosses- Let I" he
the event that a tail occurs on the third toss- Let 2' he the event that two tails occur in three tosses-
Eased on the above information- which one of the following statements is TE't-UE“.l
(A) If and I” are not independent (B) F and Z are dependent
(C) l" and-E are independent (D) If and E are independent

Options
Options::
1. fl -4
a. is
s. 3 c
4. 3 :1

Mining Engineering
Number of Questions: 55
Section Marks: 85.0

IQ.11 (1.35 carry 11 mark each & Q.36


Q.11 to Q.35 0.36 to Q.65 each.l
0.65 carry 2 marks each.

Question Number : 11 Question Type : MCQ


Out of the support categories given for an underground coal mine- identify,r the ”active supporf-

(A) wire mesh (B) shotcrete


(C) full}r grouted roof holt (D) hydraulic prop

Options
Options::
1. 1* .4
2. 3 a
s. 3 c
4.13:1

Question Number : 12 Question Type : MCQ


Massive sandstone in immediate reef delays the lcrcal fall in gnaf of a cnal mine. Under this
cnnditicm, crushing of the pillars at nutbye side is called
(A) coal bump (B) nverriding elf pillars
(C) stifi‘ening pf pillars (D) spalling crf pillars
Options :
1. fl .4
2. v" s
s. it c
4. it :1

Question Number : 13 Question Type : NAT


A hack sight an a bench mark {if RL 100-00 m on the floor of a mmiel is 3-25 m The inverse staff
reading on a reef station nf the tunnel is 1-2.5 111 The EL of the roof statinn in m is

Correct Answer:
104
104 to 105
105

Question Number : 14 Question Type : MCQ


The angle in degrees at which a ridge line intersects ccrntcrurs is

(4)11 (B) as (1:) 4:1 (n) as


Options
Options::

1. it .4
2. it s
s. it c
4. in

Question Number : 15 Question Type : MCQ


In a drum hoisting system thrnugh a vertical shaft. nverwinding is prevented by

(A) Lilly centraller


(B) detaching hnnlr
(C) caliper brake
(D) safety catch

Options
Options::
1. fl .4
2. is
s. it c
4. it :1

Question Number : 16 Question Type : MCQ


The temperature of a parcel of air decreases finm 30.2" C ten 239° C as it rises from an altitude of
213 m to 12D :11 The lapse rate fur the atmcnsphere is

(A) subadiabatic (B) adiabatic (C) superadiabatic (D) transadiahatic

Options :
1.314
sis
sec
43::

Question Number : 17 Question Type : MCQ


The excess pore pressure in bacltfill material in a cut-and—fill stnpe leads tn

(A) reduction in strength of the wall rcnclc


(B) enhancement pf bearing strength pf fill
(C) less of shear resistance of fill
(D) preventinn pf prcrgessive failure pf crntvn pillar

Options
Options::
1. fl 4
2. 3 s
s. u'c
4. fl :1

Question Number : 18 Question Type : MCQ


The priniarj.r purpnse pf cut hcrles fcrr blasting in an underground drivage is In

(A) prcrvide additinnal flee face


(B) have smnnth surface after blasting
(C) prevent ever—breakage
(D) reduce nnise

Options
Options::

1. #14
2. 3 s
s. 1* c
4. fl :1

Question Number : 19 Question Type : MCQ


In a triangle sac. the bearings (if the 4144.4 an. ac. and c4 are as“ _. 13:1”. and .1111“ respectively-
The interinr angles A- E, and C in degrees respectively.r are

(.4) 111144.311
(B) 411. 1111312:
(c1 311.411.1111
(D) 311110.44
Options
Options::

1. fl 4
2. 3 s
s. 3 c
4. in

Question Number : 20 Question Type : MCQ


In a binc-rnial distributing the pnababilit‘j.r nf success p —} fl and number of trials 14 —} as such that
H. = Hp apprcraches tcr a finite value- The variance of the distributicrn is

(.4) 44:14- [B] as. (1:?) p4. (1:1) .1


Options
Options::

1. 1* 4
2. fl 2
s. 3 c
4. 4’21

Question Number : 21 Question Type : NAT

For a function ftt‘) _ it is given that f(0) = 2 and fTD) = 4- Ignoring all other higher order
derivative terms. the value of f{0-5} is

Correct Answer :
44

Question Number : 22 Question Type : MCQ

The two sides of a parallelogram are given by the vectors A 2 El — 3! andB = 33 + E} . The
area of the parallelogram is

[A2113 (B) 12 (C) lfl (D) 5

Options :
1. u" 4
2. fl 2
s. 1* c
4. 3 :1

Question Number : 23 Question Type : MCQ


In a HUD test, 5 ml of 1arastevcater is diluted with pure water to fill a fiflfl ml BOD bottle. The
initial and final dissolved oxygen contents of the mist are 9-0 mg.-"l and TD mg-"l respectively. The
EDD of the wastewater. in mg.-'l. is
(14:12. (B) it: (c?) 12o (D) soc:

Options
Options::

1. fl 4
2. fl 2
s. u" c
4. 1* :1

Question Number : 24 Question Type : NAT


A force of 50 N is applied to a 1wrench as shown in the figure- The magnitude of the moment in
N-mm of this force about the point P is

Correct Answer:
7900 to 7920

Question Number : 25 Question Type : MCQ

lil'ilatanc},r of rock is associated with


(A) increase in surface area after fi'agmentation
(B) decrease iu volume due to compression of rock
(C) increase in shear strain due to cracking of rock
(D) increase in volume due to cracking of rock

Options
Options::

1. it a
s. it s
s. it c
4. in

Question Number : 26 Question Type : NAT


A hord and pillar panel having square pillars is designed for sass extraction during development. If
the gallerj.r width is 5 m, the side of the pillar in m is

Correct Answer:
25 to 26

Question Number : 27 Question Type : MCQ


Low shock and high gas pressure explosive is generallyr used for blasting of
(A) hard and brittle rock mass
(13) soft and jointed rock mass
(C) hard and massive intact rock mass
(D) soft and massive intact rock mass

Options :
1.4x
salts
sic
41o

Question Number : 28 Question Type : MCQ


The covariance of copper grade for a certain lag distance in an ore body is fi-D (%)2- If the sill is
10 (941):- the semivariogram for the same lag distance in (‘34:): is

(A) 4.0 (B) 115-0 (E) 2-D (D) 154-13

Options :
1. if s
2. 4 s
s. 4 c
4. 4 o

Question Number : 29 Question Type : MCQ

—4 s 2 I5 4 IS
The matrix A = 4 I5 4 5 2 IS is
E 5 —4 s it“ oi

(-4.) orthogonal (B) diagonal (C) shew-symmetric (D) symmetric

Options :
1. if s
2.43
sic
41o

Question Number : 30 Question Type : NAT


.4 gas mixture contains CH4- CgHfi and Hg with respective concentrations of lfi‘l-t- 15% and 113% by
volume. The lower explosibilitv limit of CH4- CEHfi and H: are 5-fl%- 3-3% and 4.2% respectively-
The lower explosilzrilitjrr limit of the gas mixture, in percentage, is

Correct Answer:
4.2 to 5.0

Question Number : 31 Question Type : NAT

Intake air containing [ll-2% methane enters a section of an underground mine where emission rate of
methane is CLUE nil-"s. Assuming that the threshold limit value of methane is 1.259% the minimum
quantity of fresh air required in m'l-"s is
Correct Answer :
4.6 to
to 4.9

Question Number : 32 Question Type : MCQ


In a fully mechanised bord and pillar mining system winning of coal and its transportation fi'om the
face is commonly carried out with the combination of

(A) continuous miner shuttle car. feeder breaker and belt conveyor
(B) continuous miner- LEI}, feeder breaker and chain conveyor
(C) continuous miner- SDL- feeder breaker and belt conyeyor
(D) continuous miner shuttle car. feeder breaker and chain conveyor

Options :
1. a!" x
sits
sic
4.3::

Question Number : 33 Question Type : NAT


A11 underground coal mine employing 1200 persons experiences 2 fatal injuries. 5 serious injuries
and 3 reportable injuries during the year 2013. The total injury rate per 1000 persons employed for
the year is

Correct Answer :
13.0
13.0 to 13.6
13.6

Question Number : 34 Question Type : MCQ


In self—contained chemical—oxygen self—rescuer- oxygen is produced by

(A) Hopcalite (B) potassium peroxide


(C) sodium hydroxide (D) Protosorb

Options
Options::

1. it x
2.13s
3. it c
s. it o

Question Number : 35 Question Type : NAT


The failure data of an equipment follows an exponential distribution If the mean time between
failures is 3000 hours, the reliability of the equipment for 7’50 hours is

Correct Answer:
0.75 to 0.81
0.81
Question Number : 36 Question Type : MCQ
In a 4-2 m wide and 3-D m high gallery in a coal seam- twelve shot holes are blasted per round- The
holes are charged with 2 explosive cartridges of 435 g each- If the powder factor of the blast is 2-2
tomrefl-zg and specific gravity of coal is 1.4- the pull per round of blast in m is

(A) 1.45 (a) 1.7421 (G) 1.3a (1])4106


Options
Options::

1. fl -4
s. 1* s
a. v" c
4. 3 :1

Question Number : 37 Question Type : NAT


The stadia readings with horizontal sight on a vertical staff held at 50 m from a tacheometer are
1-23:3 m and 1.230 m- The focal length of the object glass is 2:3 cm- and the distance between the
object glass and the vertical axis of the tacheometer is 15 cm- The staclia interval in mm is

Correct Answer:
2.48 to 2.52

Question Number : 38 Question Type : MCQ

In a shortwall panel- coal is extracted from the face by a continuous miner having rate of production
3U tonne-11. lCoal having specific gravity of 1-4 is transported by shuttle cars of capacity 0.9 mt3
each to a feeder breaker located at 60 m from the face- If the average speed of the LED is [1-5 m-"s_-
and total loading and unloading time of LED is 4G s_- the mnnber of LHDs required to match the
production of the continuous miner is

(4)1 (3)2 (C) 3 (1334


Options
Options::
1. fl -4
s. v" s
a. 3 c
4. 3 :1

Question Number : 39 Question Type : NAT


Vertical photographs of an area lying SUD m above the mean sea level are to be talcen at a scale of
1:2flflflfl from an aircraft- If the camera has a focal length of 210 mm. the flying height of the
aircraft above the mean sea level in m is

Correct Answer:
4700

Question Number : 40 Question Type : MCQ


Match the fnllewing lecatiens with suppert types in ccral mines.

Location Sup-pert tvpe

P- Rnadwav junctions l- Pcrwered Support


Q- Between adjacent panels 2- Check and belt
FL Lengwall face 3- Each fill
S- Geaf 4- Barrier pillar

(mean—3,114.24 fB)P-4,Q-3.R—l-S-2 (c) P—E,Q-4.R-1-S-3 {D} P—2,Q—3.R—4.a—1


Options
Options::

1. 3 a
2. 3 a
2. fr:
4. 3 D

Question Number : 41 Question Type : MCQ


4
'
|I| — '1 :1
The value ef J 16 all is
{1.

(A) 12.52 (13] 5:21.24 (C) 2:112 (D) 3-14


Options
Options::
1. a!" a
2. 3 a
2. 3 c
a. 3 D

Question Number : 42 Question Type : NAT


A rectangular field of area EUUDD n12 is to be divided intcr 5 different plots by fencing as shown in
the figure- The value of L in in fur which the total length of fencing becomes: minimum is
L. L
.J
'1 1'"
| | | | | | | | | | | |
.l -l

I I"

Correct Answer :
161
161 to 165
165

Question Number : 43 Question Type : MCQ


Match the following for a drilling system

Component Function

P. Drill l. Utilization of energy in fi'agmenting rock


Q- Drill rod 2- Reduction of energy loss due to regrinding
E. Drill hit 3- IEonversion of original form of energyr into mechanical energy
E. Flushing medium 4- Transmission of energy fi'om prime mover to applicator

(A) P-B-Q-lR-lS-dl (B) P-4,Q-1-R-3-S-E (C) P-3,Q-4-R-l-S-E (D) P-EQ-lfl-E-S-LI

Options
Options::
1. 3 a
2. 3 s
3. fr:
4. 3 o

Question Number : 44 Question Type : NAT

For the ventilation system shown- the combined resistance of the trunk airways and the shafts is
2-2 nm'fl- The resistances of splits A and B are flj l‘ilszm'3 and [LE l"»ls2m'E respectively- A
regulator of size 2.13 m2 is placed in split A. lConsidering the fan generates a pressure of lflflfl Pa-
the air flow in mlfs in split B is

D-C- shaft
LII-C. shaft

Intake trunk Return trunk


airway airway
Spl'rt B

Correct Answer :
10.2
10.2 to 10.8
10.8

Question Number : 45 Question Type : NAT

A mine fan running at 30D rpm delivers 150 m3.-"s of air at a pressure of 900 Pa. Fan and motor
efficiencies are q'lll- and gill-“E: respectively- If the fan speed is reduced to 250 rpm- the saving in
electric power input to the motor in l-IW' is

Correct Answer :
82 to 86

Question Number : 46 Question Type : NAT


Subsidence profile function- .s(-r)- along the lateral cross-section over a flat longwall panel is given
as

"3.3.1"
sfi-r) = H.3[fl395 — tanl1[ Uym
D

where I = distance (m) from the inflection point and D = depth (m) of the seam- Considering that
the inflection point lies vertically above the edge of the panel, the angle of draw in degrees for a
depth of EEG m is

Correct Answer :
20 to 21
21

Question Number : 47 Question Type : NAT

A goaf voidfiof 25D n13 is filled in 3 hours by hydraulic sand stowing method- Density of the sand is
2-6 tonne-"m:- If the filling factor of goaf void is fl-El and sand to water ratio in the stowing mixture
is l-[l| torme to 1-1 m3 - the stowing rate in mJ.-"h is

Correct Answer :
286 to 293

Question Number : 48 Question Type : NAT


A single-acting reciprocating pump delivers Ill-DIS mlfs of water when rI-oing at 45 cycles per
minute- The piston diameter is SUD mm and stroke length is 4043 mm- The volumetric efficiency of
the pinup in 'Hi is

Correct Answer :
83 to 87

Question Number : 49 Question Type : MCQ

Match the method of mining with strength of oreb-ody- type of support and orebody geometry.

Strength Support Geometry lIethod

P. Strong L- Unsupported K. Tabular and steep l. Cut—Elfld—fill


Q- Moderate M- Artificially supported Y. Tabular and flat 2- Block caving
E. ‘Wealc N- Self—supporting E- Massive and steep 3- Room and Pillar

(A) P—M—K—E- Q—N—Z—E, a_L_v_1


(B) P-L—H—l- o—N—as- R—M—Y-E
(e) P—N—v—s, o—M—a—l- a—L—a—s
(D) P—L—Z—l- Q—N—Y—l assoc—s
Options :
1.3a.
2.3a
sic
41o

Question Number : 50 Question Type : NAT


A mine air sample contains C'I-L- CC}- Hill; and U3- The mine air analysis using Haldane apparatus
gives the following results expressed in percentage of total sample volume.

Total contraction after combustion : 10.0


CD: formed after combustion : 0-0
C}: consumed in combustion : 9.5

The percentage of CH4 in the sample analysed is

Correct Answer:
3.8 to 4.2

Question Number : 51 Question Type : NAT


The initial investment for a small scale mming project is Rs. 5.0 crore. Armual cash inflow for a life
period of 4 years is gven helow.

Year Cash inflow (Rs- crore)


l 1.5
2 2.0
3 2-0
4 1-5

The net present value of the project at an arnlual discoturt rate of 10% in Rs. crore is

Correct Answer :
0.5 to 0.6

Question Number : 52 Question Type : MCQ


Given the following linear programming problenl.

Maximise E 2 3x1 + 4.1”:


Subject to
Erl + r2 i a
23:1 + 3a:2 E 9

r1 20.1:2 20

the comer point feasible solution in terms of (rhrg) is

(A) (1-5.0) (a) (1-25- 1.5} (c) (115. 1a) (0)1325. 1.5}
Options :
1.3a
21's
sic
auto

Question Number : 53 Question Type : NAT


The 3—period torque—time diagram of a statically balanced hoist is shown in the figure-

221]
It“)
131]
151]
141]
[RN-n1) 120
ion
ED
Torque
ED
tit]
21]

True [5}

The rms torque for the motor in kN—m is

Correct Answer
Answer :
106
106 to 113
113

Question Number : 54 Question Type : NAT


Airborne PMm concentration in a residential area is monitored for 24 hours by a respirable dust
sampler- Initial and final weights of the filter paper are 2-3125 g and 2.6996 g respectively- The
average airflow rate during sampling is 1.2 mlfmin The PMm concentration of the area in
_3_
new 15

Correct Answer
Answer :
220 to 228

Question Number : 55 Question Type : NAT


The assignment problem given requires four dififlent jobs to be done on four different machines-

Machine
M1 M1 M3. M4
37" 35 36
33 3'? 315
3D 2.5 23
33 2.9 3:3

The minimum cost of as signment is

Correct Answer:
116
1 16

Question Number : 56 Question Type : MCQ


Acceleration of a particle moving in a straight line is expressed by
’5'
dis _ 7'
h

{it2
where :3 denotes distance {m} and 1". time {s}- At time i": U- the distance and velocity of the particle
are l] m and 3 m-‘s respectively. The distance travelled by the particle in m after 3 s is

{A} 3 (1315 (C) 9 (D) 13


Options
Options::
1. it a
2. it a
s. it c
4. info

Question Number : 57 Question Type : NAT


Roch bolts have length I. = {1513 + X] cm.__ where X is a random variable with probability density
function
r' 1 _ — . .3
33"): 1f 2. i 1'5
f{.‘t}::1fl_
I (I. otherwise

If 95% of the bolt lengths {L} lie in the interval 150 — :3 cm to lit] + c cm the value of c is

Correct Answer :
1.88
1.88 to 1.92
1.92

Question Number : 58 Question Type : NAT


The properties for a hivariate distribution of two random variables X and l' are given below-

Eta?) = 24. EU") 2 36. 51233) = T02, Eli?!) =1524- ELEV} 21004

The correlation coefficient between I and l’ris

Correct Answer :
0.8 to 0.85

Question Number : 59 Question Type : MCQ

Eiartial stresses at a point inside a pillar are shown in the figure.

5

All stress values are in MPa
—""-.2

v
in

]—1D

1'
s
The magnitude of the mat-tinitnn shear stress in l'vIPa and its direction with the .r—aJ-tis in degrees at
the same point respectively.r are

(A) sss. stss (B) 14g stss (c) are sass to) 14152-02
Options
Options::
1. is
s. it s
s. it c
4. it o

Question Number : 60 Question Type : NAT


A circular tunnel is constructed in a biaJLial far field stress {vertical stress Jan and horizontal stress
KP”) as shown in the figure-

till

Tlll
If the ratio of the tangential stress measured at the bfllflldflfj.’ points A and E is 3:l, the 1value off: is

Correct Answer :
0.6

Question Number : 61 Question Type : MCQ


Peale particle velocityr (PPV) at points A and B are measured for a blast pattern as shown in the
figure-

4m

.H/
Connecting wires
I'TI
a

.I
i 1 -
EDI] m
I
l

C} 1 [312
Freetaee film"

m Oi: s last holes in initiation row


”__ 51.1.3-6) 2 O: : Blast holes in second row afterehslavr

Delay: 25 ms

The relevant data are:

Amount of explosives per hole in the 1H rov.r : 500 leg


Amount of explosives per hole in the 2nd row .' 4T5 leg
PPV at point A : 13 mm-"s
PPV at point B : lfl mm-"s

|Considering the following relationship.


'
—H

D
PPV=K[
.@
] _. mm"s

where D (in m) denotes the distance flom the blast row to the measuring point and Q [in leg),
maximum charge per delay- The site constants K and 11 respectively.r are
(A) 1011.1- 31.13 (311522. 2.9: (c1sss. 1.1-59
Options
Options::
1. I a
s. I s
3. fr:
«:1. I :1

Question Number : 62 Question Type : MCQ


Copper ore of average grade 0.65% is mined, milled smelted and then refined. The following
information is available:

Mill recover].r rate : 35913


Average grade in mill concentrate : 20%
Loss in smelting process : 5 leg-"tonne of concentrate
Loss in refining process : 2 leg-"tonne of blister copper

The amount of refined copper obtained per tonne of ore in leg is

(A1310 (1315-31 (c) 5.531 {1:11 6.50


Options :
1. I a
2.13s
sic
41::

Question Number : 63 Question Type : NAT


The ratio of horizontal to vertical in-sitn stresses, K. at a mine field varies with depth. D (in m) as

'J'
a=£+tss
o
If the unit weight of overburden rock is 2.5 kN.-"m3. the horizontal stress in MPa at a depth of 400 m
is

Correct Answer :
19.10
19.10 to 19.25
19.25

Question Number : 64 Question Type : NAT


A coal seam of 2 in thickness is extracted In; a longwall retreating panel with face length of 1.10 in.
"Web depth of the shearer is 0-6 111. Average manpower in the longwall face in a shift is EU. The
specific gravity.r of in—situ coal is 1.4- Ifthe shearer makes 4 full—face cuts in 3 shifts, the face EMS
in tonne is

Correct Answer :
13
13 to 14
14

Question Number : 65 Question Type : NAT

A loaded dumper of total mass T5 tonne. having wheel diameter 1250 mm__ runs on a haul road
which ofifls an average specific rolling resistance of 2.51] Nitonne. The engine develops an axle
torque of 15 kN—m. The starting acceleration of the dumper in m-"s2 is

Correct Answer :
0.055 to 0.065
GATE 2016 General Aptitude -- GA Set-2

Q. 1—
Q. 1 Q. 5
– Q. 5 carry
carry one
one mark
mark each.
each.

Q.1
Q.1 The volume of a sphere of diameter 11 unit is ________ than the volume of a cube of side 11 unit.
(A) least (B) less (C) lesser (D) low

Q2
Q.2 The unruly crowd demanded that the accused be _____________ without trial.
(A) hanged (B) hanging (C) hankering (D) hung

Q3
Q.3 Choose the statement(s) where the underlined word is used correctly:

(i) A prone is a dried plum.


(ii) He was lying prone on the floor.
floor.
(iii) People who eat a lot of fat are prone to heart disease.

(A) (i) and (iii) only (B) (iii) only (C) (i) and (ii) only (D) (ii) and (iii) only

Q4
Q.4 Fact: If
If it rains, then the field
field is wet.

Read the following statements:


(i) It rains
(ii) field is not wet
The field
(iii) field is wet
The field
(iv) It did not rain

Which one of the options given below is NOT


NOT logically possible, based on the given fact?
(A) If (iii), then (iv). (B) If (i), then (iii).

(C) If
If (i), then (ii). (D) If
If (ii), then (iv).

Q5
Q.5 A window is made up of of a square portion and an equilateral triangle portion above it. The base of
the triangular portion coincides with the upper side of the square. If the perimeter of the window is
2
6 m, the area of the window in mm2 is ___________.

(A) 1.43
(A) 1.43 (B) 2.06
(B) 2.06 (C) 2.68
(C) 2.68 (D) 2.88
(D) 2.88

1/3
1/3
GATE 2016 General Aptitude -- GA Set-2

Q. 6—
Q. 6 Q. 10
– Q. 10 carry
carry two
two marks
marks each.
each.

Q6
Q.6 Students taking an exam are divided into two groups, PP and Q
Q such that each group has the same
number of students. The performance of each of the students in a test was evaluated out of 200
marks. It was observed that the mean ofof group PP was 105,
105, while that of group Q
Q was 85. The
P was 25, while that of group Q
standard deviation of group P Q was 5. Assuming that the marks were
distributed on a normal distribution, which of the following statements will have the highest
TRUE?
probability of being TRUE?
(A) No student in group Q
Q scored less marks than any student in group P.
P.

(B) No student in group P


P scored less marks than any student in group Q.
Q.

of group Q
(C) Most students of Q scored marks in a narrower range than students in group P.
P.

(D) The median of the marks of


of group P
P is 100.
100.

Q7
Q.7 A smart city integrates all modes of transport, uses clean energy and promotes sustainable use of
resources. It also uses technology to ensure safety and security of the city, something which critics
argue, will lead to a surveillance state.

of the following can be logically inferred from the above paragraph?


Which of

(i) All smart cities encourage the formation of surveillance states.


(ii) Surveillance is an integral part of
of a smart city.
(iii) Sustainability and surveillance go hand in hand in a smart city.
(iv) There is a perception that smart cities promote surveillance.

(A) (i) and (iv) only (B) (ii) and (iii) only

(C) (iv) only (D) (i) only

Q.8
Q.8 Find the missing sequence in the letter series.

B, FH, LNP, ____


_ _ _ _.

(A) SUWY
(A) SUWY (B) TUVW
(B) TUVW (C) TVXZ
(C) TVXZ (D) TWXZ
(D) wz

Q.9
Q.9 The binary operation □I: is defined
defined as aa □
E] b = ab+(a+b),
b = ab+(a+b), where a b are any two real numbers.
a and b
The value of
of the identity element of this operation, defined
defined as the number xx such that a
a□ = a,
E] xx = a, for
a,
any a, is .

(A) 00
(A) (B) 11
(B) (C) 22
(C) (D) 10
(D) 10

2/3
2/3
GATE 2016 General Aptitude -- GA Set-2

Q.10 | (| |)|
Q“) Which of the following curves represents the function y = ln(| 1n(|e [|sin (Ix I)” |)
I) for ||x|| < 2
2n??
Here, x represents the abscissa and y represents the ordinate.

1.5

-1.5
(A)
(A)

1.5

(B)
(B) "1-5

(C)
(C)

1.2

D.“

o .

I .4

1.2

D
(D) *211 —:r: 11' 211:
(D)

END OF
END THE QUESTION
OF THE QUESTION PAPER
PAPER
3/3
3/3
GATE 2016
GATE 2016 Mining
Mining Engineering
Engineering (MN)
(MN)

Q. 1—
Q. 1 Q. 25
– Q. 25 carry
carry one
one mark
mark each.
each.

Q.1
Q.1 The differential
The differential of
of the
the equation, x 
equation, x2 y 
+ y2
2
=1,
1, with 2
with respect to x
respect to is
x is

(A) —x/y
(A) x y (B) x/y
(B) x y (C) —y/x
(C) y x (D) y/x
(D) y x

Q2
Q.2 If [[A]
If = [[I]
B] 
A][[B] I ] then
then

(A) [[B]
(A) =[A1T
B]  [ A]T (B) [[A]
(B) =[B]T
A]  [ B]T (C) [[B]
(C) =[A1‘1
B]  [ A]1 (D) B] 
(D) [[B] =[A]
[ A]

Q3
Q.3 X 44 
X + C is
is the general integral
the general integral of
of

3! x3dx
(A) 3 x 3 dx 1 3
 x dx
(C) Ix3dx 4] x3dx
(D) 4
3
x 3 dx
44 
(A) (C) (D)
(B)
(B) x dx
lIx3dx

Q4
Q.4 Sink (x)
Sinh (x) is
is

— e6”
eexx  x — e6”
eexx  x
e6"x  x
+ ee‘x e6"x  x
+ e6”
(A)
(A) (B)
(B) (C)
(C) (D)
(D)
44 22 22 44

Q5
Q.5 Identify the correct
Identify the statement.
correct statement.
NONEL is used
NONEL of the
used for surface connection of the blast holes in order to
blast holes
(A) achieve
(A) better water
achieve better resistance over
water resistance over detonating
detonating fuse
fuse
(B) have
(B) have aa precise timing
delay timing
precise delay
(C) provide
(C) noiseless shock
provide noiseless front movement
shock front movement
(D) avoid
(D) avoid deflagration
deflagration

Q6
Q.6 Identify the
Identify the pattern of surface
pattern of surface blasting given in
blasting given in the figure. The
the figure. The values of delay
values of delay time, in ms,
time, in ms,
are given
are against each
given against each blasthole.
blasthole.

H OO
100 75 50 25 25 50 7 U'l
75 100
0% CO.

.0.
50 25 0 0 00. 0 N5
25 50

(A) V- cut
(A) cut
(B) extended
(B) extended V-
V- cut
cut
(C) row
(C) to row
row to row
(D) en
(D) en echelon
echelon

MN 1/14
1/14
GATE 2016
GATE 2016 Mining
Mining Engineering
Engineering (MN)
(MN)

Q.7
Q.7 Identify the
Identify the initiation
initiation sequence
sequence which
which is
is NOT
NOT possible for surface
possible for surface blasting.
blasting.
(A) Detonating
(A) Detonating fuse
fuse NonelElectronic
9NoneléElectronic detonator
detonator
(B) Electric
(B) Electric detonatorNonelDetonating
detonatoréNonel9Detonating fuse
fuse
(C) Electric detonator
(C) Electric Detonating fuse
detonatoré Detonating fuse 
9 Nonel
Nonel
(D) Electronic
(D) Electronic detonator Detonating fuse
detonatoré Detonating fuse 
9 Nonel
Nonel

Q.8
Q.8 Parallel holes
Parallel holes at
at right
right angles
angles to
to the face with
the face with some
some holes
holes uncharged are associated
uncharged are associated with
with the
the
following shot
following shot hole
hole pattern
pattern
(A) drag
(A) drag cut
cut (B) wedge
(B) cut
wedge cut (C) pyramid
(C) cut
pyramid cut (D) burn
(D) cut
burn cut

Q.9
Q.9 Bieniawski’s Rock
Bieniawski’s Rock Mass
Mass Rating
Rating considers
considers the
the parameters: RQD, spacing
parameters: RQD, spacing of
ofjjoints,
oints, condition
condition of
of
joints, ground water
joints, ground condition, and
water condition, and
(A) tensile
(A) strength
tensile strength
(B) uniaxial
(B) compressive strength
uniaxial compressive strength
(C) shear
(C) shear strength
strength
(D) buckling
(D) strength
buckling strength

Q.10
Q.10 A rockmass
A rockmass is is subjected
subjected to
to hydrostatic
hydrostatic pressure of 6
pressure of 6 MPa.
MPa. If
If each
each of
of the
the measured strains
measured strains
2 8W
8)“xx  yy
2
 
6‘22
zz ,
, is
is 2.0
2.0 mm/m,
mm/m, then
then the
the bulk
bulk modulus,
modulus, in
in GPa,
GPa, is
is _______

Q.11
Q.11 Identify the
Identify the uniaXial compressive loading
uniaxial compressive condition from
loading condition from the four Mohr
following four
the following Mohr circles.
circles.

'E ‘l:

3» O 3» O
(1) (2}

“I: t

(3) (4}

(A) (1)
(A) (1) (B) (2)
(B) (2) (C) (3)
(C) (3) (D) (4)
(D) (4)

MN 2/14
2/14
GATE 2016
GATE 2016 Mining
Mining Engineering
Engineering (MN)
(MN)

Q.12
Q.12 Out of
Out of the
the given stress-strain curves,
given stress-strain curves, identify the rock
identify the rock type
type that is most
that is most prone
prone to
to rock
rock burst.
burst.

A (1)
(1)

(2)
(2)
σ0'

3
(3)
(4)

>
0,0 ε8
(A) (1)
(A) (1) (B) (2)
(B) (2) (C) (3)
(C) (3) (D) (4)
(D) (4)

Q.13
Q.13 A longwall
A longwall panel of width
panel of width 120
120 m
m is
is extracted
extracted at
at aa depth
depth of
of 200
200 m.
m. Critical
Critical subsidence
subsidence is
is reached
reached
when the
when the panel length becomes
panel length 150 m.
becomes 150 If the
m. If seam were
the seam to be
were to be worked at aa depth
worked at depth of
of 300
300 m, critical
m, critical
subsidence would
would be panel length, in m,
be observed at a panel m, of _______.

Q.14
Q.14 The support
The support system
system followed
followed along
along the
the goaf
goaf edge
edge in
in aa depillaring
depillaring panel is
panel is
(A) rope
(A) stitching
rope stitching
(B) cable
(B) cable bolting
bolting
(C) wooden/steel
(C) chock
wooden/steel chock
(D) hydraulic
(D) hydraulic prop
prop

Q.15
Q.15 Which one
Which one of
of the
the following
following ropes CANNOT be
ropes CANNOT an effective
be an effective cable
cable bolt?
bolt?
(A) locked
(A) locked coil
coil wire
wire rope
rope
(B) Langs
(B) Langs lay
lay wire rope
wire rope
(C) ordinary
(C) ordinary lay
lay wire
wire rope
rope
(D) bird-caged
(D) wire rope
bird-caged wire rope

Q.16
Q.16 In metalliferous
In metalliferous mines,
mines, the
the sublevel
sublevel interval
interval does
does NOT depend on
NOT depend on
(A) capacity
(A) capacity of
of drilling
drilling equipment
equipment
(B) capacity
(B) capacity of
of loading
loading equipment
equipment
(C) strength
(C) strength of rib pillar
of rib pillar
(D) strength
(D) strength of
of wall
wall rock
rock

MN 3/14
3/14
GATE 2016
GATE 2016 Mining
Mining Engineering
Engineering (MN)
(MN)

Q.17
Q.17 Jack hammer does NOT
NOT contain
(A) pawl
(A) and ratchet
pawl and ratchet
(B) gear
(B) gear box
box
(C) rifle
(C) rifle bar
bar
(D) piston
(D) piston

Q.18
Q.18 At the
At the inlet of aa mine
inlet of mine roadway,
roadway, thethe dry and wet
dry and wet bulb
bulb temperatures of air
temperatures of are 38
air are 380C
0
and 29
C and 290C
0
C ,,
respectively. At the
the outlet,
outlet, the
the corresponding
corresponding temperatures are 32
temperatures are 320C and 29290CC ,, respectively. The
0 0
respectively. At C and respectively. The
process of heat
process of heat transfer in the
transfer in airway is
the airway is described
described as
as
(A) evaporative
(A) evaporative cooling
cooling
(B) sensible
(B) sensible cooling
cooling
(C) sensible
(C) sensible heating
heating
(D) dehumidification
(D) dehumidification

Q19
Q.19 Underground coal
Underground coal mines are in
mines are in principle
principle ventilated
ventilated by exhausting system,
by exhausting system, so
so that
that
(A) spontaneous
(A) spontaneous heating
heating risk is reduced
risk is reduced
(B) fumes
(B) fumes can
can be quickly removed
be quickly removed in in case
case of
of an
an underground fire
underground fire
(C) build-up
(C) of methane
build-up of methane concentration
concentration isis decreased
decreased
(D) cool
(D) cool and
and fresh
fresh intake
intake air
air can
can enter
enter underground
underground

Q20
Q.20 Identify the
Identify the WRONG
WRONG statement.
statement.
Pit bottom
Pit air lock
bottom air lock
(A) prevents
(A) the short
prevents the short circuiting of air
circuiting of air when
when the is reversed
flow is
the flow in coal
reversed in coal mines
mines
(B) has
(B) at least
has at least three doors
three doors
(C) has
(C) at least
has at least one
one door
door that
that has
has provision for latching
provision for latching
(D) all
(D) all doors
doors are
are in
in principle designed to
principle designed to open towards high
open towards high pressure side of
pressure side of the air
the air

Q21
Q.21 Identify the
Identify the WRONG
WRONG statement.
statement.
The ‘temperature
The ‘temperature inversion’ of the
inversion’ of the atmosphere
atmosphere in surface mines
in surface aggravates the
mines aggravates the problem of
problem of
(A) airborne
(A) airborne dust
dust
(B) noise
(B) noise
(C) ground
(C) ground vibrations
vibrations
(D) visibility
(D) visibility

Q22
Q.22 In aa CO
In CO self
self rescuer,
rescuer, the
the purpose of the
purpose of calcium bromide
the calcium and lithium
bromide and lithium chloride
chloride mixture is to
mixture is to
(A) dry
(A) the incoming
dry the incoming air
air
(B) convert
(B) convert the
the CO
CO catalytically
catalytically to
to CO
CO22
(C) absorb
(C) absorb and
and thereby
thereby neutralise
neutralise CO
CO
(D) cool
(D) cool the
the inhaled
inhaled air
air from
from excess
excess exothermic
exothermic heat
heat due
due to chemical reaction
to chemical reaction

MN 4/14
4/14
GATE 2016
GATE 2016 Mining
Mining Engineering
Engineering (MN)
(MN)

Q.23
Q.23 IRR of
IRR of aa project is the
project is discount rate
the discount rate at
at which
which
(A) profit
(A) after tax
profit after taX is
is zero
zero
(B) written
(B) down value
written down of the
value of the project is zero
project is zero
(C) revenue
(C) from the
revenue from the project is zero
project is zero
(D) NPV
(D) NPV isis zero
zero

Q24
Q.24 For the
For critical path
the critical path network shown, the
network shown, slack for
the slack for the activity ‘b’,
the activity in months,
‘b’, in months, is
is

activity Duration
{months}
a 4
b 3
c 5
d 4
e 7

(A)4
(A) 4 (B) 6
(B) 6 (C) 9
(C) 9 (D) 13
(D) 13

Q.25
Q.25 The three
The three axes
axes comprising
comprising the
the numerical codification of
numerical codification of resources, as per
resources, as per the
the UNFC, are
UNFC, are
(A) Economic
(A) Economic Viability, Geological Assessment,
Viability, Geological Assessment, Geotechnical
Geotechnical Assessment
Assessment
(B) Geological
(B) Geological Assessment, Environmental Assessment,
Assessment, Environmental Feasibility Assessment
Assessment, Feasibility Assessment
(C) Feasibility
(C) Feasibility Assessment,
Assessment, Geological
Geological Assessment,
Assessment, Mining
Mining Assessment
Assessment
(D) Economic
(D) Economic Viability,
Viability, Geological
Geological Assessment, Feasibility Assessment
Assessment, Feasibility Assessment

Q. 26 —
Q. 26 Q. 55
– Q. 55 carry
carry two
two marks
marks each.
each.

Q26
Q.26 Equations of
Equations of two
two planes are Zz 
planes are = 4 and =4
and Zz  4
+33x
x .. The included angle
The included angle between
between the
the two
two planes in
planes in
degrees, is
degrees, is ____________


Q27
Q.27 A force
A P
force I3 22 —5
2iiˆ  5}ˆj 
+6 kˆ acts
6]; acts on
on aa particle.
particle. TheThe particle is moved
particle is from point
moved from point A to point
A to B, where
point B, where
 
the
the position vectors of
position vectors of A and B
A and E are
are 6 6?iˆ  —3
+ jˆj  kˆ and
3/; and 4 —3
4fiˆ  —2
3}ˆj  kˆ respectively.
2]; The work
respectively. The done is
work done is
____________

Q28
Q.28 The value
The of xx in
value of in the
the simultaneous
simultaneous equations
equations is
is ______________

x  y  2z  3
33x+y+22=3
x  3 y  z  3
22x—3y—22—3
x  2y  z  4
x+2y+z=4

MN 5/14
5/14
GATE 2016
GATE 2016 Mining
Mining Engineering
Engineering (MN)
(MN)

Q.29
Q.29 Two persons
Two persons PP and
and QQ toss
toss an
an unbiased coin alternately
unbiased coin alternately on
on an
an understanding
understanding that
that whoever gets the
whoever gets the
head first wins.
head first wins. If
If P starts the
P starts game, then
the game, then the
the probability of P
probability of P winning
winning the game is
the game is ____________

Q30
Q.30 Data pertaining
Data pertaining to
to aa surface
surface bench
bench blast is given
blast is given below:
below:

Burden =
Burden = 3.0
3.0 m
m Sub-grade drilling
Sub-grade = 1.0
drilling = 1.0 m
m
= 4.0 m
Spacing = 4.0
Spacing m Collar stemming
Collar = 4.0 m
stemming = 4.0 m
Bench height
Bench =
height = 10.0
10.0 m
m Air decking
Air decking length =
length = 1.0
1.0 m
m
Density of
Density of rock = kg/m33
2000 kg/m
rock = 2000 Linear charge
Linear charge concentration = 10
concentration = 10 kg/m
kg/m

The powder
powder factor of the
the blast,
blast, in kg/tonne,
kg/tonne, is _______

Q31
Q.31 Match the
Match following for
the following for aa typical
typical slurry
slurry explosive.
explosive.

Chemical
Chemical Purpose
Purpose

P
P. Calcium nitrate
Calcium nitrate 1.
1. Cross linking
Cross linking agent
agent
Q.
Q. Potassium dichromate
Potassium dichromate 2.
2. Gelling agent
Gelling agent
R
R. TNT
TNT 3.
3. Oxidiser
Oxidiser
S
S. Starch
Starch 4.
4. Fuel
Fuel
(A) P-1,
(A) P-l, Q-2,
Q-2, R-3,
R-3, S-4
8-4
(B) P-2,
(B) P-2, Q-4,
Q-4, R-3,
R-3, S-1
S-l
(C) P-3,
(C) Q-l, R-4,
P-3, Q-1, R-4, S-2
S-2
(D) P-4,
(D) P-4, Q-3,
Q-3, R-2,
R-2, S-1
8-1

Q32
Q.32 A 10
A 10 m
m thick
thick coal
coal block is excavated
block is excavated by
by aa contractor
contractor at
at aa cost
cost of
of Rs.
Rs. 40
40 per
per m3
m3 .. The
The excavated
excavated
2
area, measured in the
the mine
mine plan,
plan, is found to
to be cm2.. If the
be 50 cm the mine
mine plan
plan has
has been
been drawn to
to a scale
of 1:1000,
of 121000, the
the payment
payment to
to be
be made
made to the contractor,
to the contractor, in
in lakhs
lakhs ofof Rs.,
Rs., is
is _______

Q33
Q.33 Two vertical
Two vertical shafts of aa mine
shafts of have the
mine have following parameters:
the following parameters:

Shaft
Shaft Shaft-A
Shaft‐A Shaft-B
Shaft‐B
Collar RL
Collar RL (m)
(m) 0.0
0.0 0.0
0.0
Depth (m) 250 200
Northing(m)
Northing (m) 200
200 100
100
Easting (m) 100 -100
‐100
The gradient
The gradient of
of the
the drift
drift connecting
connecting the
the shaft
shaft bottoms, in degrees,
bottoms, in degrees, is_____
is

MN 6/14
6/14
GATE 2016
GATE 2016 Mining
Mining Engineering
Engineering (MN)
(MN)

Q.34
Q.34 For aa station
For station ‘A’
‘A’ on
on the
the Earth’s
Earth’s surface,
surface, as
as shown in the
shown in the figure,
figure, match the following
match the following
P
i
g A
A
!
!
Equator
_______ ____Oi
O ____________

Reference meridian
L M
i
!
E
P,
P’

Arc
Arc Description
Description
MA
Q. MA
Q. 1. Longitude
1. Longitude
R. LM
R. 2. Co-latitude
S. PA 3. Latitude

(A) Q-2,
(A) Q-2, R-3, S-1
8-1
(B) Q-3, R-1,
(B) Q-3, R-l, S-2
(C) Q-2,
(C) Q-2, R-1,
R-l, S-3
(D) Q-3,
(D) Q-3, R-2, S-1
S-l

Q35
Q.35 Match the following
Match the following for
for the
the prismatic compass shown
prismatic compass shown below
below
| x1
:
I; Horse hair
I. .
Reflecting mirror \ ,
. in Slide
Hinged sun 9133595
1 '
I Lufling pin X3 “fl . Eye vane
'
Glass cover Prism cap . "'7’"..\-.:._=in..-i

' --
'

L
Graduated circle fix§§

a
X2 "°‘_-_—._.'- 'o

m _ it“ Focussing stud Ior prism


BMW" “3 .
i‘:
LEE—fr:
—- Pivot—K
I,
:1- ,
H==-—— “ "—
r :..==- *
Jtflim" '
/ Liflinglavar Hider Hinged strap
X4 Screw for slaif or tripod Metal box

Component
Component Name
Name

P.
P. X1
X1 1.
1. Agate bearing
Agate bearing
Q.
Q. X2
X2 2.
2. Object vane
Object vane
R.
R. X3
X3 3.
3. Magnetic needle
Magnetic needle
S.
S. X4
X4 4.
4. Prism
Prism
(A) P-1,
(A) P-l, Q-2,
Q-2, R-3,
R-3, S-4
8-4
(B) P-1,
(B) P-l, Q-3,
Q-3, R-2,
R-2, S-4
S-4
(C) P-2,
(C) P-2, Q-1,
Q-l, R-4,
R-4, S-3
S-3
(D) P-3,
(D) P-3, Q-1,
Q-l, R-4,
R-4, S-2
S-2

Q36
Q.36 A ladder
ladder placed frictionless wall
against aa frictionless at an inclination of
an inclination of 60 with horizontal,
600 with is in
in aa state
state of
of
0
A placed against wall at horizontal, is
limiting equilibrium.
limiting equilibrium. The
The ladder
ladder has
has aa length
length of
of 13
13 m and aa uniform
m and mass of
uniform mass of 4
4 kg/m. The
kg/m. The
coefficient of
coefficient of friction
friction between
between the ladder and
the ladder and the
the floor
floor is
is ______

MN 7/14
7/14
GATE 2016
GATE 2016 Mining
Mining Engineering
Engineering (MN)
(MN)

Q37
Q.37 A cubical
A cubical rock
rock sample
sample isis enclosed
enclosed between
between two fixed hard
two fixed steel plates
hard steel as shown
plates as shown inin the figure below.
the figure below.
The modulus
The modulus of
of elasticity
elasticity and
and Poisson’s
Poisson’s ratio
ratio of
of the
the rock are 2
rock are 2 GPa
GPa and
and 0.25,
0.25, respectively.
respectively. If If the
the
rock is
rock is subjected
subjected to
to the stresses as
the stresses as shown in the
shown in figure, the
the figure, the strain in x-direction,
strain in X-direction, in
in mm/m,
mm/m, is is
_______.

10 MPa

Rock block
Steel plate
Steel plate9
55Mpa
Mpa

8 MPa

Q38
Q.38 In aa hydrostatic
In stress field,
hydrostatic stress field, point
point A is in
A is in the
the middle of two
middle of circular openings
two circular openings as shown in
as shown in the
the
figure. The radial
figure. radial stress, in MPa, at point
point A A is _________.

M Pa
5 MPa
i i

A
5 MPa 0 55MPa
MPa
/2.:m
2.5 m 2.5m
2.5 m

T Tm
\ 10
10mm /

5 MPa

MN 8/14
8/14
GATE 2016
GATE 2016 Mining
Mining Engineering
Engineering (MN)
(MN)

Q39
Q.39 Curves (a)
Curves (a) and
and (b)
(b) represent
represent the stress distributions
the stress distributions along
along the length of
the length of aa ‘full
‘full column
column grouted
grouted bolt’
bolt’
shown in the
shown in the figure.
figure. Curves
Curves (a)
(a) and
and (b)
(b) are
are

Stress
Stress

(a) (b)
Rock surface a .- :'-_
Grouted material
Bolt
0,0 Length
\l
Pickup length Anchor length
Anchor IN

(A) Tensile
(A) Tensile stress,
stress, Compressive
Compressive stress
stress
(B) Axial
(B) Axial stress,
stress, Shear
Shear stress
stress
(C) Compressive
(C) Compressive stress,
stress, Tensile
Tensile stress
stress
(D) Shear
(D) Shear stress,
stress, Axial
Axial stress
stress

Q.40
Q.40 Match the
Match following mechanical
the following mechanical properties with the
properties with formulae
the formulae

Mechanical property
Mechanical property Formula
Formula
P. Modulus
P. Modulus of
of elasticity
elasticity 1. c
1. c+ n tan
antangp
2. 
lateral / 
Q. Compressive
Q. Compressive strength
strength / longitudinal
2. glateral glongitudinal
R. Shear
R. Shear Strength
Strength 3. 
3. 0 // 6‘
S. Poisson’s
S. Poisson’s ratio
ratio 4.
4. Fn // 
F flrz
r2
(A) P-1,
(A) P-l, Q-2,
Q-2, R-3,
R-3, S-4
S-4
(B) P-1,
(B) P-l, Q-4,
Q-4, R-3,
R-3, S-2
S-2
(C) P-3,
(C) Q-4, R-1,
P-3, Q-4, R-l, S-2
S-2
(D) P-3,
(D) P-3, Q-2,
Q-2, R-1,
R-l, S-4
S-4

Q.41
Q.41 A skip
A skip of
of 10
10 tonne
tonne capacity
capacity hoists
hoists ore
ore through
through aa 1000
1000 m
m deep
deep shaft
shaft at
at aa speed
speed of
of 20
20 m/s. The skip
m/s. The skip
accelerates and
and decelerates
decelerates at
at 2.0
2.0 m/s
Ill/$2.. The
The loading
loading and
and unloading
unloading times for the
the skip are 2.5
skip are 2.5 min
2
accelerates times for min
and 1.5
and 1.5 min,
min, respectively. The maximum
respectively. The hourly capacity
maximum hourly capacity of
of the
the hoisting system, in
hoisting system, in tonnes, is
tonnes, is
_____________

Q.42
Q.42 the following:
Match the
Match following:

Haulage unit
Haulage unit Safety device
Safety device
P.
P. Friction winder
Friction winder 1.
1. Run-away switch
Run-away switch
Q.
Q. Drum winder
Drum winder 2.
2. Lilly controller
Lilly controller
R.
R. Direct rope
Direct rope haulage
haulage 3.
3. Regenerative braking
Regenerative braking
S.
S. Endless rope
Endless haulage
rope haulage 4.
4. Monkey/back catch
Monkey/back catch

(A) P-1,
(A) P—l, Q-2,
Q—2, R-3,
R—3, S-4
8—4
(B) P-3,
(B) P—3, Q-2,
Q—2, R-1,
R—l, S-4
S—4
(C) P-1,
(C) P—l, Q-3,
Q—3, R-4,
R—4, S-2
S—2
(D) P-2,
(D) P—2, Q-3, R—l, S-4
Q—3, R-1, S—4

MN 9/14
9/14
GATE 2016
GATE 2016 Mining
Mining Engineering
Engineering (MN)
(MN)

Q.43
Q.43 In the
In the gear
gear assembly
assembly shown,
shown, the
the rpm of Gear
rpm of Gear 11 is
is 600.
600. The
The number of teeth
number of in Gear
teeth in Gear 1,
1, Gear
Gear 2,
2, Gear
Gear
3, Gear 4, Gear 5 and Gear 6 is 30, 45, 15, 20, 10
45, 15, respectively. The rpm
10 and 30, respectively. rpm of Gear 6 is
______

Gear 1

Gear 6

Q.44
Q.44 An operating
An operating surface
surface mine
mine is
is proposed
proposed toto be deepened by
be deepened 30 m
by 30 m as
as shown
shown inin the figure. If
the figure. If the
the
density of the ore is 2.4 tonne/m3 the incremental stripping ratio for the deepening, in
3
density of the ore is 2.4 tonne m , the incremental stripping ratio for the deepening, in
3 .
m3 /tonne 1s ______.
tonne , is
Proposed line of deepening

/ Overburden

150 m

300
100 m 300 30 m Ore

30 m
300
100 m

Q.45
Q.45 From an
From an openpit sump, mine
openpit sump, water is
mine water is lifted using aa 250
lifted using 250 m long straight
m long straight pipeline laid along
pipeline laid along aa
gradient of
gradient of 340.. The
34 0
The pumping rate is
pumping rate is 500
500 gpm
gpm (1(1 gallon =
gallon = 3.8
3.8 litres).
litres). Additional
Additional head loss due
head loss due to
to
pipe friction can
pipe friction can be considered to
be considered to be 10% of
be 10% head lifted.
of head lifted. At
At anan overall efficiency of
overall efficiency of 70%,
70%, the
the
electric power
electric consumed by
power consumed by the
the pump,
pump, in in kW, is __________.
kW, is

MN 10/14
10/14
GATE 2016
GATE 2016 Mining
Mining Engineering
Engineering (MN)
(MN)

Q.46
Q.46 With reference
With to Coward
reference to Coward diagram,
diagram, match the following
match the following in
in the context of
the context of explosibility
explosibility of
of aa mixture
mixture
of ‘normal
of ‘normal air’
air’ and
and ‘methane’.
‘methane’.

(022 %, CH
(O CH44 %) Mixture status

P. 20.5,
P. 20.5, 2.4
2.4 1. Impossible
1. Impossible mixture
mixture
Q. 19.0,
Q. 19.0, 9.5
9.5 2. Non-explosive
2. Non-explosive
R. 17.0,
R. 17.0, 19.0
19.0 3. Potentially
3. Potentially explosive
explosive
S. 20.0,
S. 20.0, 19.5
19.5 4. Explosive
4. Explosive

(A) P-2,
(A) 13—2, Q-4,
Q-4, R-3,
R-3, S-1
s—1
(B) P-2,
(B) P-2, Q-3, R—l, S-4
Q-3, R-1, s—4
(C) P-2,
(C) P-2, Q-4,
Q-4, R-1,
R—l, S-3
53
(D) P-3,
(D) 13—3, Q-2,
Q-2, R-1,
R—l, S-4
s—4

Q47
Q.47 A U-tube
A manometer is
U-tube manometer is subjected
subjected to differential pressure
to differential as shown.
pressure as If specific
shown. If gravity of
specific gravity of kerosene
kerosene
is 0.8,
is 0.8, the
the value of (P1
value of P1  P2 ) ,, in
—P2 in Pa,
Pa, is_________.
is
|31 P2

Kerosene

Water

Q.48
Q.48 An air
An air stream
stream having
having anan enthalpy
enthalpy of
of 100
100 kJ/kgda,
kJ/kgda, isis flowing
flowing atat 20
20 kgda/s. It is
kgda/s. It is cooled
cooled by
by water at
water at
temperature 100 circulating
circulating in
in aa cooling
cooling coil
coil at
at aa flow
flow rate of 10.0
10.0 l/s. If the
US. If the return
return temperature of
0
temperature 10 C rate of temperature of
is 20
water is 200 C , the enthalpy of
the enthalpy of the air, in
cooled air, in kJ/kgda, is ___________.
0
water the cooled kJ/kgda, is
(Specific heat
(Specific of water:
heat of 4.18 kJ
water: 4.18 [cl/kgkg 0C
0
C ;; kgda:
kgda: kg
kg ofof dry
dry air).
air).

MN 11/14
11/14
GATE 2016
GATE 2016 Mining
Mining Engineering
Engineering (MN)
(MN)

Q49
Q.49 The static
The static pressure
pressure characteristic of aa mine
characteristic of mine fan
fan is
is as
as shown.
shown. If
If the mine resistance
the mine is 0.3
resistance is 0.3 NSZ/m8
Ns 2 m8 ,,
the quantity
quantity generated
generated by the fan,
fan, in
in m m3 /Ss , is
is _______.
3
the by the

1000
1000

P (Pa)
(Pa)
P

0.0
0.0 100
100
Q (m
Q /s) 9
(m3/s)
3

Q.50
Q.50 In the
In the context
context of
of ventilation
ventilation plan symbols, match
plan symbols, the following:
match the following:

Symbol
Symbol Description
Description

P.
P. M 1. Temporary
1. Temporary stopping
stopping
7W
Q.
Q. 2. Regulator
2. Regulator

R —B
R. R 3. Air-crossing
3. Air-crossing

S.
S. j; 4. Ventilation
4. Ventilation stopping
stopping

(A)P—3,
(A) Q———4,R2,Sl
P-3, Q-4, R-2, S-1
(B)P—2,
(B) Q——3,R1,S—4
P-2, Q-3, R-1, S-4
(C)P—l,
(C) Q—3,R——4,S2
P-1, Q-3, R-4, S-2
(D)P—3,Q——2,R
(D) l,S—4
P-3, Q-2, R-1, S-4

Q51
Q.51 A mill
A mill concentrate,
concentrate, having 25% copper,
having 25% copper, is
is proposed to be
proposed to sold at
be sold at Rs.
Rs. 1,25,000
1,25 ,000 per tonne. The
per tonne. The grade
grade
of the
of the deposit is 0.8%
deposit is 0.8% Cu
Cu and
and the overall cost
the overall of mining
cost of and milling
mining and is Rs.
milling is Rs. 2,520
2,520 per tonne of
per tonne of ore.
ore.
At a recovery of 75%, the
the estimated profit, of concentrate, is ________.
profit, in Rs./tonne of

MN 12/14
12/14
GATE 2016
GATE 2016 Mining
Mining Engineering
Engineering (MN)
(MN)

Q52
Q.52 Copper grade
Copper grade distribution
distribution in
in an
an ore
ore body
body has the probability
has the density function,
probability density f (x),
function, f x  , as in
shown in
as shown
the figure.
the The average
figure. The average grade
grade of
of the in %
deposit, in
the deposit, % Cu,
Cu, is
is ______.

fl?!)

0.0 0.5 2.0


% Cu .7:

Q.53
Q.53 The semivariogram
The shown belongs
semivariogram shown to aa bauxite
belongs to deposit. The
bauxite deposit. The expected
expected difference
difference in
in the
the A1203 (%)
Al2O3 (%)
values between
values two boreholes
between two separated by
boreholes separated by aa distance
distance of
of 200
200 m is _______.
m is

300 (%)
300 (%)22
T
γv (h)
(M

100 (%)
100 (%)Z2

0.0 300 m
Lag distance %

Q54
Q.54 A surface
A surface mine has 15
mine has 15 identical
identical dumpers
dumpers and
and two shovels. For
two shovels. shovel 1,
For shovel 1, the dumper cycle
the dumper cycle time is
time is
30 min
30 min and
and the
the shovel
shovel loading time is
loading time is 5
5 min.
min. For
For shovel
shovel 2,
2, the
the dumper
dumper cycle
cycle time is 32
time is 32 min and the
min and the
shovel loading
shovel loading time is 4.0
time is 4.0 min.
min. Based
Based on
on match factor optimisation
match factor optimisation (equitable
(equitable match
match factor),
factor), the
the
ideal allocation
ideal allocation of
of dumpers
dumpers to shovel 11 and
to shovel shovel 2,
and shovel 2, respectively is
respectively is
(A) 6,
(A) 6, 99 (B) 7,
(B) 7, 88 (C) 9,
(C) 9, 66 (D) 8,
(D) 8, 77

MN 13/14
13/14
GATE 2016
GATE 2016 Mining
Mining Engineering
Engineering (MN)
(MN)

Q.55
Q.55 The composited
The composited grade
grade value,
value, in %, between
in %, between the RLs 10
the RLs 10 m
m to 20 m
to 20 for the
m for following borehole
the following borehole
is __________.
configuration is
configuration

RL=Om
RL =0m
'
/// /// /// ////\/// /// ///

25%
25% 14m
14 m

35%
18m
18 m

END OF
END THE QUESTION
OF THE QUESTION PAPER
PAPER

MN 14/14
14/14
Q. No
Q. No Type Section
Type Section Key
Key Marks
Marks
11 MCQ
MCQ GA BB 11
22 MCQ
MCQ GA A
A 11
33 MCQ
MCQ GA DD 11
44 MCQ
MCQ GA CC 11
55 MCQ
MCQ GA BB 11
66 MCQ
MCQ GA CC 22
77 MCQ
MCQ GA CC 22
88 MCQ
MCQ GA CC 22
99 MCQ
MCQ GA A
A 22
10
10 MCQ
MCQ GA CC 22
11 MCQ
MCQ MN
MN A
A 11
22 MCQ
MCQ MN
MN CC 11
33 MCQ
MCQ MN
MN DD 11
44 MCQ
MCQ MN
MN BB 11
55 MCQ
MCQ MN
MN CC 11
66 MCQ
MCQ MN
MN BB 11
77 MCQ
MCQ MN
MN A
A 11
88 MCQ
MCQ MN
MN DD 11
99 MCQ
MCQ MN
MN BB 11
10
10 NAT
NAT MN
MN 1.00 :: 1.00
1.00 1.00 11
11
11 MCQ
MCQ MN
MN A
A 11
12
12 MCQ
MCQ MN
MN DD 11
13
13 NAT
NAT MN
MN 225.00 :: 225.00
225.00 225.00 11
14
14 MCQ
MCQ MN
MN CC 11
15
15 MCQ
MCQ MN
MN A
A 11
16
16 MCQ
MCQ MN
MN BB 11
17
17 MCQ
MCQ MN
MN BB 11
18
18 MCQ
MCQ MN
MN A
A 11
19
19 MCQ
MCQ MN
MN CC 11
20
20 MCQ
MCQ MN
MN CC 11
21
21 MCQ
MCQ MN
MN CC 11
22
22 MCQ
MCQ MN
MN A
A 11
23
23 MCQ
MCQ MN
MN DD 11
24
24 MCQ
MCQ MN
MN BB 11
25
25 MCQ
MCQ MN
MN DD 11
26
26 NAT
NAT MN
MN 100.00 :: 110.00
100.00 110.00 22
27
27 NAT
NAT MN
MN 22.00 :: 22.00
22.00 22.00 22
28
28 NAT
NAT MN
MN 1.00 :: 1.00
1.00 1.00 22
29
29 NAT
NAT MN
MN 0.66 :: 0.67
0.66 0.67 22
30
30 NAT
NAT MN
MN 0.25 :: 0.25
0.25 0.25 22
31
31 MCQ
MCQ MN
MN CC 22
32
32 NAT
NAT MN
MN 20.00 :: 20.00
20.00 20.00 22
33
33 NAT
NAT MN
MN 12.00 :: 13.00
12.00 13.00 22
34
34 MCQ
MCQ MN
MN BB 22
35
35 MCQ
MCQ MN
MN CC 22
36
36 NAT
NAT MN
MN 0.28 :: 0.30
0.28 0.30 22
37
37 NAT
NAT MN
MN 0.20 :: 0.30
0.20 0.30 22
38
38 NAT
NAT MN
MN 7.00 :: 8.00
7.00 8.00 22
39
39 MCQ
MCQ MN
MN DD 22
40
40 MCQ
MCQ MN CC 22
41
41 NAT
NAT MN
MN 100.00 :: 110.00
100.00 110.00 22
42
42 MCQ
MCQ MN DD 22
43
43 NAT
NAT MN
MN 100.00 :: 100.00
100.00 100.00 22
44
44 NAT
NAT MN
MN 0.55 :: 0.75
0.55 0.75 22
45
45 NAT
NAT MN
MN 60.00 :: 70.00
60.00 70.00 22
46
46 MCQ
MCQ MN A
A 22
47
47 NAT
NAT MN
MN -960.00 :: -930.00
-960.00 -930.00 22
48
48 NAT
NAT MN
MN 78.00 :: 80.00
78.00 80.00 22
49
49 NAT
NAT MN
MN 43.00 :: 44.00
43.00 44.00 22
50
50 MCQ
MCQ MN A
A 22
51
51 NAT
NAT MN
MN 19500.00 :: 20500.00
19500.00 20500.00 22
52
52 NAT
NAT MN
MN 0.77 :: 0.84
0.77 0.84 22
53
53 NAT
NAT MN
MN 0.00 :: 0.00
0.00 0.00 22
54
54 MCQ
MCQ MN
MN A
A 22
55
55 NAT
NAT MN
MN 31.00 :: 31.00
31.00 31.00 22

You might also like